31 Years NEETChapterwise Topicwise Solved CHEMISTRY

You might also like

Download as pdf or txt
Download as pdf or txt
You are on page 1of 296

Chapterwise - Topicwise

CHEMISTRY
EBD_7324
nd
• Corporate Office : 45, 2 Floor, Maharishi Dayanand Marg, Corner Market,
Malviya Nagar, New Delhi-110017

Tel. : 011-49842349 / 49842350

By : Preetima Bajpai

Typeset by Disha DTP Team

DISHA PUBLICATION
ALL RIGHTS RESERVED

© Copyright Publisher

No part of this publication may be reproduced in any form without prior permission of the
publisher. The author and the publisher do not take any legal responsibility for any errors or
misrepresentations that might have crept in. We have tried and made our best efforts to provide
accurate up-to-date information in this book.

For further information about the books from DISHA,


Log on to www.dishapublication.com or email to info@dishapublication.com

(ii)
Index
NEET Solved Paper 2018 2018- 1- 2018-12

1. Some Basic Concepts 17. Electrochemistry 149-158


of Chemistry 1-10 18. Chemical Kinetics 159-169
2. Structure of Atom 11-19 19. Surface Chemistry 170-173
3. Classification of Elements 20. General Principles and
and Periodicity in Properties Processes of Isolation of
Elements 174-176
20-25
21. The p-Block Elements (Group 15,
4. Chemical Bonding and 16, 17 and 18) 177-188
Molecular Structure 26-43 22. The d-and f-Block Elements
5. States of Matter 44-50 189-199
6. Thermodynamics 51-61 23. Coordination Compounds
7. Equilibrium 62-79 200-213
8. Redox Reactions 80-82 24. Haloalkanes and Haloarenes
9. Hydrogen 83-85 214-219
10. The s-Block Elements 86-91 25. Alcohols, Phenols and Ethers
11. The p-Block Elements 220-229
(Group 13 & 14) 92-95 26. Aldehydes, Ketones,
12. Organic Chemistry - Some and Carboxylic Acids 230-248
Basic Principles and 27. Amines 249-259
Techniques 96-118 28. Biomolecules 260-268
13. Hydrocarbons 119-130 29. Polymers 269-274
14. Environmental Chemistry 30. Chemistry in Everyday
131-132 Life 275-276
15. The Solid State 133-138 31. Nuclear Chemistry 277-280
16. Solutions 139-148

(iii)
EBD_7324
TREND ANALYSIS OF AIPMT/NEET PAPERS (2009-2018)
CHEMISTRY
Ch. 2009 2010 2011 2012 2013 2014 2015 2016 2016 2017 2018
Chapter Name Ph-1 Ph-2
No.
1 Same Basic Concepts of 1 1 0 0 1 2 1 0 1 1 2 (Q.9, 34)
Chemistry
2 Structure of Atom 3 1 3 2 3 2 0 1 2 1 1 (Q. 32)
3 Classification of Elements 1 2 0 1 0 1 1 1 0 1 0
and Periodicity in
Properties
4 Chemical Bonding and 4 4 2 3 4 3 5 2 3 4 2 (Q. 6, 31)
Molecular structure
5 States of Matter 1 0 4 1 2 1 0 1 0 0 2 (Q. 27, 40)
6 Thermodynamics 2 2 2 3 0 2 0 1 1 2 1 (Q.28)
7 Equilibrium 3 6 2 3 2 4 3 2 2 3 3 (Q. 25, 37, 38)
8 Redox Reactions 0 1 0 2 0 1 0 1 1 0 1 (Q. 24)
9 Hydrogen 0 0 0 0 0 1 0 1 0 0 0
10 The s-Block Elements 1 2 2 1 0 0 2 1 1 3 3 (Q. 10, 29, 35)
11 The p-Block Elements 2 1 1 1 3 0 0 0 1 1 2 (Q. 2, 4)
(gp - 13 & 14)
12 Organic Chemistry-Some 4 7 4 2 4 2 4 0 2 3 2 (Q. 16, 18)
Basic Principles and
Techniques
13 Hydrocarbons 2 2 1 1 1 1 4 3 4 3 2 (Q. 13, 14)
14 Environmental Chemistry 0 0 1 1 1 1 0 1 0 1 1 (Q. 15)
15 The Solid State 1 1 0 2 2 1 1 2 1 1 1 (Q. 30)
16 Solutions 1 3 3 1 1 0 3 2 2 1 0
17 Electrochemistry 3 0 3 1 3 2 1 1 5 1 1 (Q. 36)
18 Chemical Kinetics 4 3 2 2 2 0 2 2 1 2 2 (Q. 26, 33)
19 Surface Chemistry 0 0 1 2 0 1 1 1 2 1 1 (Q. 39)
20 General Principles and 0 0 2 3 0 0 1 1 0 1 1 (Q. 3)
Processes of Isolation of
Elements
21 The p-Block Elements 3 1 0 2 3 1 1 5 3 2 2 (Q. 1, 5)
(gp - 15, 16, 17 & 18)
22 The d and f-Block 2 2 2 1 3 5 5 2 2 2 2 (Q. 43, 45)
Elements
23 Coordination Compounds 2 3 3 1 2 2 2 1 1 4 3 (Q. 1, 5)
24 Haloalkanes and 1 0 0 0 0 1 1 2 2 0 0
Haloarenes
25 Alcohols, Phenols and 3 0 0 1 1 3 1 2 0 3 4 (Q. 12, 19, 21,
Ethers 22)
26 Aldehydes, Ketones and 1 3 1 3 2 1 3 1 1 2 1 (Q. 20)
Carboxylic Acids
27 Amines 1 2 2 0 2 2 1 3 3 2 2 (Q. 11, 17)
28 Biomolecules 2 1 1 3 0 2 0 3 3 1 2 (Q.7, 23)
29 Polymers 1 1 1 2 2 2 1 1 1 1 1 (Q.8)
30 Chemistry in Everyday 1 1 2 0 1 1 1 1 0 1 0
Life
Total Questions 50 50 50 50 45 45 45 45 45 45 45
Note : Number in brackets indicating serial number of question asked in NEET 2018 provided on page No 2018-1-2018-5
(iv)
NEET Solved Paper 2018

1. The correct order of N-compounds in its 6. In the structure of ClF3, the number of lone pair
decreasing order of oxidation states is of electrons on central atom ‘Cl’ is
(1) HNO3, NO, N2, NH4Cl (1) One (2) Two

(2) HNO3, NO, NH4Cl, N2 (3) Three (4) Four


7. The difference between amylose and
(3) NH4Cl, N2, NO, HNO3
amylopectin is
(4) HNO3, NH4Cl, NO, N2
(1) Amylopectin has 1 ® 4 a-linkage and
2. The correct order of atomic radii in group 13
1 ® 6 a-linkage
elements is
(2) Amylose has 1 ® 4 a-linkage and 1® 6
(1) B < Al < In < Ga < Tl
b-linkage
(2) B < Al < Ga < In < Tl
(3) Amylose is made up of glucose and
(3) B < Ga < Al < In < Tl galactose
(4) B < Ga < Al < Tl < In (4) Amylopectin has 1 ® 4 a-linkage and
3. Considering Ellingham diagram, which of the 1® 6 b-linkage
following metals can be used to reduce alumina? 8. Regarding cross-linked or network polymers,
(1) Fe (2) Zn which of the following statements is incorrect?
(3) Cu (4) Mg (1) They contain covalent bonds between
4. Which one of the following elements is unable various linear polymer chains.

to form MF63– ion? (2) They are formed from bi- and tri-functional
monomers.
(1) Ga (2) Al
(3) They contain strong covalents bonds in
(3) In (4) B
their polymer chains.
5. Which of the following statements is not true
(4) Examples are bakelite and melamine.
for halogens?
9. A mixture of 2.3 g formic acid and 4.5 g oxalic
(1) All form monobasic oxyacids
acid is treated with conc. H2SO4. The evolved
(2) All are oxidizing agents
gaseous mixture is passed through KOH pellets.
(3) Chlorine has the highest electron-gain Weight (in g) of the remaining product at STP
enthalpy will be
(4) All but fluorine shows positive oxidation (1) 1.4 (2) 3.0
states (3) 4.4 (4) 2.8
EBD_7324
2018 -2 CHEMISTRY
10. Which of the following oxides is most acidic in (2) o-bromotoluene
nature? (3) p-bromotoluene
(1) MgO (2) BeO (4) 3-bromo-2,4,6-trichlorotoluene
(3) CaO (4) BaO 15. Which oxide of nitrogen is not a common
11. Nitration of aniline in strong acidic medium also pollutant introduced into the atmosphere both
gives m-nitroaniline because due to natural and human activity?
(1) Inspite of substituents nitro group always (1) N2O5 (2) NO2
goes to only m-position. (3) NO (4) N2O
(2) In electrophilic substitution reactions, 16. Which of the following molecules represents
amino group is meta directive. the order of hybridisation sp2, sp2, sp, sp from
(3) In acidic (strong) medium aniline is present left to right atoms?
as anilinium ion. (1) HC º C – C º CH
(4) In absence of substituents, nitro group (2) CH2 = CH – C º CH
always goes to m-position. (3) CH3 – CH = CH – CH3
12. The compound A on treatment with Na gives B, (4) CH2 = CH – CH = CH2
and with PCl5 gives C. B and C react together to 17. Which of the following carbocations is expected
give diethyl ether. A, B and C are in the order to be most stable?
(1) C2H5OH, C2H6, C2H5Cl
(2) C2H5OH, C2H5Cl, C2H5ONa NO2 NO2
(3) C2H5OH, C2H5ONa, C2H5Cl Å

(4) C2H5Cl, C2H6, C2H5OH (1) Å


(2)
13. Hydrocarbon (A) reacts with bromine by Y H Y H
substitution to form an alkyl bromide which by
Wurtz reaction is converted to gaseous
hydrocarbon containing less than four carbon NO2 NO2

atoms. (A) is H Å
(3) Y (4) H
(1) CH º CH (2) CH2 = CH2 Y Å
(3) CH4 (4) CH3 – CH3
14. The compound C7H8 undergoes the following
18. Which of the following is correct with respect
reactions:
to – I effect of the substituents? (R = alkyl)
3Cl / D Br /Fe Zn /HCl (1) – NH2 < – OR < – F
C7 H8 ¾¾¾¾
2 ® A ¾¾¾¾
2 ® B ¾¾¾¾® C
(2) – NR2 < – OR < – F
The product 'C' is
(3) – NR2 > – OR > – F
(1) m-bromotoluene
(4) – NH2 > – OR > – F
NEET Solved Paper 2018 2018 -3

19. In the reaction


(4) CH – CH3 and I2
OH O– Na+
OH
CHO
+ CHCl3+ NaOH ¾®
22. Identify the major products P, Q and R in the
following sequence of reactions:
The electrophile involved is

(
(1) Dichloromethyl cation C HCl2
Å
) + CH3CH2CH2Cl ¾¾¾¾®
Anhydrous
AlCl3

(Å )
(2) Formyl cation C HO
(i) O
P ¾¾¾¾®
2
+ Q+R
(ii) H3O /D

(3) Dichlorocarbene (: CCl 2 )


P Q R

(
(4) Dichloromethyl anion CHCl 2
!
) CH2CH2CH3 CHO
20. Carboxylic acids have higher boiling points than (1) , ,
aldehydes, ketones and even alcohols of
comparable molecular mass. It is due to their
(1) Formation of intramolecular H-bonding CH3CH2 – OH
(2) Formation of carboxylate ion
(3) Formation of intermolecular H-bonding CH2CH2CH3 CHO COOH
(4) More extensive association of carboxylic
(2) , ,
acid via van der Waals force of attraction
21. Compound A, C8H10O, is found to react with
NaOI (produced by reacting Y with NaOH) and
yields a yellow precipitate with characteristic OH
CH(CH3)2
smell.
(3) , ,
A and Y are respectively

(1) H3C CH2 – OH and I2


CH3 – CO – CH3

CH(CH3)2 OH
(2) CH2 – CH2 – OH and I2 (4) , ,

CH3 CH3CH(OH)CH3
(3) CH3 OH and I2
EBD_7324
2018 -4 CHEMISTRY
23. Which of the following compounds can form a (3) Forces of attraction between the gas
zwitterion? molecules
(1) Aniline (2) Acetanilide (4) Electric field present between the gas
(3) Glycine (4) Benzoic acid molecules
24. For the redox reaction 28. The bond dissociation energies of X2, Y2 and
+
MnO 4– + C 2 O 2–
4 + H ¾¾
® XY are in the ratio of 1 : 0.5 : 1. DH for the
formation of XY is –200 kJ mol–1. The bond
Mn 2 + + CO2 + H 2 O
dissociation energy of X2 will be
The correct coefficients of the reactants for the (1) 200 kJ mol–1 (2) 100 kJ mol–1
balanced equation are (3) 400 kJ mol–1 (4) 800 kJ mol–1

MnO4– C2 O2– H+ 29. Magnesium reacts with an element (X) to form


4
an ionic compound. If the ground state
(1) 16 5 2
electronic configuration of (X) is 1s2 2s2 2p3,
(2) 2 5 16
the simplest formula for this compound is
(3) 5 16 2
(1) Mg2X3 (2) MgX2
(4) 2 16 5
(3) Mg3X2 (4) Mg2X
25. Which one of the following conditions will favour
30. Iron exhibits bcc structure at room temperature.
maximum formation of the product in the reaction,
Above 900°C, it transforms to fcc structure. The
ˆˆ† X 2 (g) D r H = –X kJ :
A 2 (g) + B2 (g) ‡ˆˆ
ratio of density of iron at room temperature to
(1) Low temperature and high pressure that at 900°C (assuming molar mass and atomic
(2) Low temperature and low pressure radii of iron remains constant with temperature)
(3) High temperature and low pressure is
(4) High temperature and high pressure
26. When initial concentration of the reactant is 3 4 3
(1) (2)
2 3 2
doubled, the half-life period of a zero order
reaction
(1) is halved 1 3 3
(3) (4)
(2) is doubled 2 4 2

(3) remains unchanged 31. Consider the following species :


(4) is tripled CN+, CN–, NO and CN
27. The correction factor ‘a’ to the ideal gas equation Which one of these will have the highest bond
corresponds to order?
(1) Density of the gas molecules (1) NO (2) CN–
(2) Volume of the gas molecules (3) CN (4) CN+
NEET Solved Paper 2018 2018 -5

32. Which one is a wrong statement? 35. Among CaH2, BeH2, BaH2, the order of ionic
(1) Total orbital angular momentum of electron character is
in 's' orbital is equal to zero (1) BeH2 < CaH2 < BaH2
(2) An orbital is designated by three quantum (2) CaH2 < BeH2 < BaH2
numbers while an electron in an atom is (3) BaH2 < BeH2 < CaH2
designated by four quantum numbers
(4) BeH2 < BaH2 < CaH2
(3) The value of m for dz2 is zero
36. Consider the change in oxidation state of
(4) The electronic configuration of N atom is
bromine corresponding to different emf values
as shown in the diagram below :
1s2 2s2 2p1x 2p1y 2p1z

1.82 V 1.5 V
BrO4– ¾¾¾® BrO3– ¾¾¾® HBrO

¾¾
33. The correct difference between first and second Br –¬¾¾¾ Br2 ¬¾¾¾
1.0652 V 1.595 V
order reactions is that
(1) The rate of a first-order reaction does not Then the species undergoing disproportion-
depend on reactant concentrations, the ation is
rate of a second-order reaction does (1) BrO3– (2) BrO 4–
depend on reactant concentrations (3) HBrO (4) Br2
(2) The half-life of a first-order reaction does 37. The solubility of BaSO4 in water is
not depend on [A]0, the half-life of a 2.42 × 10–3 gL–1 at 298 K. The value of its
second-order reaction does depend on solubility product (Ksp) will be
[A]0 (Given molar mass of BaSO4 = 233 g mol–1)
(3) The rate of a first-order reaction does (1) 1.08 × 10–10 mol2L–2
depend on reactant concentrations, the (2) 1.08 × 10–12 mol2L–2
rate of a second-order reaction does not (3) 1.08 × 10–8 mol2L–2
depend on reactant concentrations
(4) 1.08 × 10–14 mol2L–2
(4) A first-order reaction can be catalyzed, a
38. Following solutions were prepared by mixing
second-order reaction cannot be catalyzed
different volumes of NaOH and HCl of different
34. In which case is number of molecules of water
concentrations :
maximum?
(1) 18 mL of water M M
a. 60 mL HCl + 40 mL NaOH
10 10
(2) 0.18 g of water
(3) 10–3 mol of water
M M
(4) 0.00224 L of water vapours at 1 atm and b. 55 mL HCl + 45 mL NaOH
10 10
273 K
EBD_7324
2018 -6 CHEMISTRY
43. Which one of the following ions exhibits d-d
M M
c. 75 mL HCl + 25 mL NaOH transition and paramagnetism as well?
5 5
(1) CrO2–
4 (2) Cr2 O72–
M M
d. 100 mL HCl + 100 mL NaOH
10 10 (3) MnO 2–
4 (4) MnO4–
pH of which one of them will be equal to 1? 44. The geometry and magnetic behaviour of the
(1) b (2) a complex [Ni(CO)4] are
(3) c (4) d
(1) Square planar geometry and diamagnetic
39. On which of the following properties does the
(2) Tetrahedral geometry and diamagnetic
coagulating power of an ion depend?
(1) The magnitude of the charge on the ion (3) Tetrahedral geometry and paramagnetic
alone (4) Square planar geometry and paramagnetic
(2) Size of the ion alone 45. Match the metal ions given in Column I with
(3) The sign of charge on the ion alone the spin magnetic moments of the ions given in
(4) Both magnitude and sign of the charge Column II and assign the correct code :
on the ion
Column I Column II
40. Given van der Waals constants for NH3, H2, O2
and CO2 are respectively 4.17, 0.244, 1.36 and a. Co3+ i. 8 BM
3.59, which one of the following gases is most
b. Cr3+ ii. 35 BM
easily liquefied?
(1) NH3 (2) H2 c. Fe3+ iii. 3 BM
(3) CO2 (4) O2
41. Iron carbonyl, Fe(CO)5 is d. Ni2+ iv. 24 BM
(1) Tetranuclear (2) Mononuclear
v. 15 BM
(3) Dinuclear (4) Trinuclear
42. The type of isomerism shown by the complex a b c d
[CoCl2(en)2] is (1) iv v ii i
(1) Geometrical isomerism (2) i ii iii iv
(2) Coordination isomerism
(3) iii v i ii
(3) Linkage isomerism
(4) iv i ii iii
(4) Ionization isomerism

ANSWER KEY
1 (1) 6 (2) 11 (3) 16 (2) 21 (4) 26 (2) 31 (2) 36 (3) 41 (2)
2 (3) 7 (1) 12 (3) 17 (4) 22 (3) 27 (3) 32 (4) 37 (1) 42 (1)
3 (4) 8 (3) 13 (3) 18 (1) 23 (3) 28 (4) 33 (2) 38 (3) 43 (3)
4 (4) 9 (4) 14 (1) 19 (3) 24 (2) 29 (3) 34 (1) 39 (4) 44 (2)
5 (4) 10 (2) 15 (1) 20 (3) 25 (1) 30 (4) 35 (1) 40 (1) 45 (1)
NEET Solved Paper 2018 2018 -7

Hints & Solutions


8. (3) Cross-linked or network polymers are
+5 +2 0 -3
1. (1) HNO3 , NO, N2 , NH4Cl usually formed from bi-functional and tri-
functional monomers and contain strong
2. (3) covalent bonds between various linear
3. (4) Mg has more – DG value than alumina. So polymer chains like melamine, bakelite etc.
H SO
it will be in the lower part of Ellingham 9. (4) HCOOH ¾¾¾¾®
2 4
CO + H2O
Dehydrating
diagram. Metals which have more – DG agent

value can reduce those metal oxides which [H 2O absorbed


by H 2SO 4]
have less – DG value. 2.3 1
At start = = 0 0
4. (4) MF63- 46 20
(moles)
Boron belongs to 2nd period and it does 1 1
Final moles 0
not have vacant d-orbital. 20 20
2 4 H SO
5. (4) Due to high electronegativity and small H2C2O4 ¾¾¾® CO + CO2 + H2O
[H 2O absorbed
size, F forms only one oxoacid, HOF known
by H 2SO 4]
as fluoric (I) acid. Oxidation number of F is 4.5 1
At start = = 0 0 0
90 20
+1 in HOF. (moles)
6. (2) The structure of ClF3 is 1 1 1
Final moles 0
20 20 20
CO2 is absorbed by KOH.
F
So the remaning product is only CO.
F Cl Moles of CO formed from both reactions
1 1 1
F = + =
20 20 10

The number of lone pair of electrons on Left mass of CO = moles × molar mass
central Cl is 2.
1
7. (1) Amylose and amylopectin are polymers of = ´ 28 = 2.8 g
10
a-D-glucose, so b-link is not possible.
10. (2) In metals moving down the group metallic
Amylose is linear with 1 ® 4 a-linkage
character increases, so basic nature
whereas amylopectin is branched and has
increases hence most acidic will be BeO.
both 1 ® 4 and 1 ® 6 a-linkages.
BeO < MgO < CaO < BaO
So option (1) should be the correct option. ¾¾¾¾¾¾¾¾¾¾
®
increasingbasic character
EBD_7324
2018 -8 CHEMISTRY
11. (3) 15. (1) Nitrous oxide (N2O) occurs naturally in
environment.

NH2
+
NH3
+
NH3 In automobile engine, when fuel is burnt
+ dinitrogen and dioxygen combine to yield
+ NO2
¾¾H ¾¾® ¾¾ ¾® NO and NO2.
nitrating
mixture NO2
sp2 sp2 sp sp
16. (2) CH2 = CH – C CH
In acidic medium aniline is protonated to
17. (4) –NO2 group is meta-directing group
form anilinium ion which is meta-directing.
NO2 NO2
Na –
12. (3) C2H5OH ¾¾® C2H5O Na+ +
(A) (B) ¬¾®
+
PCl5 Y H Y H
(Less stable due to more e– withdrawing effect of
C2H5Cl –NO 2 )
(C)
– SN2
C2H5O Na+ + C2H5Cl ¾¾® C2H5OC2H5
(B) (C) NO2 NO2 NO2

So the correct option is (3) + +


¬¾® ¬¾®
H H H
13. (3)
Br2
CH 4 ¾¾¾ ® CH 3 – Br
Na
¾¾¾® Y + Y Y
hv ether

(More stable due to less e– withdrawing effect of –


CH 3 – CH 3
(less than four 'C') NO 2) greater no. of resonating structures.
18. (1) – I effect increases on increasing
electronegativity of atom. So, correct order
CH3 CCl3 of – I effect is
–NH2 < – OR < – F.
14. (1) 3Cl Br *Most appropriate answer is option (1),
¾¾®
D
2
¾¾®
2
Fe however option (2) may also be correct
answer.


19. (3) CHCl3 + NaOH ¾® CCl3 + H2O
CCl3 CH3
¾®

– Cl (a-elimination)

Zn : CCl2 dichlorocarbene
¾¾®
HCl
(electrophile)
Br Br
m-Bromotoluene
20. (3) Carboxylic acids have higher boiling points
than aldehydes, ketones and even alcohols
of comparable molecular mass.
NEET Solved Paper 2018 2018 -9

This is due to more extensive association 23. (3) ˆˆ†


HOOC – CH 2 – NH 2 ‡ˆˆ
through intermolecular H-bonding. Glycine
+
OOC – CH 2 – NH 3
O ¼¼¼H O Zwitter ion
R C C R
O H ¼¼¼O +7 +2
24. (2) ® Mn2+; 5e– gain
Mn O 4– ¾¾ ...(i)

21. (4) Haloform reaction is shown by compound +3 +4


® CO2 ; 2e - loss
C2O 42– ¾¾ ...(ii)
having
Multiplying (i) by 2 and (ii) by 5 to balance
CH3 – C – or CH3 – CH – e–
|| | Group
O OH 2 MnO 4– + 5 C 2 O 2–
4 ¾¾
®
2 Mn 2 + + 10 CO2

NaOI
¾¾¾¾®
On balancing charge;
CH – CH3 or
| NaOH + I 2
+
OH 2 MnO 4– + 5 C2 O 2–
4 + 16 H ¾¾
®

2 Mn 2 + + 10 CO2 + 8 H 2O
– +
C – ONa + CHI3 25. (1) ˆˆ† X (g); DH = - x kJ
A2 (g) + B2 (g) ‡ˆˆ 2
|| Yellow
O ppt.
On increasing pressure equilibrium shifts

22. (3) Mechanism : in a direction where number of moles


decreases i.e. forward direction.
AlCl
CH 3 – CH 2 – CH 2 – Cl ¾¾¾®
3
On decreasing temperature, equilibrium
+ –
CH3 – CH 2 – CH 2 + AlCl4 shifts in exothermic direction i.e., forward
1° Carbocation direction.
CH3 – CH – CH3 So, high pressure and low temperature
+ –
ESR
¬¾¾¾ CH 3 CH CH3 H favours maximum formation of product.
shift
2° Carbocation
(P) [A]o
CH3 26. (2) (t1/2 )zero =
2k
CH3 – CH – CH3 CH3–C– O– O–H
O +
\ If [A]o = doubled, t1/2 = doubled
¾¾®
D
2 H3O
D 27. (3) In real gas equation,
Cumene Cumene
(P) hydroperoxide
van der waal constant (a) µ forces of
OH attraction.
O
+ CH3 – C– CH3 28. (4) Let B.E of x2, y2 and xy are x kJ mol–1,
0.5 x kJ mol–1 and x kJ mol–1 respectively
Phenol Acetone
(Q) (R)
EBD_7324
2018 -10 CHEMISTRY
CN : (s1s)2, (s*1s)2, (s2s)2,(s*2s)2,
1 1 (p2px)2 = (p2py)2,(s2pz)1
x 2 + y 2 ® xy; DH = –200 kJ mol –1
2 2
9-4
DH = –200 = S (B.E)Reactants – S(B.E)Product B.O. = = 2.5
2

é1 1 ù CN+ : (s1s)2, (s*1s)2, (s2s)2,(s*2s)2,


= ê ´ (x) + ´ (0.5x) ú – [1 ´ (x)] (p2px)2 = (p2py)2
ë2 2 û
8-4
On solving, x = 800 kJ mol–1 B.O. = =2
2
29. (3) Element (X) electronic configuration
1s2 2s2 2p3 Hence, option (2) should be the right
So, valency of X will be 3. answer.
Valency of Mg is 2. 32. (4) The correct configuration of 'N' is
Formula of compound formed by Mg and X
will be Mg3X2.
1s2 2s2 2p1x 2p1y 2p1z
4r
30. (4) For bcc lattice : Z = 2, a =
3

For fcc lattice : Z = 4, a = 2 2r 33. (2) (t 1/2 )1 st order = Independent of


concentration

æ ZM ö 1
çç 3 ÷÷ (t1/2)2nd order µ
d25 °C [A]o
è NA a øbcc
\ =
d900 °C æ ZM ö
çç 3 ÷÷ 34. (1)
è NA a øfcc (1) Mass of water = 18 × 1 = 18 g

3
Molecules of water = mole × NA
2 æ 2 2r ö 3 3
= ç ÷ =
4 4r 4 2 18
ç ÷ = N = NA
è 3 ø 18 A
31. (2) NO : (s1s)2, (s*1s)2, (s2s)2,(s*2s)2,(s2pz)2,
(2) Molecules of water = mole × NA
(p2px)2 = (p2py)2,(p*2px)1 = (p*2py)0

10 - 5 0.18
= N = 10–2 NA
B.O. =
2
= 2.5 18 A

CN– : (s1s)2, (s*1s)2, (s2s)2,(s*2s)2, (3) Molecules of water = mole × NA = 10–3 NA


(p2px)2 = (p2py)2, (s2pz)2
0.00224
(4) Moles of water = = 10–4
10 - 4 22.4
B.O. = =3
2
Molecules of water = mole × NA = 10–4 NA
NEET Solved Paper 2018 2018 -11

35. (1) BeH2 < CaH2 < BaH2 39. (4) According to Hardy Schulze rule,
Smaller the size of cation, more will be its coagulating power of an ion depends on
polarising power. Hence BeH2 will be least both magnitude and sign of the charge on
ionic. the ion.
36. (3) Calculate E°cell corresponding to each 40. (1) van der waal constant ‘a’, signifies
compound undergoing disproportionation intermolecular forces of attraction.
reaction. The reaction for which E°cell comes Higher is the value of ‘a’, easier will be the
out + ve is spontaneous. liquefaction of gas.
HBrO ¾® Br2 41. (2) Fe(CO)5
EAN = Z – O.N. + 2(C.N.)
E° = 1.595 V, SRP (cathode)
= 26 – 0 + 2(5)
HBrO ¾® BrO 3– = 26 + 10
= 36
E° = –1.5 V, SOP (anode) Only one central metal atom/ion is present
2HBrO ¾® Br2 + BrO 3– and it follows EAN rule, so it is
E°cell = SRP (cathode) – SRP (anode) mononuclear.
= 1.595 – 1.5 42. (1) In the given complex, the CN of Co is 6,
= 0.095 V and the complex has octahedral
E°cell > 0 Þ DG° < 0 [spontaneous] geometry.

37. (1) Solubility of BaSO4 = 2.42 × 10–3 gL–1

Cl en
2.42 ´ 10-3
\s= = 1.038 ´ 10 -5 mol L-1
233
en Co en Co en
Ksp = s2 = (1.038 × 10–5)2 Cl
Cl Cl
= 1.08 × 10–10 mol2 L–2
trans (optically inactive) cis (optically active)

1
38. (3) Meq. of HCl = 75 ´ ´ 1 = 15
5 43. (3) CrO2–
4 Cr6+ diamagnetic
1
Meq. of NaOH = 25 ´ ´ 1= 5
5 Cr2O2–
7 Cr6+ diamagnetic
Meq. of HCl in resulting solution = 10
Molarity of [H+] in resulting mixture MnO4– Mn7+ diamagnetic
MnO 2–
4 Mn6+ paramagnetic
10 1
= =
100 10

é1ù Thus unpaired electron is present, so d–d


pH = –log[H+] = –log ê ú = 1.0
ë 10 û transition is possible.
EBD_7324
2018 -12 CHEMISTRY
For, four ‘CO’ligands hybridisation would
be sp3 and thus the complex would be
diamagnetic and of tetrahedral geometry.
45. (1) Co3+ = [Ar] 3d 6 , unpaired e–(n) = 4
Spin magnetic moment
Before After
transition transition
= 4(4 + 2) = 24 B.M.

Cr3+ = [Ar] 3d 3 , unpaired e–(n) = 3


44. (2) Ni(28) : [Ar]3d8 4s2 Spin magnetic moment
Q CO is a strong field ligand, so unpaired = 3(3 + 2) = 15 B.M.
electrons get paired. Hence, configuration
would be: Fe3+ = [Ar] 3d 5 , unpaired e–(n) = 5
Spin magnetic moment

= 5(5 + 2) = 35 B.M.

Ni2+ = [Ar] 3d 8 , unpaired e–(n) = 2


Spin magnetic moment
sp3–hybridisation
= 2(2 + 2) = 8 B.M.

CO CO CO CO
ݸ¿°¬»®

1 Some Basic Concepts of


Chemistry

̱°·½ ï æ Í·¹²·º·½¿²¬ Ú·¹«®»­ô Ô¿©­ ±º ݸ»³·½¿´ (a) 44 g CO2 (b) 48 g O3


ݱ³¾·²¿¬·±²­ ¿²¼ Ó±´» ݱ²½»°¬ (c) 8 g H2 (d) 64 g SO2
7. The number of atoms in 0.1 mol of a triatomic gas
1. A mixture of gases contains H2 and O2 gases in is : [2010]
the ratio of 1 : 4 (w/w). What is the molar ratio of (NA = 6.02 ×1023 mol–1)
the two gases in the mixture ? [2015]
(a) 6.026 × 1022 (b) 1.806 × 1023
(a) 4 : 1 (b) 16 : 1 (c) 3.600 × 10 23 (d) 1.800 × 1022
(c) 2 : 1 (d) 1 : 4
8. What volume of oxygen gas (O2) measured at
2. The number of water molecules is maximum in :
0°C and 1 atm, is needed to burn completely 1L
[2015 RS]
of propane gas (C3H8) measured under the same
(a) 18 molecules of water
conditions ? [2008]
(b) 1.8 gram of water
(c) 18 gram of water (a) 7 L (b) 6 L
(d) 18 moles of water (c) 5 L (d) 10 L
3. IfAvogadro number NA, is changed from 6.022 × 9. Number of moles of MnO4 required to oxidize
1023 mol –1 to 6.022 × 1020 mol–1 this would one mole of ferrous oxalate completely in acidic
change : [2015 RS] medium will be : [2008]
(a) the definition of mass in units of grams (a) 0.6 moles (b) 0.4 moles
(b) the mass of one mole of carbon (c) 7.5 moles (d) 0.2 moles
(c) the ratio of chemical species to each other 10. Volume occupied by one molecule of water
in a balanced equation. (density = 1 g cm–3) is : [2008]
(d) the ratio of elements to each other in a (a) 9.0 × 10–23 cm3 (b) 6.023 × 10– 23 cm3
compound (c) 3.0 × 10–23 cm3 (d) 5.5 × 10– 23 cm3
4. When 22.4 litres of H2(g) is mixed with 11.2 litres 11. The number of moles of KMnO4 that will be
of Cl2(g), each at S.T.P., the moles of HCl(g)
needed to react with one mole of sulphite ion in
formed is equal to : [2014]
acidic solution is [2007]
(a) 1 mole of HCl(g) (b) 2 moles of HCl(g)
(a) 4/5 (b) 2/5
(c) 0.5 moles of HCl(g) (d) 1.5 moles of HCl(g)
(c) 1 (d) 3/5
5. 1.0 g of magnesium is burnt with 0.56 g O2 in a
closed vessel. Which reactant is left in excess 12. An element, X has the following isotopic
and how much ? [2014] composition : [2007]
200 X : 90% 199 X : 8.0 % 202X : 2.0%
(At. wt. Mg = 24 ; O = 16)
(a) Mg, 0.16 g (b) O2, 0.16 g The weighted average atomic mass of the
(c) Mg, 0.44 g (d) O2, 0.28 g naturally occuring element X is closest to
6. Which has the maximum number of molecules (a) 201 amu (b) 202 amu
among the following ? [2011 M] (c) 199 amu (d) 200 amu
EBD_7324
î ÝØÛÓ×ÍÌÎÇ
13. The number of moles of KMnO4 reduced by one 21. In the final answer of the expression
mole of KI in alkaline medium is: [2005] ( 29.2 20.2) (1.79 10 5 )
(a) one (b) two 1.37
(c) five (d) one fifth the number of significant figures is : [1994]
(a) 1 (b) 2
14. The maximum number of molecules is present in
(c) 3 (d) 4
(a) 15 L of H2 gas at STP [2004] 22. If NA is Avogadro’s number then number of
(b) 5 L of N2 gas at STP valence electrons in 4.2g of nitride ions (N3–) is
(c) 0.5 g of H2 gas (a) 2.4 NA (b) 4.2 NA [1994]
(d) 10 g of O2 gas (c) 1.6 NA (d) 3.2 NA
23. The molecular weight of O2 and SO2 are 32 and
15. Which has maximum number of molecules? 64 respectively. At 15°C and 150 mm Hg pressure,
(a) 7 g N2 (b) 2 g H2 [2002] one litre of O2 contains ‘N’ molecules. The
(c) 16 g NO2 (d) 16 g O2 number of molecules in two litres of SO2 under
the same conditions of temperature and pressure
16. Specific volume of cylindrical virus particle is
will be : [1990]
6.02 × 10–2 cc/gm. whose radius and length 7 Å (a) N/2 (b) N
& 10 Å respectively. If NA = 6.02 × 1023, find (c) 2N (d) 4N
molecular weight of virus [2001] 24. Boron has two stable isotopes, 10B (19%) and
3 4 11B (81%). Average atomic weight for boron in
(a) 3.08 × 10 kg/mol (b) 3.08 × 10 kg/mol
(c) 1.54 × 104 kg/mol (d) 15.4 kg/mol the periodic table is [1990]
(a) 10.8 (b) 10.2
17. Assuming fully decomposed, the volume of CO2
(c) 11.2 (d) 10.0
released at STP on heating 9.85 g of BaCO3
25. The number of oxygen atoms in 4.4 g of CO2 is
(Atomic mass, Ba = 137) will be [2000] [1990]
(a) 2.24 L (b) 4.96 L (a) 1.2 × 1023 (b) 6 × 1022
(c) 1.12 L (d) 0.84 L (c) 6 × 1023 (d) 12 × 1023
18. Haemoglobin contains 0.334% of iron by weight. 26. The number of gram molecules of oxygen in
The molecular weight of haemoglobin is 6.02 × 1024 CO molecules is [1990]
(a) 10 g molecules (b) 5 g molecules
approximately 67200. The number of iron atoms
(c) 1 g molecules (d) 0.5 g molecules
(at. wt. of Fe is 56) present in one molecule of 27. What is the weight of oxygen required for the
haemoglobin are [1998] complete combustion of 2.8 kg of ethylene ?
(a) 1 (b) 6 (a) 2.8 kg (b) 6.4 kg [1989]
(c) 4 (d) 2 (c) 9.6 kg (d) 96 kg
28. Ratio of Cp and Cv of a gas ‘X’ is 1.4. The number
19. The number of significant figures for the three
of atoms of the gas ‘X’ present in 11.2 litres of it
numbers 161 cm, 0.161 cm, 0.0161 cm are
at NTP will be [1989]
(a) 3,4 and 5 respectively [1998] (a) 6.02 ×1023 (b) 1.2 × 1023
(b) 3,4 and 4 respectively (c) 3.01 × 1023 (d) 2.01 × 1023
(c) 3,3 and 4 respectively 29. 1 c.c. N2O at NTP contains : [1988]
(d) 3,3 and 3 respectively (a) 1 . 8 22
10 atoms
20. The weight of one molecule of a compound 224
6.02
C60H122 is [1995] (b) 10 23 molecules
22400
(a) 1.2 × 10–20 gram (b) 1.4 × 10–21 gram 1.32
(c) 1023 electrons
(c) 5.025 × 1023 gram (d) 6.023 × 1023 gram 224
(d) all the above
ͱ³» Þ¿­·½ ݱ²½»°¬­ ±º ݸ»³·­¬®§ í

30. At S.T.P. the density of CCl4 vapours in g/L will ̱°·½ íæ ͬ±·½¸·±³»¬®·½ Ý¿´½«´¿¬·±²­
be nearest to : [1988]
37. 20.0 g of a magnesium carbonate sample
(a) 6.87 (b) 3.42
decomposes on heating to give carbon dioxide
(c) 10.26 (d) 4.57
and 8.0 g magnesium oxide. What will be the
̱°·½ îæ л®½»²¬ ݱ³°±­·¬·±² ¿²¼ percentage purity of magnesium carbonate in
the sample ? [2015 RS]
Û³°·®·½¿´ Ú±®³«´¿
(a) 75 (b) 96
31. An organic compoun d contain s carbon, (c) 60 (d) 84
hydrogen and oxygen. Its elemental analysis 38. What is the mass of precipitate formed when 50
gave C, 38.71% and H, 9.67%. The empirical mL of 16.9% solution of AgNO3 is mixed with 50
formula of the compound would be : [2008] mL of 5.8% NaCl solution ? [2015 RS]
(a) CH3O (b) CH2O (Ag = 107.8, N = 14, O = 16, Na = 23, Cl = 35.5)
(c) CHO (d) CH4O (a) 28 g (b) 3.5 g
32. Percentage of Se in peroxidase anhydrase (c) 7 g (d) 14 g
enzyme is 0.5% by weight (at. wt. = 78.4) then 39. In an experiment it showed that 10 mL of 0.05 M
minimum molecular weight of peroxidase solution of chloride required 10 mL of 0.1 M
anhydrase enzyme is [2001] solution of AgNO3, which of the following will
(a) 1.568 × 10 3 (b) 15.68 be the formula of the chloride (X stands for the
symbol of the element other than chlorine):
(c) 2.136 × 104 (d) 1.568 × 104
[NEET Kar. 2013]
33. An organic compound containing C, H and O (a) X2Cl (b) X2Cl2
gave on analysis C – 40% and H – 6.66%. Its (c) XCl2 (d) XCl4
empirical formula would be [1999, 94]
40. 6.02 × 1020 molecules of urea are present in
(a) C3H6O (b) CHO 100 mL of its solution. The concentration of
(c) CH2O (d) CH4O solution is : [NEET 2013]
34. An organic compound containing C, H and N (a) 0.01 M (b) 0.001 M
gave the following analysis : (c) 0.1 M (d) 0.02 M
C = 40% ; H = 13.33% ; N = 46.67%
41. What is the [OH] in the final solution prepared
Its empirical formula would be [1998]
(a) C2H7N2 (b) CH5N by mixing 20.0 mL of 0.050 M HCl with 30.0 mL of
(c) CH4N (d) C2H7N 0.10 M Ba(OH)2? [2009]
35. The percentage weight of Zn in white vitriol (a) 0.40 M (b) 0.0050 M
[ZnSO 4.7H2O] is approximately equal to (c) 0.12 M (d) 0.10 M
42. 10 g of hydrogen and 64 g of oxygen were filled
( Zn 65, S 32, O 16 and H = 1) [1995]
in a steel vessel and exploded. Amount of water
(a) 33.65 % (b) 32.56 % produced in this reaction will be: [2009]
(c) 23.65 % (d) 22.65 % (a) 3 mol (b) 4 mol
36. A metal oxide has the formula Z2O3. It can be
(c) 1 mol (d) 2 mol
reduced by hydrogen to give free metal and
43. How many moles of lead (II) chloride will be
water. 0.1596 g of the metal oxide requires 6 mg
formed from a reaction between 6.5 g of PbO
of hydrogen for complete reduction. The atomic
and 3.2 g of HCl ? [2008]
weight of the metal is [1989]
(a) 0.044 (b) 0.333
(a) 27.9 (b) 159.6
(c) 79.8 (d) 55.8 (c) 0.011 (d) 0.029
EBD_7324
ì ÝØÛÓ×ÍÌÎÇ
44. Concentrated aqueous sulphuric acid is 98% 47. In the reaction
H2SO4 by mass and has a density of 1.80 g mL– 1. 4 NH3 (g) + 5 O2 (g) 4 NO(g) + 6 H2O(1)
Volume of acid required to make one litre of When 1 mole of ammonia and 1 mole of O2 are
0.1MH2SO4 solution is [2007] made to react to completion, [1998]
(a) 1.0 mole of H2O is produced
(a) 16.65 mL (b) 22.20 mL
(b) 1.0 mole of NO will be produced
(c) 5.55 mL (d) 11.10 mL (c) all the oxygen will be consumed
45. The mass of carbon anode consumed (giving (d) all the ammonia will be consumed
only carbon dioxide) in the production of 48. Liquid benzene (C6H6) burns in oxygen according
270 kg of aluminium metal from bauxite by the to the equation
Hall process is (Atomic mass: Al = 27)
2C 6 H 6 (l ) 15O 2 ( g ) 12CO 2 ( g ) 6 H 2 O( g )
[2005] How many litres of O2 at STP are needed to
(a) 270 kg (b) 540 kg complete the combustion of 39 g of liquid
(c) 90 kg (d) 180 kg benzene?(Mol. wt. of O2 = 32, C6H6 = 78)
[1996]
46. In Haber process 30 litres of dihydrogen and (a) 74 L (b) 11.2 L
30 litres of dinitrogen were taken for reaction (c) 22.4 L (d) 84 L
which yielded only 50% of the expected product. 49. A 5 molar solution of H2SO4 is diluted from
What will be the composition of gaseous mixture 1 litre to a volume of 10 litres, the normality of
under the aforesaid condition in the end? the solution will be : [1991]
[2003] (a) 1 N (b) 0.1 N
(a) 20 litres ammonia, 25 litres nitrogen, 15 litres (c) 5 N (d) 0.5 N
hydrogen 50. One litre hard water contains 12.00 mg Mg2+.
(b) 20 litres ammonia, 20 litres nitrogen, 20 litres Mili-equivalents of washing soda required to
hydrogen remove its hardness is : [1988]
(a) 1 (b) 12. 16
(c) 10 litres ammonia, 25 litres nitrogen, 15 litres
(c) 1 × 10–3 (d) 12. 16 × 10–3
hydrogen
(d) 20 litres ammonia, 10 litres nitrogen, 30 litres
hydrogen

ANSWER KEY
1 (a) 6 (c) 11 (b) 16 (d) 21 (c) 26 (b) 31 (a) 36 (d) 41 (d) 46 (c)
2 (d) 7 (b) 12 (d) 17 (c) 22 (a) 27 (c) 32 (d) 37 (d) 42 (b) 47 (c)
3 (b) 8 (c) 13 (a) 18 (c) 23 (c) 28 (a) 33 (c) 38 (c) 43 (d) 48 (d)
4 (a) 9 (b) 14 (a) 19 (d) 24 (a) 29 (d) 34 (c) 39 (c) 44 (c) 49 (a)
5 (a) 10 (c) 15 (b) 20 (b) 25 (a) 30 (a) 35 (d) 40 (a) 45 (c) 50 (a)
ͱ³» Þ¿­·½ ݱ²½»°¬­ ±º ݸ»³·­¬®§ ë

Hints & Solutions


1. (a) Ratio of weight of gases = wH : wO = 1 : 4 8
2 2 Moles of H2 = 4 , 4NA
1 4 2
Ratio of moles of gases = n H : nO = : 64
2 2 2 32 Moles of SO2 = 1 , NA
64
1 32 7. (b) The number of atoms in 0.1 mol of a triatomic
Molar Ratio = 4 :1
2 4 gas = 0.1 × 3 × 6.023 × 1023.
2. (d) No. of moles of water = 1.806 × 1023
In 1.8 g of H2O = 0.1 moles 8. (c) Writing the equation of combustion of
In 18 g of H2O = 1 moles propane (C3H8), we get
1 mole contain 6.022 × 1023 molecules of water C3 H8 5O 2 3CO 2 4H 2 O
therefore maximum number of molecules is in 18 1vol 5vol
IL 5L
moles of water.
From the above equation we find that we need
3. (b) If 6.022 × 1023 changes to 6.022 × 1020/mol
5 L of oxygen at NTP to completely burn 1 L of
than this would change mass of one mole of
propane at N.T.P.
carbon.
4. (a) H2 + Cl2 2HCl If we change the conditions for both the gases
from N.T.P. to same conditions of temperature
t=0 22.4 lit ..............................
and pressure. The same results are obtained. i.e.
t = 0 or 1 mole ..............................
5 L is the correct answer.
at time t 1 – 0.5 0.5 × 2
= 0.5 = 1 mole COO – – +
9. (b) 5 +2 MnO4 + 16 H
COO–
1 oxalate ion
5. (a) Initially Mg O2 MgO
1g 2
0.56 2+
2 Mn +10CO2+8H2O

1 0.56 From above equation 2 moles of MnO4 required


or mole mole to oxidise 5 moles of oxalate.
24 32
Thus number of moles of MnO4 required to oxidise
0.0416 mole 0.0175 mole one mole of oxalate =2/5 =0.4 moles
(0.0416 – 2 × 0.0175) (2 × 0.0175) mole Mass
10. (c) Density
= 0.0066 mole Volume
Mass of Mg = 0.0066 × 24 1gram
1 gram cm–3 =
= 0.158 0.16g cm3
6. (c) No. of molecules Mass 1gram
Volume = 3
1cm3
Density 1gram cm
44
Moles of CO2 = 1 , NA Volume occupied by 1 gram water = 1 cm3
44
or Volume occupied by
48
Moles of O3 = 1 , NA
48 6.023 1023
molecules of water = 1 cm3
18
EBD_7324
ê ÝØÛÓ×ÍÌÎÇ
(d) Similarly 10 g of O2 gas
1
[ 1g water = moles of water] 10
18 = 6.023 10 23 molecules
32
Thus volume occupied by 1 molecule of water
Thus (a) will have maximum number of molecules
1 18 3 15. (b) 2g of H2 means one mole of H2, hence
= cm = 3.0×10–23 cm3.
6.023 10 23 contains 6.023 × 1023 molecules. Others have
i.e. the correct answer is option (c). less than one mole, so have less no. of molecules.
11. (b) The balance chemical equation is : 16. (d) Specific volume (volume of 1 gm) of
cylindrical virus particle = 6.02 × 10–2 cc/gm
2MnO 4 6H 5SO32 Radius of virus (r) = 7 Å = 7 × 10–8 cm
Length of virus = 10 × 10–8 cm
2Mn 2 5SO 42 3H 2O Volume of virus =
From the equation it is clear that 22
r 2l (7 10 8 2
) 10 10 8
Moles of MnO4– require to oxidise 5 moles of 7
SO32 – are 2 = 154 × 10–23 cc
Moles of MnO4– require to oxidise 1 mole of volume
Wt. of one virus particle
SO32– are 2/5. specific volume
12. (d) Average isotopic mass of Mol. wt. of virus = Wt. of NA particle
23
200 90 199 8 202 2 154 10
X = 6.02 10 23
90 8 2 6.02 10 2
= 15400 g/mol = 15.4 kg/mol
18000 1592 404 17. (c) BaCO 3 BaO CO 2
100 197 gm
As 197 gm of BaCO3 will release
19996 22.4 litre of CO2 at STP
199.96 200 amu
100 1 gm of BaCO3 will release
7 6 22.4
13. (a) OH = litre of CO2
KMnO4 K2 MnO4 197
Change in oxidation number of Mn in basic And 9.85 gm of BaCO3 will release carbon dioxide
medium is 1. Hence mole of KI is equal to mole 22 .4
= 9.85 = 1.12 litre of CO2
of KMnO4. 197
14. (a) No. of molecules in different cases 18. (c) Given : Percentage of the iron = 0.334%;
(a) 22.4 litre at STP contains Molecular weight of the h aemoglobin
= 6.023×1023molecule of H2 = 67200 and atomic weight of iron = 56. We know
that the number of iron atoms
15
15 litre at STP contains = 6.023 10 23 Molecular wt. of haemoglobin % of iron
22.4
100 Atomic weight of iron
(b) 22.4 litre at STP contains
= 6.023×1023 molecule of N2 67200 0.334
= 4
100 56
5 19. (d) We know that all non-zero digits are
5 litre at STP contians = 6.023 10 23
22.4 significant and the zeros at the beginning of a
(c) 2 gm of H2= 6.023×1023 molecules of H2 number are not significant. Therefore number
0.5 161 cm, 0.161 cm and 0.0161cm have 3, 3 and 3
0.5 gm of H2= 6.023 10 23 significant figures respectively.
2
ͱ³» Þ¿­·½ ݱ²½»°¬­ ±º ݸ»³·­¬®§ é

20. (b) Molecular weight of C60H122 As 28 kg of C2H4 undergo complete combustion


= (12 × 60) + 122 = 842. by = 96 kg of O2
Therefore weight of one molecule 2.8 kg of C2H4 undergo complete combustion
Molecular weight of C60 H122 by = 9.6 kg of O2.
= 28. (a) Cp / Cv = 1.4 shows that the gas is diatomic.
Avogadro's number
842 22.4 litre at NTP 6.02 × 1023 molecules
21 21
1.36 10 g å 1.4 10 g
6.023 1023 11.2 L at NTP = 3.01 × 1023 molecules
21. (c) On calculation we find No. of atomic gas = 3.01 × 1023 × 2 atoms
= 6.02 × 1023 atoms
(29.2 – 20.2)(1.79 105 )
= 1.17×106 29. (d) At NTP 22400 cc of N2O
1.37
As the least precise number contains 3 = 6.02 × 1023 molecules
significant figures therefore answers should also 1 cc of N2O contain
contains 3 significant figures.
6.02 10 23
22. (a) No of moles of nitride ion = molecules
22400
4.2
= 0.3 mol 0.3 N A nitride ions.
14 3 6.02 1023 1.8
= 1022 atoms
Valence electrons = 8 × 0.3 N A = 2.4 N A 22400 224
(5 + 3 due to charge). One N3– ion contains ( N2O molecule has 3 atoms)
8 valence electrons.
23. (c) According to Avogadro's law "equal No. of electrons in a molecule of N 2O
volumes of all gases contain equal numbers of = 7 + 7 + 8 = 22
molecules under similar condition s of Hence no. of electrons
temperature and pressure". Thus if 1 L of one
gas contains N molecules, 2 L of any gas under 6.02 10 23 1.32 10 23
22 electrons
the same conditions will contain 2N molecules. 22400 224
24. (a) Average atomic mass 30. (a) 1 mol CCl4 vapour = 12 + 4 × 35.5
R.A.(1) M.No R.A.(2) M.No. = 154 g 22.4 L at STP
R.A.(1) R.A.(2) 154 1 1
Density = gL 6.875 gL
Where R.A. = relative abundance 22.4
M.No = Mass number 31. (a)
19 10 81 11
= = 10.81 Element % Atomic Atomic Simple
100 weight ratio ratio
25. (a) 1 mol of CO2 = 44 g of CO2 C 38.71 12 38.71 3.23
No. of oxygen atoms 12
3.23
3.23
1

4.4 H 9.67 1 9.67 9.67


4.4 g CO2 = = 0.1 mol CO2 9.67 3
44 1 3.23
= 6 × 1022 molecules O 100 16 51.62 3.23
(38.71 9.67) 3.23 1
16 3.23
= 2 × 6 × 1022 atoms or 1.2 × 10 23 atoms. 51.62
26. (b) 6.02 × 1023 molecules of CO =1mole of CO
6.02 × 1024 CO molecules= 10 moles CO Thus empirical formula is CH3O.
= 10 g atoms of O = 5 g molecules of O 2 32. (d) Suppose the mol. wt. of enzyme = x
Given 100 g of enzyme wt of Se = 0.5 gm
27. (c) C2H4 + 3 O2 2CO2 + 2H2O 0.5
In x g of enzyme wt. of Se = x
28 kg 96 kg 100
EBD_7324
è ÝØÛÓ×ÍÌÎÇ
0. 5 x 0.1596
Hence 78.4 26.6 g of Z2 O3
100 0.006
x = 15680 = 1.568 × 104
33. (c) Eq. wt. of Z2O3 = 26.6
(from the definition of eq. wt.)
Element % At. Relative Ratio
wt number Eq. wt. of Z + Eq. wt. of O (8) = 26.6
Eq. wt. of Z = 26.6 – 8 = 18.6
40 3.33 Valency of metal in Z2O3 = 3
C 40 12 3.33 1
12 3.33
Atomic wt.
6.66 6.66 Eq. wt.of metal ã
H 6.66 1 6.66 2 valency
1 3.33
At. wt. of Z = 18.6 × 3 = 55.8
53.34 3.33
O 53.34 16 3.33 1 37. (d)
16 3.33 MgCO 3 MgO CO 2
(% of O in organic compound 84 g of MgCO3 form 40 g of MgO
Table for empirical formula :
40 20
100 – (40 6 .66 ) 53. 34 % ) 20g of MgCO3 form g of MgO
Empirical formula of organic compound 84
= CH2O. = 9.52 g of MgO
34. (c) As the sum of the percentage of C, H & N Since 8.0 g of MgO is formed
is 100. Thus it does not contains O atom.
8
Table for empirical formula Purity of sample 100 = 84.0%
9.52
Element % At. Relative Ratio
wt. Number 38. (c) 50 ml of 16.9% solution of AgNO3
16.9
40 3.33 50 = 8.45 g of Ag NO
C 40.00 12 3.33 1 100 3
12 3.33
8.45g
13.33 13.33 nmole =
H 13.33 1 13.33 4 (107.8 14 16 3) g / mol
1 3 .33

46.67 3.33 8.45 g


N 46.67 14 3.33 1 = 0.0497 moles
14 3.33 169.8g / mol
Hence empirical formula = CH4N 50 ml of 5.8% solution of NaCl contain
35. (d) Molecular weight of ZnSO 4 .7 H 2O 5.8
NaCl = 50 2.9 g
= 65 + 32 + (4 × 16) + 7(2 × 1 + 16) = 287. 100
percentage mass of zin c (Zn)
2.9g
65 nNaCl = = 0.0495 moles
100 22.65% (23 35.5) g/ mol
287 –
36. (d) The reaction can be given as AgNO3 + NaCl AgCl + Na + Cl
1 mole 1 mole 1 mole
Z2 O3 + 3H2 2Z + 3H2O 0.049 mole 0.049 mole 0.049 mole of AgCl
0.1596 g of Z2O3 react with H2 w
= 6 mg = 0.006 g n= w = (n AgCl) × Molecular Mass
M
1 g of H2 react with
= (0.049) × (107.8 + 35.5) = 7.02 g
ͱ³» Þ¿­·½ ݱ²½»°¬­ ±º ݸ»³·­¬®§ ç

39. (c) Millimoles of solution of chloride


278
= 0.05 × 10 = 0.5 = 6.5g
223
Millimoles of AgNO3 solution = 10 × 0.1 = 1 Number of moles of PbCl2 formed
So, the millimoles of AgNO3 are double than the
chloride solution. 278 6.5
= moles = 0.029 moles.
XCl2 + 2AgNO3 2AgCl + X (NO3)2 223 278

6.02 1020 1000 6.02 1021 44. (c) Molarity of H2SO4 solution
40. (a) M = =
100 6.02 1023 6.02 1023 98 1000
1.80 18.0
= 0.01 M 98 100
41. (d) No. of milli equivalent of HCl = 20 × 0.05 Suppose V ml of this H2SO4 is used to prepare
= 1.0 1 lit. of 0.1M H2SO4
No. of milli equivalent of Ba (OH)2 V 18.0 1000 0.1
= 30 × 0.1 × 2 = 6.0
After neutralization, no. of milli equivalents in 1000 0.1
or V 5.55 ml.
50 ml. of solution = (6 – 1) = 5 18.0
Total volume of the solution = 20 + 30 = 50 ml 45. (c) 2Al 2O 3 3C Al 3CO 2
No. of milli equivalent of OH– in 50 ml is Gram equivalent of Al2O3 gm equivalent of C
5 1000 27
[OH ] 10 3 = 0.1 M Now equivalent weight of Al = =9
50 3
1 Equivalent weight of C
42. (b) H2 + O H 2O
2 2
10g 64g 12 0 4
= = 3 (C C O2 )
(5 mol) (2 mol) 4
In this reaction oxygen is the limiting agent.
Hence amount of H2O produced depends on 270 10 3
No. of gram equivalent of Al =
the amount of O2 taken 9
0.5 mole of O2 gives H2O = 1 mol = 30 × 103
2 mole of O2 gives H2O = 4 mol Hence,
43. (d) Writing the equation for the reaction, we get No. of gram equivalent of C = 30 × 103
Again,
PbO + 2HCl PbCl2 + H2O
No. of gram equivalent of C
207 + 16 2 × 36.5 207 + 71
mass in gram
= 223 g = 73g = 278g =
gram equivalent weight
From this equation we find 223 g of PbO reacts
mass
with 73 g of HCl to form 278 g of PbCl 2. 30 × 103 =
3
If we carry out the reaction between 3.2 g HCl
mass = 90 × 103 g = 90 kg
and 6.5 g PbO.
Amount of PbO that reacts with 3.2 g HCl 46. (c) N2 3H 2 2NH 3
1 vol. 3 vol. 2 vol.
223 10 litre 30 litre 20 litre
3.2 =9.77 g.
73 It is given that only 50% of the expected product
Since amount of PbO present is only 6.5 g so is formed hence only 10 litre of NH3 is formed
PbO is the limiting reagent. N2 used = 5 litres, left = 30 – 5 = 25 litres
Amount of PbCl 2 formed by 6.5 g of PbO H2 used = 15 litres, left = 30 – 15 = 15 litres
EBD_7324
ïð ÝØÛÓ×ÍÌÎÇ

15 22.4 39
47. (c) According to Stoichiometry they should = 84.0 litre
156
react as follow
49. (a) 5 M H2SO4 = 10 N H2SO4,
4 NH 3 5O 2 4 NO 6H 2 O
4 moles 5 moles 4 moles 6 moles
( Basicity of H2SO4 = 2)
0.8 moles
0 .8 moles 1 mole 1.2 moles N1V1 = N2V2,
Thus for 1 mole of O2 only 0.8 moles of NH3 is 10 × 1 = N2 × 10 or N2 = 1 N
consumed. Hence O2 is consumed completely.
50. (a) Mg 2 Na 2CO3 MgCO3 2Na
48. (d) 2C6 H 6 15O2 (g) 12CO2 (g) 6H 2 O(g)
1 g eq. 1g eq.
2(78) 15(32)
1 g eq. of Mg2+ = 12 g of Mg2+ = 12000 mg
156 gm of benzene requir ed oxygen
= 15 × 22.4 litre = 1000 milli eq. of Na 2CO3
1 gm of benzene required oxygen 12 mg Mg2+ = 1 milli eq. Na2CO3
15 22.4
= litre
156
39 gm of Benzene required oxygen
ݸ¿°¬»®

2 Structure of Atom

̱°·½ ï æ ߬±³·½ Ó±¼»´­ ¿²¼ Ü«¿´ Ò¿¬«®» ±º of the molecule per atom will be: [2009]
Û´»½¬®±³¿¹²»¬·½ ο¼·¿¬·±² (a) 2.2 × 10–19 J (b) 2.0 × 10–19 J
(c) 4.0 × 10–20 J (d) 2.0 × 10–20 J
1. Calculate the energy in joule corresponding to 6. The value of Planck's constant is 6.63 × 10–34 Js.
light of wavelength 45 nm : The velocity of light is 3.0 × 108 m s–1. Which value
(Planck’s constant h = 6.63 × 10 –34 Js; speed of is closest to the wavelength in nanometers of a
light c = 3 × 108 ms–1) [2014] quantum of light with frequency of 8 × 1015 s–1 ?
(a) 6.67 × 1015 (b) 6.67 × 1011 [2003]
(c) 4.42 × 10–15 (d) 4.42 × 10–18 (a) 3 × 107 (b) 2 × 10–25
2. According to law of photochemical equivalence (c) 5 × 10–18 (d) 4 × 101
the energy absorbed (in ergs/mole) is given as
7. If the energy of a photon is given as 3.03 × 10–19 J
(h = 6.62 × 10–27 ergs, c = 3 × 1010 cm s–1,
then, the wavelength ( ) of the photon is :
NA = 6.02 × 1023 mol–1) [NEET Kar. 2013]
[2000]
1.196 1016 1.196 108 (a) 6.56 nm (b) 65.6 nm
(a) (b) (c) 656 nm (d) 0.656 nm
8. In the photo-electron emission, the energy of
2.859 105 2.859 1016 the emitted electron is [1994]
(c) (d)
(a) greater than the incident photon
3. The value of Planck’s constant is 6.63 × 10–34 Js. (b) same as than of the incident photon
The speed of light is 3 × 1017 nm s–1.. Which (c) smaller than the incident photon
value is closest to the wavelength in nanometer (d) proportional to the intensity of incident
of a quantum of light with frequency of photon.
6 × 1015 s–1? [NEET 2013] 9. The electron was shown experimentally to have
(a) 25 (b) 50 wave properties by [1994]
(c) 75 (d) 10 (a) de Broglie
4. The energies E1 and E2 of two radiations are 25 eV (b) Davisson and Germer
and 50 eV, respectively. The relation between their
(c) N. Bohr
wavelengths i.e., 1 and 2 will be: [2011]
(d) Schrodinger.
(a) 1 = 2 (b) 1 = 2 2
10. Which of the following is never true for cathode
1 rays ? [1994]
(c) 1=4 2 (d) 1
2 2 (a) They possess kinetic energy
5. The energy absorbed by each molecule (A2) of (b) They are electromagnetic waves
a substance is 4.4 × 10–19 J and bond energy (c) They produce heat
per molecule is 4.0 × 10–19 J. The kinetic energy (d) They produce mechanical pressure.
EBD_7324
ïî ÝØÛÓ×ÍÌÎÇ
̱°·½ î æ Þ±¸®ù­ ³±¼»´ º±® ا¼®±¹»² ߬±³ 16. According to Bohr’s theory the energy required
øÛ³·­­·±² ¿²¼ ß¾­±®°¬·±² Í°»½¬®¿÷ for an electron in the Li 2+ ion to be emitted from
n = 2 state is (given that the ground state
Z2 ionization energy of hydrogen atom is 13.6 eV)
11. Based on equation E = – 2.178 × 10-18 J,
n2 [1999]
certain conclusions are written. Which of them (a) 61.2 eV (b) 13.6 eV
is not correct ? [NEET 2013] (c) 30.6 eV (d) 10.2 eV
(a) Larger the value of n, the larger is the orbit 17. The Bohr orbit radius for the hydrogen atom (n = 1)
radius. is approximately 0.530 Å. The radius for the first
(b) Equation can be used to calculate the excited state (n = 2) orbit is (in Å) [1998]
change in energy when the electron (a) 0.13 (b) 1.06
changes orbit. (c) 4.77 (d) 2.12
(c) For n = 1, the electron has a more negative
18. The radius of hydrogen atom in the ground state
energy than it does for n = 6 which mean
that the electron is more loosely bound in is 0.53 Å. The radius of Li2+ ion (atomic number = 3)
the smallest allowed orbit. in a similar state is [1995]
(d) The negative sign in equation simply (a) 0.17 Å (b) 0.265 Å
means that the energy or electron bound to (c) 0.53 Å (d) 1.06 Å
the nucleus is lower than it would be if the 19. If ionization potential for hydrogen atom is
electrons were at the infinite distance from 13.6 eV, then ionization potential for He+ will be
the nucleus. [1993]
12. According to the Bohr Theory, which of the (a) 54.4 eV (b) 6.8 eV
following transitions in the hydrogen atom will (c) 13.6 eV (d) 24.5 eV
give rise to the least energetic photon ? 20. The energy of an electron in the nth Bohr orbit of
[2011 M] hydrogen atom is [1992]
(a) n = 6 to n = 1 (b) n = 5 to n = 4 13.6 13.6
(c) n = 6 to n = 5 (d) n = 5 to n = 3 (a) – eV (b) – eV
n4 n3
13. The energy of second Bohr or bit of t he
hydrogen atom is 328 kJ mol 1; hence the 13.6 13.6
(c) – eV (d) – eV
energy of fourth Bohr orbit would be: n2 n
[2005] 21. If r is the radius of the first orbit, the radius of n th
(a) 41 kJ mol 1 (b) 82 kJ mol 1 orbit of H-atom is given by [1988]
(c) 164 kJ mol 1 (d) 1312 kJ mol 1 (a) rn2 (b) rn
14. The frequency of radiation emitted when the (c) r/n (d) r 2 n2
electron falls from n = 4 to n = 1 in a hydrogen 22. The spectrum of He is expected to be similar to
atom will be (Given ionization energy of that [1988]
H=2.18 ×10–18J atom–1and h = 6.625 × 10–34 J s ) (a) H (b) Li+
[2004] (c) Na (d) He+
(a) 1.54 1015 s 1 (b) 1.03 1015 s 1
̱°·½ íæ Ü«¿´ Þ»¸¿ª·±«® ±º Ó¿¬¬»® ¿²¼
(c) 3.08 1015 s 1 (d) 2.00 1015 s 1
Ø»·­»²¾»®¹ ˲½»®¬¿·²¬§ Ю·²½·°´»
15. In hydrogen atom, energy of first excited state is
–3.4 eV. Find out KE of the same orbit of 23. Which one is the wrong statement ? [2017]
Hydrogen atom [2002] (a) The uncertainty principle is
(a) + 3.4 eV (b) + 6.8 eV E t h/4
(b) Half filled and fully filled orbitals have
(c) – 13.6 eV (d) + 13.6 eV
greater stability due to greater exchange
ͬ®«½¬«®» ±º ߬±³ ïí

energy, greater symmetry and more (c) 8.0 × 10–26 kg ms–1


balanced arrangement. (d) 5.0 × 10–26 kg ms–1
(c) The energy of 2s orbital is less than the energy 29. The momentum of a particle having a de Broglie
of 2p orbital in case of Hydrogen like atoms wavelength of 10–17 metres is [1996]
(d) de-Broglies's wavelength is given by
(Given h = 6.625 × 10–34 Js)
h
, (a) 3.3125 × 10–7 kg ms–1
m
where m = mass of the particle, = group (b) 26.5 × 10–7 kg ms–1
velocity of the particle (c) 6.625 × 10–17 kg ms–1
24. A 0.66 kg ball is moving with a speed of 100 m/s. (d) 13.25 × 10–17 kg ms–1
The associated wavelength will be 30. Uncertainty in position of an electron
(mass = 9.1 × 10–28 g) moving with a velocity of
h 6.6 10 34 Js : [2010] 3 × 104 cm/s accurate upto 0.001% will be
(use h/4 ) in uncertainty expression where
(a) 1.0 10–32m (b) 6.6 10–32m h = 6.626 ×10–27 erg-second) [1995]
(c) 6.6 10 m –34 (d) 1.0 10–35m (a) 1.93 cm (b) 3.84 cm
25. The measurement of the electron position if (c) 5.76 cm (d) 7.68 cm
associated with an uncertainty in momentum, 31. When an electron of charge ‘e’ and mass ‘m’
which is equal to 1 × 10–18 g cm s– 1 . The moves with a velocity ‘v’ about the nuclear
uncertainty in electron velocity is, [2008] charge ‘Ze’ in circular orbit of radius ‘r’, the
(mass of an electron is 9 × 10 – 28 g) potential energy of the electrons is given by
(a) 1 × 109 cm s–1 (b) 1 × 106 cm s–1 [1994]
5 –1 (d) 1 × 1011 cm s–1
(c) 1 × 10 cm s (a) Ze 2 / r (b) Ze 2 / r
26. If uncertainty in position and momentum are 2
equal, then uncertainty in velocity is : [2008] (c) Ze 2 / r (d) mv / r
32. Which of the following statements do not form
1 h h a part of Bohr’s model of hydrogen atom ?
(a) (b) [1989]
2m 2
(a) Energy of the electrons in the orbits are
1 h h quantized
(c) (d) (b) The electron in the orbit nearest the nucleus
m
has the lowest energy
27. Given : The mass of electron is 9.11 × 10–31 kg (c) Electrons revolve in different orbits around
Plank constant is 6.626 × 10 –34 Js, the nucleus
the uncertainty involved in the measurement of (d) The position and velocity of the electrons
velocity within a distance of 0.1 Å is [2006] in the or bit cannot be determined
(a) 5.79 × 107 ms–1 (b) 5.79 × 108 ms–1 simultaneously.
5
(c) 5.79 × 10 ms –1 (d) 5.79 × 106 ms–1 ̱°·½ ì æ Ï«¿²¬«³ Ó»½¸¿²·½¿´ Ó±¼»´ ±º ߬±³
28. The position of both, an electron and a helium 33. Two electrons occupying the same orbital are
atom is known within 1.0 nm. Further the
distinguished by [2016]
momentum of the electron is known within
(a) Principal quantum number
5.0 × 10–26 kg ms–1. The minimum uncertainty
(b) Magnetic quantum number
in the measurement of the momentum of the
helium atom is [1998] (c) Azimuthal quantum number
(a) 50 kg ms–1 (d) Spin quantum number
(b) 80 kg ms–1
EBD_7324
ïì ÝØÛÓ×ÍÌÎÇ
34. What is the maximum number of orbitals that 42. Maximum number of electrons in a subshell of
can be identified with the following quantum an atom is determined by the following: [2009]
numbers? [2014] (a) 2 l + 1 (b) 4 l – 2
n = 3, = 1, m = 0
(c) 2 n2 (d) 4 l + 2
(a) 1 (b) 2
43. Consider the following sets of quantum numbers:
(c) 3 (d) 4
n l m s
35. What is the maximum numbers of electrons that
(i) 3 0 0 + 1/2
can be associated with the following set of
(ii) 2 2 1 + 1/2
quantum numbers? [NEET 2013]
n = 3, l = 1 and m = –1 (iii) 4 3 –2 – 1/2
(a) 6 (b) 4 (iv) 1 0 –1 – 1/2
(c) 2 (d) 10 (v) 3 2 3 + 1/2
36. The orbital angular momentum of a p-electron Which of the following sets of quantum number
is given as : [2012 M] is not possible? [2007]
h (a) (i), (ii), (iii) and (iv) (b) (ii), (iv) and (v)
h
(a) (b) 3 (c) (i) and (iii) (d) (ii), (iii) and (iv)
2 2
44. The orientation of an atomic orbital is governed
3h h by [2006]
(c) (d) 6.
2 2 (a) Spin quantum number
37. The correct set of four quantum numbers for the (b) Magnetic quantum number
valence electron of rubidium atom (Z = 37) is (c) Principal quantum number
[2012] (d) Azimuthal quantum number
(a) 5, 1, + 1/2 (b) 6, 0, 0 + 1/2 45. The ions O 2–, F– , Na +, Mg 2+ and Al 3+ are
(c) 5, 0, 0 + 1/2 (d) 5, 1, 0 + 1/2 isoelectronic. Their ionic radii show [2003]
38. Maximum number of electrons in a subshell with : (a) A decrease from O2– to F– and then increase
l = 3 and n = 4 is : [2012] from Na+ to Al3+
(a) 14 (b) 16 (b) A significant increase from O2– to Al3+
(c) 10 (d) 12 (c) A significant decrease from O2– to Al3+
39. If n = 6, the correct sequence for filling of (d) An increase from O2– to F – and then
electrons will be : [2011] decrease from Na+ to Al3+
(a) ns (n – 2) f (n – 1) d np 46. Which of the following is isoelectronic?[2002]
(b) ns (n – 1) d (n – 2) f np (a) CO2, NO2 (b) NO2–, CO2
(c) ns (n – 2) f np (n – 1) d –
(c) CN , CO (d) SO2, CO2
(d) ns np (n – 1) d (n – 2) f 47. The following quantum numbers are possible
40. The total number of atomic orbitals in fourth for how many orbital (s) n = 3, l = 2, m = +2 ?
energy level of an atom is : [2011] [2001]
(a) 8 (b) 16 (a) 1 (b) 3
(c) 32 (d) 4 (c) 2 (d) 4
41. Which of the following is not permissible 48. Set of isoelectronic species is [2000]
arrangement of electrons in an atom? [2009] (a) N2 ,CO2 ,CNó , O
(a) n = 5, l = 3, m = 0, s = + 1/2 (b) N, H 2S, CO
(b) n = 3, l = 2, m = – 3, s = – 1/2
(c) N 2 , CO, CN , O 22
(c) n = 3, l = 2, m = – 2, s = – 1/2
(d) Ca , Mg, Cl
(d) n = 4, l = 0, m = 0, s = – 1/2
ͬ®«½¬«®» ±º ߬±³ ïë

49. The ion that is isoelectronic with CO is [1997] 55. For azimuthal quantum number = 3, the
(a) CN– (b) O2+ maximum number of electrons will be [1991]
(c) O2 – (d) N2+ (a) 2 (b) 6
50. The orbitals are called degenerate when (c) 0 (d) 14.
[1996] 56. In a given atom no two electrons can have the
(a) they have the same wave functions same values for all the four quantum numbers.
(b) they have the same wave functions but This is called [1991]
different energies (a) Hund’s Rule
(c) they have different wave functions but (b) Aufbau principle
same energy (c) Uncertainty principle
(d) they have the same energy (d) Pauli’s Exclusion principle.
51. If electron has spin quantum number + 1/2 and 57. An ion has 18 electrons in the outermost shell, it
is [1990]
a magnetic quantum number – 1, it cannot be
(a) Cu+ (b) Th 4+
present in [1994]
(c) Cs+ (d) K+
(a) d-orbital (b) f-orbital
58. The total number of electrons that can be
(c) p-orbital (d) s-orbital. accommodated in all the orbitals having principal
52. For which one of the following sets of four quantum number 2 and azimuthal quantum
quantum numbers, an electron will have the number 1 is [1990]
highest energy? [1994] (a) 2 (b) 4
n l m s (c) 6 (d) 8
(a) 3 2 1 1/2 59. The maximum number of electrons in a subshell
(b) 4 2 –1 1/2 is given by the expression [1989]
(c) 4 1 0 –1/2 (a) 4l – 2 (b) 4l + 2
(c) 2l + 2 (d) 2n2
(d) 5 0 0 –1/2
60. Number of unpaired electrons in N2+ is [1989]
53. Which of the following species has four lone
(a) 2 (b) 0
pairs of electrons ? [1993]
(c) 1 (d) 34
(a) I (b) O 61. The number of spherical nodes in 3p orbitals are
(c) Cl– (d) He [1988]
54. The order of filling of electrons in the orbitals of (a) one (b) three
an atom will be [1991] (c) none (d) two
(a) 3d, 4s, 4p, 4d, 5s (b) 4s, 3d, 4p, 5s, 4d
(c) 5s, 4p, 3d, 4d, 5s (d) 3d, 4p, 4s, 4d, 5s

ANS WER KEY


1 (d) 8 (c) 15 (a) 22 (b) 29 (c) 36 (a) 43 (b) 50 (d) 57 (a)
2 (b) 9 (b) 16 (c) 23 (c) 30 (a) 37 (c) 44 (b) 51 (d) 58 (c)
3 (b) 10 (b) 17 (d) 24 (d) 31 (b) 38 (a) 45 (c) 52 (b) 59 (b)
4 (b) 11 (c) 18 (a) 25 (a) 32 (d) 39 (a) 46 (c) 53 (b) 60 (c)
5 (d) 12 (c) 19 (a) 26 (a) 33 (d) 40 (b) 47 (a) 54 (b) 61 (a)
6 (d) 13 (b) 20 (c) 27 (d) 34 (a) 41 (b) 48 (c) 55 (d)
7 (c) 14 (c) 21 (a) 28 (d) 35 (c) 42 (d) 49 (a) 56 (d)
EBD_7324
ïê ÝØÛÓ×ÍÌÎÇ

Hints & Solutions


6.63 10 34 3 108 8. (c) K.E. of emitted electron
hc
1. (d) E = = 9 = h h 0 (i.e. smaller than h ).
45 10
= 4.42 × 10– 18 J 9. (b) Wave nature of electron was shown by
hc Davisson and Germer. Davisson and Germer
2. (b) E = NA
demonstrated the physical reality of the wave
27 nature of electrons by showing that a beam of
6.62 10 3 1010 6.02 1023
electrons could also be diffracted by crystals
just like light of x-rays.
1.19 108 10. (b) Cathode rays are not electromagnetic waves.
= ergs mol–1
11. (c) Energy of an electron at infinite distance from
3. (b) c = the nucleus is zero. As an electron approaches
c 3 1017 the nucleus, the electron attraction increases
= = = 50 nm and hence the energy of electron decreases and
6 1015
thus becomes negative. Thus as the value of n
4. (b) Given E1 = 25eV E2 = 50 eV
decreases, i.e. lower the orbit is, more negative
hc hc E1 2 is the energy of the electron in it.
E1 E2
1 2 E2 1 12. (c) Energy of photon obtained from the
2 25 1 transition n = 6 to n = 5 will have least energy.
1 2 2
1 50 2 1 1
E 13.6Z 2
5. (d) K.E per atom n12 n22
2
4.4 ×10 –19 – 4.0 ×10 –19 z
= 13. (b) We know that En = E1
2 n
E1
0.4×10 –19 Given E2 = –328 kJ mol–1
= 2.0 10 –20 J 22
2 E2 328 1
E4 2 = kJmol = –82 kJmol–1
hc c 2 4
6. (d) E h or
1 1 1
14. (c) IE
3 10 8 8 h n12 n 22
3.75 10 m
8 1015 18
2.18 10 1 1
In nanometer = 3.75× 10 which is closest to 34 3.08 1015 s 1
4× 101 6.625 10 1 16
7. (c) The energy of photon, 15. (a) Suppose the nucleus of hydrogen atom have
hc charge of one proton +e. The electron revolves
E= 3.03 10 19 in an orbit of radius r around it. Therefore the
centripetal force is supplied by electrostatic force
6.626 10 34 3 108 of attraction between the electron and nucleus
or i.e.
3.03 10 19
19.878
or 10 7 = 6.56 × 10–7m= 656 nm m 2 Ze2
3.03
r r2
ͬ®«½¬«®» ±º ߬±³ ïé
the I.E. of H i.e., 13.6 × 4 = 54.4 eV
2 Ze2 20. (c) Energy of an electron in Bohr's orbit is given
or m
r
13.6
by the relationship. E n eV.
1 1 Ze2 n2
or m 2 K.E
2 2 r 21. (a) Radius of n th orbit = r 1 n2 . (for H-atom)
now total energy (En) = K.E + P.E 22. (b) Both He and Li+ contain 2 electrons each,
in first excited state therefore their spectrum will be similar.
23. (c) For h ydr ogen like atoms ener gy of
1 Ze 2
E mv 2 2s-orbital and 2p-orbital is equal.
2 r

1 Ze 2 Ze 2 h 6.6 10 34
24. (d) 1 10 35 m
2 r r mv 0.66 100
25. (a) p = m v
1 Ze 2
3.4 eV Substituting the given values of x and m, we get
2 r
1×10–18 gcms–1 = 9×10–28 g × v
1 Ze 2
K.E 3.4 eV 18
2 r 1 10
or v
16. (c) Energy of electron in 2nd orbit of Li +2 9 10 28
2
Z = 1.1 × 109 cm s–1 å 1×109 cm s–1
ã ó13.6
2
n h
26. (a) We know p. x
13.6 (3) 2 4
= = –30.6 eV
( 2) 2 or m. v. x
h
[ p= m v]
Energy required = 0 – (–30.6) = 30.6 eV 4
17. (d) Radius of hydrogen atom = 0.530 Å, Number since p = x (given)
of excited state (n) = 2 and atomic number of h h
hydrogen atom (Z) = 1. We know that the Bohr p. p or m v m v
4 4
radius. h
2
or v
n2 (2 ) 2 4 m2
(r) Radius of atom 0.530
Z 1 h 1 h
4 0.530 2.12 Å or v 2
4 m 2m
18. (a) State of hydrogen atom (n) = 1
h
(due to ground state) 27. (d) We know that x . p
Radius of hydrogen atom (r) = 0.53 Å. Atomic 4
h
number of Li (Z) = 3. x.m v
Radius of Li 2+ ion 4
h
v
n2 (1)2 4 xm
r 0.53 0.17Å.
Z 3 6.626 10 34
19. (a) The ionization energy of any hydrogen like v
4 0.1 10 10 9.11 10 31
species (having one electron only) is given by 66
the equation 107 5.79 106 m / sec
4 9
2 2 2
Z m e4 28. (d) By Heisenberg uncertainty Principle
I.E or I.E Z 2 h
h2 Üx lÜp ã (which is constant)
Since the atomic number of H is 1 and that of He 4°
As x for electron and helium atom is same thus
is 2, therefore, the I.E. of He+ is four times (22)
EBD_7324
ïè ÝØÛÓ×ÍÌÎÇ
momentum of electron and helium will also be 37. (c) Electronic configuration of Rb = [Kr] 5s 1
same and the uncertainty in momentum of helium Set of quantum numbers, n = 5
atom is equal to 5 × 10–26 kg. m.s–1. = 0, s-orbital
29. (c) Acc. to de-Broglie, m = 0, s = + 1/2
h h 6.626 10 34 38. (a) (n = 4, l = 3) 4f subshell
mv
mv 10 17 Since, maximum no. of electrons in a subshell
–17
p = 6.626 × 10 kg m/s = 2(2l + 1)
30. (a) Given mass of an electron(m) = 9.1 × 10–28 g;
So, total no. of electron in 4f subshell
Velocity of electron ( v) 3 10 4 cm / s;
= 2 (2 × 3 + 1) = 14 electrons.
Accuracy in velocity 0.001% 0.001 ; 39. (a) ns (n 2)f (n 1)d np [n = 6]
100
Actual velocity of the electron 40. (b) Total no. of atomic orbital in a shell = n2.
0.001 Given n = 4; hence number of atomic orbitals in
( v ) 3 10 4 0.3 cm / s . 4th shell will be 16.
100
Planck’s constant (h) = 6.626×10–27erg-sec. 41. (b) m = 2 l +1, thus for l = 2, m = 5, hence values
Uncertainty in the position of the electron of m will be – 2, –1, 0, + 1, + 2.
Therefore for l = 2, m cannot have the value – 3.
h 6.626l10ó27 l 7
(Üx) ã ã 42. (d) The number of subshell is (2 l + 1). The
4° mÜv 4 l 22 l(9.1l10ó28 ) l0.3
maximum number of electrons in the sub shell is
=1.93cm 2 (2 l + 1) = (4 l + 2)
r
Ze2 Ze 2 43. (b) (ii) is not possible for any value of n because
31. (b) P.E. = work done = dr .
r2 r l varies from 0 to (n – 1) thus for n = 2, l can be
32. (d) It is uncertainty principle which was given only 0, 1, 2.
by Heisenberg and not Bohr’s postulate (iv) is not possible because for l = 0, m = 0.
33. (d) Two electrons occupying the same orbital (v) is not possible because for l = 2, m varies
should have opposite spins i.e. they differ in from –2 to +2.
spin quantum number. 44. (b) Magnetic quantum n o. represen ts the
34. (a) Given: n = 3, l = 1, m = 0 orientation of atomic orbitals in an atom. For
Hence orbital is 3p example px, py & p z have orientation along
X-axis, Y-axis & Z-axis.
–1 0 +1 45. (c) Amongst isoelectronic species, ionic radii of
anion is more than that of cations. Further size
of anion increase with increase in –ve charge
hence the number of orbital identified by m = 0
and size of cation decrease with increase in + ve
can be one only. charge. Hence ionic radii decreases from O– to
35. (c) n = 3 3rd shell Al3+.
l = 1 p sub shell. 46. (c) Both CN– and CO have 14 electrons
m = – 1 is possible for two electrons present in 47. (a) Quantum number n = 3, l = 2, m = +2 represent
an orbital. one of the 5d-orbitals (3d) orbital with
1
36. (a) Orbital angular momentum s
2
h which is possible only for one e–.
( 1)
2 48. (c) The molecule which contains same number
For p orbital = 1 of electrons are called isoelectronic. eg.
h h
So, = 2 N2 = CO = CN = O 2 2 = 14e–
2 2
ͬ®«½¬«®» ±º ߬±³ ïç

49. (a) We know that ions which have the same n+ 4 + 0 3 + 2 4 +1 5 + 0 4 + 2


number of electrons are called isoelectronic. We value =4 5 5 5 6
also know that both CO and CN – have 14 55. (d) = 3 means f-subshell. Maximum no. of
electrons, therefore these are isoelectronic. electrons = 4 + 2 = 4 × 3 + 2 = 14
50. (d) The orbitals which have same energy are 56. (d) This is as per the definition of Pauli’s
called degenerate orbitals eg. p x , p y , p z . Exclusion principle.
51. (d) m = (2 + 1) and m = – 1 means, 57. (a) Cu+ = 29 – 1 = 28 e–
–1 = 2 + 1, = 1 thus the electronic confingration of Cu+ is
i.e least value of = 1. So it cannot be present
in s-orbital. Because for s orbital = 0. = 1s2 2s 2 2p 6 3s
ïì
2
3p6 3d10
ìîìì í
52. (b) The sub-shell are 3d, 4d, 4p and 4s, 4d has 18eó
highest energy as n + value is maximum for 58. (c) n = 2, = 1 means 2 p – orbital. Electrons that
this.
can be accommodated = 6 as p sub-shell has 3
53. (b) Outer electronic configuration of Cl
orbital and each orbital contain 2 electrons.
2 2 2 1
= 3s 3p x 3p y 3p z 59. (b) No. of orbitals in a sub-shell = 2l + 1
Outer electronic configuration of No. of electrons = 2(2l + 1) = 4l + 2
60. (c) N(7) = 1s2 2s2 2p2
Cl – 3s 2 3p2x 3p y 2 3pz 2
N 2õ ã 1s2 , 2s2 2p1x
hence Cl– contain four lone pairs of electron
54. (b) The sub-shell with lowest value of (n + ) is Unpaired electrons = 1.
filled up first. When two or more sub-shells have 61. (a) No. of radial nodes in 3p-orbital
same (n + ) value the subshell with lowest value = (n – l – 1)
of 'n' is filled up first therefore the correct order is [for p ortbital l = 1]
orbital 4s 3d 4p 5s 4d =3–1–1 =1
EBD_7324
îð ÝØÛÓ×ÍÌÎÇ

ݸ¿°¬»®
Classification of Elements
3 and Periodicity in
Properties
̱°·½ ïæ Ó±¼»®² л®·±¼·½ Ì¿¾´» (b) B < C < N < O (increasing first ionisation
enthalpy)
1. The element Z = 114 has been discovered
(c) I < Br < Cl < F (increasing electron gain
recently. [2017]
enthalpy)
It will belong to which of the following family/
group and electronic configuration ? (d) Li < Na < K < Rb (increasing metallic radius)
(a) Carbon family, [Rn] 5f 14 6d10 7s2 7p2 7. The species Ar, K+ and Ca2+ contain the same
(b) Oxygen family, [Rn] 5f 14 6d10 7s2 7p4 number of electrons. In which order do their
(c) Nitrogen family, [Rn] 5f 14 6d10 7s2 7p6 radii increase ? [2015]
(d) Halogen family, [Rn] 5f 14 6d10 7s2 7p5 (a) Ca 2+ < Ar < K + (b) Ca 2+ < K + < Ar
2. An atom has electronic configuration 1s2 2s2 2p6
3s2 3p6 3d3 4s2, you will place it in which group? (c) K + < Ar < Ca 2+ (d) Ar < K + < Ca 2+
[2002] 8. The formation of the oxide ion O 2–(g), from
(a) Fifth (b) Fifteenth oxygen atom requires first an exothermic and
(c) Second (d) Third then an endothermic step as shown below :
3. The element, with atomic number 118, will be O(g) + e– O–(g); fH
= –141 kJ mol –1
[1996]
(a) alkali (b) noble gas O– (g) + e– O2– (g); f H = +780 kJ mol –1
(c) lanthanide (d) transition element Thus process of formation of O2– in gas phase
4. The electronic configuration of an element is is unfavourable even though O 2– is
isoelectronic with neon. It is due to the fact
1s 2 2s 2 2p 6 3s 2 3p 3 . What is the atomic number
that [2015 RS]
of the element, which is just below the above (a) Electron repulsion outweighs the stability
element in the periodic table? [1995] gain ed by achieving noble gas
(a) 33 (b) 34 configuration
(c) 36 (d) 49 (b) O– ion has comparatively smaller size than
5. If the atomic number of an element is 33, it will be oxygen atom
placed in the periodic table in the [1993] (c) Oxygen is more electronegative
(a) First group (b) Third group (d) Addition of electron in oxygen results in
(c) Fifth group (d) Seventh group. larger size of the ion.
̱°·½ îæ л®·±¼·½ Ì®»²¼­ ·² Ю±°»®¬·»­ ±º 9. Which of the following orders of ionic radii is
Û´»³»²¬­ correctly represented ? [2014]
(a) H– > H+ > H (b) Na+ > F– > O2–
6. In which of the following options the order of
arrangement does not agree with the variation (c) F– > O2– > Na+ (d) Al3+> Mg2+> N3–
of property indicated against it ? [2016] 10. Which one of the following arrangements
(a) Al3+ < Mg2+ < Na+ < F– (increasing ionic represents the correct order of least negative
to most negative electron gain enthalpy for C,
size) Ca, Al, F and O? [NEET Kar. 2013]
Ý´¿­­·º·½¿¬·±² ±º Û´»³»²¬­ ¿²¼ л®·±¼·½·¬§ ·² Ю±°»®¬·»­ îï

(a) Ca < Al < C < O < F (a) NH 2 (b)F–


(b) Al < Ca < O < C < F (c) I– (d) HS–
(c) Al < O < C < Ca < F 19. Which of the following electronic configuration
(d) C < F < O < Al < Ca of an atom has the lowest ionisation enthalpy?
11. Identify the wrong statement in the following: [2007]
[2012] (a) 1s2 2s2 2p3 (b) 1s2 2s2 2p5 3s1
(a) Amongst isoelectronic species, smaller the (c) 1s2 2s2 2p6 (d) 1s2 2s2 2p5
positive charge on the cation, smaller is the 20. Identify the correct order of the size of the
ionic radius. following: [2007]
(b) Amongst isoelectronic species, greater the
negative charge on the anion, larger is the (a) Ca2+ < K+ < Ar < Cl– < S2–
ionic radius. (b) Ar < Ca2+ < K+ < Cl– < S2–
(c) Atomic radius of the elements increases as (c) Ca2+ < Ar < K+ < Cl– < S2–
one moves down the first group of the (d) Ca2+ < K+ < Ar < S2– < Cl–
periodic table. 21. Which one of the following arrangements
(d) Atomic radius of the elements decreases represents the correct order of electron gain
as one moves across from left to right in enthalpy (with negative sign) of the given
the 2nd period of the periodic table. atomic species? [2005]
12. What is the value of electron gain enthalpy of (a) S < O < Cl < F (b) Cl < F < S < O
Na+ if IE1 of Na = 5.1 eV ? [2011M] (c) F < Cl < O < S (d) O < S < F < Cl
(a) –5.1 eV (b) –10.2 eV 22. Which one of the following oxides is expected
(c) +2.55 eV (d) +10.2 eV to exhibit paramagnetic behaviour?
13. Among the elements Ca, Mg, P and Cl, the order [2005]
of increasing atomic radii is : [2010] (a) CO2 (b) SiO2
(a) Ca < Mg < P < Cl (b) Mg < Ca < Cl < P (c) SO2 (d) ClO2
(c) Cl < P < Mg < Ca (d) P < Cl < Ca < Mg 23. Ionic radii are [2004]
14. Which of the following represents the correct (a) inversely proportional to effective nuclear
order of increasing electron gain enthalpy with charge
negative sign for the elements O, S, F and Cl ? (b) inversely propor tional to square of
[2010] effective nuclear charge
(a) Cl < F < O < S (b) O < S < F < Cl (c) directly proportional to effective nuclear
(c) F < S < O < Cl (d) S < O < Cl < F charge
15. The correct order of the decreasing ionic radii (d) directly proportional to square of effective
among the following isoelectronic species are : nuclear charge
(a) Ca 2+ > K + > S2– > Cl – [2010] 24. Among K, Ca, Fe and Zn the element which can
– 2 2 form more than one binary compound with
(b) Cl S Ca K
(c) S2– > Cl – > K + > Ca 2+ chlorine is [2004]
(d) K + > Ca 2+ > Cl – > S 2– (a) Fe (b) Zn
16. Amongst the elements with following electronic (c) K (d) Ca
configurations, which one of them may have the 25. Which of the following statements is true?
highest ionization energy? [2009] [2002]
(a) Ne [3s23p2] (b) Ar [3d104s24p3 ] (a) Silicon exhibits 4 coordination number in
(c) Ne [3s23p1] (d) Ne [3s23p3] its compound
17. The stability of + 1 oxidation state increases in (b) Bond energy of F2 is less than Cl2
the sequence: [2009]
(c) Mn(III) oxidation state is more stable than
(a) Tl < In < Ga < Al (b) In < Tl < Ga < Al Mn(II) in aqueous state
(c) Ga < In < Al < Tl (d) Al < Ga < In < Tl
(d) Elements of 15th group shows only +3 and
18. Which one of the following ionic species has +5 oxidation states
the greatest proton affinity to form stable
compound? [2007]
EBD_7324
îî ÝØÛÓ×ÍÌÎÇ
26. Which of the following order is wrong? 34. Which electronic configuration of an element has
[2002] abnormally high difference between second and
(a) NH3 < PH3 < AsH3 – Acidic third ionization energy ? [1993]
(b) Li < Be < B < C – First IP (a) 1 s2, 2 s2, 2 p6, 3 s1
(c) Al2O3 < MgO < Na2O < K2O – Basic (b) 1 s2, 2 s2, 2 p6, 3 s1 3 p1
(c) 1 s2, 2 s2, 2 p6, 3 s2 3 p2
(d) Li+ < Na+ < K+ < Cs+ – Ionic radius (d) 1 s2, 2 s2, 2 p6, 3 s2
27. Correct order of first IP among following elements 35. In the periodic table from left to right in a period,
Be, B, C, N, O is [2001] the atomic volume [1993]
(a) B < Be < C < O < N (a) Decreases
(b) B < Be < C < N < O (b) Increases
(c) Be < B < C < N < O (c) Remains same
(d) Be < B < C < O < N (d) First decrease then increases
28. Of the given electronic configurations for the 36. Na+, Mg++, Al3+ and Si4+ are isoelectronic. The
elements, which electronic configuration order of their ionic size is [1993]
indicates that there will be abnormally high (a) Na+ > Mg++ < Al3 + < Si4+
difference in the second and third ionization (b) Na+ < Mg++ > Al3+ > Si4+
energy for the element? [1999] (c) Na+ > Mg++ > Al3+ > Si4+
(a) 1s2 2s2 2p6 3s2 (b) 1s2 2s2 2p6 3s1 (d) Na+ < Mg++ > Al3+ < Si4+
(c) 1s2 2s2 2p6 3s2 3p1(d) 1s2 2s2 2p6 3s2 3p2 37. One would expect proton to have very large
29. The first ionization potentials (eV) of Be and B [1993]
respectively are [1998] (a) Charge (b) Ionization potential
(c) Hydration energy (d) Radius.
(a) 8.29, 9.32 (b) 9.32, 9.32
38. Which of the following sets has strongest
(c) 8.29, 8.29 (d) 9.32, 8.29
tendency to form anions ? [1993]
30. Which of the following does not represent the (a) Ga, In, Tl (b) Na, Mg, Al
correct order of the properties indicated
(c) N, O, F (d) V, Cr, Mn
[1997]
39. Elements of which of the following groups will
(a) Ni2+ > Cr2+ > Fe2+ > Mn2+ (size) form anions most readily ? [1992]
(b) Sc > Ti > Cr > Mn (size) (a) Oxygen family (b) Nitrogen family
(c) Mn 2+ > Ni 2+ < Co2+ <Fe2+ (unpaired (c) Halogens (d) Alkali metals
electron) 40. In the periodic table, with the increase in atomic
(d) Fe2+ > Co2+ > Ni 2+ > Cu 2+ (unpaired number, the metallic character of an element
electron) [1989]
31. Which one of the following ions will be the (a) Decreases in a period and increases in a group
smallest in size? [1996] (b) Increases in a period and decreases in a group
(a) Na+ (b) Mg2+ (c) Increases both in a period and the group
(c) F– (d) O2– (d) Decreases in a period and the group.
32. Among the following oxides, the one which is 41. The electronic configuration of four elements are
most basic is [1994] given below. Which element does not belong to
(a) ZnO (b) MgO the same family as others ? [1989]
(c) Al2O3 (d) N2O5 (a) [Xe]4f145d101s2 (b) [Kr]4d10 5s 2
33. One of the characteristic properties of non-metals (c) [Ne]3s23p5 (d) [Ar] 3d10 4s2
is that they [1993] 42. Pauling’s electronegativity values for elements
(a) Are reducing agents are useful in predicting [1989]
(b) Form basic oxides (a) Polarity of the molecules
(b) Position in the E.M.F. series
(c) Form cations by electron gain
(c) Coordination numbers
(d) Are electronegative
(d) Dipole moments.
Ý´¿­­·º·½¿¬·±² ±º Û´»³»²¬­ ¿²¼ л®·±¼·½·¬§ ·² Ю±°»®¬·»­ îí

ANS WER KEY


1 (a) 6 (b) 11 (a) 16 (d) 21 (d) 26 (b) 31 (b) 36 (c) 41 (c)
2 (a) 7 (b) 12 (a) 17 (d) 22 (d) 27 (a) 32 (b) 37 (c) 42 (a)
3 (b) 8 (a) 13 (c) 18 (a) 23 (a) 28 (a) 33 (a) 38 (c)
4 (a) 9 (N) 14 (b) 19 (b) 24 (a) 29 (d) 34 (d) 39 (c)
5 (c) 10 (a) 15 (c) 20 (a) 25 (b) 30 (a) 35 (d) 40 (a)

Hints & Solutions


1. (a) Z = 114 belong to Group 14, carbon family 7. (b) In isoelectronic species the radius decrease
Electronic configuration = [Rn]5f146d107s27p2 with increase in nuclear charge hence increasing
2. (a) The electronic configuration clear ly
suggest that it is a d-block element (having order of radius is Ca+2 < K+ < Ar
configuration (n – 1) d 1– 10 ns 0 – 2 ) which starts 8. (a) Incoming electrons occupies the smaller n
from III B and goes till II B. Hence with d3 = 2 shell, also negative charge on oxygen (O–) is
configuration it would be classified in the fifth another factor due to which incoming electron
group.
feel repulsion.
3. (b) Electronic configuration of element with
atomic number 118 will be [Rn]5f 14 6d 1 0 Hence electron repulsion outweigh the stability
7s27p6. Since its elctronic configuration in the gained by achieving noble gas configuration.
outer most orbit (ns2np6) resemble with that
of inert or noble gases, therefore it will be 9. (N) All answers are incorrect.
noble gas element. 10. (a) As the nuclear charge increases, the force
4. (a) Atomic number of the given element is 15 of attraction between the nucleus and the
and it belongs to 5th group. Therefore atomic incoming electron increases and hence the
number of the element below the above elecron gain enthalpy becomes more negative,
element = 15 + 18 = 33. hence the correct order is
5. (c) Element with Z = 33 Ca < Al < C < O < F
(1s2 2s p 6 3s2 p6 d10 4s2 p3 ) lies in fifth (or 15th) 11. (a) As the positive charge increases on metal
group. cation, radius decreases. This is due to the fact
that nuclear charge in the case of a cation is
6. (b) The correct order is B < C < O < N acting on lesser number of electrons and pulls
Generally ionisation energy increases across them closer.
a per iod. But here first I.E. of O 12. (a) IE1 of Na = – Electron gain enthalpy of Na +
is less than the first I.E. of N. This is due to the half- = – 5.1 eV.
filled 2p orbital in N(1s2, 2s2, 2p3) which is more 13. (c) 12Mg 15 P 17Cl 20Ca
stable than the 2p orbital in O (1s2, 2s2, 2p4). 160 110 99 197 (pm)
So the order will be: Cl < P < Mg < Ca
(3) The correct order of electron affinity is 14. (b) O < S < F < Cl
I < Br < F < Cl Electron gain enthalpy of given elements are
– 141, – 200, – 333 and – 349 kJ mol – 1
Halogens have high electron affinities which
respectively.
decreases on moving down the group. However, 15. (c) Among the isoelectronic species, size
fluorine has lower value than chlorine which is increases with the increase in negative charge.
due to its small size and repulsion between the Thus S2– has the highest negative charge and
electron added and electrons already present. hence largest in size followed by Cl–, K+ and Ca2+.
EBD_7324
îì ÝØÛÓ×ÍÌÎÇ
16. (d) The smaller the atomic size, larger is the 24. (a) Among the given options, only Fe shows
value of ionisation potential. Further the atoms variable oxidation states so it can form two
having half filled or fully filled orbitals are chlorides, viz. FeCl2 and FeCl3.
comparatively more stable, hence more energy 25. (b) This is because of inter-electronic replusions
is required to remove the electron from such between lone pairs.
atoms. B.E. : F–F Cl – Cl
17. (d) The stability of +1 oxidation state increases (kJ mol–1) : 158.8 242.6
from aluminium to thallium i.e. .. ..
Al < Ga < In < Tl : .F. F:
..
18. (a) Proton affinity decreases in moving across 26. (b) Along the period, I.P. generally increases
the period from left to right due to increase in but not regularly. Be and B are exceptions. First I.P.
charge, within a group the proton affinities
increases in moving from left to right in a period,
decreases from top to bottom.
but I.P. of B is lower than Be.
Nitrogen family > Oxygen family > Halogens
27. (a) Be – 1s22s2; B – 1s22s22p1; C – 1s22s22p2;
19. (b)
N – 1s22s22p3; O – 1s22s22p4. IP increases along
20. (a) For isoelectronic species, size of anion
increases as negative charge increases whereas the period. But IP of Be > B. Further IP of O < N
size of cation decreases with increase in positive because atoms with fully or partly filled orbitals are
charge. Further ionic radii of anions is more than most stable and hence have high ionisation energy.
that of cations. Thus the correct order is 28. (a) Mg 1s 2 2s 2 2p 6 3s 2
Ca++ < K+ < Ar < Cl– < S– –
After the removal of 2 electrons, the magnesium
21. (d) The amount of energy released when an
electron is added to an isolated gaseous atom ion will acquire noble gas configuration hence
to produce a monovalent anion is called removal of 3rd electron will require large amount
electron gain enthalpy. of energy.
Electron affinity value generally increase on 29. (d) First ionisation potential of Be is greater
moving from left to right in a period however than boron due to following configuration
there are exceptions of this rule in the case of 2 2 2 2 1
4Be=1s ,2s 5B=1s ,2s 2p
those atoms which have stable configuration. Order of attraction of electrons towards nucleus
These atoms resist the addition of extra 2s>2p, so more amount of energy is required to
electron, therefore the low value of electron
affinity remove the electron in 2s-orbital in comparison
to 2p orbital.
O S F Cl
1.48 2.0 3.6 3.8 30. (a) In a period on moving from left to right ionic
On the other hand Cl, because of its radii decreases.
comparatively bigger size than F, allow the (a) So order of cationic radii is
addition of an extra electron more easily. Cr 2+ > Mn2+ > Fe2+ > Ni2+ and
22. (d) Due to odd number of electrons in ClO2, (b) Sc > Ti > Cr > Mn
it i s expect ed to exhibit paramagn etic
behaviour. (correct order of atomic radii)
(c) For unpaired electrons
Cl
Mn 2 (Five) Ni 2 (Two)
O O
Paramagnetic Co 2 (Three) Fe2 (Four)
23. (a) Ionic radii are inversely proportional to (d) For unpaired electrons
effective nuclear charge. Fe2 (Four) Co2 (Three)
Ionic radii in the n th orbit is given as
Ni 2 (Two) Cu 2 (One)
n 2a0 1 31. (b) Greater is the positive charge on atom, larger
rn or rn
Z Z will be the effective nuclear charge. Hence smaller
when n = principal quantum number is the size.
Z=effective nuclear change.
Ý´¿­­·º·½¿¬·±² ±º Û´»³»²¬­ ¿²¼ л®·±¼·½·¬§ ·² Ю±°»®¬·»­ îë

32. (b) N2O5 is strongly acidic, ZnO and Al2 O3 are strongest tendency to form anions by gaining
amphoteric, therefore, MgO is most basic. electrons from metal atoms.
33. (a) Non metals form oxides with oxygen and 39. (c) Electron affinity values are high in case of
halogen because halogens have seven electrons
thus reduce oxides of metals behaving as
reducing agents. ( ns 2 np 5 ) in the valence shell, they have a
34. (d) Abnormally high difference between 2nd strong tendency to acquire the nearest inert gas
configuration by gaining an electron from the
and 3rd ionization energy means that the
metallic atom and form halide ions easily.
element has two valence electrons.
40. (a) Metallic character decreases in a period and
35. (d) Atomic volume is the volume occupied by increases in a group.
one gram of an element. Within a period from 41. (c) Elements (a), (b) and (d) belong to the same
left to right, atomic volume first decreases and group since each one of them has two electrons
then increases due to increases of nuclear in the s-sub shell. In contrast, element (c) has
charge and increase in the number of electrons seven electrons in the valence shell and hence
in the valence shell. does not lie in the same group in which elements
36. (c) Amongst isoelectronic ions, the size of the (a), (b) and (d) lie.
cation decreases as the magnitude of the charge 42. (a) Pauling scale of electronegativity was
increases. helpful in predicting :
37. (c) Proton (H+) being very small in size would (i) Nature of bond between two atoms
have very large hydration energy. (ii) Stability of bond
38. (c) N, O and F (p-block elements) are highly By calculating the difference in electro-
electronegative non metals and will have the negativities, polarity of bond can be calculated.
EBD_7324
îê ÝØÛÓ×ÍÌÎÇ

ݸ¿°¬»®

4 Chemical Bonding and


Molecular Structure

̱°·½ ïæ Û´»½¬®±ª¿´»²¬ô ݱª¿´»²¬ ¿²¼ (a) Na and Cl (b) Mg and F


ݱ󱮼·²¿¬» Þ±²¼·²¹ (c) Li and F (d) Na and F
9. The weakest among the following types of bonds
1. Which of the following pairs of species have is [1994]
the same bond order ? [2017] (a) ionic (b) covalent
(a) O2, NO+ (b) CN–, CO (c) metallic (d) H–bond.
(c) N 2 , O 2 (d) CO, NO 10. Strongest bond is in between [1993]
(a) CsF (b) NaCl
2. Which of the following is electron - deficient?
(c) Both (a) and (b) (d) None of above
[NEET 2013] 11. Among the following which compound will
(a) (SiH3)2 (b) (BH3)2 show the highest lattice energy ? [1993]
(c) PH3 (d) (CH3)2 (a) KF (b) NaF
3. The correct sequence of increasing covalent (c) CsF (d) RbF
character is represented by [2005] 12. Among LiCl, BeCl2 BCl3 and CCl4, the covalent
bond character follows the order [1990]
(a) LiCl < NaCl < BeCl 2
(a) LiCl < BeCl2 > BCl3 > CCl4
(b) BeCl 2 < LiCl < NaCl (b) BeCl2 < BCl3 < CCl4 < LiCl
(c) NaCl < LiCl < BeCl2 (c) LiCl < BeCl2 < BCl3 < CCl4
(d) BeCl 2 < NaCl < LiCl (d) LiCl > BeCl2 > BCl3 > CCl4
4. Which of the following is the electr on 13. Which of the following does not apply to metallic
deficient molecule? [2005] bond ? [1989]
(a) Overlapping valence orbitals
(a) C2H6 (b) B2H6
(b) Mobile valency electrons
(c) SiH4 (d) PH3 (c) Delocalized electrons
5. Cation and anion combines in a crystal to form (d) Highly directed bonds.
following type of compound. [2000]
̱°·½ îæ ѽ¬»¬ ®«´»ô λ­±²¿²½» ¿²¼
(a) ionic (b) metallic
ا¼®±¹»² Þ±²¼·²¹
(c) covalent (d) dipole-dipole
6. Which of the following compounds has a 3- 14. Which of the following structures is the most
centre bond? [1996] preferred and hence of lowest energy for SO3?
(a) Diborane (b) Carbon dioxide [2011 M]
(c) Boron trifluroide (d) Ammonia O
7. Linus Pauling received the Nobel Prize for his S
S
work on [1994] (a) (b) O O
O O
(a) atomic structure (b) photosynthesis
(c) chemical bonds (d) thermodynamics O O
8. Which of the following pairs will form the most S S
(c) (d)
stable ionic bond ? [1994] O O O O O
ݸ»³·½¿´ Þ±²¼·²¹ ¿²¼ Ó±´»½«´¿® ͬ®«½¬«®» îé

15. What is the dominant intermolecular force or (a) NO2 group at p-position behave in a
bond that must be overcome in converting liquid different way from that at o-position.
CH3OH to a gas? [2009] (b) intramolecular hydrogen bonding exists in
(a) Dipole-dipole interaction p-nitrophenol
(c) there is intermolecular hydrogen bonding
(b) Covalent bonds in p-nitrophenol
(c) London dispersion force (d) p-nitrophenol has a higher molecular
(d) Hydrogen bonding weight than o-nitrophenol.
16. Which of the following is not a correct 23. Which one of the following is the correct order
statement? [2006] of interactions ? [1993]
(a) The canonical structures have no real (a) Covalent < hydrogen bonding < vander
existence Waals < dipole-dipole
(b) Every AB5 molecule does in fact have (b) vander Waals < hydrogen bonding < dipole
square pyramidal structure < covalent
(c) Multiple bonds are always shorter than (c) vander Waals < dipole-dipole < hydrogen
corresponding single bonds bonding < covalent
(d) The electron-deficient molecules can act (d) Dipole-dipole < vander Waals < hydrogen
as Lewis acids bonding < covalent.
17. In X — H --- Y, X and Y both are electronegative 24. Strongest hydrogen bond is shown by [1992]
(a) Water
elements
(b) Ammonia
(a) Electron density on X will increase and
(c) Hydrogen fluoride
on H will decrease [2001] (d) Hydrogen sulphide.
(b) In both electron density will decrease 25. Which one of the following formulae does not
(c) In both electron density will increase correctly represent the bonding capacities of
(d) Electron density will decrease on X and the two atoms involved ? [1990]
will increase on H
18. Among the following the electron deficient H
|
compound is : [2000] H — P—H
(a) BCl3 (b) CCl4 (a) |
(c) PCl5 (d) BeCl2 H
19. Which one of the following molecules will form (b) F F
a linear polymeric structure due to hydrogen O
bonding? [2000] O
(a) NH3 (b) H2O (c) O N
(c) HCl (d) HF O–H
20. In PO43– ion, the formal charge on each oxygen O
atom and P—O bond order respectively are (d) H – C = C
[1998] O–H
26. Which one shows maximum hydrogen bonding?
(a) –0.75, 0.6 (b) – 0.75, 1.0 (a) H2O (b) H2 Se [1990]
(c) – 0.75, 1.25 (d) –3, 1.25 (c) H2S (d) HF.
21. The low density of ice compared to water is due
to [1997] ̱°·½ íæ Ü·°±´» Ó±³»²¬ ¿²¼ Þ±²¼ б´¿®·¬§
(a) hydrogen-bonding interactions 27. Which of the following molecules has the
(b) dipole-dipole interactions maximum dipole moment ? [2014]
(c) dipole-induced dipole interactions (a) CO2 (b) CH4
(d) induced dipole-induced dipole interactions (c) NH3 (d) NF3
22. The boiling point of p-nitrophenol is higher than 28. Which of the following is a polar molecule ?
that of o-nitrophenol because [1994]
[NEET 2013]
EBD_7324
îè ÝØÛÓ×ÍÌÎÇ
(a) SF4 (b) SiF4 34. Which of the following bonds will be most polar?
(c) XeF4 (d) BF3 [1992]
29. The electronegativity difference between N and (a) N – Cl (b) O – F
F is greater than that between N and H yet the (c) N – F (d) N – N
dipole moment of NH3 (1.5 D) is larger than 35. H2O has a non zero dipole moment while BeF2
that of NF3 (0.2D). This is because [2006] has zero dipole moment because [1989]
(a) in NH3 the atomic dipole and bond dipole (a) H2O molecule is linear while BeF2 is bent
are in the same direction whereas in NF3 (b) BeF2 molecule is linear while H2O is bent
these are in opposite directions (c) Fluorine has more electronegativity than
(b) in NH3 as well as NF3 the atomic dipole oxygen
and bond dipole are in opposite directions (d) Beryllium has more electronegativity than
(c) in NH3 the atomic dipole and bond dipole oxygen.
are in the opposite directions whereas in ̱°·½ ìæ ÊÍÛÐΠ̸»±®§ ¿²¼ ا¾®·¼·­¿¬·±²
NF3 these are in the same direction
(d) in NH3 as well as in NF3 the atomic dipole 36. Which of the following pairs of compounds is
and bond dipole are in the same direction isoelectronic and isostructural ? [2017]
30. Wh ich of th e followin g woul d have a (a) TeI2,XeF2 (b) IBr2 , XeF2
permanent dipole moment? [2005]
(c) IF3, XeF2 (d) BeCl2,XeF2
(a) SiF4 (b) SF 4
37. The species, having bond angles of 120° is :-
(c) XeF 4 (d) BF3
[2017]
31. The correct order of the O–O bond length in O2,
(a) CIF3 (b) NCl3
H2O2 and O3 is [1995, 2005]
(c) BCl3 (d) PH3
(a) O2 O3 H 2O 2 38. Consider the molecules CH4, NH3 and H2O.
(b) O 3 H 2O 2 O2 Which of the given statements is false? [2016]
(a) The H–C–H bond angle in CH4, the H–N–H
(c) O 2 H 2O 2 O3 bond angle in NH3, and the H–O–H bond
(d) H 2 O 2 O 3 O 2 angle in H2O are all greater than 90°
(b) The H–O–H bond angle in H 2O is larger
32. H2O is dipolar, whereas BeF2 is not. It is because
than the H–C–H bond angle in CH4.
[2004]
(c) The H–O–H bond angle in H2O is smaller
(a) the electronegativity of F is greater than than the H–N–H bond angle in NH3.
that of O (d) The H–C–H bond angle in CH4 is larger
(b) H2O involves hydrogen bonding whereas than the H–N–H bond angle in NH3.
BeF2 is a discrete molecule 39. Predict the correct order among the following :
(c) H2O is linear and BeF2 is angular [2016]
(d) H2O is angular and BeF2 is linear (a) lone pair- lone pair > lone pair - bond pair
33. The dipole moments of diatomic molecules AB > bond pair - bond pair
and CD are 10.41D and 10.27 D, respectively (b) lone pair - lone pair > bond pair - bond
while their bond distances are 2.82 and 2.67 Å, pair > lone pair - bond pair
respectively. This indicates that [1999] (c) bond pair - bond pair > lone pair - bond
(a) bon ding is 100% ionic in both the pair > lone pair - lone pair
molecules (d) lone pair - bond pair > bond pair - bond
(b) AB has more ionic bond character than pair > lone pair - lone pair
CD 40. Which of the following pairs of ions are
isoelectronic and isostructural ? [2015]
(c) AB has lesser ionic bond character than
CD (a) ClO3– , CO32– (b) SO32– , NO 3–
(d) bonding is nearly covalent in both the
molecules
(c) ClO3– , SO32– (d) CO2– 2–
3 , SO3
ݸ»³·½¿´ Þ±²¼·²¹ ¿²¼ Ó±´»½«´¿® ͬ®«½¬«®» îç

41. Maximum bond angle at nitrogen is present in same hybridization of orbitals, NO2–, NO3– ,
which of the following ? [2015] – +
NH2 , NH4 , SCN ? – [2011]
(a) NO2– and NO3– (b) NO4+ and NO3–
(a) NO 2– (b) NO 2
(c) SCN– and NH2– (d) NO2– and NH2–
(c) NO3– (d) NO 2 51. Considering the state of hybridization of carbon
atoms, find out the molecule among the following
42. In which of the following pairs, both the species
which is linear ? [2011]
are not isostructural ? [2015 RS]
(a) CH3– CH = CH–CH3
(a) SiCl 4 , PClõ 4 (b) CH3 – C C – CH3
(b) diamond, silicon carbide (c) CH2 = CH – CH2 – C CH
(c) NH3, PH3 (d) CH3 – CH2 – CH2 – CH3
(d) XeF4, XeO4 52. In which of the following molecules the central
43. Be2+ is isoelectronic with which of the following atom does not have sp3 hybridization? [2010]
(a) NH +4 (b) CH4
ions? [2014]
(c) SF4 (d) BF4–
(a) H+ (b) Li+
53. Some of the properties of the two species, NO3
(c) Na+ (d) Mg2+
and H3O+ are described below. Which one of
44. Which one of the following species has plane
them is correct? [2010]
triangular shape ? [2014]
(a) Similar in hybridization for the central atom
(a) N3– (b) NO3– with different structures.
(c) NO2– (d) CO2 (b) Dissimilar in hybridization for the central
45. The pair of species that has the same bond order atom with different structures.
in the following is: [NEET Kar. 2013] (c) isostructural with same hybridization for
(a) O2, B2 (b) CO, NO+ the central atom.
(c) NO–, CN– (d) O2, N2 (d) Isostructural with different hybridization
46. In which of the following pair both the species for the central atom.
54. In which one of the following species the central
have sp3 hybridization? [NEET Kar. 2013]
atom has the type of hybridization which is not
(a) H2S, BF3 (b) SiF4, BeH2
the same as that present in the other three?
(c) NF3, H2O (d) NF3, BF3 (a) SF4 (b) I3– [2010]
47. XeF2 is isostructural with [NEET 2013] (c) SbCl5 2–
(d) PCl5
(a) ICl2– (b) SbCl3 55. In which of the following pairs of molecules/
(c) BaCl2 (d) TeF2 ions, the central atoms have sp2 hybridization?
48. Which of the following species contains three [2010]
bond pairs and one lone pair around the central (a) NO 2 and NH 3 (b) BF3 and NO 2
atom ? [2012] (c) NH 2 and H 2 O (d) BF3 and NH 2
(a) H2O (b) BF3 56. In which of the following molecules / ions

(c) NH2 (d) PCl3
BF3, NO 2 , NH 2 and H2O , [2009]
49. Which one of the following pairs is isostructural
(i.e., having the same shape and hybridization)? the central atom is sp2 hybridized ?
BCl3 and BrCl3 (a) NH 2 and H2O (b) NO2 and H2O
(a) [2012]
(c) BF3 and NO 2 (d) NO 2 and NH 2
(b) NH3 and NO3
57. In which of the following pairs, the two species
(c) NF3 and BF3 are isostructural? [2007]
(a) SO32– and NO3– (b) BF3 an NF3
(d) BF4 and NH 4 (c) BrO3– and XeO3 (d) SF4 and XeF4
50. Which of the two ions from the list given below 58. Which of the following is not isostructural with
that have the geometry that is explained by the SiCl4? [2006]
EBD_7324
íð ÝØÛÓ×ÍÌÎÇ
(a) SO42– (b) PO43– 69. Which one of the following has the pyramidal
(c) NH4+ (d) SCl4 shape? [1999]
59. Which of the following species has a linear shape ? (a) CO32– (b) SO3
(a) SO2 (b) NO2+ [2006] (c) BF3 (d) PF3
(c) O3 (d) NO2– 70. Which of the following molecules is planar?
60. In which of the following molecules all the bonds (a) SF4 (b) XeF4 [1998]
are not equal? [2006] (c) NF3 (d) SiF4
(a) BF3 (b) AlF3
71. The AsF5 molecule is trigonal bipyramidal. The
(c) NF3 (d) ClF3
hybrid orbitals used by the As atom for bonding
61. Which of the following molecules has trigonal
planar geometry? [2005] are [1997]
(a) BF3 (b) NH3 (a) d x 2 y2 , d 2z , s, p x , p y
(c) PCl3 (d) IF 3
62. In BrF 3 molecule, the lone pairs occupy (b) dxy, s, px, py, pz
equatorial positions to minimize [2004] (c) s, px, py, pz, dz2
(a) lone pair - bond pair repulsion only (d) d 2 2 , s, px, py, pz
x y
(b) bond pair - bond pair repulsion only 72. The cylindrical shape of an alkyne is due to the
(c) lone pair - lone pair repulsion and lone pair fact that it has [1997]
- bond pair repulsion (a) three sigma C – C bonds
(d) lone pair - lone pair repulsion only (b) two sigma C – C and one ' ' C – C bond
63. In an octahedral structure, the pair of d orbitals
(c) three ' ' C – C bonds
involved in d 2sp 3 hybridization is [2004] (d) one sigma C– C and two ' ' C – C bonds
(a) d x 2 y 2 , d z 2 (b) d xz, d x 2 y 2 73. The BCl3 is a planar molecule whereas NCl 3 is
pyramidal because [1995]
(c) d 2 d xz
z , (d) d xy, d yz (a) B-Cl bond is more polar than N-Cl bond
64. In a regular octahedral molecule, MX6 the (b) N-Cl bond is more covalent than B-Cl bond
number of X - M - X bonds at 180° is [2004] (c) nitrogen atom is smaller than boron atom
(a) three (b) two (d) BCl3 has no lone pair but NCl3 has a lone
(c) six (d) four pair of electrons
65. Which of the following has p – d bonding? 74. The distance between the two adjacent carbon
(a) NO3– (b) SO32– [2002] atoms is largest in [1994]
(c) BO33– (d) CO32– (a) benzene (b) ethene
66. Main axis of a diatomic molecule is z, molecular (c) butane (d) ethyne
orbital px and py overlap to form which of the 75. Among the following orbital bonds, the angle is
following orbital? [2001] minimum between [1994]
(a) - molecular orbital (a) sp3 bonds (b) px and py orbitals
(b) - molecular orbital (c) H – O – H in water (d) sp bonds.
(c) - molecular orbital 76. Which of the following does not have a
(d) No bond will be formed tetrahedral structure ? [1994]
67. Which of the following two are isostructural? (a) BH–4 (b) BH3
[2001] (c) NH 4 (d) H 2 O.
(a) NH3, BF3 (b) PCl5, ICl5 77. Which of the following statements is not correct ?
(c) XeF2, IF2– (d) CO3–2, SO3–2 [1993]
68. Among the following ions the p –d overlap (a) Double bond is shorter than a single bond
could be present in [2000] (b) Sigma bond is weaker than a (pi) bond
(a) (b) NO 3 (c) Double bond is stronger than a single bond
NO 2
(d) Covalent bond is stronger than hydrogen
(c) PO34 (d) CO 23 bond.
ݸ»³·½¿´ Þ±²¼·²¹ ¿²¼ Ó±´»½«´¿® ͬ®«½¬«®» íï

78. Which structure is linear ? [1992] 88. Which of the following species contains equal
(a) SO2 (b) CO2 number of - and -bonds : [2015]
(c) CO 32 (d) SO 24 (a) XeO4 (b) (CN)2
79. Which one of the following has the shortest (c) CH2(CN)2 (d) HCO3–
carbon carbon bond length ? [1992]
(a) Benzene (b) Ethene 89. Decreasing order of stability of O2, Oó õ
2 ,O 2 and
(c) Ethyne (d) Ethane O 22ó is : [2015 RS]
80. In compound X, all the bond angles are exactly

109°28; X is [1991] (a) Oõ ó
2 â O2 â O2 â O 2
(a) Chloromethane
(b) Carbon tetrachloride (b) O22ó â Oó õ
2 â O2 â O2
(c) Iodoform 2ó
(d) Chloroform.
(c) O2 â O õ ó
2 â O2 â O2
81. Which statement is NOT correct ? [1990] 2ó
(d) Oó õ
2 â O2 â O2 â O2
(a) A sigma bond is weaker than a -bond.
(b) A sigma bond is stronger than a -bond. 90. The hybridization in volved in complex
(c) A double bond is stronger than a single [Ni(CN)4]2–. is (At. No. Ni = 28) [2015 RS]
bond. (a) dsp 2 (b) sp 3
(d) A double bond is shorter than a single (c) d 2sp 2 (d) d 2sp 3
bond. 91. The outer orbitals of C in ethene molecule can
82. In which one of the following molecules the be considered to be hybridized to give three
central atom said to adopt sp2 hybridization? equivalent sp2 orbitals. The total number of sigma
(a) BeF2 (b) BF3 [1989] ( ) and pi ( ) bonds in ethene molecule is
(c) C2H2 (d) NH3 [NEET Kar. 2013]
83. Which of the following molecule does not have (a) 1 sigma ( ) and 2 pi ( ) bonds
a linear arrangement of atoms ? [1989]
(b) 3 sigma ( ) and 2 pi ( ) bonds
(a) H2S (b) C2H2
(c) 4 sigma ( ) and 1 pi ( ) bonds
(c) BeH2 (d) CO2
84. Equilateral shape has [1988] (d) 5 sigma ( ) and 1 pi ( ) bonds
(a) sp hybridisation (b) sp2 hybridisation 92. In which of the following ionisation processes
(c) sp3 hybridisaiton (d) sp 3 hybridisation the bond energy increases and the magnetic
85. The angle between the overlapping of one behaviour changes from paramagnetic to
s-orbital and one p-orbital is [1988] diamagnetic? [NEET Kar. 2013]
(a) 180° (b) 120° (a) N2 N2+ (b) O2 O2+
(c) 109 28' (d) 120°, 60° (c) C2 C2+ (d) NO NO+
̱°·½ ëæ Ê¿´»²½» Þ±²¼ ¿²¼ Ó±´»½«´¿® 93. Which of the following is paramagnetic ?
Ñ®¾·¬¿´ ̸»±®§ [NEET 2013]
(a) O 2 (b) CN–
86. The correct bond order in the following species
is: [2015] (c) NO+ (d) CO
94. Which one of the following molecules contains
(a) O 22 O2– O2 (b) O 2 O 2– O22 no bond? [NEET 2013]
(c) O 2– O 2 O 22 (d) O 22 O 2 O 2– (a) H2O (b) SO2
87. Which of the following options represents the (c) NO2 (d) CO2
correct bond order ? [2015] 95. Four diatomic species are listed below. Identify
the correct order in which the bond order is
(a) O 2– O2 O2 (b) O –2 O2 O2
increasing in them: [2008, 2012 M]
(c) O –2 O2 O2 (d) O –2 O2 O2
EBD_7324
íî ÝØÛÓ×ÍÌÎÇ
104. The angular shape of ozone molecule (O 3 )
(a) NO O2 C22 He2 consists of : [2008]
(b) O2 NO C22 He 2 (a) 1 sigma and 2 pi bonds
(b) 2 sigma and 2 pi bonds
(c) C22 He2 O2 NO (c) 1 sigma and 1 pi bonds
(d) 2 sigma and 1 pi bonds
(d) He2 O2 NO C22 105. The correct order of C–O bond length among
96. During change of O2 to O 2 ion, the electron CO, CO32 , CO2 is [2007]
adds on which one of the following orbitals ?
(a) CO < CO32– < CO2 (b) CO32– < CO2 < CO
[2012 M]
(c) CO < CO2 < CO32– (d) CO2 < CO32– < CO
(a) * orbital (b) orbital 106. The number of unpaired electrons in a
(c) * orbital (d) orbital
paramagnetic diatomic molecule of an element
97. The pair of species with the same bond order is : with atomic number 16 is [2006]
(a) O2– 2 , B2 (b) O+2 , NO+ [2012] (a) 3 (b) 4
(c) NO, CO (d) N2, O2 (c) 1 (d) 2
98. Bond order of 1.5 is shown by : [2012] 107. Among the following the pair in which the two
species are not isostructural is [2004]
(a) O 2 (b) O 2
(a) SiF4 and SF4 (b) IO3 and XeO 3
(c) O 22 (d) O2
99. The pairs of species of oxygen and their (c) BH 4 and NH 4 (d) PF6 and SF6
magnetic behaviours are noted below. Which of
108. Which of the following statements is not correct
the following presents the correct description ?
for sigma and pi-bonds formed between two
(a) O 2 ,O 22 – Both diamagnetic [2011 M] carbon atoms? [2003]
2
(b) O ,O2 – Both paramagnetic (a) Sigma-bond determines the direction
(c) O2 ,O2 – Both paramagnetic between carbon atoms but a pi-bond has
(d) O,O 22 – Both paramagnetic no primary effect in this regard
100. Which of the following has the minimum bond (b) Sigma-bond is stronger than a pi-bond
length ? [2011] (c) Bond energies of sigma- and pi-bonds are
(a) O2+ (b) O2– of the order of 264 kJ/mol and 347 kJ/mol,
(c) O22– (d) O2 respectively
101. Which one of the following species does not (d) Free rotation of atoms about a sigma-bond
exist under normal conditions? [2010] is allowed but not in case of a pi-bond
109. In NO3– ion number of bond pair and lone pair
(a) Be2 (b) Be2
of electrons on nitrogen atom respectively are
(c) B2 (d) Li 2 (a) 2, 2 (b) 3, 1 [2002]
102. According to MO theory which of the following (c) 1, 3 (d) 4, 0
lists ranks the nitrogen species in terms of 110. In which of the following the bond angle is
increasing bond order? [2009] maximum? [2001]
(a) N 2– N 2– N 2 (b) N 2 N 2– N 2– (a) NH3 (b) SCl2
2 2
(c) NH4+ (d) PCl3
(c) N 2– N 2– 2 N 2 (d) N 2– N 2 N 2– 2 111. The relationship between the dissociation
103. The correct order of increasing bond angles in energy of N2 and N2+ is : [2000]
the following triatomic species is : [2008] (a) Dissociation energy of N2+ > dissociation
(a) NO2 NO2 NO 2 energy of N2
(b) Dissociation energy of N2 = dissociation
(b) NO2 NO2 NO2 energy of N2+
(c) NO2 NO2 NO2 (c) Dissociation energy of N2 > dissociation
(d) NO2 NO2 NO energy of N2+
ݸ»³·½¿´ Þ±²¼·²¹ ¿²¼ Ó±´»½«´¿® ͬ®«½¬«®» íí
(d) Dissociation energy of N2 can either be
(c) NO 3 NO 2 N2O4 NO
lower or higher than the dissociation energy
of N2+ (d) NO N 2O4 NO 2 NO 3
112. Which one of the following arrangements rep-
resents the increasing bond orders of the given 117. The ground state electronic configuration of
species? [1999] valence shell electrons in nitrogen molecule (N2)
(a) NO+ < NO < NO– < O2– is written as KK 2s2 , * 2s 2 , 2p 2x , 2p 2y 2p 2z
(b) O2– < NO– < NO <NO+ Bond order in nitrogen molecule is
(c) NO– < O2– < NO < NO+ [1995]
(d) NO < NO+ < O2– < NO– (a) 0 (b) 1
113. Among the following which one is not (c) 2 (d) 3
paramagnetic? [Atomic numbers : Be = 4,
118. Which of the following species is paramagnetic?
Ne = 10, As = 33, Cl = 17] [1998]
(a) Cl– (b) Be+ (a) O 22 (b) NO [1995]
(c) Ne+2 (d) As + (c) CO (d) CN–
114. The number of anti-bonding electron pairs in 119. Mark the incorrect statement in the following
O 2 2 molecular ion on the basis of molecular [1994]
orbital theory is, (Atomic number of O is 8) (a) The bond order in the species O2, O2+ and
(a) 5 (b) 2 [1998] O2– decreases as O 2 O 2 O 2
(c) 3 (d) 4 (b) The bond energy in a diatomic molecule
115. N2 and O2 are converted into monoonions, N 2– always increases when an electron is lost
and O2– respectively. Which of the following (c) Electrons in antibonding M.O. contribute
statements is wrong ? [1997] to repulsion between two atoms.
(a) In N2, the N—N bond weakens (d) With increase in bond order, bond length
(b) In O2, the O—O bond order increases decreases and bond strength increases.
(c) In O2, bond length decreases 120. Linear combination of two hybridized orbitals
(d) N2– becomes diamagnetic belonging to two atoms and each having one
116. The correct order of N–O bond lengths in NO, electron leads to a [1990]
NO2–, NO3– and N2O4 is [1996] (a) Sigma bond
(b) Double bond
(a) N2O4 NO 2 NO3 NO
(c) Co-ordinate covalent bond
(b) NO NO 3 N 2O4 NO 2 (d) Pi bond.

ANSWER KEY
1 (b) 13 (d) 25 (d) 37 (c) 49 (d) 61 (a) 73 (d) 85 (a) 97 (a) 109 (d)
2 (b) 14 (d) 26 (d) 38 (b) 50 (a) 62 (c) 74 (c) 86 (c) 98 (b) 110 (c)
3 (c) 15 (d) 27 (c) 39 (a) 51 (b) 63 (a) 75 (b) 87 (a) 99 (c) 111 (c)
4 (b) 16 (b) 28 (a) 40 (c) 52 (a) 64 (a) 76 (b) 88 (a) 100 (a) 112 (b)
5 (a) 17 (a) 29 (a) 41 (b) 53 (b) 65 (b) 77 (b) 89 (a) 101 (b) 113 (a)
6 (a) 18 (a) 30 (b) 42 (d) 54 (c) 66 (a) 78 (b) 90 (a) 102 (a) 114 (d)
7 (c) 19 (d) 31 (d) 43 (b) 55 (b) 67 (c) 79 (c) 91 (d) 103 (b) 115 (b)
8 (b) 20 (c) 32 (d) 44 (b) 56 (c) 68 (c) 80 (b) 92 (d) 104 (d) 116 (c)
9 (d) 21 (a) 33 (c) 45 (b) 57 (c) 69 (d) 81 (a) 93 (a) 105 (c) 117 (d)
10 (a) 22 (c) 34 (c) 46 (c) 58 (d) 70 (b) 82 (b) 94 (a) 106 (d) 118 (b)
11 (b) 23 (b) 35 (b) 47 (a) 59 (b) 71 (c) 83 (a) 95 (d) 107 (a) 119 (b)
12 (c) 24 (c) 36 (b) 48 (d) 60 (d) 72 (d) 84 (b) 96 (a) 108 (c) 120 (a)
EBD_7324
íì ÝØÛÓ×ÍÌÎÇ

Hints & Solutions


1. (b) CN– and CO have same no. of electrons and 9. (d) H-bond is the weakest.
have same bond order equal to 3. 10. (a) According to Fajan rules, ionic character
2. (b) (BH3)2 or (B2H6) increases with increase in size of the cation and
H H decrease in size of the anion. Thus, CsF has
H higher ionic character than NaCl and hence bond
B B
strength of CsF is stronger than NaCl.
H H H
11. (b) For compounds containing ions of same
It contains two 3 centre - 2 electron bonds and charge, lattice energy increases as the size the
present above and below the plane of mol- ions decrease. Thus, NaF has highest lattice
ecules compounds which do not have energy. The size of cation is in the order Na + <
sufficient number of electrons to form normal
K+ < Rb+ < Cs+.
covalent bonds are called electron deficient
molecules. 12. (c) As we move in period from Li Be
3. (c) As difference of electronegativity increases B C, the electronegativity (EN) increases and
% ionic character increases and covalent hence the EN difference between the element
character decreases i.e., electronegativity and Cl decreases and accordingly the covalent
difference decreases covalent character increases. character increases.Thus LiCl < BeCl2 < BCl3<
Further greater the charge on the cation more CCl4 is correct.
will be its covalent character. Be has maximum 13. (d) In metallic bonds each ion is surrounded
(+2) charge. by equal no. of oppositely charged ions hence
4. (b) The compound, of which central atom is have electrostatic attraction on all sides and
octetless known as electron deficient compound. hence do not have directional characteristics.
Hence B2H6 is electron deficient compound.
5. (a) We know that the electrostatic force that 14. (d) Formal charges help in the selection of the
binds the oppositely charged ions which are lowest energy structure from a number of
formed by transfer of electron from one atom to possible Lewis structures for a given compound.
another is called ionic bond. We also know that The lowest energy structure means the structure
cation and anion are oppositely charged particles with the smallest formal charge on each atom of
therefore they form ionic bond in crystal. the compound. A Lewis dot structure is
preferable when all formal charges are zero.
6. (a) H H H 15. (d) Due to intermolecular hydrogen bonding
in methanol, it exist as assosiated molecule.
B B 16. (b) Statement (a), (c), (d) are correct. Statement
(b) is incorrect statement.
H H H AB5 may have two structures as follows :
The bond represented by dots form the 3-centred B
electron pair bond. The idea of three centred
electron pair bond B–H–B bridges is necessary B B
because diborane does not have sufficient A
electrons to form normal covalent bonds. It has B B
A
only 12 electrons instead of 14 that are required B
to give simple ethane like structure. B
7. (c) Chemical bonds. B B
B
8. (b) The stability of the ionic bond depends upon
Square Pyramidal Trigonal Bipyramidal
the lattice energy which is expected to be more
between Mg and F due to +2 charge on Mg atom.
ݸ»³·½¿´ Þ±²¼·²¹ ¿²¼ Ó±´»½«´¿® ͬ®«½¬«®» íë

17. (a) In X — H - - - Y, X and Y both are 27. (c) Dipole moment of NH3 > NF3
electronegative elements (i.e attracts the +
–..
electron pair) then electron density on X will N N
increase and on H will decrease.
18. (a) Boron in BCl3 has 6 electrons in outer most –F F–
+H H ( ) F
shell. Hence BCl3 is an electron pair deficient H ( –)
( )

compound. = 1.4D = 0.23D


19. (d) F—H-----F—H-----F—H-----F
(F is more electronegative than N)
HF form linear polymer structure due to
hydrogen bonding. 28. (a) SF 4 has 4 bond pairs and 1 lone pair
20. (c) Bond order between P – O of electrons, sp3 d hybridisation leads to
no. of bonds in all possible direction 5 F
1.25 F
total no. of resonating structures 4 irregular shape S and resultant 0.
F
F
O O–
29. (a) In NH3 the atomic dipole and bond dipole

O P O– O P O– are in the same direction whereas in NF3 these
O– O– are in opposite direction so in the former case
O– O– they are added up whereas in the latter case net

result is reduction of dipole moment. It has
O P O– –
O P O been shown in the following figure :
O O–
.. ..
N N
3
or Formal charge on oxygen = 0.75
4 F
21. (a) We know that due to polar nature, water H
molecules are held together by intermolecular H F
hydrogen bonds. The structure of ice is open H F
with large number of vacant spaces, therefore 30. (b) SF4 has permanent dipole moment.
the density of ice is less than water. SF4 has sp3d hybridization and see saw shape
22. (c) The b.p. of p-nitrophenol is higher than (irregular geometry).
that of o-nitrophenol because in p-nitrophenol F F
there is intermolecular H-bonding but in o-
nitrophenol it is intramolecular H-bonding. Si
23. (b) The strength of the interactions follows the
order vander Waal’s < hydrogen – bonding < F F
dipole-dipole < covalent. =0
24. (c) H – F shows strongest H-bonds. (see Ans.9) Whereas XeF4 shows squre planar geometry
SiF4 has tetrahedral shape and BF3 has Trigonal
O planar shape. All these are symmetric molecules.
||
Hence 0.
25. (d) H C C* O H
31. (d) The bond length of O – O in O2 is 1.21 Å,
The star marked carbon has a valency of 5 and in H2O2 it is 1.48 Å and in O3 it is 1.28 Å.
hence this formula is not correct. correct order of bond length is H2O2 > O3 > O2.
26. (d) With the increase of electronegativity and 32. (d) In a linear symmetrical molecule like BeF2,
decrease in size of the atom to which hydrogen is the bond angle between three atoms is 180°,
covalently linked, the strength of hydrogen bond hence the polarity due to one bond is cancelled
increases. As F is most electronegative thus HF by the equal polarity due to other bond. Also it
shows maximum strength of hydrogen bond. is not angular H2O.
EBD_7324
íê ÝØÛÓ×ÍÌÎÇ
33. (c) As dipole moment = electric charge × bond Note: The geometry of H2O should have been
length tetrahedral if there are all bond pairs. But due
D. M. of AB molecules to presence of two lone pairs the shape is
10 8 distorted tetrahedral. Hence bond angle reduced
4.8 10 2 .82 10 13 . 53D
D.M. of CD molecules to 104.5° from 109.5°.
39. (a) According to VSEPR theory order of
4.8 10 10 2.67 10 10 12.81D repulsion in between lp – lp, lp – bp and bp –
now % ionic character bp is as under
Actual dipole moment of the bond lp – lp > lp – bp > bp – bp
=
Dipole moment of pure ionic compound 40. (c) ClO3– and SO3–2 both have same number
% ionic character in of electrons (42) and central atom in each
10.41 being sp3 hybridised. Both are having one lone
AB 100 76.94%
13.53 pair on central atom hence they are pyramidal.
% ionic character in 41. (b) NO2+ has sp hybridisation so it is linear
10.27 with bond angle = 180°.
CD 100 80.23%
12.81 42. (d) XeF4, XeO4
34. (c) Polarity of the bond depends upon the O
electronegativity difference of the two atoms
F F Xe
forming the bond. Greater the electronegativity Xe
difference, more is the polarity of the bond. O O
N – Cl O–F N–F N–N F F O
3.0–3.0 3.5–4.0 3.0–4.0 3.0–3.0 (Square planar) [Tetrahedral]
As the electronegativity difference between N 43. (b) Be2+ = (4 – 2) = 2
and F is maximum hence this bond is most
is isoelectronic with Li+ (3 – 1 = 2)
polar.
35. (b) BeF2 is linear and hence has zero dipole Since both have same number of electrons in
moment while H2O, being a bent molecule, has their outermost shell.
a finite or non-zero dipole moment.
1
44. (b) Hybridization of NO3– = (5 0 1 0)
36. (b) IBr2 , XeF2 2
Total number of valence electrons are equal in 6
= 3 sp 2 hence geometry is trigonal
both the species and both the species exhibit 2
linear shape. planner.
37. (c) BCl3 is trigonal planar and hence the bond NO2– (nitrite ion) also has sp2 hybridization
angle is 120°. and gives a trigonal planner geometry but
because there are only two outer atoms, the
Cl molecular geometry is bent with 120º bond
angles.
120°
B 45. (b) No. of electrons in CO = 6 + 8 = 14
Cl Cl No. of electrons in NO+ = 7 + 8 – 1 = 14
38. (b) CH4 NH3 H2O CO and NO+ are isoelectronic species.
Isoelectronic species have identical bond order.
H 46. (c) Applying VSEPR theory, both NF3 and H2O
C 109°28' N are sp3 hybridized.
O
H H H 107°H H 47. (a) sp3d and Linear
H 104.5° H F Xe F
H

Tetrahedral; Trigonal Bent CI I CI sp3d and Linear
pyramidal
ݸ»³·½¿´ Þ±²¼·²¹ ¿²¼ Ó±´»½«´¿® ͬ®«½¬«®» íé

48. PCl3 56. (c) For neutral molecules:


No. of electron pairs = No. of atoms bonded
P
Cl 1
Cl Cl to it + [Group number of central atom –
2
49. (d) BF4 hybridisation sp3, tetrahedral structure. Valency of the central atom].
NH 4 hybridisation sp3, tetrahedral structure. For ions :
No. of electron pairs = No. of atoms bonded
1 1
50. (a) Hybridisation = [No. of valence to it + [Group no. of central atom –
2 2
electrons of central atom + no. of monovalent valency of the central atom ± no. of electrons]
atoms attached to it + On calcuting no. of electron pairs in given
Negative charge if any – positive charge if any] molecules. We find that in the given molecules
hybridisation is
1
NO2– , H [5 0 1 0] 3 sp 2 BF3 sp2
2
NO2 sp2
1
NO3– , H [5 0 1 0] 3 sp 2 NH 2 sp3
2
H2O sp 3
1 57. (c) Hybridization of Br in BrO3–
NH2– , H [5 2 1 0] 4 sp3
2 1
H= 7 0 0 1 4
1 2
NH4+, H [5 4 0 1] 4 sp3
2 hybridization = sp3
SCN– = sp Hybridization of Xe in XeO3.
NO2– and NO3– have same hybridisation. 1
H 8 0 0 0 4
sp 3 sp sp sp 3 2
51. (b) H3C — C C — CH3 Hybridization = sp3
linear In both cases, the structure is trigonal
52. (a) NH +4 : sp3 hybridisation pyramidal.
58. (d) In SiCl4 there is sp3 hybridisation so the
(b) CH4: sp3 hybridisation
structure is tetrahedral. In SO42–, PO43–, NH4+
(c) SF4: sp3 d hybridisation
the structure is tetrah edral with sp 3
(d) BF4– : sp3 hybridisation hybridisation. But in SCl4, sp3d hybridisation
53. (b) In NO 3 , nitrogen is in sp2 hybridisation, is present so its shape is different i.e., see saw.
59. (b) NO2+ will have linear shape as it will have
thus planar in shape. In H3 O , oxygen is in sp3 sp hybridisation.
hybridisation, thus tetrahedral in shape. 60. (d) In BF3, AlF3 & NF3 all fluoride atoms are
5+5+2 symmetrically oriented with respect to central
54. (c) SbCl52– : = 6 means sp 3 d 2 metal atom but in ClF3 three fluorine atoms are
2 arranged as follows :
hybridisation F
I3– , SF4 , and PCl5 ; all have sp3d hybridization.

3 3 Cl F
55. (b) BF3 : 3 means sp2 hybridisation
2
5 1 F
NO2 : 3 means sp2 hybridisation Here two bonds are in equitorial plane & one
2
bond is in axial plane.
EBD_7324
íè ÝØÛÓ×ÍÌÎÇ
61. (a) BF3 is sp2
hybridised. So, it is trigonal In 'S' unhybride d- orbital is present, which will
planar. NH3, PCl3 has sp3 hybridisation hence involved in bond formation with oxygen
has trigonal bipyramidal shape, IF3, has sp3d atom.
hydridization and is T-shaped. 2
F 8 O ã 1s , 2s2 2p2x 2p1y 2p1z
In oxygen two unpaired p- orbital is present in
these one is involved in bond formation
62. (c) Br F while other is used in bond formation
Thus in SO 32 , p and d orbitals are involved

F for p d bonding.
In BrF3, both bond pairs as well as lone pairs 66. (a) For -overlap the lobes of the atomic
of electrons are present. Due to the presence orbitals are perpendicular to the line joining
of lone pairs of electrons (lp) in the valence the nuclei.
shell, the bond angle is contracted and the
molecule takes the T-shape. This is due to 67. (c) In XeF2 and IF2 . Both XeF2 and IF2– are
greater repulsion between two lone pairs or sp3d hybridized and have planar shape.
between a lone pair and a bond pair than 68. (c) In P–O bond, bond is formed by the
between the two bond pairs. sidewise overlapping of d-orbital of P and p-
63. (a) Only those d orbitals whose lobes are orbital of oxygen. Hence it is formed by p
directed along X, Y and Z directions hybridise and d overlapping.
with s and p orbitals. In other three d orbitals
O
namely dxy , d yz and d xz , the lobes are at an
angle of 45° from both axis, hence the extent P
– –
of their overlap with s and p orbitals is much O – O
O
lesser than d 2 2 and d 2 orbitals.
x y z 69. (d) PF3 has pyramidal shape
64. (a) X5 Phosphorus exist in sp 3 hybridiation state
X1 hence it exist in tetrahedral shape. But due to
X4
presence of lone pair its shape is pyramidal.
1
M 70. (b) XeF4 hybridisation is (V X C A)
2
hence V = 8 (no. of valence e– )
X = 4 (no of monovalent atom)
X3 X2 1 (8 4 0 0) 6, 3 2
X6 2
Thus here bond angles between C = 0 charge on cation
X 4 M X 2 180 A = 0 (charge on anion).
X1 M X 3 180 F F
X 5 M X 6 180 Xe
The shape is Square planar shape.
65. (b) In SO3 2 F F
– 71. (c) The electronic configuration of As is
– O
2 2 6 1 1 1 1 1
O =S : – As = 1s , 2s , 2p , 3s 3p x 3p y 3p z 3d
O ïìììîìììí
S is sp3 hybridised, so q sp 3d hybridisation
So, the hybrid orbitals used by As atom in AsF5
16 S 1s 2 , 2s 2 2 p 6 , 3s 2 3p1x 3p1y 3p1z 3d1xy
ïììîììí ¥ molecule are s, px, py, pz, dz2.
sp 3hybridisation unhybride
ݸ»³·½¿´ Þ±²¼·²¹ ¿²¼ Ó±´»½«´¿® ͬ®«½¬«®» íç

72. (d) In alkynes the hybridisation is sp i.e each s–s, p–p or s–p orbitals and hybridised orbitals
carbon atom undergoes sp hybridisation to (sp, sp2, sp3, sp3d and sp3d2) hence bonds are
form two sp-hybrid orbitals. The two 2p-orbitals strong bonds where as Pi ( )-bonds are formed
remain unhybridised. Hybrid orbitals form one by side ways overlap of unhybridised p- and d-
sigma and two unhybridised orbitals form - orbitals hence bonds are weak bonds.
bonds. 82. (b) BF3 involves sp2-hybridization.
. . . . 83. (a) For linear arrangement of atoms the
hybridisation should be sp(linear shape, 180°
C . . C angle). Only H2 S has sp3-hybridization and
hence has angular shape while C2H2, BeH2 and
. . . .
CO2 all involve sp - hybridization and hence
have linear arrangement of atoms.
Hence two bond and one sigma bond between
84. (b) Equilateral or triangular planar shape
C — C lead to cylindrical shape.
73. (d) As there is no lone pair on boron in BCl 3 involves sp2 hybridization.
therefore no repulsion takes place. But there
is a lone pair on nitrogen in NCl3. Therefore 85. (a)
repulsion takes place. Thus BCl 3 is planar s-orbital p-orbital
molecule but NCl3 is a pyramidal molecule. The overlap between s- and p-orbitals occurs along
74. (c) The C–C bond distance decreases as the internuclear axis and hence the angle is 180°.
multiplicity of the bond increases. Thus, bond
86. (c) O 2+ ion - Total number of electrons
distance decreases in the order: butane (1.54
Å) > benzene (1.39 Å) > ethene (1.34) Å > (16 – 1) = 15.
ethyne (1.20 Å). Thus in butane, C – C bond Electronic configuration
distance is the largest.
1s2 1s2 2s 2 2s 2 2p 2x
75. (b) The angle between the bonds formed by
px and py orbitals is the minimum i.e. 90°. 2p 2y 2p 2z 2p1y
76. (b) BH3 has sp2 hybridization and hence does
not have tetrahedral structure while all others Nb Na 10 5 5 1
Bond order = 2
have tetrahedral structures. 2 2 2 2
77. (b) Sigma bond is stronger than -bond. The O 2– (Super oxide ion): Total number of electrons
electrons in the bond are loosely held. The
(16 1) 17 .
bond is easily broken and is more reactive than
-bond. Energy released during sigma bond Electronic configuration
formation is always more than bond because
of greater extent of overlapping. 1s2 1s2 2s 2 2s2 2p 2x
78. (b) CO 2 has sp-hybridization and is linear.. 2p 2y 2pz2 2p 2y 2p1z

SO2 and CO 32 are planar (sp2) while SO 24 is (Nb Na ) 10 7 3 1


Bond order 1
tetrahedral (sp3). 2 2 2 2
79. (c) The bond length decreases in the order O 22 ion: Total number of electrons
sp3 > sp2 > sp.
Because of the triple bond, the carbon-carbon = (16 – 2) = 14 Electronic configuration
bond distance in ethyne is shortest. 1s2 < *1s2 < 2s2 < *2s2 < 2px2 < 2py2
80. (b) CCl4 has sp3 hybridisation, tetrahedral = 2pz2
geometry and all bond angles of 109° 28
(N b – Na ) 10 – 4 6
81. (a) A bond is stronger than a -bond Bond order = 3
hence option (a) is not correct. Sigma ( ) bonds 2 2 2
are formed by head on overlap of unhybridised So bond order: O2– < O2+ < O22+
EBD_7324
ìð ÝØÛÓ×ÍÌÎÇ
87. (a) Oxygen molecule (O2) – Total number of
1
electrons = 16 and electronic configuration is Bond order for O22 (10 8) 1.0
2
1s2 1s2 2s2 2s 2 2p2x hence the correct order is

2p 2y 2p 2z 2p1y 2p1z O2 O2 O 2– O 2–
2

Nb Na 10 6 4
Bond order = 2 90. (a) Ni2 = [Ar]18 4s0 3d8
2 2 2
Valence bond theory can be used to predict
O+2 ion - Total number of electrons (16 – 1) = shape.
15. Electronic configuration
1s2 1s2 2s2 2s 2 2p2x 3d 4s 4p
2
2p2y 2p 2z 2p1y dsp hybridization
(In presence of ligand, pairing of electron occurs)
Nb Na 10 5 5 1
Bond order = 2 Square planar.
2 2 2 2
91. (d) H— C = C — H
O2– (Super oxide ion) Total n umber of | |
H H
electrons (16 1) 17 . Electr onic
configuration 92. (d) (a) N2 N2
1s 2
1s2
2s 2
2s 2
2p2x B.O. 3 2.5
Bond energy decreases
2p 2y 2p z2 2p 2y 2p1z Magnetic behaviour changes from diamagnetic
(Nb Na ) 10 7 3 1 to paramagnetic
Bond order 1
2 2 2 2 O2 O2
O (b)
B.O. 2 2.5
Bond energy increases
88. (a) Xe
Magnetic behaviour does not change.
O O O C2 C2
Number of bonds = 4 (c)
B.O. 2 2.5
Number of bonds = 4 Bond energy decreases
89. (a) According to molecular orbital theory as Magnetic behaviour changes from diamagnetic
bond order decreases stability of the molecule to paramagnetic
decreases NO NO
(d)
1 B.O. 2 2.5
Bond order (N – Na) bond energy increases
2 b
Magnetic behaviour changes from paramagnetic
1 to diamagnetic
Bond order for O2 (10 5) 2.5
2 93. (a) Molecular orbital configuration of O 2 is
1
Bond order for O 2 (10 6) 2 O 2 (17) = 1s2, *1s2, 2s2, *2s2,
2
2pz2, 2px2 = 2py2, *2px2 = *2py1
1
Bond order for O 2 (10 7) 1.5
2
ݸ»³·½¿´ Þ±²¼·²¹ ¿²¼ Ó±´»½«´¿® ͬ®«½¬«®» ìï

O 1
94. (a) Bond order 10 8 1
H H 2
98. (b) (O2) = 1s 2 1s2 2 s2 2s 2 2 p z2 2 p x2
O S O O N O O C O 2 p 2y 2 p1x 2 p1y
Nb N a 10 6 4
95. (d) Calculating the bond order of various Bond order = 2
species. 2 2 2
O 2 ion 1s 2 1s 2 2s 2 2 s 2
O 2 : kk 2s 2 2s 2 2p z2
2 pz2 2 px2 2 p 2y 2 p1x
2p 2x 2p2y 2p 2x 2p1y
Nb Na 10 5 5 1
Number of electrons in bonding Bond order = 2
2 2 2 2
Number of electrons in non-bonding
B.O. O2 1s 2 1s 2 2 s2 2 s2 2 Pz2
2
8 5
= or 1.5 2 px2 2 p y2 2 px2 2 p1y
2
( Nb Na ) 10 7 3 1
NO : k k 2s 2 2s2 2p 2x 2p 2y 2p 2z 2p1x Bond order 1
2 2 2 2
Nb Na 8 3 2 2
B.O. = or 2.5 O22 1s 2 1s2 2 s2 2 s 2 2 pz 2 px
2 2
C 22 : kk 2s 2 2s 2 2p 2x 2p 2y 2p 2z 2 p 2y 2 px2 2 p y2

Nb Na 8 3 Nb
N a 10 8 2
B.O. = or 3 Bond order 1
2 2 2 2 2
99. (c) MOT configurations of O 2 and O2+ :
He 2 1s 2 1s1
O2+ : ( 1s)2 ( *1s)2 ( 2s)2 ( *2s)2 ( 2pz)2
NbNa 2 1 ( 2p2x = 2p2y) ( *2px1 = *2p1y)
B. O. = or 0.5
2 2 Number of unpair ed electr ons = 2, so
From these values we find the correct order of paramagnetic.
increasing bond order is O2 : ( 1s)2 ( *1s)2 ( 2s)2 ( *2s)2 ( 2pz)2
He22 O2 NO C 22 ( 2p2x = 2p2y) ( *2px1 = *2p0y)
96. (a) O2 = KK ( 2s)2 ( * 2s)2 ( 2pz)2 Number of unpair ed electr ons = 1, so
( 2px)2 ( 2py)2 ( * 2px)1 ( * 2py)1 paramagnetic.

O2 = KK ( 2s)2 ( * 2s)2 ( 2pz)2 100. (a) O2 (16) : KK ( 2s)2 ( 2s)2 ( 2pz)2 ( 2px)2
( 2py)2, B.O.: 2
( 2px)2 ( 2py)2 ( * 2px)2 ( * 2py)1
O2+ (15) : Remove one electron from * 2 p y from
97. (a) Both O2–
2 and B2 has bond order equal to 1.
O2 , B.O.: 2.5
B2 10 [ 1s2 *1s2 2 s 2 * 2 s 2 2 p1 y 2 pz1 ]
O2– (17) : KK ( 2s)2 ( 2s)2 ( 2pz)2 ( 2px)2
N Na 6 4 2
Bondorder = b = 1 ( 2py)2 , B.O.: 1.5
2 2 2
B2 is known to be in the gas phase. O22– (18) : KK ( 2s)2 ( 2s)2 ( 2pz)2 ( 2px)2
( 2py)2, B.O.: 1
O2 2 1s 2 *1s2 2s2 *2s2 2p z 2
Since, the bond length decreases as the bond order
2 px 2 2 p y 2 *
2 p x2 *
2 py 2 increases, hence, O2+ have least bond length.
EBD_7324
ìî ÝØÛÓ×ÍÌÎÇ
– –
101. (b) Bond order of Be2 = 0, hence Be2 cannot O O O
exist. (ii) C C C
102. (a) Molecular orbital configuration of – – – –
O O O O O O
N 22– 1s 2
*1s 2
2s 2
* 2s – 2
..
2p 2y *2p1y (iii) : O C O : :O C O :
2p 2x .. .. ..
2p 2z * 2p1z ..
10 – 6 :O C O:
Bond order = 2 ..
2
N 2– 1s 2 *1s2 2s2 * 2s 2 More is the single bond character. More will
2p 2y * 2p1y be the bond length. Hence, the corret order is :
2p 2x CO < CO2 < CO32–
2p 2z * 2p0z 106. (d) Electronic configuration of the molecule
10 5 according to molecular orbital theory, is
Bond order = 2.5
2 1s2 *1s2 2s2 *2s2 2pz2 ( 2px2 = 2py2)
2p2y
N2 1s 2 *1s 2 2s 2 * 2s 2 , 2p 2x ( *2px2 = 2py2) *2pz2 3s2 *3s2 3pz2
2p2z
( 3px2 = 3py2) ( *3px1 = 3py1)
10 – 4
Bond order = 3 Last two electrons are unpaired. So no. of
2 unpaired electron is 2.
The correct order is = N 2–
2 N 2– N 2 107. (a) SiF4 has symmetrical tetrahedral shape which
103. (b) From the structure of three species we can is due to sp3 hybridisation of silicon atom in its
determine the number of lone pair electrons excited state while SF4 has distorted tetrahedral
on central atom (i.e. N atom) and thus the bond or sea-saw geometry which arises due to sp3d
angle. hybridisation of sulphur atom and one lone pair
N N + of e–s in one of the equatorial hybrid orbital.
O..
..O.. .. O N=O 108. (c) As sigma bond is stronger than the (pi)
O O NO+2
NO 2 NO 2 bond, so it must be having higher bond energy
We know that higher the number of lone pair than (pi) bond.
of electron on central atom, greater is the lp – lp
109. (d) N :
repulsion between Nitrogen and oxygen atoms.
Thus smaller is bond angle. 1s 2 2s 2 2p3
The correct order of bond angle is To form NO 3 , nitrogen uses one p-electron
NO 2 NO2 NO 2 for -bond formation and two p-electrons for
104. (d) The shape of ozone molecule is -bond formation. 2s electrons are used for
+ + coordinate bond formation. Thus there is no
O O lone pair on nitrogen and four bond pairs are
present.
– O O O– O
O
In it we find 2 and 1 bond, i.e., option (d) is
correct.
105. (c) All these structures exhibits resonance N
and can be represented by the following
resonating structures. O O
..
110. (c) We know that bond angles of NH3 = 107º,
(i) :C O: :C O: NH4+ = 109.5º, PCl3 < 109º. Therefore bond
angle of NH4+ is maximum.
ݸ»³·½¿´ Þ±²¼·²¹ ¿²¼ Ó±´»½«´¿® ͬ®«½¬«®» ìí

111. (c) Dissociation energy of any molecules


114. (d) Total no. of electrons in O 2 2 = 16 + 2 = 18
depends upon bond order. Bond order in N2
Distribution of electrons in molecular orbital
molecule is 3 while bond order in N 2 is 2.5.
1s 2 , * 1s 2 , 2s 2 , * 2s 2 , 2p 2x , 2 p 2y
Further we know that more the Bond order,
more is the stability and more is the BDE. 2p 2z , * 2p x2 * 2p2y
õ 2 2 2 2 2
112. (b) NO ã ­1s ­ *1s ­2s ­ * 2s ­2px Anti bonding electron = 8 (4 pairs)
° 2p y 2 ã ° 2p z2 115. (b) We know that in O2 bond, the order is 2
and in O2– bond, the order is 1.5. Therefore
1 the wrong statements is (b).
Bond order of NO+ = (N b Na )
2 116. (c) The N–O bond length decreases in the order
1 1 O
ã (10 ó 4) = 6 3
2 2
1
O N > O—
—N
Similarly, Bond order of NO = (10 ó 5) O O
2
O
1
= (5) 2.5 — N—N —
> O— — O > N—
— O
2
1 1 O
Bond order of NO– ã (10 ó 6) = ( 4) 2 117. (d) In this configuration, there are four
2 2 completely filled bonding molecular orbitals
1 1 an d one completely filled antibonding
Bond order of O 2 ã (10 ó 7) = (3) 1.5
2 2 molecular orbital. So that Nb = 8 and N a 2.
By above calculation, we get 1 1
Decreasing bond order Bond order = ( N b Na ) (8 2 ) 3.
2 2
NO+ > NO > NO– > O2 118. (b) Paramagnetism is caused by the presence
113. (a) Paramagnetic character is based upon of atoms, ions or molecules with unpaired
presence of unpaired electron electrons. In NO the presence of unpaired
electron is clear. Therefore it is paramagnetic.
Cl 1s2 , 2s 2 2p6 ,3s2 3p 2x 3p 2y 3p 2z
119. (b) The removal of an electron from a diatomic
molecule may increase the bond order as in the
Be 1s2 , 2s1
conversion O2 (2) O 2 ( 2.5) or decrease
Ne 2 1s 2 , 2s 2 2p x2 2p1y 2p1z the bond order as in the conversion, N 2 (3.0)
N 2 (2.5), As a result, the bond energy
As 1s2 , 2s 2 2p6 , 3s 2 3p6 3d10 ,
may increase or decrease. thus, statement (b) is
4s 2 4p1x 4p1y 4p z0 incorrect.
Hence only Cl– do not have unpaired electrons. 120. (a) Linear combination of two hybridized
orbitals leads to the formation of sigma bond.
EBD_7324
ìì ÝØÛÓ×ÍÌÎÇ

ݸ¿°¬»®

5 States of Matter

̱°·½ ïæ Ù¿­ ´¿©­ ¿²¼ ×¼»¿´ ¹¿­ Û¯«¿¬·±² (a) 50.00 u (b) 12.25 u
(c) 6.50 u (d) 25.00 u
1. Equal moles of hydrogen and oxygen gases are 7. A bubble of air is underwater at temperature 15°C
placed in a container with a pin-hole through and the pressure 1.5 bar. If the bubble rises to
which both can escape. What fraction of the the surface where the temperature is 25°C and
oxygen escapes in the time required for one-half the pressure is 1.0 bar, what will happen to the
of the hydrogen to escape ? [2016] volume of the bubble ? [2011M]
(a) 1/8 (b) 1/4 (a) Volume will become greater by a factor of 1.6.
(c) 3/8 (d) 1/2 (b) Volume will become greater by a factor of 1.1.
(c ) Volume will become smaller by a factor of 0.70.
2. Equal masses of H2,O2 and methane have been
(d) Volume will become greater by a factor of 2.5.
taken in a container of volume V at temperature
8. From a heated mixture of nitrogen, oxygen and
27°C in identical conditions. The ratio of the carbon, two compounds (out of the many
volumes of gases H2 : O2 : methane would be : obtained) are isolated. The rates of diffusion of
(a) 8 : 16 : 1 (b) 16 : 8 : 1 [2014] the two isolated compounds are almost identical.
(c) 16 : 1 : 2 (d) 8 : 1 : 2 The two compounds are [1999]
3. Dipole-induced dipole interactions are present (a) N2O and CO2 (b) CO and NO
in which of the following pairs : [NEET 2013] (c) CO2 and NO2 (d) N2O and CO
9. If 500 ml of gas A at 400 torr and 666.6 ml of B at
(a) Cl2 and CCl4 600 torr are placed in a 3 litre flask, the pressure
(b) HCl and He atoms of the system will be [1999]
(c) SiF4 and He atoms (a) 200 torr (b) 100 torr
(d) H2O and alcohol (c) 550 torr (d) 366 torr
4. 50 mL of each gas A and of gas B takes 150 and 10. A gaseous mixture contains H2 and O2 in the
200 seconds respectively for effusing through a molar ratio 8 : 1. The ratio of H2 : O2 by weight in
pin hole under the similar condition. If molecular this mixture would be [1999]
mass of gas B is 36, the molecular mass of gas A (a) 4 : 1 (b) 1 : 8
will be : [2012] (c) 8 : 1 (d) 1 : 2
(a) 96 (b) 128 11. 500 ml of nitrogen at 27°C is cooled to –5°C at
(c) 32 (d) 64 the same pressure. The new volume becomes
5. A certain gas takes three times as long to effuse [1995]
out as helium. Its molecular mass will be : (a) 326.32 ml (b) 446.66 ml
(a) 27 u (b) 36 u [2012 M] (c) 546.32 ml (d) 771.56 ml
(c) 64 u (d) 9 u 12. 600 c.c. of a gas at a pressure of 750 mm is
6. Two gases A and B having the same volume compressed to 500 c.c. Taking the temperature
diffuse through a porous partition in 20 and 10 to remain constant, the increase in pressure, is
seconds respectively. The molecular mass of A [1995]
is 49 u. Molecular mass of B will be : [2011] (a) 150 mm (b) 250 mm
(c) 350 mm (d) 450 mm
ͬ¿¬»­ ±º Ó¿¬¬»® ìë

13. The correct value of the gas constant ‘R’ is close


(a) 927 / 27 time the earlier value
to : [1992]
(a) 0.082 litre-atmosphere K (b) same as before
(c) halved
(b) 0.082 litre-atmosphere K–1 mol–1
(d) doubled
(c) 0.082 litre – atmosphere–1 K mol–1
20. At STP, 0.50 mol H2 gas and 1.0 mol He gas
(d) 0.082 litre –1 atmosphere – 1 K mol
(a) have equal average kinetic energies [1993]
14. At constant temperature, for a given mass of an
(b) have equal molecular speeds
ideal gas [1991]
(c) occupy equal volumes
(a) The ratio of pressure and volume always
(d) have equal effusion rates
remains constant.
21. Internal energy and pressure of a gas per unit
(b) Volume always remains constant.
volume are related as : [1993]
(c) Pressure always remains constant. 3
(d) The product of pressure and volume (a) P 2 E (b) P E
always remains constant. 3 2
15. Pressure remaining the same, the volume of a (c) P 1 E (d) P 2E
given mass of an ideal gas increases for every 2
22. The ratio among most probable velocity, mean
degree centigrade rise in temperature by definite velocity and root mean square velocity is given
fraction of its volume at [1989] by [1993]
(a) 0°C
(b) its critical temperature (a) 1 : 2 : 3 (b) 1 : 2. 3
(c) absolute zero (c) 2 : 3 : 8/ (d) 2 : 8/ : 3
(d) its Boyle temperature 23. A closed flask contains water in all its three
states solid, liquid and vapour at 0°C. In this
̱°·½ îæ Õ·²»¬·½ ̸»±®§ ±º Ù¿­»­ ¿²¼ situation, the average kinetic energy of water
Ó±´»½«´¿® Í°»»¼­
molecules will be [1992]
16. By what factor does the average velocity of a (a) the greatest in all the three states
gaseous molecule increase when the temperature (b) the greatest in vapour state
(in Kelvin) is doubled ? [2011] (c) the greatest in the liquid state
(a) 2.0 (b) 2.8 (d) the greatest in the solid state
(c) 4.0 (d) 1.4 24. In a closed flask of 5 litres, 1.0 g of H2 is heated
17. If a gas expands at constant temperature, it from 300 to 600 K. Which statement is not correct?
indicates that : [2008] (a) Pressure of the gas increases [1991]
(a) kinetic energy of molecules decreases (b) The rate of collision increases
(b) pressure of the gas increases (c) The number of moles of gas increases
(c) kinetic energy of molecules remains the same (d) The energy of gaseous molecules increases
(d) number of the molecules of gas increases 25. The root mean square speeds at STP for the
18. Which of the following expressions correctly gases H2, N2, O2 and HBr are in the order : [1991]
represents the relationship between the average (a) H2< N2< O2 < HBr (b) HBr < O2 < N2 < H2
molar kinetic energy, KE, of CO and N2 molecules (c) H2 < N2 = O2< HBr (d) HBr < O2 < H2 < N2.
at the same temperature ? [2000] 26. Root mean square velocity of a gas molecule is
(a) KECO KE N2 proportional to [1990]
(b) KE CO KE N 2 (a) m1/2 (b) m0
(c) KE CO KE N2 (c) m–1/2 (d) m
(d) cannot be predicted unless volumes of the 27. Absolute zero is defined as the temperature
gases are given (a) at which all molecular motion ceases
19. The temperature of the gas is raised from 27°C
(b) at which liquid helium boils [1990]
to 927°C, the root mean square velocity is (c) at which ether boils
[1994]
(d) all of the above
EBD_7324
ìê ÝØÛÓ×ÍÌÎÇ
̱°·½ í æ Ê¿² ¼»® É¿¿´ù­ Û¯«¿¬·±² ¿²¼ (a) High temperature, low pressure
´·¯©º¿½¬·±² ±º Ù¿­»­ (b) Low temperature, high pressure
(c) High temperature, high pressure
28. A gas such as carbon monoxide would be most (d) Low temperature, low pressure
likely to obey the ideal gas law at : [2015 RS] 35. Cyclopropane and oxygen at partial pressures
(a) high temperatures and low pressures. 170 torr and 570 torr respectively are mixed in a
(b) low temperatures and high pressures. gas cylinder. What is the ratio of the number of
(c) high temperatures and low pressures. moles of cyclopropane to the number of moles
(d) low temperatures and low pressures. of oxygen (nC3H6/nO2)? [1996]
29. Maximum deviation from ideal gas is expected 170 42
(a) 0.39
from : [NEET 2013] 570 32
(a) N2(g) (b) CH4(g) 170 170 570
(b) 0.19
(c) NH3 (g) (d) H2(g) 42 42 32
30. What is the density of N2 gas at 227°C and 170
(c) 0.23
5.00 atm pressure? (R = 0.0821 atm K–1 mol–1) 740
[NEET Kar. 2013] 170
(a) 0.29 g/ml (b) 1.40 g/ml (d) 0.30
570
(c) 2.81 g/ml (d) 3.41 g/ml 36. At which one of the following temperature -
31. For real gases van der Waals equation is written as pressure conditions the deviation of a gas from
ideal behaviour is expected to be minimum?
an 2 (V nb ) nRT
p [1996]
V2 (a) 350 K and 3 atm. (b) 550 K and 1 atm.
where ‘a’ and ‘b’ are van der Waals constants. (c) 250 K and 4 atm. (d) 450 K and 2 atm.
Two sets of gases are : 37. Under what conditions will a pure sample of an
(I) O2, CO2, H2 and He ideal gas not only exhibit a pressure of 1 atm but
(II) CH4, O2 and H2 also a concentration of 1 mole litre–1 ?
The gases given in set-I in increasing order of (R = 0.082 litre atm mol–1deg–1) [1993]
‘b’ and gases given in set-II in decreasing order (a) At STP
of ‘a’, are arranged below. Select the correct (b) When V = 22.4 litres
order from the following : [2012 M] (c) When T = 12 K
(a) (I) He < H2 < CO2 < O2 (II) CH4 > H2 > O2 (d) Impossible under any conditions
(b) (I) O2 < He < H2 < CO2 (II) H2 > O2 > CH4 38. When is deviation more in the behaviour of a
(c) (I) H2 < He < O2 < CO2 (II) CH4 > O2 > H2 gas from the ideal gas equation PV = nRT ? [1993]
(d) (I) H2 < O2 < He < CO2 (II) O2 > CH4 > H2 (a) At high temperature and low pressure
32. A gaseous mixture was prepared by taking equal (b) At low temperature and high pressure
mole of CO and N2. If the total pressure of the (c) At high temperature and high pressure
mixture was found 1 atmosphere, the partial (d) At low temperature and low high pressure
pressure of the nitrogen (N2) in the mixture is : 39. Which is not true in case of an ideal gas ? [1992]
(a) 0.5 atm (b) 0.8 atm [2011] (a) It cannot be converted into a liquid
(c) 0.9 atm (d) 1 atm (b) There is no interaction between the
33. The pressure exerted by 6.0g of methane gas in molecules
a 0.03 m3 vessel at 129°C is (Atomic masses :
(c) All molecules of the gas move with same
C = 12.01, H = 1.01 and R = 8.314 JK–1 mol –1)
speed
(a) 31684 Pa (b) 215216 Pa [2010]
(d) At a given temperature, PV is proportional
(c) 13409 Pa (d) 41648 Pa
34. Van der Waal's real gas, act as an ideal gas, at to the amount of the gas
which conditions? [2002]
ͬ¿¬»­ ±º Ó¿¬¬»® ìé

40. An ideal gas can’t be liquefied because [1992] then for an ideal gas, the density is given by
(a) its critical temperature is always above 0°C [1989]
(b) its molecules are relatively smaller in size RT P
(a) (b)
(c) it solidifies before becoming a liquid PM RT
(d) forces operated between its molecules are M PM
negligible (c) (d)
V RT
41. Select one correct statement. In the gas equation,
45. Correct gas equation is : [1989]
PV = nRT [1992]
(a) n is the number of molecules of a gas V1T2 V2 T1 P V
1 1 T1
(a) (b)
(b) V denotes volume of one mole of the gas P1 P2 P2 V2 T2
(c) n moles of the gas have a volume V P1T2 P2 V2 V1V2
(d) P is the pressure of the gas when only one (c) (d) P1P2
V1 T2 T1T2
mole of gas is present.
42. A gas is said to behave like an ideal gas when ̱°·½ ìæ Ô·¯«·¼ ͬ¿¬»
the relation PV/T = constant. When do you
expect a real gas to behave like an ideal gas ? 46. The surface tension of which of the following
(a) When the temperature is low [1991] liquid is maximum? [2005]
(a) C2H5OH (b) CH3OH
(b) When both the temperature and pressure
(c) H2O (d) C6H6
are low
47. In a pair of immiscible liquids, a common solute
(c) When both the temperature and pressure
dissolves in both and the equilibrium is reached.
are high Then the concentration of the solute in upper
(d) When the temperature is high and pressure layer is [1994]
is low (a) In fixed ratio with that in the lower layer
43. In van der Waal's equation of state for a non- (b) Same as the lower layer
ideal gas, th e term that accoun ts for (c) Lower than the lower layer
intermolecular forces is : [1990] (d) Higher than the lower layer.
(a) (V – b) (b) (RT)–1 48. A liquid can exist only : [1994]
a (a) between triple point and critical temperature
(c) P (d) RT
V2 (b) at any temperature above the melting point
44. If P, V, M, T and R are pressure, Volume, molar (c) between melting point an d critical
mass, temperature and gas constant respectively, temperature
(d) between boiling and melting temperature.

ANSWER KEY
1 (a) 6 (b) 11 (b) 16 (d) 21 (a) 26 (c) 31 (c) 36 (b) 41 (c) 46 (c)
2 (c) 7 (a) 12 (a) 17 (c) 22 (d) 27 (a) 32 (a) 37 (c) 42 (d) 47 (a)
3 (b) 8 (a) 13 (b) 18 (c) 23 (b) 28 (a) 33 (d) 38 (b) 43 (c) 48 (d)
4 (N) 9 (a) 14 (d) 19 (d) 24 (c) 29 (c) 34 (a) 39 (c) 44 (d)
5 (b) 10 (d) 15 (a) 20 (a) 25 (b) 30 (d) 35 (d) 40 (d) 45 (b)
EBD_7324
ìè ÝØÛÓ×ÍÌÎÇ

Hints & Solutions


1. (a) Given, n H2 n O2 and t H2 t O2 3r1 M2
According to Graham's law of diffusion for two
r1 4
different gases.
M2
rH 2 v1 / t1 MO 2 32 9
4
rO 2 v2 / t 2 MH 2 2 M2 = 36 u
rA MB
1/ 2 6. (b) r
16 4 B M A
1/ x
V
x MB
4 20 MB 1
2 2 49
V 49
x=8
10
Fraction of O2 = 1/8
1
2. (c) According to Avogadro’s law "At same MB = 49 12.25u
4
temperature and pressure 7. (a) Given
Volume no. of moles" P1 = 1.5 bar, T1 = 273 + 15 = 288 K, V1 = V
w w w P2 = 1.0 bar, T1 = 273 + 25 = 298K, V2 = ?
n H 2 = ; n O2 ; n
2 32 CH4 16 P1V1 P2 V2
VH : VO : VCH = n H2 : n O 2 : n CH 4
2 2 4
T1 T2
w w w 1.5 V 1 V2
= : : = 16 : 1 : 2
2 32 16 288 298
3. (b) This type of attractive force operates V2 = 1.55 V i.e., volume of bubble will be almost
between the polar molecules having permanent 1.6 time to initial volume of bubble.
dipole and the molecules lacking permanent 8. (a) Rate of diffusion depend upon molecular
dipole. weight
HCl is polar ( 0) and He is non polar ( = 0), r1 M2
thus gives dipole-induced dipole interaction. r1 = r 2 if M1 = M2
r2 M1
VA VB MB Hence compounds are N2O and CO2 as both
4. (N) =
tA tB MA have same molar mass i.e 22
9. (a) Applying Boyle's law P 1V1 = P2V2 for both
tB 200 36 4 36 gases
= =
tA 150 MA 3 MA 500 200
400 P 3 P
16 36 81 1000 3
= MA = = 20.25
9 MA 4 666.6 400
600 P 3 P
5. (b) According to Graham’s law of diffusion 1000 3
200 400 600
1 1 PT P P 200 torr
r 3 3 3
d M
Moles of H 2 8
r1 M2 10. (d) (Given)
Moles of O 2 1
r2 M1
M.W. of O 2 weight of H 2 8
.
M.W. of H 2 weight of O 2 1
ͬ¿¬»­ ±º Ó¿¬¬»® ìç

weight of H 2 8 2 1 27 273 300 1


weight of O 2 32 1 2 927 273 1200 2
11. (b) Given initial volume (V1) = 500 ml ; Initial u2 = 2u1
temperature (T1) = 27ºC = 300 K and final 20. (a) Average kinetic energy depends only on
temperature (T2) = –5ºC = 268 K.
3
From Charle’s law : temperature K.E. kT
2
V1 V2 500 V2
or 1 1
T1 T2 300 268 21. (a) PV mnu 2 Mu 2
Where V2 = New volume of gas 3 3
2 1 2 2
500 . M u2 E or P E per unit vol.
V2 268 446.66 ml. 3 2 3 3
300
12. (a) Given initial volume (V1) = 600 c.c.; Initial 2RT
22. (d) Most probable velocity ( )
pressure (P1) = 750 mm and final volume M
(V2) = 500 c.c. According to Boyle’s law, 8RT
P1V1 = P2V2 Mean velocity ( v )
M
750 × 600 = P2 × 500
3RT
750 600 Root mean square velocity (u) =
or P2 900 mm . M
500
2 RT 8RT 3RT
Therefore increase in pressure = (900 – 750) :v:u : :
M M M
= 150 mm.
13. (b) R = 0.082 litre atm K–1 mol–1 . 8
2: : 3
14. (d) According to Boyle’s law at constant
1 23. (b) Velocity and hence the K.E is maximum in
temperature, P or PV = constant the gaseous state.
V
24. (c) Here volume is constant. Again the mass of
15. (a) Charle’s Law - The volume of the given H2 is fixed so the number of moles of the gas do not
1 change. As temperature increases the pressure also
mass of a gas increases or decreases by of increases and the rate of collision among the gas
273
molecules and their energy also increases.
its volume at 0°C for each degree rise or fall of
1 1
temperature at constant pressure. 25. (b) PV m N u2 M u2
3 3
t or u 3PV / M.
Vt V0 1 at constant Pressure
273
1
8RT At STP, u
16. (d) Average velocity = M
M i.e higher will be the molar mass lower will be the
i.e., v T value of urms.
V2 2T Molecular masses of H2, N2, O2 and HBr are 2,
= 1.41 28, 32 and 81. Hence the correct order of urms
V1 T
17. (c) At any constant temperature the K.E. of will be HBr < O2 < N2 < H2
gaseous molecules remains same. (K.E. T). 26. (c) Accor ding to kinetic gas equation
3 1
18. (c) Average molar kinetic energy = kT PV = mNu 2 , u = root mean square velocity
2 3
As temperature is same hence average kinetic 1
energy of CO and N2 will be same. 3PV 1 ó
u2 or u i.e u k m 2
19. (d) u T or u1 / u 2 T1 / T2 mN m
EBD_7324
ëð ÝØÛÓ×ÍÌÎÇ
27. (a) Absolute zero is the temperature at which 40. (d) In an ideal gas, the intermolecular forces of
kinetic energy of gas molecules becomes zero attraction are negligible and hence it cannot be
i.e. all molecular motion ceases. liquefied.
41. (c) In the equation PV = nRT, n moles of the
28. (a) At high temperature and low pressure. gas have volume V.
29. (c) Higher the critical temperature more easily 42. (d) At low pressure and high temperature
will be the gas liquify. Now since most easily correction for 1 mole of a gas is negligible, i.e
liquifiable gas show larger deviation, NH3 will the effect of a/v2 and b becomes negligible.
show maximum deviation from ideal behaviour. Thus the gas equation becomes
PM 5 28 PV = nRT
30. (d) Density = 3.41g/ml
RT 0.0821 500 PV
PV = RT or 1 [Ideal gas equation]
31. (c) Van der waal’s gas constant ‘a’ represents RT
intermolecular forces of attraction b/w gaseous Z = 1, and gas shows ideal behaviour.
molecules while ‘b’ represent effective size of 43. (c) Van der Waal's equation for 1 mole :
molecules, so the order should be H2 < He < O2
< CO2 and CH4 > O2 > H2. a
P (V b ) RT
32. (a) Given nCO = n N2 V2
PCO + PN2 = 1 atm a
Here, P represent the intermolecular
Partial pressure of a gas = mole fraction of gas × V2
total pressure forces and (V – b) is the corrected volume.
n N2 n N2
PN2 = 1= 1 m
n CO n N 2 2n N2 44. (d) PV = nRT = RT
1 M
= 0.5 atm.
2 m PM
or PM = RT = dRT d=
nRT w RT V RT
33. (d) P =
V m V
PV P1V1 P2 V2
6 8.314 402 45. (b) constant or
å 41648Pa T T1 T2
16.05 0.03
34. (a) At higher temperature and low pressure
real gas acts as an ideal gas. P1V1 T1
35. (d) By Ideal gas equation P2 V2 T2
P1V n1RT 46. (c) Due to intermolecular H-bonding the
n1 P1 and n 2 P2 surface tension of H2O is more than other given
liquids. One H2O molecule is joined with 4
n1 P1 n1 170 another H2O molecule through H–bond.
n 2 P2 n 2 570 Hydrogen bonding is in order H2O > C2H5OH >
36. (b) At low pressure and high temperature real CH3OH.
gas nearly behave like ideal gas. Hence deviation 47. (a) By definition of Nernst distribution law.
is minimum from ideal behaviour. When a solute is shaken with two immiscible
n liquids, having solubility in both, the solute
37. (c) PV = nRT or P = RT = CRT.. distributes itself between the two liquids in such
V
Hence 1 = 1 × 0.082 × T a way that the ratio of its concentrations in two
1 liquids is constant at a given temperature,
T 12K provided the molecular state of the solute
0.082
38. (b) At low temperature and high pressure. remains the same in both the liquids.
48. (d) A substance exists as a liquid above its
39. (c) Molecules of an ideal gas move with
m. pt. and below its b. pt.
different speeds.
ݸ¿°¬»®

6 Thermodynamics

̱°·½ ïæ Ú·®­¬ Ô¿© ¿²¼ Þ¿­·½ Ú«²¼¿³»²¬¿´­ (a) (I) and (IV) (b) (II), (III) and (IV)
±º ̸»®³±¼§²¿³·½­ (c) (I), (II) and (III) (d) (II) and (III)
7. When 1 mol of a gas is heated at constant
1. A gas is allowed to expand in a well insulated volume, temperature is raised from 298 to 308 K.
container against a constant external pressure
If heat supplied to the gas is 500 J, then which
of 2.5atm from an initial volume of 2.50 L to a
statement is correct ? [2001]
final volume of 4.50L. The change in internal
energy U of the gas in joules will be:- [2017] (a) q = w = 500 J, U = 0
(a) – 500J (b) – 505J (b) q = U = 500 J, w = 0
(c) + 505J (d) 1136.25J (c) q = –w = 500 J, U = 0
2. The heat of combustion of carbon to CO2 is (d) U = 0, q = w = –500 J
–393.5 kJ/mol. The heat released upon formation ̱°·½ îæ Ô¿©­ ±º ̸»®³±½¸»³·­¬®§
of 35.2 g of CO2 from carbon and oxygen gas is
[2015 RS] 8. Three thermochemical equations are given
(a) –315 kJ (b) +315kJ below:
(c) –630 kJ (d) –3.15 kJ (i) C(graphite) + O2(g) CO2(g);
3. When 5 litres of a gas mixture of methane and
rH° = x kJ mol
–1
propane is perfectly combusted at 0°C and 1
atmosphere, 16 litre of oxygen at the same 1
(ii) C(graphite) + O (g) CO(g);
temperature and pressure is consumed. The 2 2
amount of heat released from this combustion rH° = y kJ mol
–1
in kJ ( H comb (CH 4) = 890 kJ mol – 1,
Hcomb (C3H8) = 2220 kJ mol–1) is 1
(iii) CO(g) + O (g) CO2(g);
[NEET Kar. 2013] 2 2
rH° = z kJ mol
–1
(a) 32 (b) 38
(c) 317 (d) 477 Based on the above equations, find out which
4. Which of the following is correct option for free of the relationship given below is correct?
expansion of an ideal gas under adiabatic [NEET Kar. 2013]
condition ? [2011] (a) x = y – z (b) z = x + y
(a) q = 0, T 0, w = 0 (c) x = y + z (d) y = 2z – x
(b) q 0, T = 0, w = 0 9. Standard enthalpy of vapourisation vap H° for
(c) q = 0, T = 0, w = 0 water at 100°C is 40.66 kJ mol–1. The internal
(d) q = 0, T < 0, w 0 energy of vaporisation of water at 100°C
5. Three moles of an ideal gas expan ded (in kJ mol–1) is : [2012]
spontaneously into vacuum. The work done will (a) + 37.56 (b) – 43.76
be : [2010] (c) + 43.76 (d) + 40.66
(a) Zero (b) Infinite (Assume water vapour to behave like an ideal gas).
(c) 3 Joules (d) 9 Joules 10. Equal volumes of two monoatomic gases, A and
6. Which of the following are not state functions ? B, at same temperature and pressure are mixed.
(I) q + w (II) q [2008] The ratio of specific heats (Cp/Cv) of the mixture
(II) w (IV) H - TS will be : [2012 M]
EBD_7324
ëî ÝØÛÓ×ÍÌÎÇ
(a) 0.83 (b) 1.50 H = – X2 kJ mol–1
(c) 3.3 (d) 1.67 (iii) CO2(g) + H2(g) = CO(g) + H2O,
11. Enthalpy change for the reaction, [2011] H = – X3 kJ mol–1
4H(g) 2H2(g) is – 869.6 kJ. 5
(iv) C 2 H 2(g) O 2(g) = 2CO2(g) + H2O(l)’
The dissociation energy of H–H bond is : 2
H = + 4X4 kJ mol–1
(a) – 434.8 kJ (b) – 869.6 kJ
Enthalpy of formation of H2O (l) is
(c) + 434.8 kJ (d) + 217.4 kJ
(a) + X3 kJ mol– 1 (b) – X4 kJ mol– 1
12. Consider the following processes :
(c) + X1 kJ mol– 1 (d) – X2 kJ mol– 1
H (kJ/mol)
17. Given that bond energies of H – H and Cl – Cl
1/2 A B +150
are 430 kJ mol– 1 and 240 kJ mol–1 respectively
3B 2C + D –125
and Hf for HCl is – 90 kJ mol– 1, bond enthalpy
E + A 2D +350
of HCl is [2007]
For B + D E + 2C, H will be : [2011M]
(a) 525 kJ/mol (b) – 175 kJ/mol (a) 380 kJ mol–1 (b) 425 kJ mol–1
(c) – 325 kJ/mol (d) 325 kJ/mol (c) 245 kJ mol–1 (d) 290 kJ mol–1
13. The following two reactions are known : [2010] 18. The enthalpy of hydrogenation of cyclohexane
is – 119.5 kJ mol–1. If resonance energy of benzene
Fe2O3(s) + 3CO (g) 2Fe(s) + 3CO2(g);
is –150.4 kJ mol–1, its enthalpy of hydrogenation
H = –26.8 kJ
would be [2006]
FeO(s) + CO(g) Fe(s) + CO2(g);
(a) – 208.1 kg mol –1 (b) – 269.9 kg mol–1
H = –16.5 kJ (c) – 358.5 kg mol –1 (d) – 508.9 kg mol–1
The value of H for the following reaction 19. Assume each reaction is carried out in an open
Fe2O3(s) + CO(g) 2FeO(s) + CO2(g) is; container. For which reaction will H = E ?
(a) + 6.2 kJ (b) + 10.3 kJ [2006]
(c) – 43.3 kJ (d) – 10.3 kJ (a) C(s) + 2H2O (g) 2H2 (g) + CO2 (g)
(b) PCl 5 (g) PCl3 (g) + Cl2 (g)
14. From the following bond energies: [2009]
(c) 2CO (g) + O2 (g) 2CO2 (g)
H – H bond energy: 431.37 kJ mol –1
(d) H2 (g) + Br 2 (g) 2 HBr (g)
C = C bond energy: 606.10 kJ mol –1
20. The absolute enthalpy of neutralisation of the
C – C bond energy: 336.49 kJ mol–1
reaction:
C – H bond energy: 410.50 kJ mol–1
MgO (s) + 2HCl (aq) MgCl2 (aq) + H2O (l)
Enthalpy for the reaction, will be: [2005]
H H H H (a) 57.33 kJ mol 1
| | | | (b) Greater than 57.33 kJ mol 1
C C H H H C C H (c) Less than 57.33 kJ mol 1
| | | |
(d) 57.33 kJ mol 1
H H H H
will be: 21. If the bond energies of H H, Br Br, and
(a) – 243.6 kJ mol–1 (b) –120.0 kJ mol–1 H-Br are 433, 192 and 364 kJ mol–1 respectively,
(c) 553.0 kJ mol–1 (d) 1523.6 kJ mol–1 the H° for the reaction [2004]
15. Bond dissociation enthalpy of H2, Cl2 and HCl H 2 (g ) Br2 (g ) 2HBr ( g) is
are 434, 242 and 431 kJ mol–1 respectively.
(a) – 261 kJ (b) + 103 kJ
Enthalpy of formation of HCl is : [2008]
(c) + 261kJ (d) – 103 kJ
(a) 93 kJ mol–1 (b) – 245 kJ mol–1
22. The work done during the expansion of a
(c) – 93 kJ mol–1 (d) 245 kJ mol–1
gas from a volume of 4 dm3 to 6 dm3 against
16. Consider the following reactions: [2007]
a constant external pressure of 3 atm is
(i) H +(aq) + OH–(aq) = H2O(l),
(1 L atm = 101.32 J) [2004]
H = – X1 kJ mol–1
(a) – 6 J (b) – 608 J
1 (c) + 304 J (d) – 304 J
(ii) H2(g) + O2(g) = H2O(l),
2
̸»®³±¼§²¿³·½­ ëí

23. The molar heat capacity of water at constant 31. The values of heat of formation of SO 2 and SO3
pressure is 75 JK–1 mol–1. When 1kJ of heat is are –298.2 kJ and –98.2 kJ. The heat of formation
supplied to 100 g of water, which is free to of the reaction [2000]
expand, the increase in temperature of water is SO 2 (1 / 2) O 2 SO 3 will be
(a) 6.6 K (b) 1.2 K [2003]
(a) –200 kJ (b) –356.2 kJ
(c) 2.4 K (d) 4.8 K
24. The densities of graphite and diamond at 298 K (c) + 200 kJ (d) – 396.2 kJ
are 2.25 and 3.31 g cm–3, respectively. If the 32. For a cyclic process, which of the following is
standard free energy difference ( Gº) is equal not true? [1999]
to 1895 J mol–1, the pressure at which graphite (a) H= 0 (b) E = 0
will be transformed into diamond at 298 K is (c) G = 0 (d) Total W = 0
[2003] 33. Adiabatic expansions of an ideal gas is
(a) 9.92 × 105 Pa (b) 9.92 × 108 Pa accompanied by [1999]
(c) 9.92 × 107 Pa (d) 9.92 × 106 Pa (a) decrease in E
25. For which one of the following equations is (b) increase in temperature
Hºreact equal to Hfº for the product? [2003] (c) decrease in S
(a) 2 CO( g) O2 (g ) 2 CO2 (g ) (d) no change in any one of the above
(b) N 2( g) O 3(g ) N 2 O 3( g ) properties
(c) CH 4(g ) 2 Cl (g ) CH 2 Cl 2( l) 2HCl (g ) 34. For a reaction in which all reactants and
products are liquids, which one of the following
(d) Xe(g) 2 F2( g) XeF4(g)
equations is most applicable ? [1999]
26. For the reaction (a) H< E (b) H = S
C3 H 8(g) 5O 2( g) 3CO 2(g ) 4 H 2 O (l)
(c) H= E (d) H = G
at constant temperature, H – E is [2003]
(a) – RT (b) + RT 35. One mole of an ideal gas at 300 K is expanded
(c) – 3 RT (d) + 3 RT isothermally from an initial volume of 1 litre to
27. In a closed insulated container, a liquid is stirred 10 litres. The E for this process is
with a paddle to increase the temperature, which (R = 2 cal. mol–1 K–1) [1998]
of the following is true? [2002] (a) 163.7 cal (b) zero
(a) E = W 0, q 0 (b) E = W = q 0 (c) 1381.1 cal (d) 9 lit. atm
(c) E = 0, W = q 0 (d) W = 0, E = q 0 36. Given that C O 2 CO 2 : H º x kJ
28. Heat of combustion Hº for C (s), H2 (g) and
CH4 (g) are –94, –68 and –213 kcal/mol, then 2 CO O 2 2 CO 2 : H º y kJ
Hº for C(s) 2H 2 (g) CH 4 (g ) is [2002] the enthalpy of formation of carbon monoxide
(a) –17 kcal (b) – 111 kcal will be [1997]
(c) –170 kcal (d) –85 kcal 2x y y 2x
1 (a) (b)
29. Enth alpy of CH 4 O2 CH 3 OH is 2 2
2 (c) 2x – y (d) y – 2x
negative. If enthalpy of combustion of CH4 and 37. Hydrogen has an ionisation energy of 1311 kJ
CH3OH are x and y respectively, then which mol –1 and for chlorine it is 1256 kJ mol –1.
relation is correct [2001] Hydrogen forms H+ (aq) ions but chlorine does
(a) x > y (b) x < y not form Cl+ (aq) ions because [1996]
(c) x = y (d) x y
(a) H+ has lower hydration enthalpy
30. What is the enthalpy change for,
(b) Cl+ has lower hydration enthalpy
2H 2 O 2 (l) 2 H 2 O(l) O 2 ( g) if heat of (c) Cl has high electron affinity
formation of H2O2 (l) and H2O (l) are –188 and (d) Cl has high electronegativity
–286 kJ/mol respectively? [2001] 38. If enthalpies of formation of C 2 H 4(g) , CO2(g)
(a) –196 kJ/mol (b) + 948 kJ/mol
(c) + 196 kJ/mol (d) –948 kJ/mol and H 2 O ( l) at 25°C and 1atm pressure are 52,
EBD_7324
ëì ÝØÛÓ×ÍÌÎÇ
– 394 and – 286 kJ/mol respectively, the change 45. Standard entropies of X2, Y2 and XY3 are 60, 40
in ethalpy is equal to [1995] and 50 JK–1mol–1 respectively. For the reaction
(a) – 141.2 kJ/mol (b) – 1412 kJ/mol 1 3
X 2 + Y2 XY3 , H = – 30 kJ
(c) + 14.2 kJ/mol (d) + 1412 kJ/mol 2 2
39. Equal volumes of molar hydrochloric acid and to be at equilibrium, the temperature should be:
sulphuric acid are neutralized by dil. NaOH [2010]
solution and x kcal and y kcal of heat are liberated (a) 750 K (b) 1000 K
respectively. Which of the following is true ? (c) 1250 K (d) 500 K
[1994] 46. For the gas phase reaction, [2008]
1 PCl5(g) PCl3(g) + Cl2(g)
(a) x = y (b) x y
2 which of the following conditions are correct ?
(c) x = 2y (d) None of these (a) H = 0 and S < 0 (b) H > 0 and S > 0
40. For the reaction [1994] (c) H < 0 and S < 0 (d) H > 0 and S < 0
N 2 3H 2 2NH3, H = ? 47. What is the entropy change (in JK–1 mol–1) when
one mole of ice is converted into water at 0º C?
(a) E + 2RT
T (b) E –2RTT (The enthalpy change for the conversion of ice
(c) H = RT
T (d) E – RT.. to liquid water is 6.0 kJ mol–1 at 0ºC) [2003]
41. During isothermal expansion of an ideal gas, its (a) 21.98 (b) 20.13
[1991, 94] (c) 2.013 (d) 2.198
(a) internal energy increases 48. 2 mole of an ideal gas at 27ºC temperature is
(b) enthalpy decreases expanded reversibly from 2 lit to 20 lit. Find the
(c) enthalpy remains unaffected entropy change (R = 2 cal/mol K) [2002]
(d) enthalpy reduces to zero. (a) 92.1 (b) 0
42. If H is the change in enthalpy and E, the (c) 4 (d) 9.2
49. Unit of entropy is [2002]
change in internal energy accompanying a
(a) JK–1 mol–1 (b) J mol –1
gaseous reaction, then [1990]
(c) J–1 K–1 mol–1 (d) JK mol–1
(a) H is always greater than E, 50. The entropy change in the fusion of one mole
(b) H < E only if the number of moles of of a solid melting at 27ºC (Latent heat of fusion,
the products is greater than the number of 2930 J mol–1) is : [2000]
moles of the reactants (a) 9.77 J K–1 mol–1 (b) 10.73 J K–1 mol–1
(c) H is always less than E (c) 2930 J K–1 mol–1 (d) 108.5 J K–1 mol–1
51. Identify the correct statement regarding
(d) H < E only if the number of moles of
products is less than the number of moles entropy: [1998]
of the reactants. (a) At absolute zero of temperature, entropy
of a perfectly crystalline substance is taken
̱°·½ íæ Û²¬®±°§ ¿²¼ Í»½±²¼ Ô¿© ±º to be zero
̸»®³±¼§²¿³·½­ (b) At absolute zero of temperature, the
entropy of a perfectly crystalline substance
43. The enthalpy of fusion of water is 1.435 kCal/mol. is +ve
The molar entropy change for the melting of ice (c) At absolute zero of temperature, the
at 0°C is : [2012] entropy of all crystalline substances is
(a) 10.52 cal / (mol K) (b) 21.04 cal / (mol K) taken to be zero
(c) 5.260 cal / (mol K) (d) 0.526 cal / (mol K) (d) At 0ºC, the entropy of a perfectly
44. If the enthalpy change for the transition of liquid crystalline substance is taken to be zero
water to steam is 30 kJ mol–1 at 27ºC, the entropy 52. According to the third law of thermodynamics
change for the process would be : [2011] which one of the following quantities for a
(a) 10 J mol –1 K–1 (b) 1.0 J mol–1 K–1 perfectly crystalline solid is zero at absolute
(c) 0.1 J mol–1 K–1 (d) 100 J mol–1 K–1 zero? [1996]
̸»®³±¼§²¿³·½­ ëë

(a) Free energy (b) Entropy 59. For vaporization of water at 1 atmospheric
(c) Enthalpy (d) Internal energy pressure, the values of H and S are 40.63
53. Given the following entropy values (in J K–1 mol–1) kJmol–1 and 108.8 JK–1 mol–1, respectively. The
at 298 K and 1 atm :H2 (g) : 130.6, Cl2 (g) : 223.0, temperature when Gibbs energy change ( G) for
HCl (g) : 186.7.The entropy change (in this transformation will be zero, is: [2010]
J K–1 mol–1) for the reaction (a) 293.4 K (b) 273.4 K
H 2 ( g ) Cl 2 ( g ) 2HCl ( g ) is [1996] (c) 393.4 K (d) 373.4 K.
60. Match List -I (Equations) with List-II (Type of
(a) +540.3 (b) +727.0
processes) and select the correct option. [2010]
(c) –166.9 (d) +19.8
List I List II
54. A chemical reaction will be spontaneous if it is Equations Type of processes
accompanied by a decrease of [1994]
(1) Kp > Q (i) Non spontaneous
(a) entropy of the system (2) G < RT In Q (ii) Equilibrium
(b) enthalpy of the system (3) Kp = Q (iii) Spontaneous and
(c) internal energy of the system endothermic
(d) free energy of the system H
(4) T (iv) Spontaneous
̱°·½ ìæ Í°±²¬¿²»·¬§ ¿²¼ Ù·¾¾ù­ Ú®»» Û²»®¹§ S
Options:
55. For a given reaction, H = 35.5 kJ mol-1 and S (1) (2) (3) (4)
= 83.6 JK-1 mol-1. The reaction is spontaneous at (a) (ii) (i) (iv) (iii)
: (Assume that H and S do not vary with (b) (i) (ii) (iii) (iv)
tempearature) [2017]
(c) (iii) (iv) (ii) (i)
(a) T > 425 K (b) All temperatures
(d) (iv) (i) (ii) (iii)
(c) T > 298 K (d) T < 425 K
61. The values of H and S for the reaction,
56. The correct thermodynamic conditions for the
spontaneous reaction at all temperatures is C(graphite) + CO2 (g) 2CO(g) are 170 kJ and
[2016] 170 JK , respectively. This reaction will be
–1

(a) H < 0 and S = 0 (b) H > 0 and S < 0 spontaneous at [2009]


(c) H < 0 and S > 0 (d) H < 0 and S < 0 (a) 910 K (b) 1110 K
57. For the reaction : [2014] (c) 510 K (d) 710 K
X2O4( ) 2XO2(g) 62. The enthalpy and entropy change for the
U = 2.1 k cal, S = 20 cal K–1 at 300 K reaction
Hence G is:- Br2(l) + Cl2 (g) 2 BrCl (g)
(a) 2.7 k cal (b) – 2.7 k cal are 30kJ mol–1 and 105 JK–1 mol–1 respectively.
The temperature at which the reaction will be in
(c) 9.3 k cal (d) – 9.3 k cal
equilibrium is [2006]
58. In which of the following reactions, standard (a) 273 K (b) 450 K
entropy change ( S°) is positive and standard (c) 300 K (d) 285.7 K
Gibb’s energy change ( G°) decreases sharply
63. Identify the correct statement for change of Gibbs
with increasing temperature ? [2012]
energy for a system ( G system) at constant
1 temperature and pressure : [2006]
(a) C graphite + O2(g) CO(g)
2 (a) If Gsystem = 0, the system has attained
1 equilibrium
(b) CO(g) + O2(g) CO2(g) (b) If Gsystem = 0, the system is still moving in
2
1 a particular direction
(c) Mg(s) + O2(g) MgO(s) (c) If Gsystem < 0, the process is not spontaneous
2 (d) If G system > 0, the process is n ot
1 1 1 spontaneous
(d) C graphite + O2(g) CO2(g)
2 2 2 64. A reaction occurs spontaneously if [2005]
EBD_7324
ëê ÝØÛÓ×ÍÌÎÇ
(a) T S < H and both H and S are + ve 68. The factor of G values is important in
(b) T S > H and H is + ve and S is ve metallurgy. The G values for the following
(c) T S > H and both H and S are + ve reactions at 800ºC are given as :
(d) T S = H and both H and S are + ve S 2 ( s ) 2O 2 ( g ) 2SO 2( g ) ; G =–544 kJ
65. Which of the following pairs of a chemical
reaction is certain to result in a spontaneous 2 Zn ( s ) S 2 (s ) 2 ZnS ( s ) ; G = –293 kJ
reaction? [2005] 2 Zn ( s ) O 2 (g ) 2 ZnO ( s ) ; G = –480 kJ
(a) Exothermic and increasing disorder Then G for the reaction :
(b) Exothermic and decreasing disorder
(c) Endothermic and increasing disorder 2 ZnS (s ) 3O 2( g ) 2 ZnO ( s ) 2SO 2 ( g )
(d) Endothermic and decreasing disorder will be : [2000]
66. Stan dard enthalpy and standard entropy (a) –357 kJ (b) –731 kJ
changes for the oxidation of ammonia at 298 K (c) –773 kJ (d) –229 kJ
are – 382.64 kJ mol–1 and -145.6 JK–1 mol–1,
69. Consider the following reaction occurring in an
respectively. Standard Gibb's energy change for
automobile [1994]
the same reaction at 298 K is [2004]
(a) –22.1 kJ mol–1 (b) –339.3 kJ mol–1 2C 8 H18 (g ) 25 O 2 (g ) 16CO 2 18H 2 O(g )
(c) –439.3 kJ mol–1 (d) –523.2 kJ mol–1
67. Considering entropy (S) as a thermodynamic the sign of H, S and G would be
parameter, the criterion for the spontaneity of (a) +,–,+ (b) –, +, –
any process is [2004] (c) –, +, + (d) +, +, –.
(a) Ssystem Ssurroundings 0
(b) Ssystem Ssurroundings 0
(c) Ssystem 0 only
(d) Ssurroundings 0 only

ANS WER KEY


1 (b) 8 (c) 15 (c) 22 (b) 29 (a) 36 (b) 43 (c) 50 (a) 57 (b) 64 (c)
2 (a) 9 (a) 16 (d) 23 (c) 30 (a) 37 (b) 44 (d) 51 (a) 58 (a) 65 (a)
3 (c) 10 (d) 17 (b) 24 (b) 31 (c) 38 (b) 45 (a) 52 (b) 59 (d) 66 (b)
4 (c) 11 (c) 18 (a) 25 (d) 32 (d) 39 (b) 46 (b) 53 (d) 60 (d) 67 (a)
5 (a) 12 (b) 19 (d) 26 (c) 33 (a) 40 (b) 47 (a) 54 (d) 61 (b) 68 (b)
6 (d) 13 (a) 20 (c) 27 (a) 34 (c) 41 (c) 48 (d) 55 (a) 62 (d) 69 (b)
7 (b) 14 (b) 21 (d) 28 (a) 35 (b) 42 (d) 49 (a) 56 (c) 63 (a)
̸»®³±¼§²¿³·½­ ëé

Hints & Solutions


1. (b) The system is in isolated state. 9. (a) vapH° = 40.66 kJ mol–1
For an adiabatic process, q = 0 T = 100 + 273 = 373 K, E = ?
U= q + w
U= w H = E + ngRT E = H – ngRT
= –p V ng = number of gaseous moles of products
= –2.5 atm × (4.5 – 2.5) L – number of gaseous moles of reactants
= –2.5 × 2 L-atm H Ol H2O(g)
= –5 × 101.3 J
= –506.5 J –505J ng = 1 – 0 = 1
2. (a) C + O2 CO2 + 393.5 kJ/mol E = H – RT
12g 44g E = (40.66 × 103) – (8.314 × 373)
44g CO2 is formed from 12g of carbon = 37559 J/mol or 37.56 kJ / mol
12 35.2 10. (d) For a monoatomic gas
35.2g is formed from g of C
44 3
CV = R
= 9.6 g of C = 9.6/12 = 0.8 mole 2
1 mole release heat 393.5 kJ 3 5
CP = R R = R
0.8 mole release heat = 393.5 × 0.8 2 2
= 314.8 kJ 315 kg 5
R
CP 2 5
3. (c) CH 4 + 2O 2 CO2 + 2H2O 1.67
x 2x
CV 3 R 3
2
C3 H8 + 5O 2 3CO2 + 4H2O 11. (c) Given
(5 x ) 5(5 x )
4H (g) 2H 2(g) ; H 869.6 kJ
2x + 5(5– x) = 16
or 2H 2(g) 4H (g) ;H 869.6 kJ
x = 3L
Heat released 869.6
H 2(g) 2H (g) ; H = 434.8 kJ
2
3 2 12. (b) Given H
= 890 2220 = 317
22.4 22.4
1
4. (c) For free expansion of an ideal gas under A B + 150 ...(a)
adiabatic condition 2
q = 0, T = 0 , w = 0 3B 2C D –125 ...(b)
5. (a) Ideal gas during spontaneous expansion E A 2D +350 ...(c)
into vacuum does not do any external work. To calculate H operate
6. (d) We know that q (heat) and work (w) are not 2 × eq. (a) + eq. (b) – eq. (c)
state functions but (q + w) is a state functions. H = 300 – 125 – 350 = – 175
H – TS (i.e. G) is also a state functions. 13. (a) Fe2O3(s)+ CO(g) 2FeO(s)+ CO2(g)
7. (b) As volume is constant hence work done in Given
this proces is zero hence heat supplied is equal Fe2O3(s)+ 3CO(g) 2Fe(s)+ 3CO2(g) ;
to change in internal energy. H = – 26.8 kJ … (i)
8. (c) Applying Hess’s law, equation (i) can be FeO(s) + CO(g) Fe(s) + CO2(g) ;
obtained by adding equations (ii) and (iii).
H = –16.5 kJ … (ii)
x=y+z H for the reaction
EBD_7324
ëè ÝØÛÓ×ÍÌÎÇ
Fe2O3(s) + CO2(g) 2FeO(s) + CO2(g) to the benzene molecule so it has to over come
can be calculate as for hydrogenation to take place. So H = – 358.5
eqn (i) — 2 × eqn (ii) – (–150.4) = –208.1 kJ
H = –26.8 + 33.0 = + 6.2 kJ 19. (d) We know that
14. (b) Enthalpy of reaction H= E + P V
= B.E(Reactant)– B.E(Product) In the reaction, H2 + Br2 2HBr this is no
change in volume or V = 0
= B.E (C C) 4 B.E.(C–H) B.E.(H H)
So, H = E for this reaction
B.E.(C C) 6 B.E.(C H) 20. (c) As MgO is a oxide of weak base hence
= [606.1 + (4 × 410.5) + 431.37)] – [336.49 some energy is lost to break MgO (s). Hence
+ (6 × 410.5)] enthalpy is less than –57.33 kJ mol–1.
= –120.0 kJ mol–1 21. (d) H 2 (g) Br2 (g ) 2HBr ( g)
15. (c) The reaction for formation of HCl can be H = (B E)reactant – (B E) product
written as
(433 192) (2 364)
H2 + Cl2 2HCI
H – H + Cl – Cl 2 (H – Cl) = 625 – 728 = – 103 kJ
Substituting the given values, we get enthalpy 22. (b) W = – P V
of formation of 3( 6 4) 6 litre atmosphere
Hf (HCl) = [(B.E)K–H + (B.E)Cl–Cl] – 6 101.32 608J
[2 × B.E. (H–Cl)]
23. (c) Given Cp = 75 JK–1 mol–1
= 434 + 242 – 2 × (431) = –186 kJ
100
Enthalpy of formation for n mole , Q = 1000 J
18
186
1 mol HCl = kJ = –93 kJ. T=?
2 Q = nCp T
16. (d) This reaction shows the formation of H2O,
1000 18
and the X2 represents the enthalpy of formation T 2.4 K
of H2O. The enthalpy of formation is the heat 100 75
evolved or absorbed when on e mole of 24. (b) G = – P V = Work done
substance is formed from its constituent atoms. V is the change in molar volume in the
conversion of graphite to diamond.
1 1
17. (b) H2 Cl2 HCl 12 12
2 2 V 10 3 L 1.91 10 3 L
H HCl B.E. of reactant 3.31 2.25
Work done = –(–1.91 × 10–3) × P × 101.3 J
B.E. of products
1895J mol –1
P 9794 atm
1 1 1.91 10 3 101.3
90 430 240 B.E. of HCl
2 2 1 atm = 105 × 1.013 Pa
B.E. of HCl = 215 + 120 + 90
= 425 kJ mol –1 P 9.92 108 Pa
25. (d)
18. (a) + H2 ; H = – 119.5 kJ 26. (c) H E nRT
n = 3 – (1 + 5)
= 3 – 6 = –3
H E ( 3RT )
+ 3 H2 ; H = 3(– 119.5)
27. (a) Internal ener gy is dependent upon
= – 348.5 kJ temperature and according to first law of
The resonance energy provides extra stability thermodynamics total energy of an isolated
̸»®³±¼§²¿³·½­ ëç

system remains same, i.e., in a system of = –98.2 + 298.2 = 200 kJ/Mole


constant mass, energy can neither be created 32. (d) For a cyclic process
nor destroyed by any physical or chemical E 0, H 0 & G 0 . As all depend upon
change but can be transformed from one form final state and initial state,w doesn’t depend on
to another path followed.
E q w 33. (a) E = Q–W
For closed insulated container, q = 0, so, For adiabatic expansion, Q = 0
E = +W, as work is done by the system E = –W
28. (a) C(s) O 2 (g ) CO 2 (g) The negative sign shows decrease in Internal
energy, which is equal to the work done on the
H 94 kCal/mole ..(i)
system by the surroundings.
1 34. (c) As all reactant and product are liquid
H 2 ( g) O2 H 2 O (g )
2 n (g) 0
H 68 kCal/mole ..(ii)
H E nRT
CH 4 2O 2 CO 2 2H 2 O, ( n 0)
H E
H 213 k Cal/mole ...(iii) 35. (b) For an isothermal process E = 0
C(s) 2H 2 CH 4 (g ), H ? ....(iv) 36. (b) Given C O 2 CO 2 , H º x kJ ....(a)
Eqn. (iv) can be obtained by 2CO 2 2CO O 2 H º y kJ … (b)
eq. (i) + eq. (ii) × 2 – eq.(iii)
or CO 2 CO 1/ 2O 2 , H º y / 2 kJ ...(c)
C( s ) O 2 CO 2 ( g) From eq. no. (a) and (c)
2H 2 O2 2H 2 O(g )
1 y 2x
CO 2 2H 2 O CH 4 (g ) 2O 2 C O 2 CO, H º y / 2 x kJ
2 2
C(s) 2H 2 (g ) CH 4 (g) 37. (b) Hydration energy of Cl + is very less than
H+ hence it doesn’t form Cl+ (aq) ion.
So, H CH 4 94 2( 68) ( 213)
38. (b) Enthalpy of formation of C 2 H 4 , CO2 and
= –94 – 136 + 213 = –17 k Cal/mole
1
H 2 O ar e 52, – 394 and – 286 kJ/ mol
29. (a) CH 4 (g ) O 2 ( g) CH 3OH ( l ) H ? respectively. (Given)
2
The reaction is
H [( H of combustion of CH3OH)
C 2 H 4 3O 2 2CO 2 2H 2 O.
( H of combustion of CH 4 )] change in enthalpy,
= [(–y) – (–x)] = –[–y+x] = x – y ( H) H products H reactants
Given H = –ve
x– y< 0 2 ( 394) 2 ( 286) (52 0)
hence x < y = – 1412 kJ/ mol.
30. (a) 2H 2 O 2 (l ) 2 H 2 O (l ) O 2 ( g ) H ? 39. (b) 1 M H2SO4 = 2g eq. of HCl
1
H [2 H f of H 2 O (l ) ( H f of O 2 ) Hence y = 2x or x = y..
2
(2 H f of H 2O 2(l ) )] 40. (b) ng = 2 – 4 = – 2, H = E – 2RT.
T.
[(2 286) (0) ( 2 188)] 41. (c) During isothermal expansion of ideal gas,
[ 572 376] 196 kJ / mole T = 0. Now H = E + PV
1 H E ( PV )
31. (c) SO 2 O2 SO 3
2 H = E + (nRT); T);
Thus if T = 0., H = E
H H of (SO ) H fo (SO ) i.e., remain unaffected
3 2
EBD_7324
êð ÝØÛÓ×ÍÌÎÇ
42. (d) As H = E + ngRT T constant absolute temperatur e
if np < nr; n g = n p – n r = – ve. Unit of entropy is JK–1 mol–1
Hence H < E. Latent heat of fusion Hy
50. (a) S
H 1.435 103 Melting po int T
43. (c) S
T 273 2930
5.260 cal/ mol K = J K–1 mol–1 = 9.77 J K–1 mol–1
300
44. (d) Given H = 30 kJ mol–1 T = 273 + 27 = 300 K 51. (a) We know from th e third law of
thermodynamics, the entropy of a perfectly
H T 3 104 cr ystalline substance at absolute zero
ST J mol–1
T 300 temperature is taken to be zero.
= 100 J mol –1 K–1 52. (b) Entropy states the r andomness or
disorderness of the system. At absolute zero,
1 3
45. (a) S for the reaction X 2 + Y2 XY3 the movement of molecules of the system or
2 2 randomenss of the system is zero, hence entropy
S = 50 – (30 + 60) = – 40 J is also zero.
For equilibrium G = 0 = H – T S 53. (d) Entropy change
H 30000 S Sproduct Sreactant
T = 750 K
S 40 = 2 (186.7) – (223 + 130.6)
46. (b) For the reaction = 373.4 – 353.6
PCl5 (g) PCl3 (g) Cl2 (g) = 19.8 JK–1 mol–1
The r eaction given is an example of 54. (d) G is negative for a spontaneous process.
decomposition reaction and we know that 55. (a) Given H 35.5 kJ mol–1
decomposition reactions are endothermic in S = 83.6 JK–1 mol–1
nature, i.e, H > 0. G= H–T S
Further For a reaction to be spontaneous, G = –ve
n = (1+1) – 1= +1 i.e., H < T S
Hence more number of molecules are present in H 35.5 103 Jmol 1
products which shows more randomness i.e. S T>
> 0 ( S is positive)
S 83.6 JK 1
So, the given reaction will be spontaneous at
H T > 425 K
47. (a) S
T
H per mole 6000 56. (c) G = H – T · S
S(per mole )
T 273 For a spontaneous reaction G = –ve (always)
which is possible only if
21.98 JK 1mol 1
H < 0 and S > 0
48. (d) For isothermal reversible expansion spontaneous at all temperatures.
v 57. (b) Given U = 2.1 k cal, S = 20 cal. K– 1
w q nRT 2.303 log 2
v1 T = 300 K
20 H = U + ngRT
2RT 2.303 log
2 Putting the values given in the equation
= 2 × 2 × T × 2.303 × 1 = 9.2 T 2
q 9.2T H = 2.1 2 300
Entropy change, S 9.2 cal. 1000
T T = 2.1 + 1.2 = 3.3 k cal.
q Now, G = H – T S
49. (a) S
T
20
q required heat per mole = 3.3 300 2.7 k cal
1000
̸»®³±¼§²¿³·½­ êï
58. (a) Since, in the first reaction gaseous products 63. (a) If Gsystem = 0 the system has attained
are forming from solid carbon hence entropy will equilibrium.
increase i.e. S = +ve. If G < 0, the process is spontaneous
1 If G > 0, the process does not occur in the forward
C (gr.) + O2(g) CO(g); S° = + ve
2 direction. It may occur in the backward direction.
Since, G° = H° – T S hence the value of G 64. (c) For a spontaneous reaction
decrease on increasing temperature. G(–ve), which is possible if S = +ve,
1atm H = +ve
59. (d) H 2 O H2O(g) and T S > H [As G = H – T S]
H = 40630 J mol –1 65. (a) Measure of disorder of a system is nothing
S = 108.8 JK–1 mol –1 but Entropy. For a spontaneous reaction,
G < 0. As per Gibbs Helmnoltz equation,
G = H T S
G= H– T S
When G = 0, H T S = 0 Thus G is –ve only
H 40630 J mol 1 When H = –ve (exothermic)
T = S
=
108.8 J mol 1
= 373.4 K. and S = +ve (increasing disorder)
60. (d) 66. (b) G H T S
(1) Kp > 0 (iv) Spontaneous 382.64 ( 298 145.6 10 3 )
(2) G < R T In Q (i) Non spontaneous 339.3 kJ mol 1
(3) Kp = 0 (ii) Equilibrium 67. (a) For a spontaneous process, Stotal is
H always positive
(4) T (iii) Spontaneous 68. (b) For the reaction
S
and endothermic 2 ZnS 2 Zn + S2 ; G1º = 293 kJ ..........(i)
2 Zn + O2 ZnO ; G2º = –480 kJ ..........(ii)
61. (b) G = H – T S
S2 + 2 O2 2 SO2 ; G3º = –544 kJ .........(iii)
At equilibrium, G = 0 Gº for the reaction
0 = (170 × 103 J) – T (170 JK– 1) 2 ZnS + 3 O2 2 ZnO + 2 SO2
T = 1000 K can be obtained by adding eqn. (i), (ii) and (iii)
For spontaneity, G is – ve, which is possible Gº = 293 – 480 – 544 = – 731 kJ
only if T > 1000 K. 69. (b) This is combustion reaction, which is always
62. (d) We know that exothermic hence
G= H– T S H = –ve
When the reaction is in equilibrium, G = 0 As the no. of gaseous molecules are increasing
hence entropy increases
H now G
0 H T S T H T S
S For a spontaneous reaction
2 30 1000 G ve
T 285.7 K
2 105 Which is possible in this case as H = –ve and
S = +ve.
EBD_7324
êî ÝØÛÓ×ÍÌÎÇ

ݸ¿°¬»®

7 Equilibrium

̱°·½ ïæ Ô¿© ±º Ó¿­­ ß½¬·±²ô Û¯«·´·¾®·«³ 1 1


ݱ²­¬¿²¬ øÕ ½ ¿²¼ Õ ° ÷ ¿²¼ ·¬­ ß°°´·½¿¬·±² N (g) + O2(g) NO(g) will be:
2 2 2
1. The equilibrium constant of the following are : 1
N2 + 3H2 2NH3 K1 [2017] 1
(a) K2 (b) K
N2 + O2 2NO K2 2
1 (c) K (d) K2
H2 O2 H 2O K3 5. Given that the equilibrium constant for the
2
The equilibrium constant (K) of the reaction : reaction 2SO2(g) + O2(g) 2SO3(g) has a
value of 278 at a particular temperature. What is
5 K the value of the equilibrium constant for the
2NH3 O2 2NO + 3H2O, will be;
2 following reaction at the same temperature ?
(a) K 2 K33 / K1 (b) K2K3/K1
SO 3 (g) SO2 (g)
1
O 2 (g) [2012 M]
2
(c) K32 K3 / K1 (d) K1K 33 / K 2 (a) 1.8 × 10–3 (b) 3.6 × 10–3
2. A 20 litre container at 400 K contains CO2(g) at (c) 6.0 × 10 –2 (d) 1.3 × 10–5
pressure 0.4 atm and an excess of SrO (neglect 6. Given the reaction between 2 gases represented
the volume of solid SrO). The volume of the by A2 and B2 to give the compound AB(g).
container is now decreased by moving the A2(g) + B2(g) 2 AB(g).
movable piston fitted in the container. The At equilibrium, the concentration
maximum volume of the container, when pressure of A2 = 3.0 × 10–3 M
of CO2 attains its maximum value, will be :- of B2= 4.2 × 10–3 M
(Given that : SrCO3 (s) SrO (s) + CO2(g), Kp of AB = 2.8 × 10–3 M
= 1.6 atm) [2017] lf the reaction takes place in a sealed vessel at
(a) 10 litre (b) 4 litre 527°C, then the value of KC will be : [2012 M]
(c) 2 litre (d) 5 litre (a) 2.0 (b) 1.9
3. If the value of an equilibrium constant for a (c) 0.62 (d) 4.5
particular reaction is 1.6 × 10 12 , then at
7. For the reaction N2(g) + O2(g) 2NO(g),
equilibrium the system will contain :- [2015]
the equilibrium constant is K1. The equilibrium
(a) mostly reactants
constant is K2 for the reaction [2011]
(b) mostly products
(c) similar amounts of reactants and products 2NO(g) + O2(g) 2NO2(g).
(d) all reactants What is K for the reaction
4. If the equilibrium constant for 1
NO2(g) N (g) + O2(g) ?
N2(g) + O2(g) 2NO(g) is K, 2 2
the equilibrium constant for [2015 RS]
Û¯«·´·¾®·«³ êí

(a) 1 / (2K1K2) (b) 1 / (4K1K2) equilibrium (a) and (b) are in the ratio :
(c) [1 / K1K2]½ (d) 1 / (K1K2) (a) 3 : 1 (b) 1 : 9
(c) 36 : 1 (d) 1 : 1
8. In which of the following equilibrium Kc and Kp
13. The value of equilibrium constant of the reaction
are not equal? [2010]
1 1
(a) 2 NO(g) N 2(g) +O 2(g) HI g H 2 (g) I 2 is 8.0 [2008]
2 2
(b) SO 2(g) +NO2(g) SO3(g) +NO(g) The equilibrium constant of the reaction
H2 g I 2 (g) 2HI(g) will be:
(c) H 2(g) +I2(g) 2 HI(g)
1 1
(d) 2C(s) +O2(g) 2 CO2(g) (a) (b)
16 64
9. The reaction 2A (g) B(g) 3C(g) D(g) is 1
(c) 16 (d)
begun with the concentrations of A and B both 8
at an initial value of 1.00 M. When equilibrium is 14. The following equilibrium constants are given:
reached, the concentration of D is measured and N 2 3H 2 2NH3 ; K1 [2003, 2007]
found to be 0.25 M. The value for the equilibrium N2 O2 2NO; K 2
constant for this reaction is given by the
expression [2010] 1
H2 O2 H 2 O; K2
(a) [(0.75)3 (0.25)] [(0.75)2 (0.25)] 2
(b) [(0.75)3 (0.25)] [(1.00)2 (1.00)] The equilibrium constant for the oxidation of
(c) [(0.75)3 (0.25)] [(0.50)2 (0.75)] NH3 by oxygen to give NO is
(d) [(0.75)3 (0.25)] [(0.50)2 (0.25)]
10. If the concentration of OH– ions in the reaction K 2 K32 K 22 K3
(a) (b)
Fe(OH)3(s) Fe3+ (aq) + 3OH– (aq) is K1 K1
1
decreased by times, then equilibrium K1 K 2 K 2 K33
4 (c) (d)
3+
concentration of Fe will increase by : [2008] K3 K1
(a) 8 times (b) 16 times 15. For the reaction [2006]
(c) 64 times (d) 4 times
CH 4 (g ) 2O 2 (g ) CO2 (g) + 2H2O(l),
11. The dissociation equilibrium of a gas AB2 can
–1
be represented as : [2008] rH = –170.8 k J mol
Which of the following statements is not true ?
2AB2 (g) 2AB(g) B2 (g)
(a) The equilibrium constant for the reaction
The degree of dissociation is ‘x’ and is small [CO 2 ]
compared to 1. The expression relating the is given by K p [ CH 4 ][O 2 ]
degree of dissociation (x) with equilibrium
(b) Addition of CH4(g) or O2(g) at equilibrium
constant Kp and total pressure P is :
will cause a shift to the right
(a) (2Kp/P) (b) (2Kp/P) 1/3
(c) The reaction is exothermic
(c) (2Kp/P)1/2 (d) (Kp/P) (d) At equilibrium, the concentrations of
12. The values of Kp1 and Kp2 for the reactions CO2(g) and H2O(l) are not equal
X Y Z ...(a) [2008] 16. K1 and K2 are equilibrium constant for reactions
(i) and (ii)
and A 2B ...(b)
N2(g) + O2 (g) 2 NO (g) .............(i)
are in the ratio of 9 : 1. If degree of dissociation 1 1
of X and A be equal, then total pressure at NO(g) N 2 (g ) O 2 (g ) ......(ii)
2 2
EBD_7324
êì ÝØÛÓ×ÍÌÎÇ
Then, [1989, 94, 2005] CH 3COOC2H 5 H CH 3COOH C 2 H 5OH is
2 (a) 4.33 (b) 5.33
1
(a) K1 = (b) K1 = K22 (c) 6.33 (d) 7.33
K2
̱°·½ îæ λ´¿¬·±² ¾»¬©»»² Õô Ï ¿²¼ Ù ¿²¼
1 Ú¿½¬±®­ Ûºº»½¬·²¹ Û¯«·´·¾®·«³
(c) K1 = (d) K1 = (K2)0
K2 22. Consider t he following liquid - vapour
17. Value of KP in the reaction equilibrium. [2016]
MgCO3(s) MgO (s) CO 2(g ) is [2000] Liquid Vapour
Which of the following relations is correct ?
(a) KP = PCO
2
PCO 2 PMgO d nG Hv d nP Hv
(b) KP PCO 2 (a) 2 2 (b)
PMgCO 3 dT RT dT RT

PCO 2 PMgO d nP Hv d nP Hv
(c) KP (c) 2 2 (d)
PMgCO 3 dT T dT RT2
23. Which of the following statements is correct for
PMgCO 3 a reversible process in a state of equilibrium ?
(d) KP
PCO 2 PMgO [2015]
(a) G = 2.30 RT log K
18. For dibasic acid correct order is [2000]
(b) Gº = –2.30 RT log K
(a) K a1 K a 2 (b) K a1 K a2
(c) Gº = 2.30 RT log K
(c) K a1 K a 2 (d) not certain (d) G = –2.30 RT log K
19. If K1 and K2 are the respective equilibrium 24. For the reversible reaction, [2014]
constants for the two reactions N2(g) + 3H2(g) 2NH3(g) + heat
XeF6 (g) + H2O (g) XeOF4 (g) + 2HF (g) The equilibrium shifts in forward direction
(a) By increasing the concentration of NH3(g)
XeO4 (g) + XeF6 (g) XeOF 4 (g) + XeO 3F 2 (g)
(b) By decreasing the pressure
(c) By decreasing concentration of N2(g) and
the equilibrium constant of the reaction
H2(g)
(d) By increasing pressure and decreasing
XeO4 (g) + 2HF (g) XeO3F 2 (g) + H 2O (g) temperature.
will be [1998] 25. For a given exothermic reaction, K p and KP are
(a) K1/(K2)2 (b) K1 . K2 the equilibrium constants at temperatures T1 and
(c) K1/K2 (d) K2/K1 T2, respectively. Assuming that heat of reaction
20. If is the fraction of HI dissociated at is constant in temperature range between T1 and
equilibrium in the reaction, 2 HI (g) H2 T2, it is readily observed that: [2014]
(g) + I2 (g), starting with 2 moles of HI, the total (a) Kp > K P (b) Kp < K P
number of moles of reactants and products at
equilibrium are [1996] 1
(c) Kp = K P (d) Kp =
(a) 2 + 2 (b) 2 Kp
(c) 1 + (d) 2 –
26. The dissociation constants for acetic acid and
21. The rate constant for forward and backward
HCN at 25°C are 1.5 × 10–5 and 4.5 × 10–10
reaction of hydrolysis of ester are 1.1 10 2 and respectively. The equilibrium constant for the
1. 5 10 3 per minute respectively. Equilibrium equilibrium [2009]
constant for the reaction [1995]
Û¯«·´·¾®·«³ êë

CN– + CH3COOH HCN + CH3COO– would (a) more cis-2 pentene is formed
be: (b) equilibrium remains unaffected
(a) 3.0 × 10– 5 (b) 3.0 × 10– 4 (c) additional trans-2 pentene is formed
(c) 3.0 × 104 (d) 3.0 × 105 (d) equilibrium is shifted in forward direction
27. The reaction quotient (Q) for the reaction 32. According to Le-chatelier’s principle, adding heat
to a solid liquid equilibrium will cause the
N2(g) + 3 H2(g) 2NH3(g)
(a) temperature to increase [1993]
(b) temperature to decrease
[ NH 3 ] 2
is given by Q . The reaction will (c) amount of liquid to decrease
[ N 2 ][ H 2 ]3 (d) amount of solid to decrease.
proceed from right to left if [2003] 33. Which one of the following information can be
(a) Q = 0 (b) Q = Kc obtained on the basis of Le Chatelier principle?
(c) Q < Kc (d) Q > Kc [1992]
where Kc is the equilibrium constant (a) Dissociation constant of a weak acid
28. For the reaction (b) Entropy change in a reaction
(c) Equilibrium constant of a chemical reaction
2 BaO2 (s) 2BaO(s) + O2(g) ;
(d) Shift in equilibrium position on changing
H = +ve. In equilibrium condition, pressure
value of a constraint.
of O2 is dependent on [2002]
(a) mass of BaO2 ̱°·½ íæ ̸»±®·»­ ±º ß½·¼­ ¿²¼ Þ¿­»­ô ×±²·½
(b) mass of BaO Ю±¼«½¬ ±º É¿¬»® ¿²¼ °Ø ͽ¿´»
(c) temperature of equilibrium 34. What is the pH of the resulting solution when
(d) mass of BaO2 and BaO both equal volumes of 0.1 M NaOH and 0.01 M HCl
29. In a two-step exothermic reaction are mixed ? [2015 RS]
A2(g) + B2(g) 3C(g) D(g) (a) 12.65 (b) 2.0
Step 1 Step 2 (c) 7.0 (d) 1.04
Steps 1 and 2 are favoured respectively by 35. Which of the following salts will give highest
[1997] pH in water ? [2014]
(a) high pressure, high temperature and low (a) KCl (b) NaCl
pressure, low temperature (c) Na2CO3 (d) CuSO4
(b) high pressure, low temperature and low 36. Which of these is least likely to act as Lewis
pressure, high temperature base? [NEET 2013]
(c) low pressure, high temperature and high (a) F– (b) BF3
pressure, high temperature (c) PF3 (d) CO
(d) low pressure, low temperature and high 37. Which of the following is least likely to behave
pressure, low temperature as Lewis base ? [2011]
30. The equilibrium constant for the reaction (a) H2O (b) NH3
A2 2A at 500 K and 700 K are 1 × 10–10 (c) BF3 (d) OH–
and 1 × 10–5 respectively. The given reaction is 38. Which one of the following molecular hydrides
[1996] acts as a Lewis acid? [2010]
(a) exothermic (b) slow (a) NH3 (b) H2O
(c) endothermic (d) fast (c) B2H6 (d) CH4
31. Stan dard Gibb’s free energy change for 39. Which of the following molecules acts as a Lewis
isomerization reaction [1995] acid ? [2009]
cis-2 pentene trans-2-pentene (a) (CH3)2 O (b) (CH3)3 P
is – 3.67 kJ/mol at 400 K. If more trans-2 pentene (c) (CH3)3 N (d) (CH3)3 B
is added to the reaction vessel, then
EBD_7324
êê ÝØÛÓ×ÍÌÎÇ
40. The ionization constant of ammonium hydroxide 46. Which one of the following orders of acid
is 1.77 × 10–5 at 298 K. Hydrolysis constant of strength is correct? [2003]
ammonium chloride is: [2009] (a) RCOOH > HC CH > HOH > ROH
(a) 6.50 × 10– 12 (b) 5.65 × 10–13 (b) RCOOH > ROH > HOH > HC CH
(c) 5.65 × 10–12 (d) 5.65 × 10–10 (c) RCOOH > HOH > ROH > HC CH
41. Equal volumes of three acid solutions of pH 3, 4 (d) RCOOH > HOH > HC CH > ROH
and 5 are mixed in a vessel. What will be the H+ 47. Which one of the following compounds is not a
ion concentration in the mixture ? [2008] protonic acid? [2003]
(a) 1.11 × 10–4 M (b) 3.7 × 10–4 M (a) SO2 (OH)2 (b) B (OH)3
(c) 3.7 × 10– 3 M (d) 1.11× 10–3 M (c) PO (OH)3 (d) SO (OH)2
42. Calculate the pOH of a solution at 25°C that 48. In HS–, I–, RNH2 and NH3, order of proton
contains 1× 10– 10 M of hydronium ions, i.e. accepting tendency will be [2001]
H3O+. [2007] (a) I– > NH3 > RNH2 > HS–
(a) 4.000 (b) 9.0000 (b) HS– > RNH2 > NH3 > I–
(c) 1.000 (d) 7.000 (c) RNH2 > NH3 > HS– > I–
43. The hydrogen ion concentration of a 10–8 M (d) NH3 > RNH2 > HS– > I–
HCl aqueous solution at 298 K (Kw = 10–14) is 49. A base when dissolved in water yields a solution
[2006] with a hydroxyl ion concentration of 0.05 mol litre–1.
(a) 11 × 10–8 M (b) 9.525 × 10–8 M The solution is [2000]
(c) 1.0 × 10–8 M (d) 1.0 × 10–6 M (a) basic (b) acidic
44. What is the correct relationship between the (c) neutral (d) either 'b' or 'c'
pHs of isomolar solutions of sodium oxide
50. Conjugate acid of NH 2 is : [2000]
(pH 1 ), sodiu m sulph ide (p H2 ), sodium
selenide (pH3) and sodium telluride (pH4)? (a) NH4+ (b) NH3
[2005] (c) NH2 (d) NH
(a) pH1 > pH2 > pH3 > pH4 51. Which of the following statements about pH
(b) pH1 > pH2 pH3 > pH4 and H+ ion concentration is incorrect? [2000]
(c) pH1 < pH2 < pH3 < pH4 (a) Addition of one drop of concentrated HCl in
(d) pH1 < pH2 < pH3 pH4 NH4OH solution decreases pH of the solution.
45. The rapid change of pH near the stoichiometric (b) A solution of the mixture of one equivalent of
point of an acid-base titration is the basis of each of CH3COOH and NaOH has a pH of 7
indicator detection. pH of the solution is related (c) pH of pure neutral water is not zero
to ratio of the concentrations of the conjugate (d) A cold and concentrated H2SO4 has lower
acid (HIn) and base (In –) forms of the indicator H+ ion concentration than a dilute solution
by the expression [2004] of H2SO4
52. Among boron trifluoride, stannic chloride and
[ In ] stannous chloride, Lewis acid is represented by
(a) log pK In pH
[ HIn ] [1999]
(a) only stannic chloride
[ HIn ]
(b) log pK In pH (b) boron trifluoride and stannic chloride
[ In ] (c) boron trifluoride and stannous chloride
(d) only boron trifluoride
[ HIn ] 53. What is the H+ ion concentration of a solution
(c) log pH pK In
[ In ] prepared by dissolving 4 g of NaOH (Atomic
weight of Na = 23 amu) in 1000 ml? [1999]
[ In ] (a) 10–10 M (b) 10–4 M
(d) log pH pK In
[ HIn ] (c) 10–1 M (d) 10–13 M
Û¯«·´·¾®·«³ êé

54. The pH value of a 10 M solution of HCl is [1995] (a) 2.66 × 10–12 (b) 4.5 × 10–11
(a) less than 0 (b) equal to 0 (c) 5.3 × 10–12 (d) 2.42 × 10–8
(c) equal to 1 (d) equal to 2 62. MY and NY3, two nearly insoluble salts, have
̱°·½ ìæ ×±²·­¿¬·±² ±º É»¿µ ß½·¼­ ¿²¼ the same Ksp values of 6.2 × 10–13 at room
Þ¿­»­ ¿²¼ λ´¿¬·±² ¾»¬©»»² Õ ¿ ¿²¼ Õ ¾ temperature. Which statement would be true in
regard to MY and NY3 ? [2016]
55. At 100°C the Kw of water is 55 times its value at (a) The molar solubilities of MY and NY3 in
25°C. What will be the pH of neutral solution? water are identical.
(log 55 = 1.74) [NEET Kar. 2013] (b) The molar solubility of MY in water is less
(a) 6.13 (b) 7.00 than that of NY3
(c) 7.87 (d) 5.13 (c) The salts MY and NY3 are more soluble in
56. Accumulation of lactic acid (HC3H5 O3 ), a 0.5 M KY than in pure water.
monobasic acid in tissues leads to pain and a (d) The addition of the salt of KY to solution
feeling of fatigue. In a 0.10 M aqueous solution, of MY and NY3 will have no effect on their
lactic acid is 3.7% dissociated. The value of
solubilities.
dissociation constant, Ka, for this acid will be:
63. Consider the nitration of benzene using mixed
[NEET Kar. 2013]
conc of H2SO4 and HNO3. If a large amount of
(a) 2.8 × 10–4 (b) 1.4 × 10–5
(c) 1.4 × 10–4 (d) 3.7 × 10–4 KHSO4 is added to the mixture, the rate of
57. A weak acid, HA, has a Ka of 1.00 × 10–5. If nitration will be [2016]
0.100 mol of this acid is dissolved in one litre of (a) faster (b) slower
water, the percentage of acid dissociated at (c) unchanged (d) doubled
equilbrium is closest to [2007] 64. The Ksp of Ag2 CrO4, AgCl, AgBr and AgI
(a) 1.00% (b) 99.9% are respectively, 1.1 × 10–12, 1.8 × 10–10, 5.0 ×
(c) 0.100% (d) 99.0% 10–13, 8.3 × 10–17. Which one of the following
58. At 25 C, the dissociation constant of a base, salts will precipitate last if AgNO3 solution is
BOH, is 1.0 10 12. The concentration of added to the solution containing equal moles of
hydroxyl ions in 0.01 M aqueous solution of NaCl, NaBr, NaI and Na2CrO4? [2015]
the base would be [2005] (a) AgCl (b) AgBr
(a) 1.0 10 5 mol L 1 (b) 1.0 10 6 mol L 1 (c) Ag2CrO4 (d) AgI
(c) 2.0 10 6 mol L 1 (d) 1.0 10 7 mol L 1 65. Which one of the following pairs of solution is
59. Ionisation constant of CH3COOH is 1.7 × 10–5 if not an acidic buffer ? [2015 RS]
concentration of H+ ions is 3.4 × 10–4M, then (a) HClO4 and NaClO4
find out initial concentration of CH3COOH (b) CH3COOH and CH3 COONa
molecules [2001] (c) H2CO3 and Na2CO3
(a) 3.4 × 10–4M (b) 3.4 × 10–3M
(d) H3PO4 and Na3PO4
(c) 6.8 × 10–3M (d) 6.8 × 10–4M
66. Using the Gibbs energy change, G° = + 63.3kJ,
60. Aqueous solution of acetic acid contains [1991]
for the following reaction, [2014]
(a) CH3COO– and H+
(b) CH3COO–, H3O+ and CH3COOH Ag2CO3 2Ag+ (aq) + CO32–(aq)
(c) CH3COO–, H3O+and H+ the Ksp of Ag2CO3(s) in water at 25°C is:-
(d) CH3COOH, CH3COO– and H+ (R = 8.314 J K–1 mol–1)
(a) 3.2 × 10–26 (b) 8.0 × 10–12
̱°·½ ëæ ݱ³³±² ×±² Ûºº»½¬ô Í¿´¬ ا¼®±´§­·­ô –3
(c) 2.9 × 10 (d) 7.9 × 10–2
Þ«ºº»® ͱ´«¬·±²­ ¿²¼ ͱ´«¾·´·¬§ Ю±¼«½¬
67. Identify the correct order of solubility in aqueous
61. Concentration of the Ag+ ions in a saturated medium: [NEET 2013]
solution of Ag2C2O4 is 2.2 × 10–4 mol L–1. (a) ZnS > Na2S > CuS (b) Na2S > CuS > ZnS
Solubility product of Ag2C2O4 is :- [2017] (c) Na2S > ZnS > CuS (d) CuS > ZnS > Na2S
EBD_7324
êè ÝØÛÓ×ÍÌÎÇ
68. The values of Ksp of CaCO3 and CaC2O4 are (b) [Ag+] = 1.8 × 10–11 M ; [Pb2+] = 8.5 × 10–5 M
4.7 × 10–9 and 1.3 × 10–9 respectively at 25°C. If (c) [Ag+] = 1.8 × 10–9 M ; [Pb2+] = 1.7 × 10–3 M
the mixture of these two is washed with water, (d) [Ag+] = 1.8 × 10–11 M ; [Pb2+] = 8.5 × 10–4 M
what is the concentration of Ca2+ ions in water? 75. If pH of a saturated solution of Ba (OH) 2 is 12,
[NEET Kar. 2013]
(a) 7.746 × 10–5 M (b) 5.831 × 10–5 M the value of its K (sp) is : [2010]
(c) 6.856 × 10–5 M (d) 3.606 × 10–5 M (a) 4.00 × 10 6
M 3
(b) 4.00 × 10–7 M3
69. The dissociation constant of a weak acid is
6 3 7
1 × 10– 4. In order to prepare a buffer solution (c) 5.00 × 10 M (d) 5.00 × 10 M3
with a pH = 5 the [Salt]/[Acid] ratio should be 76. What is [H ] in mol/L of a solution that is 0.20
[NEET Kar. 2013]
(a) 1 : 10 (b) 4 : 5 M in CH 3COONa and 0.10 M in CH3 COOH ?
(c) 10 : 1 (d) 5 : 4 5
Ka for CH3 COOH = 1.8 × 10 . [2010]
70. pH of a saturated solution of Ba(OH)2 is 12.
The value of solubility product (KSP)of Ba(OH)2 4 5
(a) 3.5 × 10 (b) 1.1 × 10
is : [2012] 5
(a) 3.3 × 10– 7 (b) 5.0 × 10–7 (c) 1.8 × 10 (d) 9.0 × 10 6
(c) 4.0 × 10–6 (d) 5.0 × 10–6 77. In a buffer solution containing equal
concentration of B– and HB, the Kb for B–
71. Equimolar solutions of the following substances
is 10–10. The pH of buffer solution is : [2010]
were prepared separately. Which one of these (a) 10 (b) 7
will record the highest pH value ? [2012] (c) 6 (d) 4
(a) BaCl2 (b) AlCl3 78. Equimolar solutions of the following were
(c) LiCl (d) BeCl2 prepared in water separately. Which one of the
72. Buffer solutions have constant acidity and solutions will record the highest pH ? [2008]
alkalinity because [2012] (a) SrCl2 (b) BaCl2
(a) these give unionised acid or base on (c) MgCl2 (d) CaCl2
reaction with added acid or alkali. 79. Which of the following pairs constitutes a
(b) acids and alkalies in these solutions are buffer? [2006]
shielded from attack by other ions. (a) NaOH and NaCl
(c) they have large excess of H+ or OH– ions (b) HNO3 and NH4NO3
(d) they have fixed value of pH (c) HCl and KCl
73. A buffer solution is prepared in which the (d) HNO2 and NaNO2
concentration of NH 3 is 0.30M and the 80. H2S gas when passed through a solution of
concentration of NH4 + is 0.20 M. If the cations contain in g HCl precipitates th e
equilibrium constant, K b for NH3 equals cati on s of second group of qualitative
1.8 × 10–5, what is the pH of this solution ? analysis but not those belonging to the fourth
group. It is because [2005]
(log 2.7 = 0.433). [2011]
(a) presence of HCl decreases the sulphide
(a) 9.08 (b) 9.43
ion concentration.
(c) 11.72 (d) 8.73 (b) solubility product of group II sulphides
74. In qualitative analysis, the metals of Group I can is more than that of group IV sulphides.
be separated from other ions by precipitating (c) presence of HCl increases the sulphide
them as chloride salts. A solution initially ion concentration.
contains Ag+ and Pb2+ at a concentration of (d) sulph ides of group IV cations a r e
0.10 M. Aqueous HCl is added to this solution unstable in HCl.
until the Cl– concentration is 0.10 M. What will 81. The solubility product of a sparingly soluble
the concentrations of Ag+ and Pb2+ be at 11 . Its solubility ( in moles/
salt AX 2 is 3.2 10
equilibrium?
(Ksp for AgCl = 1.8 × 10–10, litre) is [2004]
Ksp for PbCl2 = 1.7 × 10–5) [2011M] (a) 5.6 10 6 (b) 3.1 10 4
(a) [Ag+] = 1.8 × 10–7 M ; [Pb2+] = 1.7 × 10–6 M
Û¯«·´·¾®·«³ êç
89. A physician wishes to prepare a buffer solution
(c) 2 10 4 (d) 4 10 4
of pH = 3.58 that efficiently resists a change in
82. The solubility product of AgI at 25ºC is pH yet contains only small concentrations of
1.0 × 10–16 mol 2 L–2. The solubiliy of AgI in the buffering agents. Which one of the following
10–4 N solution of KI at 25ºC is approximately weak acids together with its sodium salt would
(in mol L–1 ) [2003] be the best to use ? [1997]
(a) 1.0 × 10–8 (b) 1.0 × 10–16 (a) m-chlorobenzoic acid (pKa = 3.98)
(c) 1.0 × 10–12 (d) 1.0 × 10–10 (b) p-chlorocinnamic acid (pKa = 4.41)
83. Solubility of MX2-type eletrolytes is 0.5 × 10–4 (c) 2, 5-dihydroxy benzoic acid (pKa = 2.97)
mole/lit, then find out Ksp of electrolytes [2002] (d) Acetoacetic acid (pKa = 3.58)
90. The pH value of blood does not appreciably
(a) 5 × 10–12 (b) 25 × 10–10
change by a small addition of an acid or a base,
(c) 1 × 10–13 (d) 5 × 10–13 because the blood [1995]
84. Which has the highest value of pH? [2002] (a) is a body fluid
(a) CH3COOK (b) Na2CO3 (b) can be easily coagulated
(c) NH4Cl (d) NaNO3 (c) contains iron as a part of the molecule
85. Solution of 0.1 N NH4OH and 0.1 N NH4Cl has (d) contains serum protein which acts as buffer
pH 9.25. Then find out pKb of NH4OH [2002] 91. Which of the following is most soluble ? [1994]
(a) 9.25 (b) 4.75 (a) Bi2 S3 (Ksp= 1 × 10–17)
(c) 3.75 (d) 8.25 (b) MnS (Ksp= 7 × 10–16)
86. Solubility of a M2S salt is 3.5 × 10–6, then its (c) CuS (Ksp= 8 × 10–37)
solubility product will be [2001] (d) Ag2 S (Ksp= 6 × 10–51).
92. 0.1 M solution of which one of these substances
(a) 1.7 × 10–16 (b) 1.7 × 10–6
will be basic ? [1992]
(c) 1.7 × 10–18 (d) 1.7 × 10–12
(a) Sodium borate
87. The solubility product of a sparingly soluble (b) Ammonium chloride
salt BA2 is 4 × 10–12. The solubility of BA2 is (c) Calcium nitrate
[1999] (d) Sodium sulphate.
(a) 4 × 10–4 (b) 4 × 10–12 93. In which of the following solvents, AgBr will
(c) 4 × 10–3 (d) 1 × 10–4 have the highest solubility ? [1992]
88. The solubility products of CuS, Ag2S and HgS 3
(a) 10 M NaBr 3
(b) 10 M NH 4 OH
are 10 –31, 10 –44 , 10 –54 respectively. The
(c) Pure water (d) 10 3 M HBr
solubilities of these sulphides are in the order
[1997] 94. The compound whose aqueous solution has the
(a) Ag2S > HgS > CuS (b) Ag2S > CuS > HgS highest pH is [1988]
(a) NaCl (b) NaHCO3
(c) HgS > Ag2S > CuS (d) CuS > Ag2S > HgS
(c) Na2CO3 (d) NH4Cl.

ANSWER KEY
1 (a) 11 (b) 21 (d) 31 (a) 41 (b) 51 (b) 61 (c) 71 (a) 81 (c) 91 (b)
2 (d) 12 (c) 22 (d) 32 (d) 42 (a) 52 (c) 62 (b) 72 (a) 82 (c) 92 (a)
3 (b) 13 (b) 23 (b) 33 (d) 43 (a) 53 (d) 63 (b) 73 (b) 83 (d) 93 (b)
4 (a) 14 (d) 24 (d) 34 (a) 44 (a) 54 (a) 64 (c) 74 (c) 84 (b) 94 (c)
5 (c) 15 (a) 25 (a) 35 (c) 45 (d) 55 (a) 65 (a) 75 (d) 85 (b)
6 (c) 16 (a) 26 (c) 36 (b) 46 (c) 56 (c) 66 (b) 76 (d) 86 (a)
7 (c) 17 (a) 27 (d) 37 (c) 47 (b) 57 (a) 67 (c) 77 (d) 87 (d)
8 (d) 18 (b) 28 (c) 38 (c) 48 (c) 58 (d) 68 (a) 78 (b) 88 (d)
9 (c) 19 (d) 29 (d) 39 (d) 49 (a) 59 (c) 69 (c) 79 (d) 89 (d)
10 (c) 20 (b) 30 (b) 40 (d) 50 (b) 60 (b) 70 (b) 80 (a) 90 (d)
EBD_7324
éð ÝØÛÓ×ÍÌÎÇ

Hints & Solutions


1 1
1. (a) N2 õ O2 NO
2 2
2
NH3 [NO]
(i) N 2 3H 2 2NH3 ; K1 K~ ã 12 12
ã K1 2
3 [N 2 ] [O 2 ]
N2 H2
2 5. (c) 2SO2 O 2 2SO3 K = 278 (given)
NO
(ii) N 2 O 2 2NO; K 2
N2 O2 1 1
SO3 SO 2 O2 K'
1 H2 O 2 K
(iii) H 2 O2 H 2 O; K 3 1/2
2 H2 O2 1
Applying (II + 3 × III – I) we will get = = 6 × 10–2
278
5 K
2NH 3 O2 2NO 3H 2O; [AB]2
2 6. (c) A2 + B2 2AB ; K c
3 2
[A2 ][B2 ]
2
NO H2O NH3
K= 3 3/2 3 (2.8 10 3 ) 2 (2.8) 2
N2 O 2 H2 O2 N2 H2 Kc 0.62
3 3 3 4.2
3 10 4.2 10
K = K2 × K33 / K1 7. (c) For the reaction
2. (d) Max. pressure of CO2 = Pressure of CO2 at N 2 (g) O 2 (g) 2NO(g)
equilibrium
[NO2 ]2
For reaction, K1 =
SrCO3(s) [N 2 ][O 2 ]
SrO(s) + CO2
For the reaction
Kp = PCO2 = 1.6 atm = maximum pressure of CO2
volume of container at this stage. 2NO(g) O 2 (g) 2NO 2 (g)
nRT
V= …(i) [NO 2 ]2
P K2 =
Since container is sealed and reaction was not [NO]2 [O2 ]
earlier at equilibrium. For the reaction
n = constant. 1
NO 2 (g) N 2 (g) O 2 (g)
PV 0.4 20 2
n= …(ii)
RT RT 1
Put equation (ii) in equation (i) [N2 ] 2 [O2 ]
0.4 20 RT K=
V= 5L [NO2 ]
RT 1.6
3. (b) Equilibrium constant for reaction: 1 1
Hence, K = K1 K2
[Product]
K = 1.6 × 1012 = 1 1 1
[Reactant]
The value of K is very high so the system will [N2 ] 2 [O2 ] 2 [NO][O2 ] 2
K=
contain mostly products at equilibrium. [NO] [NO2 ]
4. (a) N2(g) + O2(g) 2NO(g) 1
2 [N2 ] 2 [O2 ]
[NO] =
Kã [NO2 ]
[N2 ][O 2 ]
Û¯«·´·¾®·«³ éï

8. (d) 2C(s) +O2(g) 2 CO 2(g) Thus, we get


PAB = (1 – x) × P PAB = x × P
n =2–1=+1 2
x
Kc and Kp are not equal. PB P
2 2
9. (c) 2A(g) + B(g) 3C(g) + D(g) x
PAB
2
PB2 (x) 2 P 2 .P
Mole ratio 2 1 3 1 2
Now, K P =
Molar concentration 1 1 0 0 2
(1 x) 2 P2
PAB2
at t = 0
Molar 0.50 0.75 0.75 0.25 x 3 .P3
= [ 1–x å 1]
concentration at equilibrium 2 1 P2
1
[C ]3[ D ] (0.75) 3 (0.25) x 3 .P 2.K p 2K p 3
Kc = or x3 = or x =
[ A]2 [ B ] (0.50) 2 (0.75) 2 P P
10. (c) For this reaction Keq. is given by 12. (c) Given reaction are
3 X Y Z ..... (i)
Fe3 OH
K and A 2B ......(ii)
Fe(OH) 3 Let the total pressure for reaction (i) and (ii) be
P1 and P2 respectively, then
= (Fe3+) (OH–)3 [ [solid]=1].
KP 9
1
1 (given)
If (OH–) is decreased by times then for reaction KP2 1
4
After dissociation,
equilibrium constant to remain constant, we
have to increase the concentration of [Fe3+] by
X Y Z
a factor of 43 i.e 4× 4 × 4 = 64. Thus option (c) is At equilibrium (1– )
correct answer. [Let 1 mole of X dissociate with as degree of
dissociation]
11. (b) For the reaction
Total number of moles = 1– + +
2AB2 (g) 2AB(g) B 2 (g) = (1+ )
initially 2 0 0
at equi 2(1 x ) 2x x 1
Thus PX = . P1 ; PY = P1;
2 1 1
AB B2 (2x) 2 x
Kc or K c
AB2
2
{2(1 x)}2 PZ = .P1
1
= x3 [(1–x) can be neglected in denominator
(1– x) å 1] KP .P1 .
The partial pressure at equilibrium are calculated
1 1 1
on the basis of total number of moles at 1
P1 / .P1 ....... (i)
equilibrium. 1
Total number of moles
Similarly for A 2B
= 2 (1–x) + 2x + x = (2 + x)
At equilibrium (1– ) 2
2(1 x) We have,
PAB = P where P is the total
2 (2 x) 2 P2
2
1
pressure. K P2 / P2 ........(ii)
1 1
2x x Dividing (i) by (ii), we get
PAB = P , PB P
(2 x) 2 (2 x) KP1 2
.P1 KP 1 P1
or 1 .
Since x is very small so can be neglected in KP 2
4 . P2 KP 4 P2
denominator 2 2
EBD_7324
éî ÝØÛÓ×ÍÌÎÇ

1 P1 KP1 9 K 2 . K33
or 9 . Now operate,
4 P2 K P2 1 K1
P1 36 [NO]2 [H 2 O]3 [N ][H2 ]3
or or P1 : P2 = 36 : 1 . 2
P2 1 3
[N2 ] [O2 ] [H 2 ] [O2 ] 3 / 2
[NH 3 ]2
13. (b) Given : Equilibrium constant (K1) for the
reaction [NO]2 [H 2 O]3
K
K1 1 1 [NH3 ]2 [O2 ]5 / 2
HI(g) H 2 (g) I 2 (g); K1 = 8; … (i)
2 2
To find equilibrium constant for the following K 2 . K 33
K
reaction K1
H 2 (g) I 2 (g) 2HI(g); K2 = ? .....(ii) 15. (a) First option is incorrect as the value of KP
multiply (i) by 2, we get given is wrong. It should have been
2HI(g) H 2 (g) I 2 (g); PCO2
KP
K1 = 82 = 64. … (iii) PCH4 [PO2 ]2
[ Note: When the equation for an equilibrium is 16. (a) For reaction (i)
multiplied by a factor, the equilibrium constant
must be raised to the power equal to the factor] [NO]2
K1 ã
Now reverse equation (iii), we get [N2 ][O2 ]
1 and for reaction (ii)
H 2 (g) I 2 (g) 2HI(g); K .....(iv)
64 [N2 ]½ [O2 ]½ therefore 1
[Note: For a reversible reaction, the equilibrium K2 ã K1 ã
constant of the backward reaction is inverse of [NO] K 22
the equilibrium constant for the forward 17. (a) MgCO3 (s) MgO(s) CO 2 (g )
reaction.]
Equation (iv) is the same as the required equation MgO & MgCO3 are solid and they do not exert
any pressure and hence only pressure exerted
1
(ii), thus K2 for equation (ii) is i.e. option (b) is by CO2. Therefore KP = PCO2
64 18. (b) In polyprotic acids the loss of second
is correct.
proton occurs much less readily than the first.
14. (d) Given,
Usually the Ka values for successive loss of
N 2 3H 2 2NH 3 ; K1 ....(i) protons from these acids differ by at least a factor
N2 O2 2NO; K 2 ....(ii) of 10–3 i.e., K a1 K a2
1
H2 O2 H 2 O; K 3 ....(iii) H2 X Hõ õ HXó øK a1 ÷
2
We have to calculate
HXó H õ õ X 2ó K a 2ø ÷
4NH3 5O 2 4NO 6H 2O; K ?
5 19. (d) For the reaction
or 2NH3 O2 2NO 3H 2 O
2 XeF6(g ) H 2 O ( g) XeOF4 ( g) 2HF(g )
2 3
[NO] [H 2 O]
For this equation, K [XeOF4 ][HF]2
K1 ....(a)
[NH3 ]2 [O 2 ]5 / 2 [XeF6 ][ H 2 O]
[NH3 ] 2
[NO] 2 and for the reaction
but K1 3
, K2
[N2 ] [H2 ] [N 2 ] [O 2 ] XeO 4( g) XeF6 (g ) XeOF4 (g ) XeO3 F2 (g )
[H 2O] [H 2O]3 [XeOF4 ][ XeO 3F2 ]
& K3 or K 3 K2
½ ....(b)
[H 2 ] [O 2 ] [H 2 ]3 [O 2 ]3 / 2 [ XeO 4 ][XeF6 ]
Û¯«·´·¾®·«³ éí

For reaction :
1.5×10 –5 1
XeO 4 (g ) 2 HF(g) XeO3F2( g ) H 2 O(g) = 105 3.33 104
4.5×10 –10 3
[ XeO3F2 ][H 2O] 27. (d) For reaction to proceed from right to left
K
[XeO 4 ][HF] 2 Q Kc
backward forward i.e the reaction
From eq. no. (a) and (b) rate rate
K K 2 / K1 will be fast in backward direction i.e r b > r f.
20. (b) According to equation 28. (c) For the reaction
BaO 2(s) BaO (s) õ O 2 (g); H = +ve.
2HI H2 + I2
At t = 0 (2 moles) 0 0 At equilibrium K p ã PO2
At equilibrium (2 – 2 ) moles mole mole
Total moles at equilibrium = 2 – 2 + + = 2 [For solid and liquids concentration term is taken
mole as unity]
21. (d) Rate constant of forward reaction (K f) = Hence, the value of equilibrium constant
1.1 × 10–2 and rate constant of backward reaction depends only upon partial pressure of O2 .
(Kb) = 1.5 × 10–3 per minute. Equilibrium constant Further on increasing temperature formation of
(Kc) O2 increases as this is an endothermic reaction.
Kf 1.1 10 2 29. (d) A 2 (g) B 2(g ) 3C(g) D(g)
7.33 step-1 step-2
Kb 3
1.5 10
since the steps 1 and 2 are exothermic hence low
22. (d) Clausius – Clapeyron's equation temprature will favour both the reactions. In step
d ln P Hv - 1 moles are increasing hence low pressure will
dT RT2 favour it. In step 2 moles are decreasing, hence
23. (b) G° = –2.30RTlog K high pressure will favour it.
because at equilibrium G = 0 30. (b) A2 2A Equilibrium constant is given
24. (d) Given reaction is exothermic reaction. Hence by
according to Le-Chatelier's principle low [A]2
temperature favours the forward reaction and Kc
on increasing pressure equilibrium will shift, [A 2 ]
towards lesser number of moles i.e. forward Since the value given is very small, hence conc.
direction. of products is less. It means the reaction is slow.
31. (a) If more trans-2-pentene is added, then its
25. (a) In exothermic reactions on increasing
concentration in right hand side will increase.
temperature value of Kp decreases
But in order to maintain the K constant,
Kp > Kp (Assuming T 1 < T2)
concentration of cis-2-pentene will also increase.
26. (c) Given, CH3COOH CH3COO– + H+ ; Therefore more cis-2-pentene will be formed.
Ka1 , = 1.5 × 10– 5 ....(i) 32. (d) Solid Liquid
It is an endoth er mic pr ocess. So when
HCN H++ CN–; K a 2 = 4.5 × 10–10
temperature is raised, more liquid is formed.
or H+ + CN– HCN; Hence adding heat will shift the equilbrium in
1 1 the forward direction.
K 'a 2 ...(ii) 33. (d) According to Le-chatelier's principle
K a 2 4.5 10 –10
whenever a constraint is applied to a system in
From (i) and (ii), we find that the equilibrium
equilibrium, the system tends to readjust so as
constant (Ka) for the reaction ,
to nullify the effect of the constraint.
CN– + CH3COOH CH3COO– + HCN, is
34. (a) gm eq of NaOH = 0.1 × V = 0.1V
Ka K a1 K a' 2 gm eq of HCl = 0.01 × V = 0.01V
EBD_7324
éì ÝØÛÓ×ÍÌÎÇ
gm eq of NaOH > gm eq. HCl = 0.00037 M = 3.7 ×10–4 M.
hence resultant solution should be basic, hence 42. (a) Given [H3O+] = 1 × 10–10 M
from the eqn at 25º [H3O+] [OH–] = 10–14
M1V1 – M2V2 = MV 14
10 4
0.1V – 0.01V = MV [OH ] 10
10
0.09 10
M= = 0.045 = 4.5 × 10–2
2 Now, [OH ] 10
pOH
= 10–4
Now pOH = – log [OH–]
pOH = 4
= – log 4.5 × 10–2 = 1.34
43. (a) For a solution of 10–8 M HCl [H+] = 10–8
pH + pOH = 14
[H+] of water = 10–7
pH = 14 – 1.34 = 12.65 Total [H+] = 10–7 + 10–8 = 10 × 10–8 + 10–8
35. (c) Na2CO3 is a salt of strong base (NaOH) and 10–8 (10 + 1) = 11 × 10–8
weak acid (H2CO3). On hydrolysis this salt will 44. (a) The solution formed from isomolar solutions
produce strongly basic solution. i.e. pH will be of sodium oxide, sodium sulphide, sodium se-
highest (pH > 7) for this sotluion. Others are lenide H2O, H2S, H2Se & H2Te respectively. As
combination of the acidic strengh increases from H2O to H2Te
KCl = Strong acid + Strong base thus pH decreases and hence the correct of pHs
neutral solution (pH 7) is
NaCl = Strong acid + Strong base pH1 > pH2 > pH3 > pH4.
neutral solution (pH 7) 45. (d) For an acid-base indicator
CuSO4 = Strong acid + wake base HIn H In
Acidic solution (pH < 7) [H ][In ]
36. (b) BF3 acts as Lewis acid. K In [HIn ]
or [H ] K In
[HIn ] [In ]
37. (c) BF3 behaves as lewis acid as it is an electron
deficient species. [HIn ]
or log H log K In log
38. (c) Boron in B2H6 is electron deficient. [In ]
39. (d) (CH3)3 B – is an electron deficient, thus Taking negative on both sides
behave as a lewis acid. [HIn]
40. (d) Ammonium chloride is a salt of weak base ó log[H õ ] ã ó log K In ó log ó
and strong acid. In this case hydrolysis constant [In ]
Kh can be calculated as [In ]
or we can write pH pK In log
[HIn ]
Kw 1 10 14 [In ó ]
Kh 5.65 10 10 or log ã pH ó pK In
Kb 1.77 10 5 [HIn]
41. (b) [H3O]+ for a solution having pH = 3 is given by
[H3O]+ = 1×10–3 moles/litre [ [H3O]+ = 10–pH] 46. (c) The higher is the tendency to donate
Similarly for solution having pH = 4, proton, stronger is the acid. Thus the correct
[H3O]+ = 1 × 10–4 moles/ litre and for pH=5 order is R – COOH > HOH > R – OH > CH CH
[H3O+] = 1×10–5 moles/ litre depending upon the rate of donation of proton.
Let the volume of each solution in mixture be L1, 47. (b) B(OH)3 does not provide H+ ions in water
then total volume of mixture solution instead it accepts OH– ion and hence it is Lewis
acid
L = (1 + 1 + 1) L =3L
Total [H3O]+ ion present in mixture solution = B(OH )3 H 2O [B(OH ) 4 ] H
(10–3 + 10–4 + 10–5) moles 48. (c) Strong base has higher tendency to accept
Then [H3O] + ion concentration of mixture the proton. Increasing order of base and hence
solution the order of accepting tendency of proton is
3
10 10 4 10 5
0.00111 I HS NH3 RNH 2
= M= M
3 3
Û¯«·´·¾®·«³ éë

49. (a) Given : Hydroxyl ion concentration At 25ºC


[OH–] = 0.05 mol L–1. We know that the Kw = [H+] [OH–] = 10–14
14 At 100°C (given)
[ H ][OH ] 1 10 Kw = [H+] [OH–] = 55 × 10 –14
14 for a neutral solution
or [H ] 1 10
–1
2 10 13 mol L
0.05 [H+] = [OH–]
We also know that [H+]2 = 55 × 10–14
13 or [H+] = (55 × 10–14)1/2
pH log[H ] log[2 10 ]
pH = – log [H+]
13
= log 2 log10 log 2 ( 13) log10 On taking log on both side
0.3010 13.0000 12.6990. – log [H+] = –log (55 × 10–14)1/2
Since the value of pH > 7, therefore the solution 1
pH = log 55 14log10
is basic. 2
pH = 6.13
50. (b) BecauseNH3 after losing a proton (H+) gives Ka Ka
NH2– 56. (c) = 0.037 ;
c 0.10
NH 3 H 2 O NH 2– H 3O
(Conjugate acid-base pair differ only by a proton) Ka (0.037)2 0.10 = 1.37 ×10–4
å 1.4 × 10–4
51. (b) CH3COOH is weak acid while NaOH is 57. (a) Given Ka = 1.00×10–5, C= 0.100 mol
strong base, so one equivalent of NaOH can for a weak electrolyte,
not be neutralized with one equivalent of degree of dissociation
CH3COOH. Hence the solution of one equivalent
K a 1 10 –5
of each does not have pH value as 7. Its pH will be = 10 2 1%
towards basic side as NaOH is a strong base C 0.100
58. (d) Given Kb = 1.0 × 10–12
hence conc. of OH– will be more than the conc.
[BOH] = 0.01 M [OH] = ?
of H+.
52. (c) Lewis acid is that compound which have BOH Bõ õ OHó
electron deficiency. eg. BF3, SnCl2. t=0 c 0 0
teq c(1–x) cx cx
4 2 2 2
53. (d) No. of moles of NaOH = 0.1 c x cx
40 Kb
c(1 x) (1 x )
[Molecular weight of NaOH = 40] 2
0.01x
No. of moles of OH– = 0.1
12
10. 10
0.01(1 x)
0.1 On calculation, we get, x = 1.0 × 10–5
Concentration of OH– = 1 litre 0.1Mole / L
Now [OH–] =cx
As we know that, [H ] [OH ] 10 14 = 0.01 × 10–5
= 1 × 10–7mol L–1
13
[H ] 10 – +
59. (c) CH3 COOH CH3 COO + H
– –1
( OH 10 )
54. (a) Molarity (M) = 10M. HCl is a strong acid [CH 3COO ][H ]
Ka
and it is completely dissociated in aqueous [CH 3COOH ]
solutions as : HCl (10) H+(10) + Cl–.
Given that,
So, for every moles of HCl, there is one H+.
Therefore [H+] = [HCl] or [H+] = 10. [CH 3 COO ] [ H ] 3.4 10 4
M
pH = – log[H+] = – log [10] = – 1.
Ka for CH3COOH = 1.7 × 10–5
55. (a) Kw at 25°C = 1 × 10–14
EBD_7324
éê ÝØÛÓ×ÍÌÎÇ
CH3COOH is weak acid, so in it [CH3COOH] is AgCl
equal to initial concentration. Hence Ksp = [Ag+] [Cl–] = 1.8 × 10–10
(3.4 10 4 )(3.4 10 4 ) 1.8 10 –10
1.7 10 5 [Ag+] =
[CH 3 COOH] [Cl – ]
3.4 10 4
3.4 10 4 AgBr
[ CH 3COOH ] Ksp = [Ag+] [Br –] = 5.0 × 10–13
1.7 10 5 5.3 10 –13
= 6.8 × 10–3M [Ag ]
60. (b) An aqueous solution of acetic acid [Br – ]
AgI
dissociates as Ksp = [Ag+] [I–] = 8.3 × 10–17
CH3COOH + H2O CH3COO– + H3O+ 8.3 10–17
[Ag ]
61. (c) Ag 2C 2 O4 (s) 2Ag aq C 2O 42 aq [I – ]
2s s –2
If we take Cr2O 4 [Cl – ] [Br – ] [I – ] 1
KSP = [Ag+]2 [C2O42–]
than maximum [Ag+] will be required in case of
[Ag+] = 2.2 × 10–4 M
Given that: Ag2CrO4.
Concentration of C2O42– ions, 65. (a) Among the given acids, HClO4 is a very
4
strong acid, completely dissociates
2.2 10 [HA] 0 in this case, hence cannot be used
C 2 O24 M
2 for acidic buffer.
1.1 10 4 M 66. (b) G = – 2.303 log K
KSP = (2.2 × 10–4)2 (1.1 × 10–4) here K = [Ag+]2 [CO3– –] = Ksp
= 5.324 × 10–12 63.3 × 10– 3 = – 2.303 × 8.314
62. (b) MY M+ + Y– × 298 log Ksp
KSP = s2 = 6.2 × 10–13 63.3 10 3
log Ksp = 11.09
s = 6.2 10 13 5705.8
Ksp = Antilog (– 11.09) = 8 × 10– 12
s = 7.87 × 10–7 mol L–1
67. (c) Solubility of alkali metal is maximum among
NY3 N3+ + 3Y– the following. Among ZnS (1.7 × 10–5) & CuS
K = s × (3s)3 = 27s4 = 6.2 × 10–13
SP (8 × 10–37) ZnS has higher value of K sp.
1/4 68. (a) CaCO3 Ca 2+ CO32
6.2 10 13 x
s= x
27 2+
CaC2O4 Ca C 2O 42
s = 3.89 × 10–4 mol L–1 y y
molar solubility of NY3 is more than MY in [Ca2+] = x + y
water.
63. (b) The presence of large amount of KHSO4 Now, Ksp (CaCO3) = [Ca2+] [CO32-]
will decrease ionisation of H2SO4 that result in or 4.7 × 10 –9 = (x + y) x
lesser ionisation of nitric acid and lesser similarly, Ksp (CaC2O4) = [Ca2+] [C2O42–]
formation of nitronium ion [NO2+]. Hence the or 1.3 × 10 –9 = (x + y) y
rate of nitration will be slower. On solving, we get
64. (c) Ag2CrO4 [Ca2+] = 7.746 × 10–5 M
Ksp = [Ag +]2 Cr2O 4–2 = 1.1 × 10–12 [Salt]
69. (c) pH = pKa + log
[Acid]
1.1 10 –12 [Salt]
[Ag ] 5 = 4 + log [ pKa = – log Ka]
[Acid]
Cr2 O4–2
Û¯«·´·¾®·«³ éé

Given, Ka = 1 × 10– 4 [salt]


pKa = – log (1× 10– 4) = 4 pOH = pK b log
[base]
Now from Handerson equation [pKb = –log Kb; pKb = –log 1.8 × 10–5]
[Salt] pKb = 4.74
pH = pKa + log
[Acid]
Putting the values 0.2
pOH 4.74 log
0.3
[Salt]
5 = 4 + log = 4.74 + 0.3010 – 0.4771 = 4.56
[Acid]
pH = 14 – 4.56 = 9.436
[Salt]
log = 5–4=1 74. (c) Ksp = [Ag+] [Cl–]
[Acid]
1.8 × 10–10 = [Ag+] [0.1]
Taking antilog
[Ag+] = 1.8 × 10–9 M
[Salt]/[Acid] = 10 = 10 : 1
Ksp = [Pb+2] [Cl–]2
70. (b) Given pH = 12
or [H+] = 10–12 1.7 × 10–5 = [Pb+2] [0.1]2
Since, [H+] [OH–] = 10–14 [Pb+2] = 1.7 × 10–3 M

10 14
75. (d) Ba (OH) 2(s) Ba 2(aq) + 2 OH (aq)
[OH–] = = 10–2 pH = 12 or pOH = 2
10 12
Ba(OH) 2 Ba 2+ + 2OH [OH ] = 10 2
M
s 2s Ba(OH) 2 Ba 2 + 2 OH–
[OH–] = 10–2 2 2
2s = 10–2 0.5× 10 10

10 2 [ Concentration of Ba 2 is half of OH ]
s 5 10 3 m
2 K sp = [Ba 2 ] [OH ]2
Ksp = 5 × 10–7
71. (a) (AlCl3, LiCl & BeCl2) ) all these solutions = [0.5 × 10 2 ] [1 × 10 2 ]2
6
are acidic due to cationic hydrolysis, whereas = 0.5 × 10 = 5 × 10 7 M3
BaCl2, is salt of strong base (Ba(OH)2) and strong Salt
acid (HCl), hence it will have maximum pH. 76. (d) pH p Ka+ log
Acid
72. (a) Lets take an example of an acidic buffer
CH3COOH and CH3COONa. log H log Ka – log Salt
Acid
CH3COOH CH3COO – + H +;
CH3COONa CH3COO– + Na+ log H = log Ka + log Acid
when few drops of HCl are added to this buffer, Salt
the H+ of HCl immediatly combine with Acid
H = Ka
CH3COO– ions to form undissociated acetic Salt
acid molecules. Thus there will be no H+ ions to
0.1
combine with CH 3 COO – ion s to form = 1.8 × 10 5 = 9 × 10 6
M
undissociated acetic acid molecules. Thus there 0.2
will be no appreciable change in its pH value. 77. (d) K b = 10 10
; Ka 10 4
or pK a = 4
Like wise if few drops of NaOH are added, the
OH – ions will combine with H+ ions to form For the buffer solution containing equal
unionised water molecule. Thus pH of solution concentration of B– and HB
will remain constant. pH = pK a + log 1
73. (b) Given : [NH3] = 0.3 M, [NH4+] = 0.2 M,
pH = pK a = 4
Kb = 1.8 × 10–5.
EBD_7324
éè ÝØÛÓ×ÍÌÎÇ
78. (b) The highest pH will be recorded by the most but pOH+ pH = 14 or pOH = 14 – pH
basic solution. The basic nature of hydroxides [Salt ]
of alkaline earth metals increase as we move from 14– pH – log pK b
[ Base]
Mg to Ba and thus the solution of BaCl2 in water
0.1
will be most basic and so it will have highest pH. 14 9.25 log pK b
79. (d) HNO2 is a weak acid and NaNO2 is salt of 0.1
that weak acid and strong base (NaOH). 14 – 9.25 – 0 = pKb
80. (a) IV th g r ou p n eeds h ig h er S 2– i on pKb = 4.75
concentration . In presen ce of HCl, the
86. (a) M 2S 2M õ õ S––
dissociation of H2S decreases hence produces 2s s
less amount of sulphide ions due to common
Solubility product = (2s)2 (s) = 4s3
ion effect, thus HCl decreases the solubility
= 4 (3.5 × 10–6)3 = 1.7 × 10–16
of H2S which is sufficient to precipitate IInd
group radicals. 87. (d) BA 2 B 2A
s 2s
81. (c) For AX 2 ; K sp 4s3 3.2 10 11
4s 3 Solubility product = [s] [2s]2 = 4s3

3.2 10 11 4l10ó12
or s 3 2 10 4 4 × 10–12 = 4s3 or s ã 3
4 4
82. (c) Ksp for AgI = 1 × 10–16 s = 10–4
88. (d) For Binary salts like CuS & HgS, solubilty,
In solution of KI, I– would be due to the both
AgI and KI, 10 –4 solution KI would provide = s K sp
10–4 I–
54
AgI would provide, say = x I– (x is solubility of sCuS 10 31 , s HgS 10
AgI)
Total I– = (10–4 + x) , Ksp of AgI = (10–4 + x)x For Ag 2S 2 Ag+ S2
2s s
Ksp = 10–4x + x2
as x is very small K sp 44
4s3 or SAg 2S 3 3 10
x2 can be ignored K sp
10–4 x = 10–16 4 4
10 16 The order is CuS > Ag2S > HgS
or x 10 12 (mol 1)
(solubility ) 10 4 [Salt]
89. (d) pH pK a log10
83. (d) Given s = 0.5 × 10–4 moles/lit [ Acid]
2+ For small concentration of buffering agent and
[MX2 M + 2X–]
[salt]
For MX2, Ksp = s × (2s)2 = 4s3 for maximum buffer capacity 1
Ksp = 4 × (0.5 × 10–4)3 = 4 × 0.125 ×10–12 [acid]
= 0.5 × 10–12 = 5 × 10–13 pH = pKa
84. (b) Na2CO3 is a salt of weak acid H2CO3 and 90. (d) The buffer system present in serum is
strong base NaOH, therefore, its aqueous H2CO3 + NaHCO3 and as we known that a buffer
solution will be basic hence has pH more than 7. solution resist the change in pH therefore pH
value of blood does not change by a small
Na 2 CO 3 2H 2 O 2NaOH 2H 2 CO 3 addition of an acid or a base.
strong base weak acid 91. (b) For Bi2S3 . Ksp– = (2s)2.(3s)3
[Salt] = 4s2. 27s3 = 108s5
85. (b) pOH pK b log
[Base]
K sp 1 10 –17
[salt] or s ã 5 5

or pKb = pOH–log 108 108


[Base] For MnS. Ksp = s2
Û¯«·´·¾®·«³ éç

or s ã Ksp ã 7l10ó16 94. (c) NaCl is a salt of strong acid and strong base
hence its aqueous solution will be neutral ie pH
for CuS s ã Ksp ã 8l10ó37 = 7. NaHCO3 is an acidic salt hence pH < 7 .
Na2CO3 is a salt of weak acid and strong base.
For Ag2S Ksp = 2s2. s = 4s3
Hence its aqueous solution will be strongly
K sp basic ie. pH > 7.
or s ã 3 NH4Cl is salt of weak base and strong acid, hence
4
thus MnS has maximum solubility. its aqueous solution will be strongly acidic i.e.
92. (a) Sodium borate is a salt of strong base pH < 7.
(NaOH) and weak acid (H3BO3 ). Hence its
aqueous solution will be basic
93. (b) AgBr has the highest solubility in
10 3 M NH 4 OH .AgBr is less soluble in NaBr
and HBr due to presence of common ion (Br)
EBD_7324
èð ÝØÛÓ×ÍÌÎÇ

ݸ¿°¬»®

8 Redox Reactions

̱°·½ ïæ Ѩ·¼¿¬·±² ¿²¼ λ¼«½¬·±² λ¿½¬·±²­ 6. A compound contains atoms of three elements
A, B and C. If the oxidation number of A is +2, B
1. Zn gives H2 gas with H2SO4 and HCl but not is +5, and that of C is –2, the possible formula of
with HNO3 because [2002] the compound is : [2000]
(a) Zn acts as an oxidising agent when it reacts
(a) A2(BC3)2 (b) A3(BC4)2
with HNO3
(c) A3(B4C)2 (d) ABC2
(b) HNO3 is weaker acid than H2SO4 and HCl
7. The oxidation number of phosphorus in pyro-
(c) In electrochemical series, Zn is above
phosphoric acid is [1999]
hydrogen
(a) +3 (b) +1 (c) +4 (d) +5
(d) NO 3 is reduced in preference to hydronium 8. The oxidation number of chromium in potassium
ion dichromate is [1995]
2. Which of the following involves a redox reaction? (a) + 6 (b) – 5 (c) – 2 (d) + 2
(a) Reaction of H2SO4 with NaOH [1997] 9. Phosphorus has the oxidation state of + 3 in
(b) Production of ozone from oxygen in the [1994]
atmosphere by lightning (a) Phosphorous acid
(c) Production of nitrogen oxides from nitrogen (b) Orthophosphoric acid
and oxygen in the atmosphere by lightning (c) Hypophosphorous acid
(d) Evaporation of water (d) Metaphosphoric acid.
3. The loss of electron is termed as [1995] 10. The oxidation state of Cr in is [1988]
(a) oxidation (b) reduction (a) + 5 (b) + 3
(c) combustion (d) neutralization (c) + 6 (d) + 7
̱°·½ îæ Ѩ·¼¿¬·±² Ò«³¾»® ̱°·½ íæ Ü·­°®±°±®¬·±²¿¬·±² ¿²¼ Þ¿´¿²½·²¹
±º λ¼±¨ λ¿½¬·±²­
4. Oxidation numbers of P in PO3– 2–
4 , of S in SO 4
11. A mixture of potassium chlorate, oxalic acid and
and that of Cr in Cr 2 O72– are respectively sulphuric acid is heated. During the reaction
[2009] which element undergoes maximum change in
(a) + 3, + 6 and + 5 (b) + 5, + 3 and + 6 the oxidation number ? [2012]
(c) – 3, + 6 and + 6 (d) + 5, + 6 and + 6 (a) S (b) H
5. The oxidation states of sulphur in the anions (c) Cl (d) C
SO32–, S2O42– and S2O62– follow the order [2003] 12. When Cl2 gas reacts with hot and concentrated
(a) S 2O 6 2 S2 O 4 2 SO 3 2 sodium hydroxide solution, the oxidation number
of chlorine changes from : [2012]
(b) S 2O 4 2 SO 3 2 S 2O 6 2
(a) zero to +1 and zero to –5
(c) SO32 S2 O4 2 S2O 6 2 (b) zero to –1 and zero to +5
(d) S 2O 4 2 (c) zero to –1 and zero to +3
S 2O 6 2 SO 3 2
(d) zero to +1 and zero to –3
λ¼±¨ λ¿½¬·±²­ èï

13. The following redox reaction is balanced by ̱°·½ ìæ Û´»½¬®±¼» ᬻ²¬·¿´ ¿²¼ Ѩ·¼·­·²¹ô
which set of coefficients ? [1999] λ¼«½·²¹ ß¹»²¬­
aZn bNO3 cH dNH 4 eH 2 O fZn 2
16. Standard reduction potentials of the half
a b c d e f
(a) 1 1 10 1 3 1 reactions are given below :
(b) 2 2 10 2 3 2 F2(g) + 2e– 2F– (aq); E° = + 2.85 V
(c) 4 2 10 1 3 4 Cl2(g) + 2e– 2Cl–(aq); E° = + 1.36 V
(d) 4 1 10 1 3 4 Br2(l) + 2e– 2Br–(aq); E° = + 1.06 V
14. In which of the following reactions, there is no I2(s) + 2e– 2I–(aq); E° = + 0.53 V
change in valency ? [1994] The strongest oxidising and reducing agents
(a) 4 KClO3 3KClO4 + KCl respectively are : [2012 M]
(b) SO2 + 2H2S 2H2O + 3S (a) F2 and I – (b) Br 2 and Cl–
(c) BaO2 + H2SO4 BaSO4 + H2O2 (c) Cl2 and Br – (d) Cl2 and I2
(d) 3 BaO + O2 2 BaO2. 17. The oxide, which cannot act as a reducing agent,
15. Which substance serves as a reducing agent in is [1995]
the following reaction ? [1994] (a) NO2 (b) SO2 (c) CO2 (d) ClO2
14H+ + Cr2 O 27 + 3Ni
2Cr3+ + 7H2O + 3Ni2+
(a) H2O (b) Ni
(c) H+ (d) Cr2 O72ó

ANS WER KEY


1 (d) 3 (a) 5 (c) 7 (d) 9 (a) 11 (c) 13 (d) 15 (b) 17 (c)
2 (c) 4 (d) 6 (b) 8 (a) 10 (c) 12 (b) 14 (c) 16 (a)
EBD_7324
èî ÝØÛÓ×ÍÌÎÇ

Hints & Solutions


1. (d) Zinc gives H2 gas with dil H2SO4/HCl but 8. (a) Let x = oxidation no. of Cr in K2Cr2O7.
not with HNO3 because in HNO3, NO3– ion is (2 × 1) + (2 × x) + 7 (– 2) = 0
reduced and give NH4NO3, N2O, NO and NO2
or 2 + 2x – 14 = 0 or x = + 6.
(based upon the concentration of HNO3)
9. (a) O.N. of P in H3PO3 (phosphorous acid)
[ Zn 2HNO 3 Zn ( NO 3 ) 2 2H] 4 3 × 1 + x + 3 × (– 2) = 0 or x = + 3
( nearly 6%)
In orthophosphoric acid (H3PO4) O.N. of P is + 5,
HNO3 + 8H NH3 + 3H2O in hypophosphorous acid (H3PO2) it is + 1 while in
NH3 + HNO3 NH4NO3 metaphosphoric acid (HPO3), it is + 5.
4Zn+10HNO3 4Zn(NO3)2+NH4NO3+3H2O 10. (c) = + 2 + 2x – 14 = 0 or x = + 6
Zn is on the top position of hydrogen in +5 +6 +6
electrochemical series. So Zn displaces H2 from 11. (c) KClO3 H 2C 2O 4 H 2SO 4 K 2SO 4
dilute H2SO4 and HCl with liberation of H2.
–1
Zn + H2SO4 ZnSO4 + H2 KCl CO 2 H 2O
i.e. maximum change in oxidation number is
2. (c) 2NaOH+H 2SO 4 Na 2SO 4 +2H 2O
(neutralization)
observed in Cl (+5 to –1).
0 0 12. (b) On reaction with hot and concentrated alkali
Light
3 O2 2 O3 (not redox reaction) a mixture of chloride and chlorate is formed
Light Hot
N 2 O2 2 N O (redox reaction) 3Cl2 + 3 NaOH(excess)
0 0 2 2
1 5
here oxidation of N2 & reduction of O2 is taking 5NaCl NaClO3 3H 2O
place
2
13. (d) Zn Zn 2e ....(1)
H 2 O (l ) H 2 O ( g ) (not redox reaction)
8e 10H NO 3 NH 4 3H 2O .....(2)
3. (a) Losing of electron is called oxidation.
operate eq. (1) × 4 + eq. (2) × 1
4. (d) PO3–
4 = x + 4 (– 2) = – 3; x – 8 = – 3; x = + 5 2
4Zn 10H NO3 4Zn NH 4 3H 2 O
SO 2–
4 = x + 4 (– 2) = – 2; x – 8 = – 2; x = + 6
2 1 1 6 2 2 6 2 1 1
14. (c) Ba O 2 H 2 S O 4 Ba S O 4 H 2 O 2
Cr 2 O72– = 2x + 7 (– 2) = – 2; 2x – 14 = – 2;
2x =12; x = + 6 In this reaction, none of the elements undergoes a
change in oxidation number or valency.
5. (c) SO 32 S is in 4 oxidation state
15. (b) The element undergo oxidation itself and
S2 O 24 S is in 3 oxidation state reduces others is known as reducing agent. In
S2 O 62 S is in 5 oxidation state this reaction O. N. of Ni Changes from 0 to + 2
6. (b) Oxidation number of a compound must be and hence Ni acts as a reducing agent.
0. Using the values for A, B and C in the four 16. (a) Higher the value of reduction potential
options we find that A3(BC4)2 is the answer. higher will be the oxidising power whereas the
Check : (+2)3 + [(+5)+4(–2)]2 = 6 + (5–8)2 = 0 lower the value of reduction potential higher will
7. (d) Pyrophosphoric acid H4P2O7 be the reducing power.
Let oxidation state of phosphorus is x 17. (c) Carbon has the maximum oxidation state
(4 × 1 + (– 2) × 7 + 2 x) = 0 of + 4, therefore carbon dioxide (CO2) cannot
2x = 10 or x = +5 act as a reducing agent.
ݸ¿°¬»®

9 Hydrogen

̱°·½ ïæ Ю»°¿®¿¬·±² ¿²¼ Ю±°»®¬·»­ ±º (a) H (aq ) H 2 O( l) H 3O (aq)


ا¼®±¹»²
(b) H (aq) H 2O(l) OH (aq) H 2 (g )
1. Which of the following statements about
hydrogen is incorrect ? [2016] (c) H (aq) H 2 O(l)
(a) hydrogen has three isotopes of which OH (aq) 2 H (aq) 2e –
tritium is the most common.
(b) Hydrogen never acts as cation in ionic salts (d) H– (aq) + H2O(l) reaction
(c) Hydronium ion, H3O+ exists freely in 5. The ionization of hydrogen atom would give rise
solution to [1990]
(d) Dihydrogen does not act as a reducing agent (a) Hydride ion (b) hydronium ion
2. When a substance A reacts with water it (c) Proton (d) hydroxyl ion.
produces a combustible gas B and a solution of
̱°·½ îæ Ю»°¿®¿¬·±² ¿²¼ Ю±°»®¬·»­ ±º É¿¬»®
substance C in water. When another substance
D reacts with this solution of C, it also produces 6. Some statements about heavy water are given
the same gas B on warming but D can produce below:
gas B on reaction with dilute sulphuric acid at (a) Heavy water is used as a moderator in
room temperature. A imparts a deep golden nuclear reactors.
yellow colour to a smokeless flame of Bunsen (b) Heavy water is more associated than
burner. A, B, C and D respectively are [1998] ordinary water.
(a) Na , H2, NaOH, Zn (c) Heavy water is more effective solvent than
(b) K, H2, KOH, Al ordinary water.
(c) Ca, H2, Ca(OH)2, Sn Which of the above statements are correct?
(a) (a) and (c) (b) (a) and (b)[2010]
(d) CaC2, C2H2, Ca(OH)2, Fe
(c) (a), (b) and (c) (d) (b) and (c)
3. Which one of the following pairs of substances
7. Which of the following groups of ions makes
on reaction will not evolve H2 gas? [1998]
the water hard ? [1994]
(a) Iron and H2SO4 (aqueous)
(a) Sodium and bicarbonate
(b) Iron and steam (b) Magnesium and chloride
(c) Copper and HCl (aqueous) (c) Potassium and sulphate
(d) Sodium and ethyl alcohol (d) Ammonium and chloride.
4. The hydride ion, H–, is a stronger base than the 8. The dielectric constant of H2 O is 80. The
hydroxide ion, OH–. Which one of the following electrostatic force of attraction between Na+ and
reactions will occur if sodium hydride (NaH) is Cl– will be [1994]
dissolved in water? [1997]
EBD_7324
èì ÝØÛÓ×ÍÌÎÇ
(a) reduced to 1/40 in water than in air (a) Oxidizing in (i) and reducing in (ii)
(b) reduced to 1/80 in water than in air (b) Reducing in (i) and oxidizing in (ii)
(c) will be increased to 80 in water than in air (c) Reducing in (i) and (ii)
(d) will remain unchanged. (d) Oxidizing in (i) and (ii)
9. At its melting point ice is lighter than water 12. When H2O2 is oxidised the product is [1999]
because [1992]
(a) OH– (b) O2
(a) H2O molecules are more closely packed in
(c) O2– (d) HO2–
solid state
13. The volume strength of 1.5 N H2O2 solution is
(b) Ice cr ystals have hollow hexagonal
arrangement of H2O molecules. (a) 4.8 L (b) 5.2 L [1996]
(c) 8.4 L (d) 8.8 L
(c) On melting of ice the H2O molecule shrinks
in size
(d) Ice froms mostly heavy water on first 14. The O – O – H bond angle in H2O2 is [1994]
melting. (a) 106° (b)
10. Calgon used as a water softener is [1989] (c) 120° (d) 97°
15. Which of the following is the true structure of
(a) Na 2 [ Na 4 (PO 3 ) 6 ]
H2O2 ? [1989]
(b) Na 4 [ Na 2 (PO 3 ) 6 ] H
|
(c) Na 4 [Na 4 (PO4 )5 ] (a) H– O – O – H (b) O O
|
(d) Na 4 [ Na 2 (PO 4 ) 6 ] H
H H
̱°·½ íæ Ю»°¿®¿¬·±² ¿²¼ Ю±°»®¬·»­ ±º (c) O=O (d) O O=0
H H
ا¼®±¹»² л®±¨·¼» 16. The reaction of H2O2 with sulphur is an example
of ........reaction [1988]
11. (i) H2O2 + O3 H2O + 2O2
(a) Addition (b) Oxidation
(ii) H2O2 + Ag2O 2Ag + H2O + O2
(c) Reduction (d) Redox
Role of hydrogen peroxide in the above reactions
is respectively - [2014]

ANS WER KEY


1 (a, d) 3 (c) 5 (c) 7 (b) 9 (b) 11 (c) 13 (c) 15 (b)
2 (a) 4 (b) 6 (b) 8 (b) 10 (a) 12 (b) 14 (d) 16 (d)
ا¼®±¹»² èë

Hints & Solutions


1. (a, d) Among the three isotopes of hydrogen, results an open cage like structure. As a result
Protium (1H1) is most common. It is an energetic there are a number of 'hole' or open spaces. In
reducing agent. It reduces oxides, chlorides and such a structure lesser number of molecules are
packed per ml. When ice melts a large no. of
sulphides of certain metals and produce free
hydrogen bonds are broken. The molecules
metals at ordinary temperature.
therefore move into the holes or open spaces
CuO + 2H Cu + H2O and come closer to each other than they were in
2. (a) 2 Na 2H 2 O 2 NaOH H 2 solid state. This result sharp increase in the
'A ' 'C ' ' B' density. Therefore ice has lower density than
Zn 2 NaOH Na 2 ZnO 2 H2 water.
'D ' 'C ' ' B' 10. (a) The complex salt of metaphosphoric acid
Zn dil. H 2SO 4 ZnSO 4 H2 sodium hexametaphosphate (NaPO3)6, is known
'D ' ' B' as calgon. It is represented as Na 2[Na 4(PO3)6]
Na produces golden yellow colour with Reduction
smokeless flame of Bunsen burner.
–1 –2
3. (c) Fe dil. H 2SO 4 FeSO 4 H2 11. (c) (i) H2O2 + O3 H2 O2 + 2O2

3Fe 4H 2O Fe 3O 4 4H 2
Steam Oxidation
Cu + dil. HCl No reaction Oxidation
Copper does not evolve H2 from acid as it is
–1
below hydrogen in electrochemical series. (ii) H2O2 + Ag 2O –2
2Ag + H2O +2O2
2 Na C 2 H 5OH 2C 2 H 5ONa H 2
Reduction
4. (b) H (aq) H 2 O(l) OH (aq) H 2 (g) Hence in both the reactions H2O2 is acting as
base 1 acid 1 base 2 acid2 an oxidising agent.
In this reaction H– acts as bronsted base as it
Oxidation
accepts one proton (H+) from H2O for H2. 12. (b) H 2 O 2 [O ] H2O O2
5. (c) H (g ) H (g ) e . 13. (c) Volume strength = 5.6 × Normality
6. (b) Heavy water is used for slowing down the = 5.6 × 1.5 = 8.4 L
speed of neutrons and used as moderators. 14. (d) O – O – H bond angle in H2O2 is 97°.
Boiling point of heavy water is more than that of
15. (b) H is the true structure of
ordinary water, so it is more associated.
O–O
7. (b) Temporary hardness is due to presence of
H
bicarbonates of calcium and magnesium and
H2O2.
permanent hardness is due to the sulphates or
chlorides of both of calcium and magnesium. 16. (d) oxidation
8. (b) Electrostatic forces of attraction are H2S + H2O2 2H2O + S
reduced to 1/80th in water.
9. (b) In the structure of ice each molecule of H2O reduction
is surrounded by three H 2 O molecules in In this reaction H2S is oxidised to sulphur and
hexagonal honey comb manner. On the other H2O2 is reduced to H2O, hence this reaction
hand in water, each molecule is surrounded by show oxidation-reduction both i.e., redox
four neighbouring molecules randomly which reaction.
EBD_7324
èê ÝØÛÓ×ÍÌÎÇ

ݸ¿°¬»®

10 The s-Block Elements

̱°·½ ïæ Ю»°¿®¿¬·±² ¿²¼ Ю±°»®¬·»­ ±º (a) CsH > RbH > KH > NaH > LiH
ß´µ¿´· Ó»¬¿´­ ¿²¼ ¬¸»·® ݱ³°±«²¼­ (b) KH > NaH > LiH > CsH > RbH
1. The function of "Sodium pump" is a biological (c) NaH > LiH > KH > RbH > CsH
process operating in each and every cell of all (d) LiH > NaH > KH > RbH > CsH
animals. Which of the following biologically 7. The correct order of the mobility of the alkali metal
important ions is also a consituent of this ions in aqueous solutions is [2006]
pump : [2015] (a) Na+ > K+ > Rb+ > Li+
(a) Mg2+ (b) K+ (b) K+ > Rb+ > Na+ > Li+
(c) Fe2+ (d) Ca2+ (c) Rb+ >K+ > Na+ > Li+
2. Which one of the alkali metals, forms only, the (d) Li+ > Na+ > K+ > Rb+
normal oxide, M2O on heating in air ? [2012] 8. In crystals of which one of the following ionic
(a) Rb (b) K compounds would you expect maximum distance
(c) Li (d) Na between centres of cations and anions? [1998]
3. The ease of adsorption of the hydrated alkali (a) LiF (b) CsF
metal ions on an ion-exchange resins follows the (c) CsI (d) LiI
order : [2012] 9. Which of the following metal ions plays an
(a) Li+ < K+ < Na+ < Rb+ important role in muscle contraction ? [1994]
(b) Rb+ < K+ <Na+ < Li+ (a) K+ (b) Na+
(c) Mg 2+ (d) Ca2+
(c) K+ < Na+ < Rb+ < Li+
(d) Na+ < Li + < K+ < Rb+ 10. Which of the following statement is false ? [1994]
4. In the replacement reaction (a) Strontium decomposes water readily than
beryllium
CI + MF CF + MI (b) Barium car bonate melts at a higher
The reaction will be most favourable if M happens temperature than calcium carbonate
to be : [2012 M] (c) Barium hydroxide is more soluble in water
(a) Na (b) K than magnesium hydroxide
(c) Rb (d) Li (d) Beryllium hydroxide is more basic than
5. The sequence of ionic mobility in aqueous barium hydroxide.
solution is : [2008] 11. Which of the following has largest size ?[1993]
(a) K+ > Na+ > Rb+ > Cs+ (a) Na (b) Na+
(b) Cs+ > Rb+ > K+ > Na + (c) Na – (d) Can’t be predicted
(c) Rb+ > K+ > Cs+ > Na + 12. Compared with the alkaline earth metals, the alkali
(d) Na+ > K+ > Rb+ > Cs+ metals exhibit [1990]
6. The alkali metals form salt-like hydrides by the (a) Smaller ionic radii
direct synthesis at elevated temperature. The (b) Highest boiling points
thermal stability of these hydrides decreases in (c) Greater hardness
which of the following orders ? [2008] (d) Lower ionization energies.
̸» ­óÞ´±½µ Û´»³»²¬­ èé
13. Which one of the following properties of alkali ̱°·½ íæ Ю»°¿®¿¬·±² ¿²¼ Ю±°»®¬·»­ ±º
metals increases in magnitude as the atomic ß´µ¿´·²» Ó»¬¿´­ ¿²¼ ¬¸»·® ݱ³°±«²¼­
number rises ? [1989]
(a) Ionic radius 21. Which of the following statements is false ?
(b) Melting point [2016]
(c) Electronegativity (a) Mg2+ ions form a complex with ATP
(d) First ionization energy. (b) Ca2+ ions are important in blood clotting
(c) Ca2+ ions are not important in maintaining
̱°·½ îæ ͱ³» ׳°±®¬¿²¬ ݱ³°±«²¼­ ±º the regular beating of the heart.
ͱ¼·«³ (d) Mg2+ ions are important in the green parts
of plants.
14. In Castner-Kellner cell for production of sodium
22. Solubility of the alkaline earth's metal sulphates
hydroxide: [NEET Kar. 2013]
in water decreases in the sequence :- [2015]
(a) Brine is electrolyzed with Pt electrodes
(a) Ca > Sr > Ba > Mg (b) Sr > Ca > Mg > Ba
(b) Brine is electrolyzed using graphite
(c) Ba > Mg > Sr > Ca (d) Mg > Ca > Sr > Ba
electrodes
23. Which of the following compounds has the
(c) Molten sodium chloride is electrolysed
lowest melting point ? [2011]
(d) Sodium amalgam is formed at mercury
cathode (a) CaCl2 (b) CaBr2
15. Which of the following statements is incorrect? (c) CaI2 (d) CaF2
[2011M] 24. Which of the following alkaline earth metal
sulphates has hydration enthalpy higher than the
(a) Pure sodium metal dissolves in liquid
lattice enthalpy? [2010]
ammonia to give blue solution.
(b) NaOH reacts with glass to give sodium silicate (a) CaSO 4 (b) BeSO 4
(c) Aluminium reacts with excess NaOH to give (c) BaSO 4 (d) SrSO 4
Al(OH)3 25. Property of the alkaline earth metals that increases
(d) NaHCO3 on heating gives Na2CO3 with their atomic number [2010]
16. Which of the following oxides is not expected to (a) Solubility of their hydroxides in water
react with sodium hydroxide? [2009] (b) Solubility of their sulphates in water
(a) CaO (b) SiO2 (c) Ionization energy
(c) BeO (d) B2O3 (d) Electronegativity
17. In which of the following processes, fused 26. The correct order of increasing thermal stability
sodium hydroxide is electrolysed at a 330ºC of K2CO3, MgCO3, CaCO3 and BeCO3 is [2007]
temperature for extraction of sodium? [2000] (a) BeCO3< MgCO3 < CaCO3 < K2CO3
(a) Castner's process (b) Down's process
(b) MgCO3 < BeCO3 < CaCO3 < K2CO3
(c) Cyanide process (d) Both 'b' and 'c'
18. Aqueous solution of sodium carbonate absorbs (c) K2CO3 < MgCO3 < CaCO3 < BeCO3
NO and NO2 to give [1996] (d) BeCO3 < MgCO3 < K2CO3 < CaCO3
(a) CO2 + NaNO3 (b) CO2 + NaNO2 27. In which of the following the hydration energy is
(c) NaNO2 + CO (d) NaNO3 + CO higher than the lattice energy? [2007]
(a) MgSO4 (b) RaSO4
19. Which of the following is known as fusion
(c) SrSO4 (d) BaSO4
mixture? [1994]
28. Calcium is obtained by the [1997]
(a) Mixture of Na2CO3 + NaHCO3 (a) electrolysis of solution of calcium chloride
(b) Na2CO3.10H2O in water
(c) Mixture of K2CO3 + Na 2CO3 (b) electrolysis of molten anhydrous calcium
(d) NaHCO3 chloride or fused calcium chloride
20. Washing soda has formula [1990] (c) roasting of limestone
(a) Na2CO3.7H2O (b) Na2CO3.10H2O (d) reduction of calcium chloride with carbon
(c) Na2CO3.3H2O (d) Na2CO3 29. For two ionic solids CaO and KI, identify the
wrong statement amongst the following : [1997]
EBD_7324
èè ÝØÛÓ×ÍÌÎÇ
(a) The lattice energy of CaO is much large 37. Which one of the following is present as an active
than that of KI ingredient in bleaching powder for bleaching
(b) KI is more soluble in water action ? [2011]
(a) CaOCl2 (b) Ca(OCl)2
(c) KI has higher melting point (c) CaO2Cl (d) CaCl2
(d) CaO has higher melting point 38. Match List – I with List –II for the compositions
30. Which one is the correct statement with reference of substances and select the correct answer using
to solubility of MgSO4 in water? [1996] the code given below the lists : [2011M]
(a) SO 4 2– ion mainly contributes towards List - I List - II
hydration energy Substances Composition
(b) Sizes of Mg2+ and SO42– are similar (1) Plaster of paris (i) CaSO4.2H2O
(c) Hydration energy of MgSO 4 is higher in (2) Epsomite (ii) CaSO4.½ H2O
comparison to its lattice energy (3) Kieserite (iii) MaSO4.7 H2O
(d) Ionic potential (charge/radius ratio) of Mg2+ (4) Gypsum (iv) MgSO4. H2O
is very low (v) CaSO4
Code :
31. Sodium is made by the electrolysis of a molten
(1) (2) (3) (4)
mixture of about 40% NaCl and 60% CaCl 2
(a) (iii) (iv) (i) (ii)
because [1995] (b) (ii) (iii) (iv) (i)
(a) Ca can reduce NaCl to Na (c) (i) (ii) (iii) (v)
(b) Ca can displace Na from NaCl (d) (iv) (iii) (ii) (i)
39. The compound A on heating gives a colourless
(c) CaCl 2 helps in conduction of electricity gas and a residue that is dissolved in water to
(d) this mixture has a lower melting point than NaCl obtain B. Excess of CO2 is bubbled through
32. All the following substances react with water. aqueous solution of B, C is formed which is
The pair that gives the same gaseous product is recovered in the solid form. Solid C on gentle
[1994] heating gives back A. The compound is [2010]
(a) K and KO2 (b) Na and Na2O2 (a) CaSO4.2H2O (b) CaCO3
(c) Ca and CaH2 (d) Ba and BaO2 (c) Na2CO3 (d) K2CO3
33. Which one of the following has minimum value 40. A solid compound ‘X’ on heating gives CO2 gas
of cation/anion ratio. [1993] and a residue. The residue mixed with water forms
(a) NaCl (b) KCl ‘Y’. On passing an excess of CO2 through ‘Y’ in
(c) MgCl2 (d) CaF2 water, a clear solution ‘Z’, is obtained. On boiling
34. Electronic configuration of calcium atom can be ‘Z’, a compound ‘X’ is reformed. The compound
written as [1992] ‘X’ is [2004]
(a) Ca(HCO3)2 (b) CaCO3
(a) [Ne], 4p2 (b) [Ar], 4s2
(c) Na2CO3 (d) K2CO3
(c) [Ne], 4s 2 (d) [kr], 4p2
41. Identify the correct statement [1995]
35. Which of the following atoms will have the
(a) gypsum is obtained by heating plaster of
smallest size ? [1989]
Paris
(a) Mg (b) Na
(c) Be (d) Li (b) plaster of Paris can be obtained by hydration
of gypsum
̱°·½ ìæ ͱ³» ׳°±®¬¿²¬ ݱ³°±«²¼­ ±º (c) plaster of paris is obtained by partial
Ý¿´½·«³ oxidation of gypsum
36. On heating which of the following releases CO2 (d) gypsum contains a lower percentage of
most easily ? [2015 RS] calcium than plaster of Paris
(a) K2CO3 (b) Na2CO3
(c) MgCO3 (d) CaCO3
̸» ­óÞ´±½µ Û´»³»²¬­ èç

ANS WER KEY


1 (b) 6 (d) 11 (c) 16 (a) 21 (c) 26 (a) 31 (d) 36 (c) 41 (d)
2 (c) 7 (c) 12 (d) 17 (a) 22 (d) 27 (a) 32 (c) 37 (b)
3 (b) 8 (c) 13 (a) 18 (c) 23 (c) 28 (b) 33 (c) 38 (b)
4 (c) 9 (d) 14 (d) 19 (c) 24 (b) 29 (c) 34 (b) 39 (b)
5 (b) 10 (d) 15 (c) 20 (b) 25 (a) 30 (c) 35 (c) 40 (b)

Hints & Solutions


1. (b) K+ ion is a constituent of sodium pump. M–H bonds becomes weaker with increase in
2. (c) All the alkali metals when heated with size of alkali metals as we move down the group
oxygen form different types of oxides for example from Li to Cs. Thus the order of stability of
lithium forms lithium oxide (Li2O), sodium forms hydrides is
sodium peroxide (Na2O2), while K, Rb and Cs LiH > NaH > KH > RbH > CsH
form their respective superoxides. 7. (c) Ionic radii of alkali metals in water follows
3. (b) All alkali metal salts are ionic (except the order Li+ > Na+ > K+ > Rb+ > Cs+
Lithium) and soluble in water due to the fact Thus in aqueous solution due to larger ionic
that cations get hydrated by water molecules. radius Li+ has lowest mobility and hence the
The degree of hydration depends upon the size correct order of ionic mobility is
of the cation. Smaller the size of a cation, greater is its
hydration energy. Li Na K Rb
Relative ionic radii : 8. (c) As Cs+ ion has larger size than Li+ and I –
has larger size than F–, therefore maximum
Cs Rb K Na Li distance between centres of cations and anions
Relative ionic radii in water or relative degree of is in CsI.
hydration: 9. (d) Ca2+ ions is an essential element for the
contraction of muscles.
Li Na K Rb Cs
10. (d) Be(OH)2 is amphoteric, but the hydroxides
4. (c) Tertiary halide can show ionic reaction with
of other alkaline earth metals are basic. The basic
MF so, MF should be most ionic for reaction to strength increases gradually.
proceed forward. Hence ‘M’ should be ‘Rb’.
11. (c) A cation is always much smaller than the
5. (b) Smaller the ion more is its ionic mobility in corresponding atom, whereas an anion is always
aqueous solution. Ionic radii of the given alkali larger than the corresponding atom, hence the
metals is in the order Na+ < K+ < Rb+ < Cs+ and size decreases in the order
thus expected ionic mobility will be in the same
order Cs+ < Rb+ < K+ < Na+. However due to Na Na Na
high degree of solvation (or hydration) because 12. (d) Because of larger size and smaller nuclear
of lower size or high charge density, the hydrated charge, alkali metals have low ionization potential
ion size follows the same order Cs+ < Rb+ < K+ relative to alkaline earth metals.
< Na + and thus conductivity order is Cs+ > Rb+ 13. (a) Within a group, ionic radius increases with
> K+ > Na+ . increase in atomic number. The melting point
6. (d) The stability of alkali metal hydrides decrease down the group due to weakening of
decreases from Li to Cs. It is due to the fact that
EBD_7324
çð ÝØÛÓ×ÍÌÎÇ
metallic bond. The electronegativity and the 1st group 2 because their hydroxide are much basic
ionization energy also decreases down the than group 2 hydroxides therefore, the order of
group. thermal stability is
14. (d) In castner kellner cell, sodium amalgam is BeCO3 < MgCO3< CaCO3< K2CO3.
formed at mercury cathode. 27. (a) The solubility and the hydration energy of
15. (c) 2Al(s) + 2NaOH (aq) + 2H2O (l) sulphates of alkaline earth metals decreases as
2NaAlO2 + 3H2 we move down the group from Be to Ba due to
sod. meta aluminate the reason that ionic size increases down the
group. The lattice energy remains constant
16. (a) NaOH is a strong alkali. It combines with
because sulphate ion is so large, so that small
acidic and amphoteric oxides to form salts. Since
change in cationic sizes do not make any
CaO is a basic oxide hence does not reacts with
difference. Thus the order will be:
NaOH.
17. (a) In Castner process, for production of (Na) BeSO 4 MgSO 4 CaSO 4 SrSO 4 BaSO4
Sodium metal, Sodium hydroxide (NaOH) is 28. (b) Calcium is obtained by electrolysis of a
electrolysed at 330ºC. fused mass consisting six parts calcium chloride
18. (c) Na 2 CO3 + NO + NO2 2 NaNO 2 + CO 2 and one part calcium fluoride at about 700°C in
an electrolytic cell.
19. (c) Mixture of K2CO3 and Na2CO3 is called as
29. (c) CaO has higher lattice energy because of
fusion mixture.
higher charge on Ca2+ and O2–, which results in
20. (b) Washing soda is Na2CO3. 10H2O. higher attraction. KI is more soluble in water
21. (c) because of low lattice energy and higher
22. (d) Solubility of alkaline earth metal sulphates hydration energy. Clearly (c) is wrong because
decreases down the group due decrease in CaO has higher melting point as compared to
hydration energy. KI.
MgSO 4 > CaSO 4 > SrSO 4 > BaSO 4 30. (c) MgSO4 is the only alkaline earth metal
sulphate which is soluble in water and for
Hydration Solubility
energy solubility hydration energy should be greater
than lattice energy.
23. (c) Melting point of metal halides decreases
31. (d) Sodium is obtained by electr olytic
as the size of the halogen increases. The correct
reduction of its chloride. Melting point of
order is
chloride of sodium is high (803°C) so in order to
CaF2 > CaCl2 > CaBr2 > CaI2 lower its melting point(600°C), calcium chloride
24. (b) Be 2+ is very small, hence its hydration is added to it.
enthalpy is greater than its lattice enthalpy. Also, 32. (c) Ca and CaH2 both react with water to form
hydration energy decreases down the group. H2 gas,
25. (a) The solubility of an ionic depends on the Ca 2H 2 O Ca (OH ) 2 H2
lattice and hydration energy. In alkaline earth
metal hydroxides, the hydration en ergy CaH 2 2H 2O Ca (OH ) 2 2H 2
decreases down the group so, their solubility
also decreases. whereas
26. (a) As the basicity of metal h ydroxides K gives H2 while KO2 gives O2 and H2O2
increases down the group from Be to Ba, the
2K + 2 H2O 2KOH + H2
thermal stability of their carbonates also
increases in the same order. Further group 1
2KO2 2H 2O 2KOH O2 H 2O 2
compounds are more thermally stable than
̸» ­óÞ´±½µ Û´»³»²¬­ çï

Similarly, Na gives H2 while Na2O2 gives H2O2 Ca OH 2CO2 + H2O Ca HCO3


2 2
(c)
2Na 2H 2 O 2 NaOH H 2
Ca HCO3 2(s)
CaCO3(s) CO 2(g) H2 O
Na 2 O 2 2H 2 O 2 NaOH H 2O 2 (A)
Also, Ba gives H2 while BaO2 gives H2O2

Ba + 2 H2O Ba (OH)2 + H2
40. (b) X heat
CO2 + Residue
BaO2 + 2H2O Ba (OH)2 + H2 O2 Solid
H2 O
33. (c) Atomic size of K+ > Ca2+ > Mg2+ and that boil
of Cl– > F–. Therefore, Mg2+/Cl– ratio has the excess CO2 Y
minimum value. Z
Clear solution
34. (b) 20Ca = 1s22s22p63s23p6 = [Ar] 4s2
The given properties coincide with CaCO3
35. (c) Within a period, the atomic size decreases
from left to right.Further atomic size increases
heat
down the group. Hence the correct order is CaCO3 CO2 CaO
i,e. Na > Mg > Li > Be. Residue
heat 'X '
36. (c) Carbonates becomes more thermally stable
H 2O
down the group, therefore MgCO3 will leave
CO2 easily. excess
Ca(HCO3 ) 2 Ca(OH)2 H 2 O
37. (b) Active ingredient in bleaching powder for 'Z' CO 2
'Y '
bleaching action is Ca (OCl)2.
1
38. (b) (A) Plaster of paris = CaSO4. H2 O
2
41. (d) Gypsum is CaSO4. 2H2O and plaster of
(B) Epsomite = MgSO4.7H2O
(C) Kieserite = MgSO4.H2O Paris is (CaSO 4 ) 2 .H 2 O. Therefore gypsum
(D) Gypsum = CaSO4.2H2O contains a lower percentage of calcium than
plaster of Paris.
39. (b) CaCO3(s) CO 2(g) CaO(s)
(A) colourless residue
CaO(s) H2O Ca OH 2
(B)
EBD_7324
çî ÝØÛÓ×ÍÌÎÇ

ݸ¿°¬»®

11 The p-Block Elements


(Group 13 & 14)

̱°·½ ïæ Þ±®±² Ú¿³·´§ 7. Which of the following statements about H3BO3


is not correct ? [1994]
1. The stability of +1 oxidation state among Al, Ga,
In and Tl increases in the sequence :[2015 RS] (a) It is a strong tribasic acid
(a) Ga < In < Al < Tl (b) Al < Ga < In < Tl (b) It is prepared by acidifying an aqueous
(c) Tl < In < Ga < Al (d) In < Tl < Ga < Al solution of borax
2. Which of the following structure is similar to (c) It has a layer structure in which planar BO3
graphite? [NEET 2013]
units are joined by hydrogen bonds
(a) B (b) B4C
(c) B2H6 (d) BN (d) It does not act as proton donor but acts as
3. Aluminium is extracted from alumina (Al2O3 ) a Lewis acid by accepting a lone pair of
by electrolysis of a molten mixture of : [2012] electrons
(a) Al2O3 + HF + NaAlF4 8. Which of the following elements is extracted
(b) Al2O3 + CaF2 + NaAlF4 commercially by the electrolysis of an aqueous
(c) Al2O3 + Na3AlF6 + CaF2 solution of its compound ? [1993]
(d) Al2O3 + KF + Na3AlF6 (a) Cl (b) Br
4. The tendency of BF3, BCl3 and BBr 3 to behave (c) Al (d) Na
as Lewis acid decreases in the sequence: [2010] 9. An example of a double salt is [1989]
(a) BCl3 > BF3 > BBr3 (a) Bleaching powder (b) K4[Fe(CN)6]
(b) BBr3 > BCl3 > BF3 (c) Hypo (d) Potash Alum
(c) BBr3 > BF3 > BCl3
(d) BF3 > BCl3 > BBr3 ̱°·½ îæ Ý¿®¾±² Ú¿³·´§
5. Al2O3 can be converted to anhydrous AlCl 3 by 10. It is because of inability of ns2 electrons of the
heating [2006] valence shell to participate in bonding that:-
(a) Al2O3 with NaCl in solid state [2017]
(b) a mixture of Al2O3 and carbon in dry Cl2 (a) Sn2+ is oxidising while Pb4+ is reducing
gas (b) Sn 2+ and Pb2+ are both oxidising and
(c) Al2O3 with Cl2 gas reducing
(d) Al2O3 with HCl gas (c) Sn4+ is reducing while Pb4+ is oxidising
6. In borax bead test which compound is formed? (d) Sn2+ is reducing while Pb4+ is oxidising
11. Which of these is not a monomer for a high
[2002]
molecular mass silicone polymer?[NEET 2013]
(a) Ortho-borate (b) Meta-borate
(a) Me2SiCl2 (b) Me3SiCl
(c) Double oxide (d) Tetra-borate (c) PhSiCl3 (d) MeSiCl3
̸» °óÞ´±½µ Û´»³»²¬­ øÙ®±«° ïí ú ïì÷ çí

12. The basic structural unit of silicates is : 17. Glass reacts with HF to produce [2000]
[NEET 2013] (a) SiF4 (b) H2SiF6
(a) SiO44 (b) SiO32 (c) H2SiO3 (d) Na3AlF6
18. In graphite, electrons are [1994]
(c) SiO24 (d) SiO
(a) Localised on every third C-atom
13. Which statement is wrong? [NEET Kar. 2013]
(a) Feldspars are not aluminosilicates (b) Present in anti-bonding orbital
(b) Beryl is an example of cyclic silicate (c) Localised on each C-atom
(c) Mg2SiO4 is orthosilicate (d) Spread out between the structure
(d) Basic structural unit in silicates is the SiO4 19. In graphite electrons are : [1993]
tetrahedron (a) localised on each carbon atom
14. Name the type of the structure of silicate in which
(b) spread out between the sheets
one oxygen atom of [SiO4]4– is shared ?[2011]
(c) localised on every third carbon atom
(a) Linear chain silicate
(d) present in antibonding orbital.
(b) Sheet silicate
20. Which of the following types of forces bind
(c) Pyrosilicate
(d) Three dimensional together the carbon atoms in diamond ?[1992]
15. The straight chain polymer is formed by [2009] (a) Ionic (b) Covalent
(a) hydrolysis of CH 3 SiCl 3 followed by (c) Dipolar (d) van der Waals.
condensation polymerisation 21. Water gas is produced by [1992]
(b) hydrolysis of (CH 3 ) 4 Si by addition (a) Passing steam through a red hot coke bed
polymerisation (b) Saturating hydrogen with moisture
(c) hydrolysis of (CH3)2SiCl2 followed by (c) Mixing oxygen and hydrogen in the ratio
condensation polymerisation of 1 : 2
(d) hydrolysis of (CH3) 3SiCl followed by
(d) Heating a mixture of CO2 and CH4 in
condensation polymerisation
petroleum refineries.
16. Which one of the following statements about
22. Glass is a [1991]
the zeolites is false ? [2004]
(a) Liquid
(a) They are used as cation exchangers
(b) Solid
(b) They have open structure which enables
(c) Supercooled liquid
them to take up small molecules
(c) Zeolites are aluminosilicates having three (d) Transparent organic polymer
dimensional network 23. The substance used as a smoke screen in
warfare is [1989]
(d) Some of the SiO44 units are replaced by
(a) SiCl4 (b) PH3
AlO 54 and AlO 96 ions in zeolites (c) PCl5 (d) Acetylene

ANSWER KEY
1 (b) 4 (b) 7 (a) 10 (d) 13 (a) 16 (c) 19 (b) 22 (c)
2 (d) 5 (b) 8 (c) 11 (b) 14 (c) 17 (b) 20 (b) 23 (a)
3 (c) 6 (b) 9 (d) 12 (a) 15 (c) 18 (d) 21 (a)
EBD_7324
çì ÝØÛÓ×ÍÌÎÇ

Hints & Solutions


1. (b) Lower oxidation state become more stable
on moving down the group 4AlO33 2Al2 O 3 3O 2 12e (at anode)
Al < Ga < ln < Tl Aluminium of 99.8% purity is obtained from this
process.
2. (d) Boron nitride (BN) is known as inorganic 9. (d) Potash Alum, K2SO4. Al2(SO4). 24H2O is a
graphite. The most stable form is hexagonal one. double salt.
It has layered structure similar to graphite. 10. (d) Inertness of ns2 electrons of the valence
3. (c) Fused alumina (Al2O3) is a bad conductor shell to participate in bonding on moving down
of electricity. Therefore, cryolite (Na 3AlF6) and the group in heavier p-block elements is called
fluorspar (CaF2) are added to purified alumina inert pair effect.
which not only make alumina a good conductor As a result, Pb(II) is more stable than Pb(IV)
of electricity but also reduce the melting point Sn(IV) is more stable than Sn(II)
of the mixture to around 1140 K. Pb(IV) is easily reduced to Pb(II) and can
4. (b) In BF3, p-p overlap between B and F is acts as an oxidising agent whereas Sn(II) is easily
maximum due to identical size and energy of oxidised to Sn(IV) and can acts as a reducing
p-orbitals, so electron deficiency in boron of BF 3 agent.
is neutralized partially to the maximum extent by
back donation. Also, the tendency to back
11. (b) Since Me3SiCl contains only one Cl,
donate decreases from F to I. So the order will
therefore it can’t form high molecular mass
be:
silicon polymer. It can form only dimer.
BF3 < BCl3 < BBr3.
12. (a) SiO44– is basic structural unit of silicates.
5. (b) Al2O3 can be converted to anhydrous AlCl 3 13. (a) Feldspars are 3-dimensional alumino-
by heating a mixture of Al 2O3 and carbon in dry silicates.
Cl2 gas.
1000°C O– O–
Al2O3 + 3C + 3Cl2 2AlCl3 + 3CO
vapours
Solid anhydrous 14. (c)
cooled
aluminium Si Si

O–
chloride O O
O
– O–
6. (b) Na2B4O7. 10 H2O 10H 2 O
Pyrosilicate (Si2O7)6–
Na 2 B4O7 2 NaBO 2 B 2O3
anhydrous sod.metaborate Boric anhydride
15. (c) Hydrolysis of substituted chlorosilanes
CuO B2 O3 Cu(BO 2 ) 2 yield corresponding silanols which undergo
cupric meta borate(Blue beed) polymerisation.
7. (a) H3BO3 is a weak monobasic acid. CH 3 Cl H OH – 2HCl
8. (c) Aluminium can be extracted by electrolysis Si +
of pure alumina (Al2O3). Alumina ionises as CH 3 Cl H OH

Al 2O 3 Al3 AlO33 CH3 OH


cathode anode Si
CH3 OH
3 Dimethylsilanol
Al 3e Al (at cathode)
̸» °óÞ´±½µ Û´»³»²¬­ øÙ®±«° ïí ú ïì÷ çë

Polymerisation of dimethylsilanol yields linear 18. (d) In graphite, each carbon is sp2 -hybridized
thermoplastic polymer. and the single occupied unh ybridized
CH3 CH3 p-orbital of C-atoms overlap side ways to give
-electron cloud which is delocalized and thus
the electrons are spread out between the
HO — Si — OH + H O —Si — OH
structure.
19. (b) In graphite, the electrons are spread out
CH3 CH3
between the sheets.
CH3 CH3 20. (b) In diamond each carbon atom is sp 3
hybridized and thus forms covalent bonds with
four other carbon atoms lying at the corners of a
HO — Si — O — Si — OH
regular tetrahedron.
21. (a) Water gas is made by blowing steam through
CH3 CH3
the layer of incandescent coal.
16. (c) Zeolites have SiO4 and AlO4 tetrahedrons
linked together in a 3-D open structure in which H 2O C H 2 CO
4 or 6 membered rings predominate. Due to open Steam Re d hot water gas
change structure they have cavities and can take 22. (c) Glass is a super cooled liquid.
up water and other small molecules. 23. (a) SiCl4 gets hydrolysed in moist air and gives
17. (b) 6 HF SiO 2 H 2SiF6 2 H 2O white fumes which are used as a smoke screen
in warfare.
EBD_7324
çê ÝØÛÓ×ÍÌÎÇ

ݸ¿°¬»®

12
Organic Chemistry - Some
Basic Principles and
Techniques
̱°·½ ïæ Ý´¿­­·º·½¿¬·±² ¿²¼ Ò±³»²½´¿¬«®» ±º (a) CH 3 CH CH 2 CH 3
Ñ®¹¿²·½ ݱ³°±«²¼­
(b) CH 3 CH 2 CH 2 CH 2
1. The IUPAC name of the compound is : [2017]
CH3
O O |
(c) CH3 C
H–C
CH3
CH 3
(d) CH CH 2
(a) 5-formylhex-2-en-3-one CH3
(b) 5-methyl-4-oxohex-2-en-5-al 4. Which nomenclature is not according to IUPAC
(c) 3-keto-2-methylhex-5-enal system? [2012]
(d) 3-keto-2-methylhex-4-enal
2. Structure of the compound whose IUPAC name (a) Br CH 2 CH CH 2 ,
is 3-ethyl-2-hydroxy-4-methylhex-3-en-5-ynoic 1-Bromo-prop-2-ene
acid is : [NEET 2013]
CH3
OH
(b) CH3 CH2 C CH2 CHCH3
(a) COOH
Br CH3
4 -Bromo-2, 4 -dimethylhexane
OH
(c) CH3 CH CH CH2 CH3,
COOH
(b)
CH3
2-Methyl-3-phenylpentane
(d) CH3 C CH2 CH2 CH2 COOH
COOH
(c)
O
OH
5-oxohexanoic acid
OH 5. The correct IUPAC name of the compound
[2011]
(d) COOH

is
3. The structure of isobutyl group in an organic
compound is : [NEET 2013]
Ñ®¹¿²·½ ݸ»³·­¬®§ ó ͱ³» Þ¿­·½ Ю·²½·°´»­ ¿²¼ Ì»½¸²·¯«»­ çé

(a) 4-Ethyl-3-propyl hex-1-ene 11. The correct order regarding the electronegativity
(b) 3-Ethyl-4-ethenyl heptane of hybrid orbitals of carbon is [2006]
(c) 3-Ethyl-4-propyl hex-1-ene (a) sp > sp2 > sp3 (b) sp < sp2 > sp3
(d) 3-(1-ethylpropyl) hex-1-ene (c) sp < sp2 < sp3 (d) sp > sp2 < sp3
6. The IUPAC name of the following compound is 12. Names of some compounds are given. Which
one is not correct in IUPAC system? [2005]
[2011 M]
CH3
Cl CH2CH3 |
(a) CH 3 CH 2 CH 2 C H CH CH 2CH 3
|
CH 2CH 3
CH3 I 3óMethyló4óethylheptane

(a) trans-2-chloro-3-iodo-2-pentene (b) CH3 CH CH CH3


| |
(b) cis-3-iodo-4-chloro-3-pentene OH CH3
3 Methyl 2 butanol
(c) trans-3-iodo-4-chloro-3-pentene
(c) CH3 CH 2 C CH CH 3
(d) cis-2-chloro-3-iodo-2-pentene || |
7. The state of hybridization of C2, C3, C5 and C6 CH 2 CH3
2 Ethyl 3 methyl but 1 ene
of the hydrocarbon, [2009]
(d) CH 3 C C CH (CH 3 ) 2
CH3 CH3 4 Methyl 2 pentyne
| |
CH3 C CH = CH CH C CH 13. Name of the compound given below is [2003]
7 6| 5 4 3 2 1 CH3
CH3
H 3C
is in the following sequence: CH3
(a) sp3, sp2, sp2 and sp
(b) sp, sp2, sp2 and sp3
CH3
(c) sp, sp2, sp3 and sp2
(a) 5-ethyl-6-methyloctane
(d) sp, sp3, sp2 and sp3
(b) 4-ethyl-3-methyloctane
8. In the hydrocarbon
(c) 3-methyl-4-ethyloctane
CH3 – CH = CH – CH2 – C CH
(d) 2, 3-diethylheptane
6 5 4 3 2 1 14. IUPAC name of the following is [2002]
The state of hybrization of carbons 1, 3 and 5
are in the following sequence : [2008] CH 2 CH CH 2 CH 2 C CH
(a) sp2, sp, sp3 (b) sp, sp3, sp2
(a) 1, 5-hexenyne (b) 1-hexene- 5-yne
(c) sp, sp2, sp3 (d) sp3, sp2, sp
(c) 1-hexyne- 5-ene (d) 1, 5-hexynene
9. The general molecular formula, which represents
the homologous series of alkanols is [2006] 15. The incorrect IUPAC name is [2001]
(a) CnH2nO (b) CnH2n + 1O (a) CH 3 C C H CH 3
|| |
(c) CnH2n + 2O (d) CnH2n O2 O CH 3
2-Methyl-3-butanone
Cl
10. The IUPAC name of is (b) CH 3 CH C H CH 3
| |
O CH3 CH 2CH 3
[2006]
2,3-Dimethylpentane
(a) 1- chloro -1-oxo - 2, 3- dimethyl pentane (c) CH 3 C H C H CH 3
(b) 2-ethyl -3- methylbutanoyl chloride | |
(c) 2, 3-dimethylpentanoyl chloride Cl Br
2-Bromo-3-chlorobutane
(d) 3, 4-dimethyl pentanoyl chloride
(d) CH 3 C CCH (CH 3 ) 2
4-Methyl-2-pentyne
EBD_7324
çè ÝØÛÓ×ÍÌÎÇ
16. The structural formula of a compound is 23. When the hybridization state of carbon atom
CH3 – CH = C = CH2. The types of hybridization changes from sp3 to sp2 and finally to sp, the
at the four carbons from left to right are angle between the hybridized orbitals [1993]
(a) sp2, sp2, sp2, sp3 (b) sp2, sp3, sp2, sp2 (a) decreases gradually
(c) sp3, sp2, sp, sp2 (d) sp3, sp2, sp2, sp2 (b) decreases considerably
17. IUPAC name for the compound [1998] (c) is not affected
CH 2CH3 (d) increases progressively.
Cl
24. The IUPAC name of [1992]
C=C is
H3C I CH 3 CH CH C CHO
| |
(a) trans-3-iodo-4-chloro-3-pentene OH CH 3
(b) cis-2-chloro-3-iodo-2-pentene (a) 4-Hydroxy-1-methylpentanal
(c) trans-2-chloro-3-iodo-2-pentene (b) 4-Hydroxy-2-methylpent-2-en-1-al
(d) cis-3-iodo-4-chloro-3-pentene (c) 2-Hydroxy-4-methylpent-3-en-5-al
18. The IUPAC name of [1996] (d) 2-Hydroxy-3-methylpent-2-en-5-al
H 3 C — C H — CH — CH 2 —CH 2— C H — CH 3 25. 2-Methyl-2-butene will be represented as [1992]
| | |
CH3 CH3 CH3 CH 3
|
is (a) CH 3 CH CH 2 CH 3
(a) 1, 3-isopropyl-3-methylpropane
(b) CH 3 C CH CH 3
(b) 2, 3, 6-trimethylheptane |
(c) 2, 5, 6-trimethylheptane CH 3
(d) 2, 6, 3-trimethylheptane (c) CH 3 CH 2 C CH 2
19. Huckel's r ule states that a monocyclic |
conjugated compound will be aromatic if it CH 3
contains [1996] (d) CH 3 CH CH CH 2
(a) (4n + 2 ) electrons |
(b) (4 + 2n) electrons CH 3
(c) 4 electrons 26. A straight chain hydrocarbon has the molecular
(d) (4n + 2) electrons formula C8H10. The hybridization of the carbon
20. The first organic compound, synthesized in the atoms from one end of the chain to the other are
laboratory, was [1995] respectively sp3, sp2, sp2, sp3, sp2, sp2, sp and
(a) alcohol (b) acetic acid sp. The structural formula of the hydrocarbon
(c) urea (d) none of these would be : [1991]
21. Which of the following IUPAC names is correct (a) CH3C CCH2 – CH = CHCH = CH2
for the compound? [1994] (b) CH3CH2– CH = CHCH = CHC CH
(c) CH3CH = CHCH2 – C CCH = CH2
H3C C H CH CH 2 CH3 (d) CH3CH = CHCH2 – CH = CH – C CH.
| |
CH3 CH2CH3 27. An sp3 hybrid orbital contains [1991]
(a) 1/4 s-character (b) 1/2 s-character
(a) 2-Methyl-3-ethylpentane (c) 1/3 s-character (d) 2/3 s-character.
(b) 3-Ethyl-2-methylpentane 28. The shortest C – C bond distance is found in
(c) 2-Ethyl-3-methylpentane [1991]
(d) 3-Methyl-2-ethylpentane (a) Diamond (b) Ethane
22. Which is the correct symbol relating the two (c) Benzene (d) Acetylene
Kekule structures of benzene ? [1993] 29. An organic compound X (molecular formula
(a) (b) C6H7O2N) has six carbon atoms in a ring system,
(c) (d) two double bonds and a nitr o gr oup as
substituent, X is [1990]
Ñ®¹¿²·½ ݸ»³·­¬®§ ó ͱ³» Þ¿­·½ Ю·²½·°´»­ ¿²¼ Ì»½¸²·¯«»­ çç

(a) Homocyclic but not aromatic because staggered conformation has


(b) Aromatic but not homocyclic torsional strain
(c) Homocyclic and aromatic (b) The eclipsed conformation of ethane is
(d) Heterocyclic and aromatic more stable than staggered conformation,
30. Which of the following possesses a sp-carbon because eclipsed conformation has no
in its structure ? [1989]
torsional strain
(a) CH2 = CCl – CH = CH2
(c) The eclipsed conformation of ethane is
(b) CCl2 = CCl2 more stable than staggered conformation
(c) CH2 = C = CH2 even though the eclipsed conformation has
(d) CH2 = CH – CH = CH2 torsional strain
31. Cyclic hydrocarbon ‘A’ has all the carbon and (d) The staggered conformation of ethane is
hydrogen atoms in a single plane. All the carbon more stable than eclipsed conformation,
carbon bonds have the same length, less than because staggered conformation has no
1.54 Å, but more than 1.34 Å. The C – C – C torsional strain.
bond angle will be [1989] 35. Two possible ster eo-structur es of
(a) 109 28' (b) 100° CH3CHOH.COOH, which are optically active, are
(c) 180° (d) 120° called. [2015 RS]
32. The Cl – C – Cl angle in 1,1,2,2- tetrachloroethene (a) Diastereomers (b) Atropisomers
and tetrachloromethane respectively will be
(c) Enantiomers (d) Mesomers
about [1988]
(a) 120° (b) 90° and 109.5° 36. Given
(c) 109.5° and 90° (d) 120° and 109.5° CH3 CH3 CH 3

̱°·½ îæ ×­±³»®·­³ ·² Ñ®¹¿²·½ ݱ³°±«²¼­ CH3 CH3


33. Which of the following biphenyls is optically CH3
active ? [2016] O O O
(a) O2N (I) (II) (III)
Which of the given compounds can exhibit
tautomerism? [2015]
(a) I and III (b) II and III
I
Br Br (c) I, II and III (d) I and II
37. Given
(b) Br Br H
I I H Br
I CH3 H

(c) H CH3 and CH3


CH3
Br
I
CH3
I II
(d) I and II are [NEET Kar. 2013]
CH3 (a) A pair of optical isomers
34. The correct statement regarding the comparison (b) Identical
of staggered and eclipsed conformation of (c) A pair of conformers
ethane, is [2016] (d) A pair of geometrical isomers
(a) The staggered conformation of ethane is 38. Which of the following acids does not exhibit
less stable than eclipsed conformation, optical isomerism ? [2012]
EBD_7324
ïðð ÝØÛÓ×ÍÌÎÇ
(a) Maleic acid (b) -amino acids (a) Butanol (b) 2-Butyne
(c) Lactic acid (d) Tartaric acid (c) 2-Butenol (d) 2-Butene
39. In the following the most stable conformation 42. How many stereoisomers does this molecule
of n-butane is: [2010] have ? [2008]
CH3 CH3 CH 3CH CHCH 2CHBrCH 3
H CH3 H H
(a) 4 (b) 6
(a) (b) (c) 8 (d) 2
43. If there is no rotation of plane polarised light by
H H H H
a compound in a specific solvent, though to be
H CH3
chiral, it may mean that [2007]
CH3 CH3
H (a) the compound is certainly meso
CH3
(b) there is no compound in the solvent
(c) (d) (c) the compound may be a racemic mixture
H (d) the compound is certainly achiral.
H
H H H 44. CH3 – CHCl – CH2 – CH3 has a chiral centre.
HH H3 C
Which one of the following represents its
40. Which of the following conformers for ethylene R-configuration? [2007]
glycol is most stable? [2010]
C2 H 5 C2 H5
OH | |
H OH (a) H C CH3 (b) Cl C CH 3
| |
Cl H
(a)
CH3 C2 H 5
| |
H H (c) H C Cl (d) H3C C Cl
H | |
C2 H5 H
OH
45. Which of the following is not chiral ? [2006]
H H
(a) 2, 3- Dibromopentane
(b) 3-Bromopentane
(b)
(c) 2-Hydroxypropanoic acid
H H (d) 2-Butanol
OH 46. The chirality of the compound [2005]
OH Br
OH
(c) C
H is
H3C Cl
H
H H H (a) R (b) S
OH (c) E (d) Z
H 47. Which one of the following pairs represents
stereoisomerism? [2005]
(d)
(a) Structural isomerism and Geometrical
isomerism
HO (b) Optical isomerism and Geometrical isomerism
H H H
(c) Chain isomerism and Rotational isomerism.
41. Which of the following compounds will exhibit
(d) Linkage isomerism and Geometrical
cis-trans (geometrical) isomerism? [2009]
isomerism
Ñ®¹¿²·½ ݸ»³·­¬®§ ó ͱ³» Þ¿­·½ Ю·²½·°´»­ ¿²¼ Ì»½¸²·¯«»­ ïðï

48. Number of chiral carbons in 53. A compound of molecular formula of C7H16


D ( ) glucose is [2004] shows optical isomerism, compound will be
(a) five (b) six [2001]
(c) three (d) four (a) 2, 3-Dimethylpentane
49. The molecular formula of diphenylmethane, (b) 2,2-Dimethylbutane
CH2 , is C13H12. (c) 3-Methylhexane
(d) None of the above
How many structural isomers are possible when
one of the hydrogens is replaced by a chlorine 54. Correct order of stability is : [2000]
atom? [2004] (a) cis -2- butene > 1-butene > trans -2-butene
(a) 6 (b) 4 (b) trans-2-butene > cis-2-butene > 1-butene
(c) 8 (d) 7 (c) 1-butene > cis-2-butene > trans-2- butene
50. Which of the following pairs of compounds are (d) cis-2-butene > trans-2-butene > 1-butene
enantiomers? [2003] 55. The correct structure of trans-2 hexenal is [1999]
CHO
CH3 CH3
(a) (b)
(a) HO H
and
HO H CHO
H OH HO H (c) CHO (d)
CH3 CH3 CHO
56. Which is a chiral molecule? [1999]
CH3 CH3
(a) 2, 2-dimethylbutanoic acid
(b) H OH HO H
and (b) 4-methylpentanoic acid
HO H H HO
CH3 CH3 (c) 3-methylpentanoic acid
(d) 3,3-dimethylbutanoic acid
CH3 CH3
57. Which of the following compounds is not chiral?
(c) H OH HO H
and [1998]
HO H HO H
CH3 CH3 (a) DCH2CH2CH2Cl (b) CH3CH2CHDCl
(c) CH3CHDCH2Cl (d) CH3CHClCH2D
CH3 CH3
58. Tautomerism will be exhibited by [1997]
H OH H OH
(d) and (a) (CH3)2NH (b) (CH3)3CNO
HO H H OH
(c) R3CNO2 (d) RCH2NO2
CH3 CH3
59. The most stable conformation of n-butane is
..
51. C H2 C CH 3 and CH 2 [1997]
C CH 3 are
|| | (a) skew boat (b) gauche
O . .:
:O (c) staggered-anti (d) eclipsed
60. Which of the following will not show cis-trans
[2002] isomerism? [1996]
(a) Resonating structures (a) CH 3 — CH CH — CH 3
(b) Tautomers
(b) CH 3 — CH 2 — CH CH — CH 2 — CH 3
(c) Geometrical isomers
(d) Optical isomers (c) CH 3 — CH CH — CH 2 — CH 3
|
52. Geometrical isomers differ in [2002] CH3
(a) position of functional group
(b) position of atoms (d) CH 3— CH — CH CH — CH 2 — CH 3
|
(c) spatial arrangement of atoms CH3
(d) length of carbon chain
EBD_7324
ïðî ÝØÛÓ×ÍÌÎÇ
61. Which of the following will exhibit chirality? ̱°·½ íæ ݱ²½»°¬ ±º λ¿½¬·±² Ó»½¸¿²·­³ ·²
[1996]
Ñ®¹¿²·½ ݱ³°±«²¼­
(a) 2-Methylhexane (b) 3-Methylhexane
(c) Neopentane (d) Isopentane 70. The most suitable method of separation of 1 : 1
62. The number of possible isomers of the compound mixture of ortho and para-nitrophenols is :
with molecular formula C7H8O is [1995] [2017]
(a) 3 (b) 5
(a) Chromatography
(c) 7 (d) 9
63. An important chemical method to resolve a racemic (b) Crystallisation
mixture makes use of the formation of [1994] (c) Steam distillation
(a) a meso compound (b) enantiomers (d) Sublimation
(c) diasteromers (d) racemates 71. The correct statement regarding electrophile is:-
64. The process of separation of a racemic [2017]
modification into d and -enantiomers is called
(a) Electrophile is a negatively charged species
[1994]
(a) Resolution and can form a bond by accepting a pair of
(b) Dehydration electrons from another electrophile
(c) Revolution (b) Electrophiles are generally neutral species
(d) Dehydrohalogenation and can form a bond by accepting a pair of
65. The restricted rotation about carbon carbon electrons from a nucleophile
double bond in 2-butene is due to [1993] (c) Electrophile can be either neutral or
(a) Overlap of one s- and sp2 - hybridized positively charged species and can form a
orbitals bond by accepting a pair of electrons from
(b) Overlap of two sp2 - hybridized orbitals a nucleophile
(c) Overlap of one p- and one sp2 - hybridized (d) Electrophile is a negatively charged species
orbitals
and can form a bond by accepting a pair of
(d) Sideways overlap of two p- orbitals.
electrons from a nucleophile
66. Isomers of a substance must have the same
[1991] 72. The pair of electron in the given carbanion,
(a) Structural formula CH3C C , is present in which of the following
(b) Physical properties orbitals ? [2016]
(c) Chemical properties (a) 2p (b) sp 3
(d) Molecular formula (c) sp 2 (d) sp
67. Which one of the following can exhibit cis-trans 73. Which of the following statements is not correct
isomerism ? [1989] for a nucleophile? [2015 RS]
(a) CH 3 CHCl COOH (a) Nucleophile is a Lewis acid
(b) H C C Cl (b) Ammonia is a nucleophile
(c) (c) Nucleophiles attack on less e– density sites
ClCH CHCl
(d) Nucleophiles are not electron seeking.
(d) ClCH 2 CH 2Cl 74. Consider the following compounds [2015]
68. How many chain isomers could be obtained from
the alkane C6H14? [1988] CH3 Ph
(a) Four (b) Five CH3—C—CH— Ph—C—PH
(c) Six (d) Seven CH3
69. Which of the following is an optically active CH3
compound ? [1988] (I) (II) (III)
(a) 1-Butanol
Hyperconjugation occurs in :
(b) 1-Propanol
(c) 2-Chlorobutane (a) II only (b) III only
(d) 4-Hydroxyheptane (c) I and III (d) I only
Ñ®¹¿²·½ ݸ»³·­¬®§ ó ͱ³» Þ¿­·½ Ю·²½·°´»­ ¿²¼ Ì»½¸²·¯«»­ ïðí
75. Which of the following is the most correct 79. Some meta-directing substituents in aromatic
electron displacement for a nucleophilic reaction substitution are given. Which one is most
to take place? [2015] deactivating? [NEET 2013]
H H2 (a) –SO3H (b) –COOH
(a) H3C—C = C – C – Cl (c) –NO2 (d) –C N
H
H H2 80. Which of the following compounds will not
(b) H3C—C = C – C – Cl undergo Friedal-Craft’s reaction easily :
H
[NEET 2013]
H H2
(c) H3C—C = C – C – Cl (a) Xylene (b) Nitrobenzene
H (c) Toluene (d) Cumene
H H2 81. Arrange the following in increasing order of
(d) H 3C—C = C – C – Cl
H stability [NEET Kar. 2013]
76. In Duma's method for estimation of nitrogen,
0.25 g of an organic compound gave 40 mL of
(A) (CH3 )2 C CH2CH3 (B) (CH3 )3 C
nitrogen collected at 300 K temperature and 725
mm pressure. If the aqueous tension at 300 K is
25 mm, the percentage of nitrogen in the (C) (CH3 ) 2 CH (D) CH 3 CH 2
compound is : [2015]
(a) 18.20 (b) 16.76
(c) 15.76 (d) 17.36 (E) CH 3
77. Which of the following compounds will undergo (a) E < D < C < B < A (b) E < D < C < A < B
racemisation when solution of KOH hydrolyses? (c) D < E < C < A < B (d) A < E < D < C < B
[2014]
82. Homolytic fission of the following alkanes forms
CH2Cl free radicals CH3 – CH3, CH3 – CH2 – CH3,
(i) (CH3)2 CH – CH3, CH3 – CH2 – CH (CH3)2.
Increasing order of stability of the radicals is
(ii) CH3CH2CH2Cl [NEET Kar. 2013]
(a) (CH3)3 C < (CH3)2 C – CH2CH3 <
CH3
| CH3 – C H – CH3 < CH3 – C H2
(iii) H3C C H CH2 Cl
(b) (CH3)2 C – CH2CH3 < CH3 – C H – CH3 <
CH3 CH3 – C H2 < (CH3)3 C

C (c) CH3 – C H2 < CH3 – C H – CH3 <


(iv) H Cl
C2H3 (CH3)2 C – CH2 – CH3 < (CH3)3 C

(a) (i) and (ii) (b) (ii) and (iv) (d) CH3 – C H2 < CH3 – C H – CH3 < (CH3)3
(c) (iii) and (iv) (d) (i) and (iv) C < (CH3)2 C – CH2CH3
78. In the Kjeldahl’s method for estimation of 83. What is the hybridisation state of benzyl
nitrogen present in a soil sample, ammonia +
evolved from 0.75 g of sample neutralized 10 mL carbonium ion —CH2?
of 1 M H2SO4. The percentage of nitrogen in
the soil is : [2014] [NEET Kar. 2013]
(a) 37.33 (b) 45.33 (a) sp3 (b) sp2
(c) 35.33 (d) 43.33 (c) spd 2 (d) sp2 d
EBD_7324
ïðì ÝØÛÓ×ÍÌÎÇ
84. Nitrogen detection in an organic compound is Cl
carried out by Lassaigne’s test. The blue colour Cl
formed corresponds to which of the following (a) (b)
formulae? [NEET Kar. 2013] NO2
(a) Fe3[Fe(CN)6]3 (b) Fe3[Fe(CN)6]2
CH3
(c) Fe4[Fe(CN)6]3 (d) Fe4[Fe(CN)6]2
85. Among the following compounds the one that Cl
is most reactive towards electrophilic nitration is: Cl
[2012] (c) (d)
(a) Benzoic acid (b) Nitrobenzene
(c) Toluene (d) Benzene OCH3
90. In Duma's method of estimation of nitrogen
86. Which one of the following is most reactive 0.35 g of an organic compound gave 55 mL of
towards electrophilic reagent ? [2011] nitrogen collected at 300 K temperature and
CH3 CH3 715 mm pressure. The percentage composition
of nitrogen in the compound would be :
(a) (b) (Aqueous tension at 300 K = 15 mm) [2011]
OCH3 OH
(a) 15.45 (b) 16.45
CH3 CH3
(c) 17.45 (d) 14.45
(c) (d) 91. The Lassaigne’s extract is boiled with conc.
NHCOCH3 CH2OH
HNO3 while testing for halogens. By doing so it
87. The correct order of increasing bond length of [2011]
C – H, C – O, C – C and C = C is : [2011] (a) decomposes Na2S and NaCN, if formed.
(a) C – H < C = C < C – O < C – C (b) helps in the precipitation of AgCl.
(b) C – C < C = C < C – O < C – H (c) increases the solubility product of AgCl.
(c) C – O < C – H < C – C < C = C (d) increases the concentration of NO 3– ions.
(d) C – H < C – O < C – C < C = C 92. Given are cyclohexanol (I) acetic acid (II),
88. Which one is a nucleophilic substitution 2, 4, 6 – trinitrophenol (III) and phenol (IV). In
reaction among the following ? [2011] these the order of decreasing acidic character
will be : [2010]
(a) CH3 – CH = CH2 + H2O (a) III > II > IV > I (b) II > III > I > IV
(c) II > III > IV > I (d) III > IV > II > I
CH3 – CH – CH3 93. The correct order of increasing reactivity of
C – X bond towards nucleophile in the following
OH compounds is: [2010]
X X
(b) RCHO + R MgX R – CH – R NO2
OH (CH3)3 C – X, (CH3)2CH – X
CH 3
(c) CH 3 – CH2–CH–CH2Br NO2
(I) (II) (III) (IV)
CH3 (a) I < II < IV < III (b) II < III < I < IV
(c) IV < III < I < II (d) III < II < I < IV
CH 3– CH 2–CH–CH 2NH 2 94. Among the given compounds, the most
susceptible to nucleophilic attack at the carbonyl
(d) CH3CHO + HCN CH3CH (OH) CN group is: [2010]
89. Which of the following compounds undergoes (a) CH 3COOCH3 (b) CH3CONH2
nucleophilic substitution reaction most easily ?
[2011 M] (b) CH 3COOCOCH3 (d) CH3COCl
Ñ®¹¿²·½ ݸ»³·­¬®§ ó ͱ³» Þ¿­·½ Ю·²½·°´»­ ¿²¼ Ì»½¸²·¯«»­ ïðë

95. Which one is most reactive towards electrophilic 101. Consider the following compounds. [2007]
reagent? [2010] (i) C6H5COCl
CH3 CH3
(ii) O2N COCl
OH CH2OH
(a) (b)
(iii) H3C COCl
CH3
CH3
(iv) OHC COCl
NHCOCH3 (d)
OCH3
(c) The correct decreasing order of their reactivity
towards hydrolysis is
96. Which of the following species is not (a) (i) > (ii) > (iii) > (iv)
electrophilic in nature? [2010] (b) (iv) > (ii) > (i) > (iii)
(c) (ii) > (iv) > (i) > (iii)
(a) NO 2 (b) Cl (d) (ii) > (iv) > (iii) > (i)
102. For (i) I–, (ii) Cl–, (iii) Br–, the increasing order of
(c) BH3 (d) H3O nucleophilicity would be [2007]
97. Which one of the following is most reactive (a) Cl– < Br– < I– (b) I– < Cl– < Br–
towards electrophilic attack ? [2008] (c) Br– < Cl– < I– (d) I– < Br– < Cl–
CH 2OH NO2
103. The order of decreasing reactivity towards an
electrophilic reagent, for the following would be
(a) (b) [2007]
(i) benzene (ii) toluene
Cl (iii) chlorobenzene (iv) phenol
OH (a) (ii) > (iv) > (i) > (iii) (b) (iv) > (iii) > (ii) > (i)
(c) (d) (c) (iv) > (ii) > (i) > (iii) (d) (i) > (ii) > (iii) > (iv)
104. Which of the following undergoes nucleophilic
substitution exclusively by SN1 mechanism?
98. Base strength of : [2008] [2005]
– –
(A) H 3 C – CH 2 (B) H 2 C CH and (a) Ethyl chloride
– (b) Isopropyl chloride
(C) H C C (c) Chlorobenzene
is in the order of : (d) Benzyl chloride
(a) (B) > (A) > (C) (b) (C) > (B) > (A) 105. Which amongst the following is the most stable
(c) (A) > (C) > (B) (d) (A) > (B) > (C) carbocation? [2005]
99. A strong base can abstract an -hydrogen
from : [2008] (a) CH 3 (b) CH3 CH 2
(a) alkene (b) amine CH3
(c) ketone (d) alkane |
100. The stability of carbanions in the following : (c) CH 3 C H (d) CH3 C
| |
CH 3 CH3
(a) RC C (b)
106. Which one of the following compounds is most
acidic? [2005]
(c) R 2 C CH (d) R 3C CH 2 (a) Cl–CH2–CH2–OH
is in the order of : [2008]
(a) (a) > (b) > (c) > (d) (b) (b) > (c) > (d) > (a) (b)
(c) (d) > (b) > (c) > (a) (d) (a) > (c) > (b) > (d)
EBD_7324
ïðê ÝØÛÓ×ÍÌÎÇ
112. In steam distillation of toluene, the pressure for
toluene in vapour is [2001]
(c) (d) (a) Equal to pressure of barometer
(b) Less than pressure of barometer
107. The best method for the separati on of (c) Equal to vapour pressure of toluene in
naphthalene and benzoic acid from their simple distillation
mixture is [2005] (d) More than vapour pressure of toluene in
(a) distillation (b) sublimation simple distillation
(c) chromatography (d) crystallisation 113. Which of the following compounds reacts
108. Which of the following is least reactive in a slower in electrophilic substitution? [2000]
nucleophilic substitution reaction. [2004] (a) C6H5NO2 (b) C6H5OH
(c) C6H5CH3 (d) C6H5NH2
(a) (CH 3 ) 3 C Cl (b) CH 2 CHCl 114. Consider the following phenols :
(c) CH 3CH 2Cl (d) CH 2 CHCH 2 Cl OH OH OH OH
109. The correct order of reactivity towards the
electrophilic substitution of the compounds
aniline (I), benzene (II) and nitrobenzene (III) is
[2003] NO2
(a) I > II > III (b) III > II > I CH3 NO2
I II III IV
(c) II > III > I (d) I < II > III
110. Which one of the following is a free-radical The decreasing order of acidity of the above
substitution reaction? [2003] phenols is [1999]
(a) III > IV > II > I (b) II > I > IV > III
(a) CH 3CHO HCN CH 3CH ( OH) CN
(c) I > IV > II > III (d) III > IV > I > II
CH3 CH2Cl 115. Which one of the following compounds will be
(b) +Cl2
Boiling
most easily attacked by an electrophile?[1998]
Cl
CH3
Anh. AlCl3
(c) +CH3Cl (a) (b)

CH3 OH
CH2Cl
(d) + AgNO 2 (c) (d)
CH2 NO2
116. Which one of the following compounds is
resistant to nucleophilic attack by hydroxyl ions?
[1998]
111. The correct order of acidic strength for following
(a) Methyl acetate (b) Acetonitrile
compounds will be [2001]
(c) Diethyl ether (d) Acetamide
OH OH OH
117. Which one of the following order is correct
regarding the – I effect of the substituents ?
[1998]
(a) – NR2 < – OR > – F
(b) – NR2 > – OR > – F
CH3 NO2 (c) – NR2 < – OR < – F
I II III (d) – NR2 > – OR < – F
(a) III > I > II (b) I > III > II
(c) II > III > I (d) I > II > III
Ñ®¹¿²·½ ݸ»³·­¬®§ ó ͱ³» Þ¿­·½ Ю·²½·°´»­ ¿²¼ Ì»½¸²·¯«»­ ïðé

118. Which one of the following is a technique most 124. An example of electrophilic substitution reaction
suitable for purification of cyclohexanone from is [1994]
a mixture containing benzoic acid, isoamyl (a) Chlorination of methane
alcohol, cyclohexane and cyclohexanone? [1997] (b) Conversion of methyl chloride to methyl alcohol
(a) Crystallization (c) Nitration of benzene
(b) Sublimation (d) Formation of ethylene from ethyl alcohol.
(c) IR spectroscopy 125. Lassaigne’s test for the detection of nitrogen
(d) Gas chromatography fails in [1994]
119. The reaction: (a) NH2CONHNH2.HCl
CH3 (CH2)5 (CH2)5 CH 3 (b) NH2NH2.HCl
H H
OH
– (c) NH2CONH2
C – Br HO – C
(d) C6H5NHNH2.HCl
H3C CH 3
126. The most suitable method for separtion of a 1 : 1
is described as [1997] mixture of ortho and para nitrophenols is [1994]
(a) SE2 (b) SN1 (a) Sublimation (b) Chromatography
(c) SN0 (d) SN2 (c) Crystallization (d) Steam distillation
120. Which one of these is not compatible with 127. A is a lighter phenol and B is an aromatic
arenes? [1997] carboxylic acid. Separation of a mixture of A and
(a) Greater stability B can be carried out easily by using a solution
(b) Delocalisation of -electrons of [1992]
(c) Electrophilic additions (a) Sodium hydroxide
(d) Resonance (b) Sodium sulphate
121. Among the following compounds (I-III), the (c) calcium chloride
ease of their reaction with electrophiles is, (d) Sodium bicarbonate
[1997] 128. The most reactive compound for electrophilic
OCH3 NO2 nitration is [1992]
(a) Benzene (b) Nitrobenzene
(c) Benzoic acid (d) Toluene.
129. Which of the following is the most stable
carbocation (carbonium ion) ? [1991]
I II III

(a) II > III > I (b) III > II > I (a) CH 3CH 2 (b) ( CH 3 ) 2 C H
(c) II > I > III (d) I > II > III
122. Decr easing order of reactivity towar ds
(c) ( CH 3 ) 3 C (d) C6 H 5 C H 2
nucleophilic addition to carbonyl group among
cyclopentanone, 3-pentanone and n-pentanal is 130. In sodium fusion test of organic compounds,
[1996] the nitrogen of the organic compound is
(a) 3-pentanone, cyclopentanone, n-pentanal converted into [1991]
(b) n-pentanal, 3-pentanone, cyclopentanone (a) Sodamide (b) Sodium cyanide
(c) n-pentanal, cyclopentanone, 3-pentanone (c) Sodium nitrite (d) Sodium nitrate
(d) cyclopentanone, 3-pentanone, n-pentanal 131. Kjeldahl’s method is used in the estimation of
123. Correct increasing order of acidity is as follows: [1990]
[1994] (a) Nitrogen (b) Halogens
(a) H2O, C2H2, H2CO3, Phenol (c) Sulphur (d) Oxygen
(b) C2H2, H2O, H2CO3, Phenol 132. Lassaigne’s test is used in qualitative analysis
(c) Phenol, C2H2, H2CO3, H2O to detect [1989]
(a) Nitrogen (b) Sulphur
(d) C2H2, H2O, Phenol, H2CO3
(c) Chlorine (d) All of these
EBD_7324
ïðè ÝØÛÓ×ÍÌÎÇ

ANSWER KEY
1 (d) 15 (a) 29 (a) 43 (a) 57 (a) 71 (c) 85 (c) 99 (c) 113 (a) 127 (d)
2 (a) 16 (c) 30 (c) 44 (b) 58 (d) 72 (d) 86 (b) 100 (d) 114 (a) 128 (d)
3 (d) 17 (c) 31 (d) 45 (b) 59 (c) 73 (a) 87 (a) 101 (c) 115 (d) 129 (c)
4 (a) 18 (b) 32 (d) 46 (a) 60 (c) 74 (b) 88 (c) 102 (a) 116 (c) 130 (b)
5 (a) 19 (d) 33 (b) 47 (b) 61 (b) 75 (b) 89 (a) 103 (c) 117 (c) 131 (a)
6 (a) 20 (c) 34 (d) 48 (a) 62 (b) 76 (b) 90 (b) 104 (d) 118 (c) 132 (d)
7 (d) 21 (b) 35 (c) 49 (b) 63 (c) 77 (N) 91 (a) 105 (d) 119 (d)
8 (b) 22 (d) 36 (c) 50 (b) 64 (a) 78 (a) 92 (a) 106 (c) 120 (c)
9 (c) 23 (d) 37 (c) 51 (a) 65 (d) 79 (c) 93 (a) 107 (b) 121 (d)
10 (c) 24 (b) 38 (a) 52 (c) 66 (d) 80 (b) 94 (d) 108 (b) 122 (c)
11 (a) 25 (b) 39 (b) 53 (a) 67 (c) 81 (b) 95 (d) 109 (a) 123 (d)
12 (a) 26 (d) 40 (a) 54 (b) 68 (b) 82 (c) 96 (d) 110 (b) 124 (c)
13 (b) 27 (a) 41 (d) 55 (a) 69 (c) 83 (b) 97 (c) 111 (a) 125 (b)
14 (b) 28 (d) 42 (a) 56 (c) 70 (c) 84 (c) 98 (d) 112 (b) 126 (d)

Hints & Solutions


O O 4 5
6. (a) Cl CH2CH3
C 2 C=C
1. (d) H 3 4 2 3
1 CH3 I
5 1
6 Correct IUPAC name of above compound is
3-k eto-2-methylhex-4-enal trans-2-chloro-3-iodo-2-pentene
Aldehydes get higher priority over ketone and
alkene in numbering of principal carbon chain. CH3 CH 3
| 3 2
sp3 | sp
OH
7. (d) CH3 sp C sp CH CH CH — C C H
7 6| 5 4 3 2 1
3 1
H 3C 2 COOH CH3
2. (a) 4
6
5 CH 3 8. (b) C – 1 is sp hybridized C C
IUPAC name of the structure is C – 3 is sp3 hybridized (C– C)
3-ethyl-2-hydroxy-4-methylhex-3-en-5-ynoic acid C – 5 is sp2 hybridized (C = C)
CH3 Thus the correct sequence is sp, sp 3, sp2.
3. (d) CH CH 2 (isobutyl group) 9. (c) General molecular formula of alkanols is
CH 3 CnH2n + 2O , (CnH2n + 1OH)
4. (a) Br CH2 CH CH 2 4
3 2 1
The correct name is 3-Bromoprop-1-ene. 3 2 1 Cl
10. (c) 5 C
5. (a) The given compound is ||
2 1 O
CH = CH2
It is 2, 3-dimethylpentanoyl chloride.
CH3 — CH2 — CH2 — C — C — CH2 — CH3 11. (a) Among the three given hybrid orbitals, sp
3 4 hybr id or bital is most electr onegative.
CH2 – CH3 Contribution of s in sp hybrid orbital is maximum
5 6 (50%) so this orbital is closer to nucleus. Naturally
4 ethyl – 3-propyl hex – 1 – ene
Ñ®¹¿²·½ ݸ»³·­¬®§ ó ͱ³» Þ¿­·½ Ю·²½·°´»­ ¿²¼ Ì»½¸²·¯«»­ ïðç

it will have greater tendency to pull electron As in this compound the common groups i.e
towards it. Hence it becomes more electronegative highly electronegative halogen atoms are on
and sp3 becomes least electronegative as it has opposite side, hence it is a trans isomer.
only 25% s-character. Thus its name is trans-2-chloro-3-iodo-2-pentene.
12. (a) Correct IUPAC name of 18. (b) When many substituents are present, the
CH3 numbering is done from the end where the sum of
| locants is the lowest (lowest sum rule)
CH 3CH 2 CH 2 C H C H CH 2 CH 3 7 6 5 4 3 2 1
|
CH 2CH3 C H3 CH CH C H 2 C H 2 C H C H3
| | |
is 4-Ethyl-3-methylheptane CH3 CH3 CH3
13. (b) CH3 CH3 2, 5, 6 – trimethylheptane (wrong)
1 3 5 7
4 2 + 5 + 6 = 13
2 8 CH3
6 1 2 3 4 5 6 7
CH3 — CH — CH — CH 2 — CH 2 — CH — CH 3
CH3 | | |
4–ethyl-3 –methyloctane CH3 CH3 CH3
14. (b) The IUPAC name of 2 + 3 + 6 = 11
2, 3, 6-trimethylheptane (correct)
1 2 3 4 5 6 19. (d) Huckel’s rule states that for aromaticity
C H2 C H — C H2 — C H2 — C C H
there must be (4n + 2) electrons present in a
is Hex-1-en-5-yne or 1-hexene-5-yne compound, where n is an integer.
The lowest number is given to the C = C double 20. (c) The vital force theory suffered first death
bond. blow in 1828 when Wohler synthesized the Ist
1 2 3 4 organic compound urea in the laboratory from
15. (a) CH3 C CH CH3 inorganic compounds reported below :
O CH3 to isomeric charge
3 Methyl-2- butanone NH 4 CNO NH 2 CONH 2
rearrangement leading
Urea
1 2 3 Later on a further blow to vital force theory was
CH3 CH CH CH3
4 5 given by Kolbe (1845) who prepared acetic acid,
CH3 CH2 CH3 the first organic compound, in laboratory from
its elements.
2, 3 dimethyl pentane
21. (b) 1 CH 3 2
CH 3CH 4 CH 2 5CH 3
4 3 2 1 | |
CH3 CH CH CH3
H 3C CH 2 CH 3
Cl Br 3-Ethyl-2-methylpentane
2 Bromo-3-chlorobutane 22. (d) Resonance structures are separated by a
5CH double headed arrow ( )
3 23. (d) Angle increases progressively
1 2 3 4
CH3 C C CH sp3 ( 109 28' ), sp2 (120°), sp (180°)
CH3 24. (b) 5 CH 3 4
CH 3CH 2 C 1CHO
4-Methyl-2-pentyne | |
OH CH 3
sp3 sp 2 sp sp 2
16. (c) CH CH C CH 4-Hydroxy-2-methylpent-2-en-1-al
3 2
Cl CH2 – CH3 CH 3
17. (c) C C |
H 3C I 25. (b) 1 CH 3 2 C 3CH 4 CH 3
2-Methyl-2-butene
EBD_7324
ïïð ÝØÛÓ×ÍÌÎÇ
sp3 sp2 sp2 sp3 sp2 sp sp repulsion between the electron clouds of -
26. (d) CH 3 CH CH CH 2 CH– C CH bonds of two non-bonded H-atomic (torsional
27. (a) sp3 orbital has 1/4(25%) s-character & 75% strain)
p-character. H HH
28. (d) Shortest C – C distance (1.20 Å) is in H H
acetylene, as acetylene has sp hybridisation.
The bond length increases in the order
H
C C C C C C H H H H
sp(1.20Å) 2 sp(1.54Å) H
sp (1.34Å) H
Staggered form Eclipsed form
NO 2
No torsional strain
NO 2
29. (a) or 35. (c) H

Hence it is homocyclic (as the ring system is made C COOH


of one type of atoms, i.e. carbon) but not aromatic. CH3
OH
As it does not follow (4n +2) electron rule of
aromaticity. Four different Substituent, only one chiral centre.
Hence only enantiomers are possible.
sp2 sp sp2 36. (c) All of these compounds show tautomerism
30. (c) CH 2 C CH 2
31. (d) All the properties mentioned in the question H3C CH3 H3C CH3
suggest that it is a benzene molecule. Since in
benzene all carbon s ar e sp 2 –hybridized,
therefore, C – C – C angle is 120° H
H H O OH
1.09Å 1.40Å CH3 CH3
C C CH3 CH3
120°
H C 120° C H H
C C O OH
H
H H
32. (d) Tetrachloroethene being an alkene has sp 2 - CH3 CH3
hybridized C– atoms and hence the angle
Cl – C – Cl is 120° while in tetrachloromethane, CH3 CH3
carbon is sp3 hybridized, therefore the angle Cl – O OH
C – Cl is 109.5°. 37. (c) Conformers are form of stereoisomers in
which isomers can be interconverted by rotation
Br Br about single bonds. I and II are staggered and
eclipsed conformers respectively.
38. (a) H H
33. (b) is optically active C— —C
HOOC COOH
(cis)
I I maleic acid
due to absence of plane of symmetry and center HOOC H
of symmetry. C——C
34. (d) In staggered confor mation an y two H COOH
(Trans)
hydrogen atoms on adjacent carbon atoms are
It shows geometrical isomerism but does not
as far apart as possible there by minimising
show optical isomerism.
Ñ®¹¿²·½ ݸ»³·­¬®§ ó ͱ³» Þ¿­·½ Ю·²½·°´»­ ¿²¼ Ì»½¸²·¯«»­ ïïï
39. (b) The bulky methyl groups are maximum away
from each other. 2
40. (a) Due to hydrogen bonding between the two C2H 5
OH groups, gauche conformation of ethylene 3
glycol (a) is the most stable conformation. 44. (b) Cl — C — CH3
H 1
O
O H 4
H
H R-configuration
45. (b)
| | | | |
H H (a) C – C*– C*– C – C – [Chiral]
| | | | |
H Br Br
41. (d) Alkenes with double bonds cannot undergo | | | | |
free rotation and can have different geometrical (b) – C – C – C – C – C – [Not chiral]
| | | | |
shapes with two different groups on each end of Br
the double bond. OH
CH3 CH3 CH3 H | |
C=C C=C (c) – C – C*– COOH [Chiral]
H H H CH3 | |
OH
cis-But-2-ene trans-But-2-ene | | | |
42. (a) In the molecule *
(d) – C – C – C – C – [Chiral]
CH 3CH CHCH 2 CH CH 3, | | | |
| * marked are chiral carbons.
Br 46. (a) Clockwise rotation.
the number of stereoisomers is given by sum of
geometrical isomers (because of presence of C = 1 Br
H4
C) and optical isomers (because of presence of
chiral carbon atom). C
H3 C Cl 2
Number of geometrical isomers = 2 (one C = C is 3
present). Hence configuration is R.
Number of optical isomers = 2 (one chiral carbon
If the eye travel in a clockwise direction, the
atom).
Total number of stereoisomers = 2 + 2 = 4 configuration is (R) as the order of priority is
43. (a) Compounds which do not show optical Br > Cl > CH3 > H.
activity inspite of the presence of chiral carbon 47. ø¾÷ Ñ°¬·½¿´ ¿²¼ ¹»±³»¬®·½¿´ ·­±³»®·­³ °¿·® «°
atoms are called meso-compounds. The absence ¬± »¨¸·¾·¬ ­¬»®»±·­±³»®·­³ò ̸·­ ·­ ¾»½¿«­» ¬¸»
of optical activity in these compound is due to ·­±³»®­ ¼·ºº»® ±²´§ ·² ¬¸»·® ±®·»²¬¿¬·±² ·² ­°¿½»ò
the presence of a plane of symmetry in their 1
48. (a) HO
molecules. e.g. meso-tartaric acid is optically C H
inactive. 2
H OH
COOH 3
HO H
H OH 4 O – D –(+)– Glucose
(Plane of symmetry) H OH
5
H OH H
CH2OH
COOH
Carbon atoms from C1 to C 5 are chiral
EBD_7324
ïïî ÝØÛÓ×ÍÌÎÇ
49. (b) In diphenylmethane monochlorination at 53. (a) A compound is said to exhibit optical
following positions will produce structured isomerism if it atleast contains one chiral carbon
isomers atom, which is an atom bonded to 4 different atoms
or groups.
CH2 CH3
|
(CH 3 ) 2 CH C* CH 2 CH3
|
1 H
2 3 4
54. (b) Stability of an alkene depends upon the heat
of hydrogenation of an alkene. The lower the
Cl
– heat of hydrogenation of an alkene higher will be
CH2
stability.
Order of stability Heat of hydrogenation
(1) (kJ/mol)
trans-2-butene 115.5
CH2 cis-2-butene 119.6 and
1-butene 126.8 respectively.

Cl 55. (a) When similar atoms are on the opposite side,
(2) the compound is in trans-form.
H CHO
CH2 C C

C3H7 H
Cl
56. (c) Chiral molecules are those molecules which
(3) have atleast one symmetric carbon atom (a carbon

Cl atom attached to 4 different groups). This is true
in case of 3-methylpentanoic acid.
CH2
H
|
(4) C2H5 – C CH2 COOH
|
50. (b) Compound which are mirror image of each
CH3
other and are not superimposable are termed as
enantiomers. 57. (a) Due to absence of a symmetric (chiral)
CH3 CH3 C-atom. D — CH 2 — CH 2 — CH 2 Cl
H OH molecule is not a chiral molecule.
HO H
and 58. (d) Tautomerism is exhibited by the oscillation
HO H H OH of hydrogen atom between two polyvalent atoms
CH3 CH3 present in the molecule. As option (d) has -
– hydrogen atom. Therefore it shows tautomerism
51. (a) CH2 — C — CH3 CH2 = C — CH3 whereas other structures do not.
O OH
O O R CH2 N R CH N

I II Nitroform O Aciform O
both are resonating structures. 59. (c) Order of stability : staggered (anti) > gauche
52. (c) Geometrical isomers differ in spatial > skew boat > eclipsed.
arrangement of atoms. Newman projection of n-butane is given as
Ñ®¹¿²·½ ݸ»³·­¬®§ ó ͱ³» Þ¿­·½ Ю·²½·°´»­ ¿²¼ Ì»½¸²·¯«»­ ïïí

CH3 CH3 CH3


67. (c) Such isomers, which possess the same
molecular and structural formula but differ in the
H H H3C H
arrangement of atoms around the double bonded
H
H3C carbon atoms are known as geometrical isomers.
H H H H H H
CH3
H
H H C Cl H C Cl
staggered (Anti) Eclipsed
|| ||
Skew or gauche
H C Cl Cl C H
The staggered conformation is most stable in
(cis) (trans)
which methyl groups are far apart as for as
68. (b) Five chain isomers are possible which are –
possible, due to minimum repulsion between
(i) CH 3 CH 2 CH 2 CH 2 CH 2 CH 3
methyl groups and is also called anti
n-hexane
conformation.
CH3 (ii) CH 3 CH CH 2 CH 2 CH 3
|
60. (c) CH CH CH2 CH3 CH 3
CH3 2-methyl pentane
Due to presence two similar methyl group at same CH 3
carbon atom, above compound doesn’t show |
(iii) CH 3 C CH 2 CH 3
geometrical isomerism. |
61. (b) H 3C — CH 2 — *CH — CH 2 — CH 2 — CH3 CH 3
| 2, 2-dimethyl butane
CH3 (iv) CH 3 CH CH CH 3
3-Methylhexane | |
Due to presence of four different groups on CH 3 CH 3
carbon,(C*) it is chiral 2, 3-dimethyl butane
62. (b) The possible isomers of the compound with (v) CH 3 CH 2 CH CH 2 CH 3
molecular formula C 7H8 O is 5. These are |
CH 3
C6H5OCH3, C6H5CH2OH and
(Anisole) benzyl alcohol 3-methyl pentane
CH3 CH3 CH3 H
|
OH 69. (c) H3C CH 2 C* CH 3
|
Cl
o- cresol OH The compound containing a chiral carbon atom
OH
p-cresol m-cresol i.e., (a carbon atom which is attached to four
63. (c) Diastereomers since they have different different substituents is known as a chiral carbon
atom) is optically active.
melting points, boiling points, solubilities etc.
64. (a) Resolution. 70. (c) Steam distillation is the most suitable method
65. (d) Rotation around bond is not possible. If of separation of 1 : 1 mixture of ortho and para
any attempt is made to rotate one of the carbon nitrophenols as there is intramolecular hydrogen
atoms, the lobes of -orbital will no longer remain bonding in o-nitrophenol.
coplanar i.e no parallel overlap will be possible 71. (c)
and thus -bond will break . This is known as 72. (d) CH3—C C
concept of restricted rotation. In other words the
No.of bp 1
presence of -bonds makes the position of two 2 & hybridisation is sp
p 1
carbon atom.
66. (d) Organic compounds having same molecular 73. (a) Nucleophile is a species that provide electron
formula but differ from each other in physical or while species which are deficient of electrons are
chemical properties or structural formula are termed as lewis acid, hence nucleophiles are
known as isomers. usually lewis bases.
EBD_7324
ïïì ÝØÛÓ×ÍÌÎÇ
74. (b) Only structure (III) has H in conjugation with 83. (b) In the carbonium ion the carbon atom
free radical. carrying the positive charge is sp2 hybridized.
So, hyperconjugation is possible in III only. 84. (c) The prussian blue colour is of Fe4 [Fe(CN)6]3

75. (b) bond is transferred after leaving Cl ferric ferrocyanide
85. (c) Electrophilic rate order
CH 3 – CH=CH.CH2 —Cl CH 3 – CH=CH—CH2 CH3
CH3—CH—CH=CH 2
76. (b) Wt. of organic substance = 0.25 g > >
V1 = 40 mL, T1 = 300 K
P1 = 725 – 25 = 700 mm of Hg Toluene Benzene
P2 = 760 mm of Hg (at STP) COOH NO2
T2 = 273 K
P1V1 P2 V2 >
T1 T2
V2 (Volume of nitrogen at STP) Benzoic acid Nitrobenzene
273 700 40 Since nitration is an electrophilic substitution
= 33.52 mL
300 760 hence presence of electron releasing group like
Percentage of nitrogen CH3 in the nucleus facilitates nitration.
86. (b) Due to + M effect of – OH group and
28 × volume of N 2 at STP ×100 hyperconjugation of – CH3 group.
= 22400 × wt. of organic substance 87. (a) Bond length order is
28 33.52 100 C H C C C O C C
= 16.76% 1.10A 1.34A 1.40A 1.54A
22400 0.25 88. (c) Because of high electronegativities of the
77. (N) Out of the given four compounds only (iv) halogen atom, the carbon halogen (C – X) is highly
compound is chiral a nd hen ce only this polarised covalent bond. Thus, the carbon atom
compound will undergo racemisation. Therefore of the C – X bond becomes a good site for attack
by nucleophiles (electron rich species).
none of the given option is correct.
Nucleophilic substitution reactions are the most
78. (a) 10 ml, 1 M H2SO4 = 20 ml, 1 M NH3 common reactions of alkyl halides.
wt of N in one mole NH3 = 14 89. (a) In SN reactions, a carbanion is formed as an
20 × 10– 3 mol NH3 intermediate, so any substituent that increases the
20 × 10– 3 × 14 nitrogen stability of carbanion and hence the transition state
0.75 g of sample contains leading to its formation will enhance the SN reactions.
To compare the rates of substitution, in
14 20 10 3 chlorobenzene, electron-withdrawing group, and
= 100 = 37.33%
0.75 chlorobenzene having electron-releasing group, we
79. (c) Decreasing order of deactivating effect of compare the structures carbanion I (from
the given m-directing group is chlorobenzene), II (from chlorobenzene containing
> NO2 > – CN > – SO3H > – COOH electron-withdrawing group) and III (from
—NO2 group is most deactivating group due chlorobenzene containing electron-releasing group).
to strong – E, – I and – M effects. Z Cl Z Cl
80. (b) –NO2 is a powerful electron withdrawing
group. Its presence on ring makes the ring less – –
active.
81. (b) Greater the number of e– donating alkyl
groups (+I effect), greater will be the stability of G
I II
carbocations.
G withdraws electrons, neutralises (disperses) –
82. (c) Stability depends on number of
hyperconjugative structure. ve charge of the ring, stabilises carbanion,
facilitates SN reaction (activation effect)
Ñ®¹¿²·½ ݸ»³·­¬®§ ó ͱ³» Þ¿­·½ Ю·²½·°´»­ ¿²¼ Ì»½¸²·¯«»­ ïïë
Z Cl (II) (III)
OH OH

>
III G
G releases electrons, intensifies –ve charge, (IV) (I)
destabilizes carbanion, retards SN reaction Explanation: Presence of three — NO2 groups in
(deactivation) o–, p– positions to phenolic groups (in III) makes
NO2 is activating group and CH3 and OCH3 are phenol strongly acidic because its corresponding
deactiving group. phenate ion (conjugate base) is highly stabilised
Hence, the cor rect order of nucleophilic
due to resonance. Conjugate base of CH3COOH,
substitution reaction is :
Cl Cl Cl II (i.e. CH3 COO ) is resonance hybrid of two
Cl equivalent structures. The conjugate base of
phenol, IV is stabilized due to resonance note
â â â that here all resonating structures are not
equivalent). The conjugate base of cyclohexanol,
NO2 CH3 OCH3 I does not exhibit resonance, hence not formed.
90. (b) Given wt of compound taken (w) = 0.35 g 93. (a) Tertiary alkyl halide is most reactive towards
Volume of nitrogen collected (V) = 55 ml nucleophilic substitution because th e
Room temperature (t K) = 300 K corresponding carbocation (3°) is most stable.
Atmospheric pressure (P) = 715 mm Aryl halide is least reactive due to partial double
Aq. tension ( ) = 15 mm bond character of the C – Cl bond.
Calculation - Presence of — NO 2 groups in ortho and para
Volume of N2 at NTP
positions increases the reactivity of the – Cl
(P ) V 273 towards nucleophiles.
= ml
t 760 (CH3)3 – C – X > (CH3)2 – CH – X >
III IV
(715 15) 55 273
= = 46.098 ml X
300 760 X
% of nitrogen NO2
28 vol.of N 2 at NTP 100 >
= 22400 wt of organic compound
NO2
II I
28 46.098 100 or I < II < IV < III
= 16.46%
22400 0.35 94. (d) Cl– is the weakest base and hence better
91. (a) Conc. HNO3 decomposes NaCN and Na2S leaving group.
to avoid their interference. CH3
NaCN HNO 3 NaNO 3 HCN OCH3
95. (d)
Na 2S 2HNO 3 2NaNO 3 H 2S
Among –OH, –CH2OH, –NHCOCH3 and –OCH3,
OH methoxy group has the highest +M effect.
92. (a) N2O NO2 > CH3COOH > H
96. (d) H3O H – O+– H has a lone pair of
NO2 electrons on oxygen atom, thus it is not an
electrophile. Also the octet is complete.
EBD_7324
ïïê ÝØÛÓ×ÍÌÎÇ
97. (c) Out of the given compounds the most which NO2 exhibit more –I effect than –CHO. On
reactive towards nucleophilic attack is the other hand CH3 is a electron releasing group
OH therefore the order of r eactivity towar ds
hydrolysis is
COCl COCl COCl COCl

> > >


Phenoxide ion is stable due to resonance.
98. (d) The amount of s-character in various hybrid NO2 CHO CH3
orbitals is as follows.
102. (a) Nucleophilicity increases down the periodic
sp = 50%, sp2 = 33% and sp3 = 25%
Therefore s-character of the C – H bond in table : I Br Cl F
acetylene (sp) is greater than that of the 103. (c) Electrophiles have high affinity for electrons.
C – H bond in alkene (sp 2 hybridized) which in They attack at the site where electron-density is
turn has greater s-character of the C – H bond in highest. Electron donating groups increases the
alkanes. Thus owing to a high s-character of the electron density. The electron donating tendency
C – H bond in alkynes, the electrons constituting decreases in the order :
this bond are more strongly held by the carbon –OH > –CH3 > –H > –Cl
nucleus with the result the hydrogen present on Therefore, the correct order of reactivity towards
such a carbon atom can be easily removed as electrophile is
proton. The acidic nature of three types of C6H5OH > C6H5CH3 > C6H6 > C6H5Cl
C – H bonds follows the following order 104. (d) SN 1 reaction is favoured by heavy groups
C H C H C H. on the carbon atom attached to halogen i.e
Ben zyl > a ll yl > ter tiar y > pr i mar y >
Further, as we know that conjugate base of a secondary > primary > alkyl halides
strong acid is a weak base, hence the correct
order of basicity is
( ) ( ) ( )
H C C CH 2 CH CH 2 CH 3
99. (c) A strong base can abstract an -hydrogen
from a ketone. Obtained from SN 1 path.
100. (d) The stability of carbanions is effected due This molecule is resonance stabilised.
to resonance, inductive effect and s- character 105. (d) More the number of alkyl groups, greater
of orbitals. Greater the number of groups having will be the dispersal of positive charge and
+ I group (alkyl group) lesser stable would be the therefore more the stability of carbocation.
carbanion.
CH3
Further stability of carbanion decreases with |
decrease in s-character. Benzene carbanions are CH3 – C
Thus | is most stable
stablized due to resonance, hence the correct
order is CH3
106. (c) Phenols are more acidic than alcohols as
they are resonance stabilised whereas alcohols
R C C R 2 C = CH> >R 3 C – CH2
are not.
The correct order of stability of given carbanion Further –NO2 is an electron withdrawing group
is in the order a > c > b > d. which increases acidic character and facilitates
101. (c) The degree of hydrolysis increases as the release of proton, whereas –CH3 is an electron
magnitude of positive charge on carbonyl group donating group which decr eases acidic
increases. Electron withdrawing group increases character, thus removal of H+ becomes difficult.
the positive charge and electron releasing group ïðéò ø¾÷ ß³±²¹ ¬¸ » ¹·ª»² ½ ±³°±«²¼­
decreases the positive charge. Among these NO2 ²¿°¸¬¸¿´»²» ·­ ª±´¿¬·´» ¾«¬ ¾»²¦±·½ ¿½·¼ ·­
& CHO are electron withdrawing group from ²±²óª±´¿¬·´» ø·¬ º±®³­ ¿ ¼·³»®÷ò ͱô ¬¸»
Ñ®¹¿²·½ ݸ»³·­¬®§ ó ͱ³» Þ¿­·½ Ю·²½·°´»­ ¿²¼ Ì»½¸²·¯«»­ ïïé
¾»­¬ ³»¬¸±¼ º±® ¬¸»·® ­»°¿®¿¬·±² ·­ Toluene boils when total pressure is equal to
­«¾´·³¿¬·±²ô ©¸·½¸ ·­ ¿°°´·½¿¾´» ¬± atmospheric pressure p i.e p = p1 + p2
½±³°±«²¼­ ©¸·½¸ ½¿² ¾» ½±²ª»®¬»¼ or p2 = p – p1
¼·®»½¬´§ ·²¬± ¬¸» ª¿°±«® °¸¿­» º®±³ ·¬­
As a result when toluene boil in the presence of
­±´·¼ ­¬¿¬» ±² ¸»¿¬·²¹ ¿²¼ ¾¿½µ ¬± ¬¸» ­±´·¼
­¬¿¬» ±² ½±±´·²¹ò Ø»²½» ·¬ ·­ ¬¸» ³±­¬ steam its partial pressure p 2 is less than
¿°°®±°®·¿¬» ³»¬¸±¼ò atmospheric pressure.
113. (a) CH3 and NH2 and OH are electron donating
108. (b) H 2 C CHCl is capable of showing
groups, whereas NO2 is an electron withdrawing
resonance which develops a partial double bond, group and leaves the benzene ring deactivated.
character to C–Cl bond, thereby making it less Due to stronger electron attracting (–I effect)
reactive towards nucleophilic substitution. effect of NO 2 group C6 H5 NO2 shows least
_ reactivity towards electrophilic substitution.
H 2 C== CH – C l H 2C – CH=Cl + 114. (a) Electron withdrawing group (–NO 2 )
109. (a) Amino group is ring activating while nitro increases the acidity while electron releasing
group is deactivating. Hence, correct order is group (–CH3, –H) decreases acidity. Also effect
aniline > benzene > nitrobenzene. will be more if functional group is present at para
I > II > III position then ortho and meta position.
NH2 NO2 115. (d) –Cl atom shows o/p-directive influence
but deactivate th e benzen e r in g, while
[–OH/–CH3] groups show o/p influence and
activate the benzene ring but –OH group is more
activating than –CH3.
I II III Hence order of electrophilic substitution
–NO2 is an electron attracting group hence OH CH3 Cl
decrease the electron density on ring, whereas –
NH2 group is electron releasing group hence
increase electron density on ring. Benzene is also
e– rich due to delocalization of electrons. 116. (c) The compound is diethyl ether (CH3CH2)2O
110. (b) In the presence of UV rays or energy O 2 by which is resistant to nucleophilic attack by
boiling, free radical is generated which attack the hydroxyl ion due to absence of double or triple
methyl carbon atom of toluene. bond, whereas all other compounds given are
unsaturated.
CH3 CH2 O
+H ||
C2 H5 OC2 H5 CH3 C OCH3
ether Methyl acetate
benzyl free
radical O
||
2Cl•
h
Cl 2 CH3 C N CH3 C NH2
•CH CH2Cl
Acetonitrile acetamide
2 • 117. (c) The atom or group which has more power to
+ Cl attract electrons in comparision to hydrogen is
said t o have -I effect. Thus high er the
111. (a) Electron releasing group (–CH3)decreases electronegativity of atom stronger will be the -I
acidity while electron withdrawing group effect. As electronegativity of N, O & F follow
the order N < O < F hence based upon
(–NO2)increases acidity.
electronegative character order of-I effect is –
112. (b) The principle of steam distillation is based
NR2 < – OR < – F.
on Dalton's law of partial pressures. Suppose p1 118. (c) IR spectroscopy is used for the purification of
and p2 be the vapour pressures of water vapour cyclohexanone from a mixture of benzoic
and the toluene at the distillation temperature. acid, isoamyl alcohol, cyclohexane and
EBD_7324
ïïè ÝØÛÓ×ÍÌÎÇ
cyclohexanone. Because in this method, each substitution reaction.
functional group appear at a certain peak. So, 125. (b) Hydrazine (NH2 NH2 ) does not contain
cyclohexanone can be identified by carbonyl peak. carbon and hence on fusion with Na metal, it
119. (d) When (–) 2 bromooctane is allowed to react cannot form NaCN; so it does not sh ow
with sodium hydroxide under given conditions, Lassaigne’s test for nitrogen.
where second order kinetics is followed, the 126. (d) The boiling point of o-nitrophenol is less
product obtained is (+) 2-octanol. than para-nitrophenol due to presence of
C6H13 H C6H13 C6H13


intramolecular h ydrogen bonding. Sin ce
OH
H C Br HO -----C -----Br HO C H + Br p-nitrophenol is less volatile than o-nitrophenol
CH3 CH3 CH3 due to presence of inter-molecular hydrogen
(–) 2-Bromooctane Transition state (+) 2–octanol bonding hence they can be separated by steam
In this reaction Walden Inversion takes place so distillation.
it is an example of S N2-reaction.
120. (c) In arenes, electrons are delocalised, hence 127. (d) Carboxylic acids dissolve in NaHCO 3 and
arenes do not undergo addition reactions easily. evolve CO2 gas but phenols do not.
Aromatic compounds (Arenes) are highly stable NaHCO3
RCOOH RCOONa + H 2O + CO2
and show resonance eg. Benzene is the simplest
example. 128. (d) Due to + I-effect of the CH3 group, toluene
121. (d) –OCH3 activates the benzene ring. –NO2 has much higher electron density in the ring than
deactivates the ring. Hence the reaction of the benzene. Nitrobenzene and benzoic acid as nitro
given compounds with electrophiles is in the and carboxylic group show- I-effect and hence
order, I > II > III. toluene is most reactive towards nitration.
122. (c) More is the electron - deficiency of the 129. (c) Higher the possibility of delocalisation of
carbonyl carbon, greater will be the reactivity of the positive charge, greater is stability of the
the carbonyl compounds towards nucleophilic species. Thus
addition.
(CH 3 )3 C (CH3 ) 2 C H C6 H 5 C H 2CH3 C H 2
O
Also, primary benzyl carbocation have almost
CH 3CH 2 CH 2 CHO > > CH3CH2COCH2CH3 the same stability as 2°-alkyl carbocations.
123. (d) Such questions can be solved by considering 130. (b) Sodium cyanide (Na + C + N NaCN).
the relative basic character of their conjugated bases (Lassaigne's test)
which for H2O, C2H2, H2CO3 and C6H5OH are 131. (a) Kjeldal's method is suitable for estimating
nitrogen in those compounds in which nitrogen
OH, HC C : , HCO 3 , C 6 H 5O
is linked to carbon and hydrogen. The method is
More the possibility for the dispersal of the not used in case of nitro, azo and azoxy
negative charge, weaker will be the base. Thus
compound. This method is basically used for
the relative basic character of the four bases is
estimating nitrogen in food fertilizers and
HCO 3 < C 6 H 5 O < OH < HC C
agricultural products.
Equivalent Non-equivalent Oxygen can accommodate
resonating structures (–) charge easily 132. (d) Nitrogen, sulphur and halogens are tested
in an organic compound by Lassaigne's test. The
Due to resonance
organic compound is fused with sodium metal as
Thus the acidic char acter of the four to convert these elements into ionisable inorganic
corresponding acids wil be substances,
HC CH H 2O C6 H5OH H 2CO3 Na C N NaCN
124. (c) Chlorination of methane proceeds via free
radical mechanism. Conversion of methyl 2Na S Na 2S
chloride to methyl alcoh ol pr oceeds via 2Na X 2 2NaX
nucleophilic substitution. Formation of ethylene
from ethyl alcohol proceeds via dehydration The cyanide, sulphide or halide ions can be
reaction. Nitration of benzene is electrophilic confirmed in aqueous solution by usual test.
ݸ¿°¬»®

13 Hydrocarbons

̱°·½ ïæ ß´µ¿²»­ 6. In the free radical chlorination of methane, the


chain initiating step involves the formation of
1. With respect to the conformers of ethane, which
of the following statements is true ? [2017] (a) Chlorine free radical [1994]
(a) Bond angle changes but bond length (b) Hydrogen chloride
remains same (c) Methyl radical
(b) Both bond angle and bond length change (d) Chloromethyl radical.
(c) Both bond angles and bond length remains
7. Reactivity of hydrogen atoms attached to
same
different carbon atoms in alkanes has the order
(d) Bond angle remains same but bond length
changes [1993]
2. Liquid hydrocarbons can be converted to a (a) Tertiary > Primary > Secondary
mixture of gaseous hydrocarbons by : [2010] (b) Primary > Secondary > Tertiary
(a) Oxidation (c) Both (a) and (b)
(b) Cracking (d) Tertiary > Secondary > Primary
(c) Distillation under reduced pressure ̱°·½ îæ ß´µ»²»­
(d) Hydrolysis
3. The reaction of ethyl magnesium bromide with 8. 2,3-Dimethyl-2-butene can be prepared by
water would give [1999] heating which of the following compounds with
(a) Ethane (b) Ethyl alcohol a strong acid ? [2015 RS]
(c) Ethyl bromide (d) Ethyl ether (a) (CH3)2 CH – CH – CH = CH2
4. In commercial gasolines the type of |
CH3
hydrocarbons which are more desirable, is
(a) branched hydrocarbons [1997] (b) (CH3)3 C – CH = CH2
(b) straight-chain hydrocarbons (c) (CH3)2C = CH – CH2 – CH3
(c) aromatic hydrocarbons such as toluene (d) (CH3)2CH – CH2 – CH = CH2
(d) linear unsaturated hydrocarbons 9. In the reaction with HCl, an alkene reacts in
5. Which one of the following reactions is accordance with the Markovnikov's rule, to give
expected to readily give a hydrocarbon product a product 1-chloro-1-methylcyclohexane. The
in good yields ? [1997] possible alkene is : [2015 RS]
(a) Electrolyt ic CH2 CH3
RCOOK
oxidation
(b) RCOO Ag
Br2 (a) (b)
Cl 2
(c) CH 3 CH 3 CH3
h
(d) C2 H5 OH
(CH 3 ) 3 CCl
(c) (a) and (b) (d)
EBD_7324
ïîð ÝØÛÓ×ÍÌÎÇ
10. A single compound of the structure : [2015] 13. In the following reaction :
CH3 CH3 CH3
+
H2O/H
OHC C C C CH —
— CH2
C H C O
H2 H2
CH3
is obtainable from ozonolysis of which of the
following cyclic compounds ? A B
Minor Product + Minor Product
H3C
The major product is : [2012]
(a)
H3C CH3

H3C CH3 (a) C CH CH3


(b) OH CH3

CH3 CH3
(c) (b)
CH3 C CH2 CH3

H3C OH CH3
(d) CH3
CH3
11. The total number of -bond electrons in the
following structure is : [2015] (c) C CH CH3
H H H
H3C CH3 OH
H3C CH3

CH3
H2C H CH3
(a) 8 (b) 12
(d) C CH2 CH2
(c) 16 (d) 4
12. The reaction of C6H5CH = CHCH3 with HBr CH3 OH
produces: [2015]
(a) C6 H5CH 2CHCH3 14. In the following reactions, [2011]
|
Br CH3
H + /Heat
(i) CH3–CH–CH–CH3 A + B
(b) C6 H5CH 2CH 2CH 2Br Major Minor
OH products products
CH=CHCH3
HBr,dark
(ii) A C D
in absenceof peroxide
(c)
A C D
Major Minor
Br product product
(d) C 6 H 5 C HCH 2CH 3 the major products (A) and (C) are respectively:
|
Br
ا¼®±½¿®¾±²­ ïîï
CH3 18. Which one of the following alkenes will react
(a) CH2 = C – CH2– CH3 2 2 3
faster with H2 under catalytic hydrogenation
CH3 conditions? [2005]
(R = Alkyl Substituent)
2 2 3 and CH2– CH– CH2– CH3
R R R H
Br
CH3 (a) (b)
H H R H
(b) CH3– C = CH–CH3 R R
R R
CH3 (c) (d)
and CH3– C – CH2– CH3 R H R R
Br 19. Reaction of HBr with propene in the presence of
CH3 peroxide gives [2004]
(c) CH3– C = CH – CH3 (a) isopropyl bromide (b) 3-bromo propane
(c) allyl bromide (d) n-propyl bromide
CH 3
CH 3
and CH 3 – CH – CH – CH 3 |
Br 20. The compound CH 3— C CH — CH 3
CH3 on reaction with NaIO4 in the presence of
(d) CH2 = C – CH2 – CH3 KMnO4 gives [2003]
3 2 3
(a) CH3CHO + CO2
CH3
(b) CH3COCH3
2 and CH3– C – CH2 – CH3 (c) CH3COCH3 + CH3COOH
Br (d) CH3COCH3 + CH3CHO
15. The IUPAC name of the compound having the 21. Which alkene on ozonolysis gives CH3CH2CHO
formula CH C – CH = CH2 is : [2009] and CH3CCH 3 [2001]
(a) 1-butyn-3-ene (b) but-1-yne-3-ene
O
(c) 1-butene-3-yne (d) 3-butene-1-yne
CH3
16. H 3 C CH CH CH 2 HBr A
| (a) CH3CH2CH = C
CH 3 CH3
A(predominantly) is : [2008] (b) CH3CH2CH = CHCH2CH3
(a) CH3 CH CH2 CH 2 Br (c) CH3 CH2 CH = CH CH3
| (d) CH 3 C CHCH 3
CH3 |
Br CH3
| 22. In preparation of alkene from alcohol using Al2O3
(b) CH3 C CH 2 CH3 which is effective factor? [2001]
| (a) Temperature
CH3 (b) Concentration
(c) CH3 CH CH CH 3 (c) Surface area of Al2O3
| |
(d) Porosity of Al2O3
Br CH3
23. Which of the following reagents convert
(d) CH3 CH CH CH 3
| | propene to 1-propanol? [2000]
CH3 Br (a) H2O, H2SO4
17. Which of the compounds with molecular formula (b) aqueous KOH
C5H10 yields acetone on ozonolysis? (c) MgSO4, NaBH4/H2O
(d) B2H6, H2O2, OH–
[2007]
24. A hydrocarbon ‘A’ on chlorination gives ‘B’
(a) 3-methyl-1-butene (b) cyclopentane
which on heating with alcoholic potassium
(c) 2-methyl-1-butene (d) 2-methyl-2-butene. hydroxide changes into another hydrocarbon
EBD_7324
ïîî ÝØÛÓ×ÍÌÎÇ
‘C’. The latter decolourises Baeyer's reagent and 30. When hydrochloric acid gas is treated with
on ozonolysis forms formaldehyde only. ‘A’ is propene in presence of benzoyl peroxide, it gives
[1998] [1993]
(a) Ethane (b) Butane (a) 2-Chloropropane (b) Allyl chloride
(c) Methane (d) Ethene (c) No reaction (d) n-Propyl chloride.
25. In reaction sequence [1997]
CH2OH ̱°·½ íæ ß´µ§²»­
Hypochlorous R
CH 2 CH 2 M |
31. Which one is the correct order of acidity ? [2017]
acid CH2OH
(a) CH CH > CH3 – C CH > CH 2
molecule 'M' and reagent 'R' respectively are
= CH2 > CH3 – CH3
(a) CH3CH2Cl and NaOH (b) CH CH > CH2 = CH2 > CH3 – C CH >
(b) CH3CH2OH and H2SO4 CH3 – CH3
(c) CH2Cl – CH2OH and aqueous NaHCO3 (c) CH3 – CH3 > CH2 = CH2 > CH3 – C CH>
(d) CH2 — CH2 and heat CH CH
(d) CH2 = CH2 > CH3 – CH = CH2 > CH3 – C
O CH > CH CH
26. In the presence of platin um catalyst, 32. Predict the correct intermediate and product in
hydrocarbon A adds hydrogen to form n-hexane. the following reaction : [2017]
When hydrogen bromide is added to A instead
H 2O, H 2SO 2
of hydrogen, only a single bromo compound is H3 C C CH Intermediate product
HgSO 4 (A)
formed. Which of the following is A? [1996] (B)

(a) CH 3 — CH 2 — CH CH — CH 2 — CH 3 (a) A : H3C – C = CH B : H3C – C = CH2


| |
(b) CH 3 — CH 2 — CH 2 — CH CH — CH 3 OH SO4
(c) CH 3 — CH CH — CH 2 — CH 2 — CH 3
(b) A : H3C – C = CH3 B : H3C – C CH
(d) CH 2 CH — CH 2 — CH 2 — CH 2 — CH 3 ||
27. When 3,3-dimethyl –2-butanol is heated with O
H2SO4, the major product obtained is [1995] (c) A : H3C – C = CH2 B : H3C – C = CH 3
(a) 2,3-dimethyl –2-butene | ||
OH O
(b) 3, 3-dimethyl –1- butene
(c) 2, 3-dimethyl –1- butene (d) A : H3C – C = CH2 B : H3C C CH 3
| ||
(d) cis & trans isomers of 2, 3-dimethyl – 2-butene SO4 O
28. The alkene R – CH = CH2 reacts readily with 33. In the reaction
B2H6 and formed the product B which on (1)NaNH2 / liq.NH3
oxidation with alkaline hydrogen peroxides H C CH X
(2)CH3CH2Br
produces [1995]
(a) R – CH2 – CHO (b) R– CH2 – CH2 – OH (1)NaNH 2 / liq.NH3
(2)CH3CH 2 Br Y
(c) R C O (d) R CH C H 2
| | | X and Y are [2016]
CH 3
OH OH (a) X = 1-Butyne ; Y = 3-Hexyne
29. Which of the following compounds has the (b) X = 2-Butyne ; Y = 3-Hexyne
lowest boiling point ? [1994] (c) X = 2-Butyne ; Y = 2-Hexyne
(a) CH3CH2CH2CH2CH3 (d) X = 1-Butyne ; Y = 2-Hexyne
(b) CH3CH = CHCH2CH3 34. Which of the following organic compounds has
same hybridization as its combustion product
(c) CH3CH = CH – CH = CH2
(CO2)? [2014]
(d) CH3CH2CH2CH3
ا¼®±½¿®¾±²­ ïîí

(a) Ethane (b) Ethyne 41. Reduction of 2-butyne with sodium in liquid
(c) Ethene (d) Ethanol ammonia gives predominantly [1993]
35. In the following reaction : (a) cis-2-Butene (b) No reaction
H 2SO 4 (c) trans-2-Butene (d) n-Butane.
HC CH 'P'
Hg 2+ Reagent
42. R CH 2 CCl 2 R R C C R
Product ‘P’ will not give [NEET Kar. 2013]
(a) Iodoform test The reagent is [1993]
(b) Tollen’s reagent test (a) Na (b) HCl in H2O
(c) Brady’s reagent test (c) KOH in C2H5OH (d) Zn in alcohol.
(d) Victor Meyer test 43. Which is the most suitable reagent among the
36. Which of the following reagents will be able to following to distinguish compound (3) from rest
distinguish between 1-butyne and 2-butyne? of the compounds ? [1989]
(a) NaNH2 (b) HCl [2012 M] 1. CH 3 C C CH 3
(c) O2 (d) Br2 2. CH 3 CH 2 CH 2 CH 3
37. The IUPAC name of the compound
3. CH 3 CH 2 C CH
CH3CH = CHC CH is [2010]
(a) Pent-l-yn-3-ene (b) Pent-4-yn-2-ene 4. CH 3 CH CH 2.
(c) Pent-3-en-1-yne (d) Pent-2-en-4-yne (a) Bromine in carbon tetrachloride
38. Predict the product C obtained in the following (b) Bromine in acetic acid
reaction of butyne-1. [2007] (c) Alk. KMnO4
HI (d) Ammoniacal silver nitrate.
CH 3CH 2 C CH HCl BC
I 44. Acetylenic hydrogens are acidic because [1989]
| (a) Sigma electron density of C – H bond in
(a) CH3 CH 2 CH 2 C H acetylene is nearer to carbon, which has
| 50% s-character
Cl
I (b) Acetylene has only open hydrogen in each
| carbon
(b) CH3 CH 2 CH CH 2 Cl (c) Acetylene contains least number of
I hydrogens among the possible
| hydrocarbons having two carbons
(c) CH3CH 2 C CH3
| (d) Acetylene belongs to the class of alkynes
Cl with molecular formula, CnH2n – 2.
(d) CH3 CH CH 2 CH 2 I
| ̱°·½ ìæ ß®±³¿¬·½ ا¼®±½¿®¾±²­
Cl
39. Products of the following reaction: [2005] 45. Given:
H3 C CH3 H3 C CH2 H2 C CH2
(1) O3
CH3C C·CH2 CH3 ...... are:
( 2) Hydrolysis
(a) CH3COOH + CO2
(b) CH3COOH + HOOC CH2CH3 CH3 CH3 CH2
(c) CH3CHO + CH3CH2CHO (I) (II) (III)
(d) CH3COOH + CH3COCH3 The enthalpy of the hydrogenation of these
40. A compound is treated with NaNH2 to give compounds will be in the order as : [2015]
sodium salt. Identify the compound [1993] (a) III > II > I (b) II > III > I
(c) II > I > III (d) I > II > III
(a) C2H2 (b) C6H6
46. The oxidation of benzene by V2O5 in presence
(c) C2H6 (d) C2H4. of air produces : [2015 RS]
EBD_7324
ïîì ÝØÛÓ×ÍÌÎÇ
(a) benzoic anhydride (b) maleic anhydride
(a) H 3 C CH 2 OH C 6 H 6
(c) benzoic acid (d) benzaldehyde
. (b) CH 3 CH CH 2 C6 H 6
47. The radical, CH2 is aromatic because it
(c) H 2 C CH 2 C 6 H 6
has : [NEET 2013]
(a) 7 p-orbitals and 6 unpaired electrons (d) H 3 C CH 3 C 6 H 6
(b) 7 p-orbitals and 7 unpaired electrons 51. For the formation of toluene by Friedal Craft
(c) 6 p-orbitals and 7 unpaired electrons reaction , reactants used in presence of
(d) 6 p-orbitals and 6 unpaired electrons anhydrous AlCl3 are [2000]
48. Which of the following chemical system is non (a) C2H2 and CCl4 (b) CH4 and CaCN2
aromatic? [NEET Kar. 2013] (c) C6H6 and CH3Cl (d) C2H5 OH and Zn
52. When CH3Cl and AlCl3 are used in Friedel-Crafts
(a) (b)
reaction, the electrophile is [1994]
(a) Cl+ (b) AlCl4–
(c) (d) (c) CH3+ (d) AlCl2+
S 53. Select the true statement about benzene amongst
49. In the followin g reaction, C 6 H 5 CH 2 Br the following [1992]
1.Mg, Ether
X, the product ‘X’ is [2010] (a) Because of unsaturation benzene easily
2.H 3O
(a) C6H5CH2CH2C6H5 undergoes addition
(b) C6H5CH2OCH2C6H5 (b) There are two types of C – C bonds in
(c) C6H5CH2OH benzene molecule
(d) C6H5CH3 (c) There is cyclic delocalisation of pi-electrons
50. Using anhydrous AlCl3 as catalyst, which one in benzene
of th e following react ions pr oduces (d) Monosubstitution of benzene gives three
ethylbenzene (PhEt)? [2004] isomeric products.

ANSWER KEY
1 (c) 7 (d) 13 (a) 19 (d) 25 (c) 31 (a) 37 (c) 43 (d) 49 (d)
2 (b) 8 (b) 14 (b) 20 (c) 26 (a) 32 (c) 38 (c) 44 (a) 50 (c)
3 (a) 9 (c) 15 (c) 21 (a) 27 (a) 33 (a) 39 (b) 45 (a) 51 (c)
4 (a) 10 (d) 16 (b) 22 (a) 28 (b) 34 (b) 40 (a) 46 (b) 52 (c)
5 (a) 11 (a) 17 (d) 23 (d) 29 (d) 35 (d) 41 (c) 47 (d) 53 (c)
6 (a) 12 (d) 18 (a) 24 (a) 30 (a) 36 (a) 42 (c) 48 (a)
ا¼®±½¿®¾±²­ ïîë

Hints & Solutions


1. (c) There is no change in bond angles and bond 8. (b) CH 3C C CH 3
| |
lengths in the conformations of ethane.
CH 3 CH3
2. (b) During cracking higher hydrocarbons
(liquid) are converted to lower gaseous 2,3-Dimethyl-2-butene
hydrocarbons. (CH3)2CH – CH – CH = CH2 contain 7 carbon
|
Br CH3
3. (a) C 2 H 5 MgBr H 2O C2 H 6 Mg
atoms
OH
CH3 CH3
4. (a) Gasoline (petrol) is a mixture of alkanes,
alkenes and aromatic hydrocarbons. The quality CH3 – C – CH = CH2+ H CH3 – C – CH – CH3
of a gasoline is determined by the amount of CH3 CH3
br an ched chai n hydr ocar bon s (2,2,4- Rearrangement
trimethylpentane, commonly known as iso- CH3
octane) present in it. CH3 CH3 CH3
C=C CH3 – C – C
5. (a) Electrolysis of a concentrated aqueous CH3 CH3 CH3
solution of either sodium or potassium salts of H
2, 3 - dimethyl – 2 - butene (Stable 3° carbocation)
saturated carboxylic acids yields higher alkane 9. (c) 1-chloro-1-methylcyclohexane.
at anode.
CH3
Electrolyt ic Cl
2RCOOK 2 RCOO 2K
Oxidation Anode Cathode

At anode 2RCOO 2RCOO 2e –


R — R 2CO 2 CH2 CH3 CH3
Cl

At Cathode 2K 2e 2K + H – Cl + Cl

2K H 2 O 2KOH H 2
(Kolbe's Method) CH3
CH3 CH3
hv • Cl
6. (a) Cl 2 Chain initiation step
2Cl H H
+ H – Cl + Cl
7. (d) The reactivity of H-atoms depends upon
the stability of free radicals, follows the order
Tertiary > secondary > primary. 10. (d)
Therefore, reactivity of H-atoms follows the same H3C
order, i.e., tertiary > secondary > primary. H3 C
O3 O=C
CH3 CH3
R R H H Zn + H2O
OHC
| | | |
R – C• R C• R C• H C• 11. (a) No. of double bonds = 4
| | | | No. of bond electrons
R H H H = 2 × no. of double bond
Tertiary Secondary primary free methyl
free radical free radical radical free radical =2× 4=8
EBD_7324
ïîê ÝØÛÓ×ÍÌÎÇ
CH3 CH3
12. (d) C6H5 —CH=CH—CH3 + HBr H2O
C CH CH3 C CH CH3

C6 H5—CH—CH2—CH3 CH3 H 3C OH
(Benzyl carbocation) 1,2-methyl shift (Minor)

Br–
(A)
OH CH3
C6H 5—CH—CH 2—CH 3
Br C CH CH3 C CH CH3
CH3
+
CH3 CH3 CH3
H 3° cabocation (Major)
13. (a) C CH CH2 (more stable) B

CH3

CH3 CH 3 CH3
H/+ Heat
14. (b) (i) CH3— CH — CH — CH 3CH3— C = CH — CH 3 + CH3— CH — CH = CH 2
2-methyl-but-2-ene 3-methyl
OH butene-1 (20%)
(80%)
(A) (B)
In this case dehydration is governed by Saytezeff’s rule according to which hydrogen is preferentially
eliminated from the carbon atom with fewer number of hydrogen atoms i.e., poor becomes poorer. Thus,
2-methyl but-2-ene is the major product.
CH3 CH 3
HBr, dark
(ii) CH3 — C = CH — CH3 (CH3)2 — CH — CH — CH3 + CH3 — C — CH2 – CH3
in absence
(A) of peroxide Br Br
(Minor) (Major)
This reaction is governed by Markownikoff’s rule according to which when an unsymmetrical reagent
e.g. HBr adds to an unsymmetrical alkene, then the negative part of the reagent is added to that carbon
atom of the double bond which bears the least number of hydrogen atom. Thus, in above case 2-methyl-
2-bromobutane will be the major product.
15. (c) If both the double and triple bonds are H 3C
present in the compound, it is regarded as CH – CH = CH2
H3C 3– methyl-1-butene
derivative of alkyne. Further if double and triple
CH3 +
bonds are at equidistance from either side, the CH – CH2 – CH2 (1°)
preference is given to double bond. So, the + CH3
IUPAC name of the compound is 1-butene-3-yne. +H
CH3 +
16. (b) We know that in case of an unsymmetrical CH – CH – CH3 (2°)
alkene there is the possibility of forming two CH 3
products. In such cases the formation of major Of the two possibilities 2° carbocation is more
pr oduct is decided on the basis of stable so the product of the reaction is expected
Markownikoffs rule which is rationalized in to be predominantly formed by 2° carbocation
terms of stability of the carbocation. Also 3° i.e.
carbocation is more stable than 2° carbocation CH 3 CH CH CH 3
and 2° carbocation is more stable than 1° | |
carbocation. CH 3 Br
i.e. 2– Bromo-3-Methylbutane
ا¼®±½¿®¾±²­ ïîé
However some electrophilic addition reaction Peroxide
H 3CCH CH 2 HBr H 3C CH 2CH 2 Br
form products that are clearly not the result of Propene n propyl bromide
the addition of electrophile to the sp2 carbon Kharasch observed that the addition of HBr to
bonded with the most hydrogen and the unsymmetrical alkene in the presence of organic
addition of a nucleophile to the other sp2 carbon. peroxides follows an opposite course to that
In the above case the major product formed is suggested by Markownikoff . This is termed
2- Bromo-2-methylbutane.
anti-Markownikoff or peroxide effect.
The unexpected product r esults from a
rearrangement of carbocation intermediate. CH 3
|
Please note that all carbocation do not rearrange. 20. (c) CH 3 C CH CH 3
NaIO 4
KMnO 4
CH3
| O
CH3 – C – CH – CH = CH2 + HBr ||
CH 3 C CH 3 CH 3COOH
CH3 CH3 21. (a)
| | H CH3
CH3 – CH – CH – CH3 + CH3 – C – CH2 – CH3 H O CH3
| | O
| | CH 3 – CH2– C = C
3
CH3 – CH2– C C
Br Br | | |
2-Bromo-3-methylbutane 2-Bromo-2-methylbutane CH3 O–O CH3
O
17. (d) H3C — C = CH — CH3 ||
CH3 CH3– C – CH3+ CH3CH2CHO
(–H 2O)
22. (a) Reaction condition s for alcohol in
H3 C O H preparation of alkened is :
O3
C C
CH3 Al 2 O3
H3 C R — CH 2 — CH 2 OH
O O 350 º 380 º C

O O R — CH CH 2 H 2O
H2 O
CH3 – C – CH3 + CH3 – C – H
while at 220º – 250ºC it forms ether.
–H2O2 acetone acetaldehyde 23. (d) We know that
18. (a) Heat of hydrogenation of alkene B2 H6
6 CH 3 CH CH 2
1 Propene ether, 0°C
Stability of alkene
H2O2
Hence the alkene which will react fastest with 2(CH3CH 2CH2 )3 B
OH
H2 will be the least stable. Among the given
options the compound having least number of 6CH3CH 2 CH 2OH 2H 3BO 3
alkyl groups (R) will be the least stable. Propanol
Further the relative rates of hydrogenation 24. (a) Since hydrocarbon C give only CH2O, on
decrease with increase of steric hindrance ozonolysis, C should be CH2 = CH2 hence going
R R R R backward A should be ethane. Thus the reactions are
C=C > C=C Cl 2 / h
R R R H CH 3CH 3 CH 3CH 2Cl alc.
( A) (B) KOH
R H R R
> C=C > C=C CH 2 CH 2 O3 / H 2 O HCHO
R H H H (C ) ( D)

R R 25. (c) We know that


Hence C=C will react fast with H2 NaHCO3
H H CH 2 CH 2 HOCl CH2 OH CH2OH
| |
19. (d) In presence of peroxide, HBr adds on CH 2Cl CH2 OH
alkenes according to anti–markownikoff’s rule.
M CH 2 OH CH 2 Cl and R NaHCO 3
EBD_7324
ïîè ÝØÛÓ×ÍÌÎÇ
H2 CH 3 CH3
26. (a) CH3 CH 2 CH CH CH 2 CH 3 Platinum +
A
CH3 – CH – C – CH 3 CH3 – C = C – CH3
CH 3 CH 2 CH 2 CH 2 CH 2
n-hexane | CH3 CH3
CH 3 2, 3-dimethyl-2-butene
CH3 CH 2 CH CH 2 CH 2 CH 3 B2 H 6
| 28. (b) 6 R CH CH2 2(RCH 2CH 2 )3 B
Ether,0 C B
Br
27. (a) When 3, 3 dimethyl – 2-butanol is heated H 2O2
6RCH 2CH 2OH 2H3BO3.
with H2SO4 the major product obtained is 2, 3- NaOH
dimethyl – 2-butene. 29. (d) Of all the options listed CH3CH2CH2CH3
CH 3 has the least number of C-atoms and hence has
| the lowest b.p.
Conc.H 2SO4
CH 3 CH C CH 3
| | 30. (a) Peroxide effect is observed only in case of
OH CH 3 HBr. Therefore, addition of HCl to propene even
3,3-dimethyl -2-butanol in the presence of benzyoyl peroxide occurs
CH3 CH3 according to Markovnikov’s rule :
+
CH3 – CH – C – CH3 CH3 – CH – C – CH3
HCl
CH 3 CH CH 2
+ OH2 CH3 CH3
(C6 H5 CO) 2 O 2

CH 3 CHCl CH 3

31. (a) Correct order is H C C H H3C C C H H 2C CH 2 CH3 CH3


Two acidic One acidic
hydrogens hydrogen
32. (c) Hydration of alkynes give ketones.
OH
H3 C C CH H3 C C CH 2
(A)
O
Tautomerism
H3 C C CH3
(B)

33. (a) NaNH 2 H3C–CH 2 · Br


HC CH HC CNa HC C – CH 2 – CH3
liq. NH 3
(X)
1-Butyne NaNH2 liq. NH3

H3C CH 2 – Br
H3 C – CH 2 – C C – CH2 – CH3 H3C – CH 2 – C C N a
3-Hexyne (Y)

34. (b) The combustion reaction of ethylene is 36. (a) 1-Butyne and 2-butyne are distinguish by
5 NaNH2 because 1-Butyne react with NaNH2 due
C2 H 2 O2 2CO2 H 2 O
2 to presence of terminal hydrogen.
Both HC CH and CO2 have sp hybridization. CH3 CH2 C CH + NaNH2
1– Butyne
O CH3 CH2 C CNa + NH3
Hg 2 / H 2SO 4
37. (c) When both double and triple bonds are
35. (d) CH CH CH3 C H present, then triple bond is considered as the
CH3CHO does not give Victor Meyer test. principal group. So, the compound name is pent-
3-en-1-yne.
ا¼®±½¿®¾±²­ ïîç

38. (c) CH3 CH 2 C CH HCl AgNO3 (d) reacts only with terminal alkynes,
i.e., 3 an d hence compoun d 3 can be
CH 3 CH 2C CH 2 distinguished from 1, 2 and 4 by. ammoniacal
| AgNO3 (d).
Cl 44. (a) The acidity of acetylene or 1–alkynes can
I be explained on the basis of molecular orbital
HI
| concept according to which formation of
CH 3 CH 2 C CH3
| C–H bond in acetylene involves sp-hybridised
Cl carbon atom. Now since s electrons are closer
According to Markownikoff’s rule which states to the nucleus than p electrons, the electrons
that when an unsymmetrical alkene undergo present in a bond having more s-character will
hydrohalogenation, the negative part goes to be correspondingly more closer to the nucleus.
the C-atom which contain lesser no. of H-atom. Thus owing to high s character of the C—H
bond in alkynes (s = 50%), the electrons constituting
O3
39. (b) CH 3 C C CH 2 CH 3 this bond are more strongly held by the carbon
O nucleus (i.e., the acetylenic carbon atom or the
2H2 O sp orbital acts as more electronegative species
CH3 – C – C – CH2 – CH3 than the sp2 and sp 3 with the result the
O O hydrogen present on such a carbon atom
(O ) ( C—H) can be esily removed as a proton.
CH 3 CO H 2O2
| 45. (a) Enthalpy of hydrogenation
CH 3CH 2CO
Methylethylglyoxal 1
CH 3COOH + CH3CH 2COOH stability of alkene
Acetic acid Propionic acid
The glyoxal formed as an intermediate is III > II > I
oxidised by H2O2 to give the acids.` V2O5
46. (b) 2C6 H 6(g) 9O2(g)
410 C
40. (a) Only C2H2 (acetylene) has acidic H-atoms
O
and hence reacts with NaNH2 to form sodium
salt, i.e.,
CH—C
HC CH + NaNH2 HC CNa + NH3. O + 4CO2(g) + 4H2O (g)
41. (c) Reduction of alkynes with Na/liq. NH3 CH—C
gives trans-alkenes. This reaction is called Birch
reduction O
CH3 C C CH 3 H 2 Maleic anhydride

H3C H
Na / liq. NH 3
C=C 47. (d) Presence of 6p orbitals, each containing one
H CH3 unpaired electron, in a six membered cyclic
trans-Butene or trans-2-Butene structure is in accordance with Huckel rule of
(Birch reduction)
aromaticity.
42. (c) On heating ethylene chloride (1, 1 dichloro 48. (a) Huckel rule is not obeyed. It has only four
ethane) with alcoholic KOH followed by electrons. Further it does not have continous
sodamide, alkyne is obtained conjugation.
ether
R CH 2 CCl 2 R 49. (d) C6H5CH2Br + Mg
alc.KOH C6H5CH2MgBr
R CH CCl R
NaNH 2 +
R C C R H2O/H
C 6 H 5CH 2 MgBr
43. (d) Br 2 in CCl4 (a), Br2 in CH3 COOH (b) and H OH
alk. KMnO4 (c) will react with all unsaturated C6H5CH3 + Mg Br(OH)
compounds, i.e., 1, 3 and 4 while ammoniacal
EBD_7324
ïíð ÝØÛÓ×ÍÌÎÇ
AlCl 3 53. (c) Benzene do not show addition reaction like
50. (c) C 6 H 6 CH 2 CH 2 C 6 H 5 CH 2 CH 3
other unsaturated hydrocar bons. Due to
CH3 resonance all the C – C bonds have the same
nature, which is possible because of the cyclic
anhydrous AlCl3
51. (c) + CH3Cl + HCl delocalisation of -electrons in benzene.
Monosubstitution will give only a single product.

52. (c) CH3Cl + AlCl3 CH3– + AlCl4–


Electrophile
Û²ª·®±²³»²¬¿´ ݸ»³·­¬®§ ïíï

ݸ¿°¬»®

14 Environmental Chemistry

̱°·½ ïæ ß·® б´´«¬·±² 6. Roasting of sulphides give the gas X as a by


1. Which of the following is not a common product. This is colorless gas with choking smell
component of Photochemical Smog? [2014] of burnt sulphur and caused great damage to
(a) Ozone respiratory organs as a result of acid rain. Its
(b) Acrolein aqueous solution is acidic, acts as a reducing
(c) Peroxyacetyl nitrate agent and its acid has never been isolated. The
(d) Chlorofluorocarbons gas X is : [NEET 2013]
2. Which one of the following statements regarding (a) SO2 (b) CO2
photochemical smog is not correct? [2012] (c) SO3 (d) H2S
(a) Carbon monoxide does not play any role in 7. Which one of the following statements is not
photochemical smog formation.
true? [NEET Kar. 2013]
(b) Photochemical smog is an oxidising agent in
(a) Dissolved oxygen (DO) in cold water can
character.
reach a concentration upto 10 ppm.
(c) Photochemical smog is formed through
(b) Clean water would have a BOD value of 5
photochemical reaction involving solar energy. ppm.
(d) Photoch emical smog does not cause (c) Fluoride deficiency in drinking water is
irritation in eyes and throat. harmful. Soluble fluoride is often used to
3. The greenhouse effect is because of the bring its concentration upto 1 ppm.
(a) presence of gases, which in general are (d) When the pH of rain water is higher than 6.5,
strong infrared absorbers, in the atmosphere it is called acid rain.
(b) presence of CO2 only in the atmosphere 8. Which one of the following statement is not
(c) pressure of O3 and CH4 in the atmosphere true ? [2011]
(d) N2O and chlorofluorohydrocarbons in the (a) pH of drinking water should be between
atmosphere [1996] 5.5 – 9.5.
4. Which of the following is/are the hazardous (b) Concentration of DO below 6 ppm is good
pollutant(s) present in automobile exhaust gases? for the growth of fish.
(i) N2 (ii) CO (c) Clean water would have a BOD value of less
(iii) CH4 (iv) Oxides of nitrogen than 5 ppm.
(a) (ii) and (iii) (b) (i) and (ii) [1996] (d) Oxides of sulphur, nitrogen and carbon are
(c) (ii) and (iv) (d) (i) and (iii) the most widespread air pollutant.
9. Green chemistry means such reactions which
̱°·½ îæ É¿¬»® ¿²¼ ͱ·´ б´´«¬·±² (a) produce colour during reactions [2008]
5. Which of the following is a sink for CO ? (b) reduce the use and production of hazardous
[2017] chemicals
(a) Microorganism present in the soil (c) are related to the depletion of ozone layer
(b) Oceans
(d) study the reactions in plants
(c) Plants
(d) Haemoglobin
EBD_7324
ïíî ÝØÛÓ×ÍÌÎÇ

ANSWER KEY
1 (d) 2 (d) 3 (a) 4 (c) 5 (a) 6 (a) 7 (d) 8 (b) 9 (b)

Hints & Solutions


1. (d) The oxidised hydrocarbons and ozone in 4. (c) CO and oxides of Nitrogen are poisonous
presence of humidity cause photochemical gases present in automobile exhaust gases.
smong. 5. (a) Microorganisms present in the soil is a sink
Hydrocarbons + O 2 , NO 2 , NO, O, O 3 for CO.
Peroxides, formaldehyde, peroxyacetyl-nitrate 6. (a) Based on the features given gas must be
(PAN), acrodein etc. Hence chlorofluoro carbons SO2 .
are not common component of photochemical 7. (d) Acid rain is the rain water containing
smog. sulphuric acid and nitric acid which are formed
2. (d) The oxidised hydrocarbons and ozone in from the oxides of sulphur and nitrogen present
presence of humidity cause photochemical in the air as pollutants and rain water has a pH
smog. range of 4-5.
Hydrocarbons + O 2 , NO 2 , NO, O, O 3
Peroxides, formaldehyde, peroxyace-tylnitrate 8. (b) The growth of fishes get hindered if the
(PAN), acrolein etc. concentration of D.O. is below 6 ppm.
It is oxidising in nature and causes irritation to 9. (b) Green chemistry may be defined as the
eyes, lungs, nose, asthamatic attack and damage programme of developing new ch emical
plants. products and chemical processes or making
3. (a) Green house gases such as CO2, ozone, improvements in the already existing
methane, the chlorofluoro carbon compounds compounds and processes so as to make them
and water vapour form a thick cover around the less harmful to human health and environment.
earth which prevents the IR rays emitted by the This means the same as to reduce the use and
earth to escape. It gradually leads to increase in production of hazardous chemicals.
temperature of atmosphere.
̸» ͱ´·¼ ͬ¿¬» ïíí

ݸ¿°¬»®

15 The Solid State

̱°·½ ïæ Ю±°»®¬·»­ ¿²¼ ̧°»­ ±º ͱ´·¼­ (a) 5.96 × 10–3 (b) 5.96
(c) 5.96 × 10–2 (d) 5.96 × 10–1
1. A solid with high electrical and thermal
7. When molten zinc is converted into solid state,
conductivity from the following is [1994]
it acquires hcp structure. The number of nearest
(a) Si (b) Li
(c) NaCl (d) Ice neighbours will be [2001]
2. The pure crystalline substance on being heated (a) 6 (b) 12
gradually first forms a turbid liquid at constant (c) 8 (d) 4
temperature and still at higher temperature 8. In the solid state, MgO has the same structure
turbidity completely disappears. The behaviour as that of sodium chloride. The number of
is a characteristic of substance forming [1993] oxygens surrounding each magnesium in MgO
(a) Allotropic crystals is [1999]
(b) Liquid crystals (a) 6 (b) 1
(c) Isomeric crystals (c) 2 (d) 4
(d) Isomorphous crystals. 9. For orthorhombic system axial ratios are
3. Most crystals show good cleavage because their
atoms, ions or molecules are [1991] a b c and the axial angles are [1991]
(a) weakly bonded together (a) 90
(b) strongly bonded together
(b) 90
(c) spherically symmetrical
(d) arranged in planes. (c) 90 , 90
4. The ability of a substances to assume two or
(d) 90
more crystalline structures is called [1990]
(a) Isomerism (b) Polymorphism ̱°·½ íæ Ý«¾·½ ͧ­¬»³ ¿²¼ Þ®¿¹¹ù­ Û¯«¿¬·±²
(c) Isomorphism (d) Amorphism
10. Which of the following statements about the
̱°·½ îæ Ý®§­¬¿´ ͬ®«½¬«®» ±º ͱ´·¼­ interstitial compounds is incorrect ?
5. A solid compound XY has NaCl structure. If the [NEET 2013]
radius of the cation is 100 pm, the radius of the (a) They are chemically reactive.
anion (Y–) will be : [2011 M] (b) They are much harder then the pure metal.
(a) 275.1 pm (b) 322.5 pm (c) They have higher melting points than the
(c) 241.5 pm (d) 165.7 pm pure metal.
6. The pyknometric density of sodium chloride (d) They retain metallic conductivity.
crystal is 2.165 × 103 kg m–3 while its X-ray 11. The number of carbon atoms per unit cell of
density is 2.178 × 103 kg m–3. The fraction of diamond unit cell is : [NEET 2013]
unoccupied sites in sodium chloride crystal is (a) 8 (b) 6
[2003]
(c) 1 (d) 4
EBD_7324
ïíì ÝØÛÓ×ÍÌÎÇ
12. A metal has a fcc lattice. The edge length of the 20. If ‘a’ stands for the edge length of the cubic
unit cell is 404 pm. The density of the metal is systems : simple cubic, body centred cubic and
2.72 g cm–3 . The molar mass of the metal is : face centred cubic, then the ratio of radii of the
(NA, Avogadro’s constant = 6.02 × 1023 mol-1) spheres in these systems will be respectively,
[NEET 2013] [2008]
(a) 30 g mol–1 (b) 27 g mol–1
1 3 1
(c) 20 g mol–1 (d) 40 g mol–1 (a) a: a: a
13. Structure of a mixed oxide is cubic close-packed 2 4 2 2
(c.c.p). The cubic unit cell of mixed oxide is 1 1
composed of oxide ions. One fourth of the (b) a : 3a : a
2 2
tetrahedral voids are occupied by divalent metal
A and the octahedral voids are occupied by a 1 3 3
(c) a: a: a
monovalent metal B. The formula of the oxide is : 2 2 2
[2012 M] (d) 1a : 3a : 2a
(a) ABO2 (b) A2BO2
(c) A2 B3 O4 (d) AB2O2 21. The fraction of total volume occupied by the
atoms present in a simple cube is [2007]
14. The number of octahedral void(s) per atom
present in a cubic close-packed structure is :
(a) (b)
[2012] 3 2 4 2
(a) 1 (b) 3
(c) 2 (d) 4
(c) (d)
15. A metal crystallizes with a face-centered cubic 4 6
lattice. The edge length of the unit cell is 408 22. CsBr crystallises in a body centered cubic lattice.
pm. The diameter of the metal atom is : [2012] The unit cell length is 436.6 pm. Given that the
(a) 288 pm (b) 408 pm atomic mass of Cs = 133 and that of Br = 80 amu
(c) 144 pm (d) 204 pm and Avogadro number being 6.02 × 1023 mol–1,
16. AB crystallizes in a body centred cubic lattice the density of CsBr is [2006]
with edge length ‘a’ equal to 387 pm. The (a) 0.425 g/cm3 (b) 8.25 g/cm3
distance between two oppositely charged ions (c) 4.25 g/cm 3 (d) 42.5 g/cm3
in the lattice is : [2010] 23. In faceócentred cubic lattice, a unit cell is
(a) 335 pm (b) 250 pm shared equally by how many unit cells? [2005]
(c) 200 pm (d) 300 pm (a) 2 (b) 4
17. Copper crystallises in a face-centred cubic lattice
(c) 6 (d) 8
with a unit cell length of 361 pm. What is the
radius of copper atom in pm? [2009] 24. A compound formed by elements X and Y
(a) 157 (b) 181 crystallizes in a cubic structure in which the X
(c) 108 (d) 128 atoms are at the corners of a cube and the Y
18. Lithium metal crystallises in a body centred cubic atoms are at the face-centres. The formula of the
crystal. If the length of the side of the unit cell of compound is [2004]
lithium is 351 pm, the atomic radius of the lithium (a) XY3 (b) X3Y
will be: [2009] (c) XY (d) XY2
(a) 151.8 pm (b) 75.5 pm 25. A compound is formed by elements A and B.
(c) 300.5 pm (d) 240.8 pm
The crystalline cubic structure has the A atoms
19. Percentage of free space in a body centred cubic
at the corners of the cube and B atoms at the
unit cell is : [2008]
body centre. The simplest formula of the
(a) 30% (b) 32%
compound is [2000]
(c) 34% (d) 28%
̸» ͱ´·¼ ͬ¿¬» ïíë

(a) AB (b) A6B 32. If NaCl is doped with 10– 4 mol % of SrCl2, the
(c) AB6 (d) A8B4 concentration of cation vacancies will be[2007]
26. The second order Bragg diffraction of X-rays (NA = 6.02 × 1023 mol–1)
with = 1.00 Å from a set of parallel planes in a
metal occurs at an angle 60º. The distance (a) 6.02 × 1016 mol–1 (b) 6.02 × 1017 mol–1
between the scattering planes in the crystal is (c) 6.02 × 1014 mol–1 (d) 6.02 × 1015 mol–1
[1998] 33. The appearance of colour in solid alkali metal
(a) 0.575 Å (b) 1.00 Å halides is generally due to [2006]
(c) 2.00 Å (d) 1.15 Å (a) Schottky defect
27. The intermetallic compound LiAg crystallizes in (b) Frenkel defect
a cubic lattice in which both lithium and silver (c) Interstitial positions
atoms have coordination number of eight. To (d) F-centres
what crystal class does the unit cell belong ? 34. Schottky defect in crystals is observed when
(a) Simple cubic [1997] [1998]
(b) Face-centred cubic (a) an ion leaves its normal site and occupies
(c) Body-centred cubic an interstitial site
(d) None
(b) unequal number of cations and anions are
28. The edge length of a face centred unit cubic cell
missing from the lattice
is 508 pm. If the radius of the cation is 100 pm,
the radius of the anion is [1996] (c) density of the crystal is increased
(a) 288 pm (b) 398 pm (d) equal number of cations and anions are
(c) 154 pm (d) 618 pm missing from the lattice
29. In the fluorite structure, the coordination number 35. When electrons are trapped into the crystal in
of Ca2+ ion is : [1993] anion vacancy, the defect is known as [1994]
(a) 4 (b) 6 (a) Schottky defect
(c) 8 (d) 3 (b) Frenkel defect
30. The number of atoms contained in a fcc unit cell (c) Stoichiometric defect
of a monoatomic substance is [1993] (d) F-centres
(a) 1 (b) 2 36. On doping Ge metal with a little of In or Ga, one
(c) 4 (d) 6 gets [1993]
(a) p-type semi conductor
̱°·½ ìæ ׳°»®º»½¬·±² ·² ͱ´·¼­ (b) n-type semi conductor
31. With which one of the following elements silicon (c) insulator
should be doped so as to give p-type (d) rectifier
semiconductor ? [2008]
(a) Germanium (b) Arsenic
(c) Selenium (d) Boron

ANSWER KEY
1 (b) 5 (c) 9 (b) 13 (d) 17 (d) 21 (d) 25 (a) 29 (c) 33 (d)
2 (b) 6 (a) 10 (a) 14 (a) 18 (a) 22 (c) 26 (d) 30 (c) 34 (d)
3 (d) 7 (b) 11 (a) 15 (a) 19 (b) 23 (c) 27 (c) 31 (d) 35 (d)
4 (b) 8 (a) 12 (b) 16 (a) 20 (a) 24 (a) 28 (c) 32 (b) 36 (a)
EBD_7324
ïíê ÝØÛÓ×ÍÌÎÇ

Hints & Solutions


1. (b) Out of the given substances, only Li has 12. (b) Density is given by
high electrical and thermal conductivity.
Z M
2. (b) Liquid crystals on heating first become turbid d ; where Z = number of formula units
and on further heating they become clear. NA a 3
3. (d) Crystals show good cleavage because their present in unit cell, which is 4 for fcc
constituent particles are arranged in planes. a = edge length of unit cell. M = Molecular mass
4. (b) The phenomenon of occurren ce of a
substance in two or more crystalline structures 4 M
2.72 = 3
is called polymorphism. 6.02 1023 404 10 10
5. (c) Radius ratio of NaCl like crystal
10
r ( 1pm 10 cm)
= 0.414
r 2.72 l 6.02 l (404)3
M= = 26.99
100 4 l107
r 241.5 pm (r+ = 100)
0.414 = 27 g mol–1
6. (a) Fraction of unoccupied sites in NaCl crystal
1
13. (d) No. of A 2 ions 8 2
x-ray density–pyknometric density 4
=
x-ray density No. of B+ ions = 4 × 1 = 4
No. of atoms in ccp,
2.178l103 ó 2.165l103 0.013l103
ã ã 1 1
2.178l103 2.178l103 O2 8 6 4
8 2
= 5.96 × 10–3 Hence the formula of the oxide will be
7. (b) hcp is a closed packed arrangement in which A2B4O4 or AB2O2.
the unit cell is hexagonal and coordination
14. (a) Number of octahedral voids in ccp is equal
number is 12.
to effective number of atoms.
8. (a) Since MgO has a rock salt structure. In this In ccp, effective number of atoms are 4 so, 4
structure each cation is surrounded by six octahedral voids.
anions and vice versa. So, 1 octahedral voids per atom.
9. (b) For orthorhombic system 90
15. (a) For fcc structure, 2a 4r
10. (a) In interstitial compounds small atoms like
H, B & C enter into the void sites between the 2 408
2r (2r = Diameter)
packed atoms of crystalline metal. They retain 2
metallic conductivity and are chemically inert. Diameter = 288.5 å 288 pm
11. (a) Diamond is like ZnS. In diamond cubic unit 16. (a) For bcc lattice, diagonal = a 3 .
cell, there are eight corner atoms, six face cen-
The distance between the two oppositely
tered atoms and four more atoms inside the struc-
ture (tetrahedral voids). a
charged ions = 3
Total no. of atoms present per unit cell 2
1 1
= 8l õ 6l õ 4 ã 8 387 1.732
8 2 = å 335pm
(corners) (face (inside 2
centered) body)
̸» ͱ´·¼ ͬ¿¬» ïíé

17. (d) Since Cu metal crystallises in a face centred Volume of cell = (436.6 × 10–12)3
cubic lattice
z l at.wt.
a Density ã
r= Av.no.l vol.of unit all cell
2 2
1 213
361 Density =
r= 127.6 å 128pm 6.02 10 23
(436.6)3 10 30
2 2
18. (a) Since lithium metal crystallises in a body
213 107
centred cubic crystal 4.25 gm / cm3
6.02 (436.6)3
a 3 351 1.732
r =151.98 pm 23. (c)
4 4
19. (b) Percentage of occupied space in a bcc is
68%.
Percentage of free space in a bcc is
(100 – 68) = 32% .
20. (a) Following generalization can be easily
derived for various types of lattice arrangements
in cubic cells between the edge length (a) of the
cell and r the radius of the sphere.
a
For simple cubic : a = 2r or r
2
For body centred cubic :
4 3
a r or r a
3 4
For face centred cubic :
1
a 2 2r or r a
2 2
An isolated fcc cell is shown here. Each face
Thus the ratio of radii of spheres will be simple :
of the cell is common to two adjacent cells.
bcc : fcc
Therefore, each face centre atom contributes
a 3 1 only half of its volume and mass to one cell.
= : a: a Arranging six cells each sharing the remain-
2 4 2 2
21. (d) Number of atoms per unit cell = 1 ing half of the face centred atoms, constitutes
Atoms touch each other along edges. Hence fcc cubic lattice. e.g., Cu and Al.
24. (a) For the given cubic structure,
a
r= 1
2 No. of X atoms at the corners 8 1
( r = radius of atom and a = edge length) 8

4 1
r No. of Y atoms at the face-centres 6 3
Therefore % fraction = 3 3 0.52 2
(2r) 6 Formula of the compound = XY 3
22. (c) In body centred cubic lattice one molecule 25. (a) Given: Atoms are present at the corners of
of CsBr is within one unit cell. the cube = A and atoms present at body
Atomic mass of unit cell centre = B. We know that a cubic unit cell has 8
= 133 + 80 = 213 a.m.u corners. Therefore contribution of each atom at
EBD_7324
ïíè ÝØÛÓ×ÍÌÎÇ
1 31. (d) The semiconductors formed by the
the corner = . Since number of atoms per unit introduction of impurity atoms containing one
8
elecron less than the parent atoms of insulators
1 are termed as p-type semiconductors. Therefore
cell is 8, therefore total contribution = 8 1.
8 silicon containing 14 electrons has to be doped
We also know the atoms in the body centre, with boron containing 13 electrons to give a
therefore number of atoms per unit cell = 1. Thus p-type semi-conductor.
formula of the compound is AB. 32. (b) Since each Sr++ ion provides one cation vacancy,
26. (d) Given : Order of Bragg diffraction (n) = 2 ; hence concentration of cation vacancies
Wavelength ( ) = 1 Å and angle ( ) = 60º. We = mole % of SrCl2 added = 10–4 mole %
know from the Bragg’s equation n = 2d sin
10 4
or 2 × 1 = 2d sin 60º = 6.023 1023 6.023 10 7
100
3 2 33. (d) The appearance of colour in solid alkali metal
2 1 2.d. d 1.15Å
2 3 halide is due to presence of F-centres found as
(where d = difference between the scattering defect in the crystal structure.
planes) 34. (d) If in an ionic crystal of the type A+, B–, equal
27. (c) A body-centred cubic system consists of number of cations and anions are missing from
eight atoms at the corners plus one atom at the their lattice sites so that the electrical neutrality
centre of cube. is maintained. The defect is called Schottky
28. (c) For fcc, the edge length of the unit cell, defect.
a = r + 2R + r 35. (d) When electrons are trapped in anion
where, R = Radius of anion & r = radius of cation vacancies, these are called F-centres.
508 = 2 × 100 + 2R R = 154 Pm
29. (c) In fluorite structure each F– ion is surrounded +ve –ve
by four Ca ++ ions whereas each Ca ++ is ion ion
surrounded by eight F– ions, giving a body
centred cubic arrangement . Thus the co- F- centre in crystal
ordination number of Ca++ = 8.
36. (a) p-type of semiconductors are produced
30. (c) The no. of atoms is a unit cell may be
calculated by the the formula (a) due to metal deficiency defects
(b) by adding impurity containing less electrons
n n n n (i.e. atoms of group 13).
Zã c õ b õ f õ e
8 1 2 4 Ge belongs to Group 14 and In to Group 13. Hence
where nc = no. of atom at the corner n b = no. on doping, p-type semiconductor is obtained.
of atoms at body centre nf = no. of atoms at face
centre ne = no. of atoms at edge centre.
An Fcc crystal contains
8 6
ã õ ã 4 atoms in a unit cell.
8 2
ͱ´«¬·±²­ ïíç

ݸ¿°¬»®

16 Solutions

̱°·½ ïæ ͱ´«¾·´·¬§ ¿²¼ ݱ²½»²¬®¿¬·±² ±º 7. The mole fraction of the solute in one molal
ͱ´«¬·±²­ aqueous solution is: [2005]
(a) 0.009 (b) 0.018
1. If molality of the dilute solutions is doubled, the (c) 0.027 (d) 0.036
value of molal depression constant (Kf) will be:-
8. 1 M, 2.5 litre NaOH solution is mixed with another
[2017]
0.5 M, 3 litre NaOH solution. Then find out the
(a) halved (b) tripled
molarity of resultant solution [2002]
(c) unchanged (d) doubled
2. Which of the following is dependent on (a) 0.80 M (b) 1.0 M
temperature? [2017] (c) 0.73 M (d) 0.50 M
(a) Molarity (b) Mole fraction 9. Molarity of liquid HCl will be, if density of
(c) Weight percentage (d) Molality solution is 1.17 gm/cc [2001]
3. What is the mole fraction of the solute in a 1.00 (a) 36.5 (b) 32.05
m aqueous solution ? [2015 RS] (c) 18.25 (d) 42.10
(a) 0.177 (b) 1.770 10. Which of the following statements, regarding
(c) 0.0354 (d) 0.0177 the mole fraction (x) of a component in solution,
4. How many grams of concentrated nitric acid is incorrect? [1999]
solution should be used to prepare 250 mL of (a) 0 < x < 1
2.0M HNO3 ? The concentrated acid is 70% (b) x < 1
HNO3 [NEET 2013] (c) x is always non-negative
(a) 90.0 g conc. HNO3 (d) –2 < x < 2
(b) 70.0 g conc. HNO3
11. A 5% solution of cane sugar (mol. wt. =342) is
(c) 54.0 g conc. HNO3
isotonic with 1% solution of a substance X. The
(d) 45.0 g conc. HNO3
molecular weight of X is [1998]
5. Mole fraction of the solute in a 1.00 molal
aqueous solution is : [2011] (a) 34.2 (b) 171.2
(a) 0.1770 (b) 0.0177 (c) 68.4 (d) 136.8
(c) 0.0344 (d) 1.7700 12. The number of moles of oxygen in one litre of air
containing 21% oxygen by volume, in standard
6. 25.3 g of sodium carbonate, Na 2 CO3 is
conditions, is [1995]
dissolved in enough water to make 250 mL of (a) 0.186 (b) 0.21
solution. If sodium carbonate dissociates
(c) 0.0093 (d) 2.10
completely, molar concentration of sodium ions,
13. Which of the following modes of expressing
Na + and carbonate ions, CO32– are respectively concentration is independent of temperature ?
(Molar mass of Na2CO3 = 106 g mol–1) [2010] [1992, 1995]
(a) 0.955 M and 1.910 M
(a) Molarity (b) Molality
(b) 1.910 M and 0.955 M
(c) Formality (d) Normality
(c) 1.90 M and 1.910 M
(d) 0.477 M and 0.477 M
EBD_7324
ïìð ÝØÛÓ×ÍÌÎÇ
̱°·½ îæ Ê¿°±«® Ю»­­«®»ô Ô¿©­ ±º 200 mm Hg and 41.5 mm Hg respectively. Vapour
ͱ´«¬·±²­ ¿²¼ ×¼»¿´ô Ò±²ó·¼»¿´ ͱ´«¬·±²­ pressure of the solution obtained by mixing
25.5 g of CHCl3 and 40 g of CH2Cl2 at the same
14. Which of the following statement about the temperature will be : (Molecular mass of CHCl3
composition of the vapour over an ideal 1 : 1 = 119.5 u and molecular mass of CH2Cl2 = 85 u).
molar mixture of benzene and toluene is correct? [2012 M]
Assume that the temperature is constant at 25°C. (a) 173.9 mm Hg (b) 615.0 mm Hg
(Given : Vapour Pressure Data at 25°C, benzene (c) 347.9 mm Hg (d) 285.5 mm Hg
= 12.8 kPa, toluene = 3.85 kPa) [2016] 20. An aqueous solution is 1.00 molal in KI. Which
(a) The vapour will contain a higher change will cause the vapour pressure of the
percentage of benzene solution to increase? [2010]
(b) The vapour will contain a higher (a) Addition of NaCI
percentage of toluene (b) Addition of Na 2SO4
(c) The vapour will contain equal amounts of (c) Addition of 1.00 molal KI
benezene and toluene (d) Addition of water
(d) Not enough information is given to make a 21. A solution of acetone in ethanol [2006]
predication (a) shows a positive deviation from Raoult’s law
15. At 100°C the vapour pressure of a solution of (b) behaves like a non ideal solution
6.5g of a solute in 100 g water is 732 mm. If (c) obeys Raoult’s law
Kb = 0.52, the boiling point of this solution will (d) shows a negative deviation from Raoult’s law
be [2016]
22. The vapour pressure of two liquids ‘P’ and
(a) 101°C (b) 100°C ‘Q’ are 80 and 60 torr, respectively. The total
(c) 102°C (d) 103°C vapour pressure of solution obtained by
16. Which one is not equal to zero for an ideal mixing 3 mole of P and 2 mole of Q would be
solution: [2015] [2005]
(a) Smix (a) 72 torr (b) 140 torr
(b) Vmix (c) 68 torr (d) 20 torr
(c) P = Pobserved - PRaoult 23. Formation of a solution from two components
(d) Hmix can be considered as [2003]
17. Which condition is not satisfied by an ideal (i) Pure solvent separated solvent
solution? [NEET Kar. 2013] molecules, H1
(a) mix H = 0 (ii) Pure solute separated solute
(b) mix V = 0 molecules, H2
(c) mix S = 0 (iii) Separted solvent
(d) Obeyance to Raoult’s Law & solute molecules Solution, H3
18. PA and PB are the vapour pressure of pure liquid Solution so formed will be ideal if
components, A and B, respectively of an ideal
binary solution. If X A represents the mole (a) ÜHsoln ã ÜH3 óÜH1 óÜH 2
fraction of component A, the total pressure of (b) ÜHsoln ã ÜH1 õ ÜH 2 õ ÜH3
the solution will be [2012]
(a) PA + XA (PB – PA) (c) ÜHsoln ã ÜH1 õ ÜH 2 óÜH3
(b) PA + XA (PA – PB) (d) ÜHsoln ã ÜH1 óÜH 2 óÜH3
(c) PB + XA (PB – PA) 24. A solution containing components A and B
(d) PB + XA (PA – PB) follows Raoult's law when [2002]
19. Vapour pr essur e of chloroform (CHCl 3 ) (a) A – B attraction force is greater than A – A
and dichloromethane (CH2Cl 2 ) at 25ºC are and B – B
ͱ´«¬·±²­ ïìï

(b) A – B attraction force is less than A – A and 30. The relative lowering of the vapour pressure is
B–B equal to the ratio between the number of
(c) A – B attraction force remains same as [1991]
A–A and B –B (a) solute molecules to the solvent molecules
(d) Volume of solution is different from sum of (b) solute molecules to the total molecules in
volume of solute and solvent the solution
25. The beans are cooked earlier in pressure cooker, (c) solvent molecules to the total molecules in
because [2001] the solution
(a) Boiling point increases with increasing (d) solvent molecules to the total number of
pressure ions of the solute.
(b) Boiling point decreases with increasing 31. An ideal solution is formed when its components
pressure [1988]
(c) Internal energy is not lost while cooking in (a) have no volume change on mixing
pressure cooker (b) have no enthalpy change on mixing
(d) Extra pressure of pressure cooker, softens (c) have both the above characteristics
the beans (d) have high solubility.
26. The vapour pressure of a solvent decreased by 32. All form ideal solution except [1988]
10mm of mercury when a non-volatile solute was (a) C6H6 and C6H5 CH3
added to the solvent. The mole fraction of the
(b) C2H6 and C2H5I
solute in the solution is 0.2. What should be the
(c) C6H5Cl and C6H5 Br
mole fraction of the solvent if the decrease in
the vapour pressure is to be 20mm of mercury? (d) C2H5 I and C2H5 OH.
[1998] ̱°·½ íæ ݱ´´·¹¿¬·ª» Ю±°»®¬·»­ ¿²¼
(a) 0.8 (b) 0.6 ß¾²±®³¿´ Ó±´»½«´¿® Ó¿­­»­
(c) 0.4 (d) 0.2
33. Which one of the following electrolytes has the
27. The vapour pressure at a given temperature of
same value of van't Hoff's factor (i) as that of the
an ideal solution containing 0.2 mol of a non-
Al2(SO4)3 (if all are 100% ionised) ? [2015]
volatile solute and 0.8 mol of solvent is 60 mm of
Hg. The vapour pressure of the pure solvent at (a) K3[Fe(CN)6] (b) Al(NO3)3
the same temperature is [1996] (c) K4[Fe(CN)6] (d) K2SO4
(a) 150 mm of Hg (b) 60 mm of Hg
34. The boiling point of 0.2 mol kg–1 solution of X
(c) 75 mm of Hg (d) 120 mm of Hg in water is greater than equimolal solution of Y
28. Vapour pressure of benzene at 30°C is 121.8 mm. in water. Which one of the following statements
When 15 g of a non volatile solute is dissolved is true in this case ? [2015]
in 250 g of benzene its vapour pressure (a) Molecular mass of X is greater than the
decreased to 120.2 mm. The molecular weight of molecular mass of Y.
the solute (Mo. wt. of solvent = 78) [1995] (b) Molecular mass of X is less than the
(a) 356.2 (b) 456.8 molecular mass of Y.
(c) 530.1 (d) 656.7 (c) Y is undergoing dissociation in water while
29. According to Raoult's law, relative lowering of X undergoes no change.
vapour pressure for a solution is equal to (d) X is undergoing dissociation in water.
[1995] 35. Of the following 0.10 m aqueous solutions,
(a) moles of solute which one will exhibit the largest freezing point
(b) moles of solvent depression? [2014]
(c) mole fraction of solute (a) KCl (b) C6H12O6
(d) mole fraction of solvent (c) Al2(SO4)3 (d) K2SO4
EBD_7324
ïìî ÝØÛÓ×ÍÌÎÇ
36. The freezing point depression constant for water 43. During osmosis, flow of water through a
is – 1.86ºC m–1. If 5.00 g Na2SO4 is dissolved in semipermeable membrane is [2006]
45.0 g H2O, the freezing point is changed by (a) from both sides of semipermeable
– 3.82ºC. Calculate the van’t Hoff factor for membrane with equal flow rates
Na2SO4. [2011] (b) from both sides of semipermeable
(a) 2.05 (b) 2.63 membrane with unequal flow rates
(c) 3.11 (d) 0.381 (c) from solution having lower concentration
37. The van’t Hoff factor i for a compound which only
undergoes dissociation in one solvent and (d) from solution having higher concentration
association in other solvent is respectively : only
[2011] 44. A solution containing 10 g per dm3 of urea
(a) less than one and greater than one. (molecular mass = 60 g mol–1) is isotonic with a
(b) less than one and less than one. 5% solution of a non-volatile solute. The
(c) greater than one and less than one. molecular mass of this non-volatile solute is
(d) greater than one and greater than one. [2006]
38. A 0.1 molal aqueous solution of a weak acid is (a) 300 g mol–1 (b) 350 g mol–1
30% ionized. If Kf for water is 1.86°C/m, the (c) 200 g mol–1 (d) 250 g mol–1
freezing point of the solution will be : [2011 M] 45. 1.00 g of a non-electrolyte solute (molar mass
(a) – 0.18°C (b) – 0.54°C 250 g mol–1) was dissolved in 51.2 g of benzene.
(c) – 0.36°C (d) – 0.24°C If the freezing point depression constant, Kf of
39. 200 mL of an aqueous solution of a protein benzene is 5.12 K kg mol–1, the freezing point of
contains its 1.26 g. The osmotic pressure of this benzene will be lowered by [2006]
solution at 300 K is found to be 2.57 × 10–3 (a) 0.3 K (b) 0.5 K
bar. The molar mass of protein will be (c) 0.4 K (d) 0.2
(R = 0.083 L bar mol–1 K–1) [2011 M] 46. A solution of urea (mol. mass 56 g mol 1) boils
(a) 51022 g mol–1 (b) 122044 g mol–1
at 100.18 C at the atmospheric pressure. If Kf
(c) 31011 g mol–1 (d) 61038 g mol–1 and Kb for water are 1.86 and 0.512 K kg mol 1
40. A solution of sucrose (molar mass = 342 g mol –1)
respectively, the above solution will freeze at
has been prepared by dissolving 68.5 g of
sucrose in 1000 g of water. The freezing point of [2005]
the solution obtained will be ( Kf for water (a) 0.654 C (b) 0.654 C
(c) 6.54 C (d) 6.54 C
= 1.86 K kg mol–1). [2010]
(a) – 0.372°C (b) – 0.520°C 47. Camphor is often used in molecular mass
(d) + 0.372°C (d) – 0.570°C determination because [2004]
41. A 0.0020 m aqueous solution of an ionic (a) it is readily available
compound Co(NH 3 ) 5 (NO 2 )Cl freezes at (b) it has a very high cryoscopic constant
– 0.00732 °C. Number of moles of ions which 1 (c) it is volatile
mol of ionic compound produces on being (d) it is solvent for organic substances
dissolved in water will be (Kf = – 1.86°C/m) 48. A solution contains non-volatile solute of
[2009] molecular mass M2. Which of the following can
(a) 3 (b) 4 be used to calculate the molecular mass of solute
(c) 1 (d) 2 in terms of osmotic pressure ? [2002]
42. 0.5 molal aqueous solution of a weak acid (HX)
(a) m2 m 2 RT
is 20% ionised. If Kf for water is 1.86 K kg M2 VRT (b) M 2
mol–1 ,the lowering in freezing point of the V
solution is [2007] m2 m2
(c) M2 RT (d) M 2
(a) 0.56 K (b) 1.12 K V V RT
(c) – 0.56 K (d) – 1.12 K
ͱ´«¬·±²­ ïìí

49. Which of the following colligative property can 54. If 0.1 M solution of glucose and 0.1 M solution
provide molar mass of proteins (or polymers or of urea are placed on two sides of th e
colloids) with greatest precision ? [2000] semipermeable membrane to equal heights, then
(a) Osmotic pressure it will be correct to say that [1992]
(b) Elevation of boiling point (a) There will be no net movement across the
(c) Depression of freezing point membrane
(d) Relative lowering of vapour pressure (b) Glucose will flow towards urea solution
(c) urea will flow towards glucose solution
50. Which of the following 0.10 m aqueous solutions (d) water will flow from urea solution to glucose
will have the lowest freezing point ? [1997]
(a) Al2(SO4)3 (b) C6H12O6 55. Which of the following aqueous solution has
(c) KCl (d) C12H22O11 minimum freezing point ? [1991]
51. At 25°C, the highest osmotic pressure is (a) 0.01 m NaCl
exhibited by 0.1 M solution of [1994] (b) 0.005 m C2H5OH
(a) CaCl2 (b) KCl (c) 0.005 m MgI2
(c) Glucose (d) Urea. (d) 0.005 m MgSO4.
52. Which one of the following salts will have the 56. Blood cells retain their normal shape in solution
same value of van’t Hoff factor (i) as that of which are [1991]
K4[Fe (CN)6]. [1994] (a) hypotonic to blood
(a) Al2(SO4)3 (b) isotonic to blood
(b) NaCl (c) hypertonic to blood
(c) Al (NO3)3 (d) equinormal to blood.
(d) Na2SO4.
53. Which one is a colligative property ? [1992]
(a) boiling point (b) vapour pressure
(c) osmotic pressure (d) freezing point

ANSWER KEY
1 (c) 7 (b) 13 (b) 19 (N) 25 (a) 31 (c) 37 (c) 43 (c) 49 (a) 55 (a)
2 (a) 8 (c) 14 (a) 20 (d) 26 (b) 32 (d) 38 (d) 44 (a) 50 (a) 56 (b)
3 (d) 9 (b) 15 (a) 21 (a) 27 (c) 33 (c) 39 (d) 45 (c) 51 (a)
4 (d) 10 (a) 16 (a) 22 (a) 28 (a) 34 (d) 40 (a) 46 (b) 52 (a)
5 (b) 11 (c) 17 (c) 23 (b) 29 (c) 35 (c) 41 (d) 47 (b) 53 (c)
6 (b) 12 (c) 18 (d) 24 (c) 30 (b) 36 (b) 42 (b) 48 (b) 54 (a)
EBD_7324
ïìì ÝØÛÓ×ÍÌÎÇ

Hints & Solutions


1. (c) K f (molal depression constant) only
[Na + ] = 2 × 0.955 = 1.91 M
depends on the nature of the solvent and is
independent of the concentration of the
CO32– = 0.955 M
solution.
2. (a) Molarity depends on the volume of a 7. (b) One molal solution means one mole solute
solution which can be changed with change in present in 1 kg (1000 g) solvent
temperature. i.e., mole of solute = 1
W 1 1000 g 1000
3. (d) Molality ã l Mole of solvent (H2O) = =
M V(kg) 18g 18
n 1
ã (where n solute = W/M) Mole fraction of solute =
V(kg) 1000
1
n 18
1.00 m ã i.e., 1 mole in 1 kg of water 18
V(kg) = = 0.01768 å 0.018.
Moles of 1 kg H2O 1018
8. (c) From molarity equation
1000g
ã ã 55.55 mole M1V1 + M2V2 = MV
18g / mol 1× 2.5 + 0.5 × 3 = M × 5.5
Moles of solute = 1
4
Mole fraction Mã ã 0.727 å 0.73M
5.5
nsolute 1
ã ã 9. (b) Density = 1.17 gm/cc
n solute õ n water (1 õ 55.55)
= 0.01768 = 0.0177 Mass
As d ã
wt 1000 Volume
4. (d) Molarity (M) = volume = 1cc mass = 1.17g
mol. wt. vol (ml)
wt. 1000 No. of moles obsolute
2= × Now molarity ã
63 250 Volume of solution in litre
63
wt. = gm
2 1.17 l1000 1170
ã ã =32.05M
100 36.5l1 36.5
wt. of 70% acid = 31.5 = 45 gm
70 10. (a) Mole fraction of any component A
5. (b) 1 molal solution means 1 mole of solute
dissolved in 1000 gm solvent. No. of moles of A
x
nsolute = 1 wsolvent = 1000g Total No. of moles
1000 As total no. of moles > no. of moles of A
nsolvent = = 55.56
18 thus x can never be equal to one on zero.
11. (c) 5% cane sugar solution means 100ml of
1 solution contain cane sugar = 5g
Xsolute = = 0.0177
1 55.56 1000 ml of solution contain cane sugar
6. (b) Concentration of
5
ã l1000 ã 50g / L
25.3 1000 100
Na 2 CO3 = = 0.955 M
106 250
ͱ´«¬·±²­ ïìë

1l1000 P Ps n W1 M2
Similarily 1% solution contains ã 15. (a)
100 P N M1 W2
= 10g/L cane sugar Where, W1 = wt of solute
50 g / L W2 = wt of solvent
( 1) = C × S × T = 0.0821 T M1 = Mass of solute
342
M2 = Mass of solvent
Osmotic pressure of 1% solution of
at 100°C, P° = 760 mm
susbtance ( 2)
10 g / L 760 732 6.5 18
= 0.0821 T 760 M1 100
M
As both are isotonic, M1 = 31.75 g mol–1
So 1= 2
50 10 W1 1000
or 0.0821 T 0.0821 T Tb = m × Kb = × Kb
342 M M1 W2
342
M (mol. wt. of X) = 68.4 0.52 6.5 1000
5 Tb = = 1.06°C
31.75 100
12. (c) Percentage volume of oxygen = 21%.
So, 100 ml of air contains = 21ml of O2 boiling point of solution
Volume of oxygen in one litre of air = 100°C + 1.06°C = 101°C
21 16. (a) For an ideal solution Smix > 0
= 1000 210ml .
100 17. (c) An ideal solution is that solution in which
each component obeys Raoult’s law under all
Therefore no. of moles 210
0.0093 conditions of temperatures and concentrations.
22400 For an ideal solution.
( volume of 1 litre of gas at S.T.P. is 22400 ml) Hmix = 0 and Vmix = 0.
13. (b) The molality involves weights of the solute
18. (d) P = PAXA + PBXB
and the solvent. Since weight does not change
with the temperature, therefore molality does = PAXA + PB (1 – XA)
not depend upon the temperature. (for binary sol. XA + XB = 1)
14. (a) Let us consider that A is benzene and B is PAXA + PB – PBXA
toluene
PB+XA (PA – PB)
1 : 1 molar mixture of A and B
1 1 25.5
xA = and xB = 19. (N) n CHCl3 0.213
2 2 119.5
Total pressure of solution (P) = PA0 x A PB0 x B
40
1 1 n CH 2Cl2 0.47
P = 12.8 × + 3.85 × = 8.325 kPa 85
2 2
1
0 12.8 PT PA X A PB X B
YA = PA x A 2 0.768
P 8.325
0.213 0.47
yB = 1 – yA = 1 – 0.768 = 0.232 200 41.5
0.683 0.683
so, the vapour will contain higher percentage of
benzene. = 62.37 + 28.55
= 90.92
EBD_7324
ïìê ÝØÛÓ×ÍÌÎÇ
20. (d) When the aqueous solution of one molal Po ó P
KI is diluted with water, concentration decreases, ã xB
therefore the vapour pressure of the resulting Po
solution increases. .2 1
xB Mole fraction of solute
21. (a) A solution of acetone in ethanol shows .2 .8 5
positive deviation from Raoult's law. It is because P 60mm of Hg
ethanol molecules are strongly hydrogen
bonded. When acetone is a dded, t hese Po ó P 1
molecules break the hydrogen bonds and ã or 4Po ã (P)l5
Po 5
ethanol becomes more volatile. Therefore its
vapour pressure is increased. 60l5
B Po ã ã 75mm of Hg
22. (a) Given V.PP = 80 torr 4
V.PQ = 60 torr 28. (a) Given vapour pressure (p°) = 121.8 mm;
Ptotal = V·PP × XP + V·PQ × XQ Weight of solute (w) = 15 g
3 2 Weight of solvent (W) = 250 g
= 80 60 = 16 × 3 + 12 × 2 From Raoult’s law,
5 5
Ptotal = 48 + 24 = 72 torr Po P w M 121.8 120.2 15 78
23. (b) For an ideal solution, Hmixing = 0 Po m W 121.8 m 250
H = H1 + H2 + H3 (Accroding to Hess's law)
15 78 121.8
i.e., for ideal solutions there is no change in or m 356.2
magnitude of the attractive forces in the two 250 1.6
components present. 29. (c) Relative lowering of vapour pressure
24. (c) These two components A and B follows the depends upon the mole fraction of solute.
condition of Raoult’s law if the force of attraction
Po
P
between A and B is equal to the force of attraction i.e., mole fraction of solute
between A and A or B and B. Po
25. (a) The beans are cooked earlier in pressure 30. (b) According to Raoult's law, the relative
cooker because boiling point increases with lowering in vapour pressure of a dilute solution
increasing pressure. is equal to the mole fraction of the solute present
26. (b) According to Raoult's law in the solution.
p p n p p n
(mole fraction of solute) = mole fraction of solute ã
p n N p nõN
10 31. (c) For ideal solution,
0.2 p° = 50 mm
p Vmixing 0 and H mixing 0.
For other solution of same solvent
32. (d) C2H5I and C2H5OH form non-ideal solution.
20 n
p n N
(mole fraction of solute) 33. (c) K4[Fe(CN)6] 4K+ + [Fe(CN)6]–
and Al2(SO4)3 2Al3+ + 3SO 2–
4
20
mole fraction of solute van’t Hoff factor is 5 for both Al2 (SO4)3 and
50
K4[Fe(CN)6]
Mole fraction of solute = 0.4
34. (d) Tb = iKb m
Hence, mole fraction of solvent = 1 – 0.4
= 0.6 Given, ( Tb)x > ( Tb)y
27. (c) According to Raoult's law ix Kb m > iyKb m
ͱ´«¬·±²­ ïìé

(Kb is same for same solvent) 41. (d) Tf = 0 – (– 0.00732°) = 0.00732


ix > iy Tf = i × Kf × m
So, x is undergoing dissociation in water. Tf 0.00732
35. (c) Colligative properties no. of particles. i= = 2
K f × m 1.86× 0.002
Since Al2(SO4)3 contains maximum number of
particles, hence will have the largest value of 42. (b) As Tf = i Kf m
freezing point depression. For
36. (b) Given Kf = – 1.86º cm–1, mass of solute
= 5.00 g, mass of solvent = 45.0 g HX H X
Tf = i × Kf. m t 0 1 0 0
5 1000 t eq (1 0.20) 0.20 0.20
3.82 = i × 1.86 ×
142 45
Total no. of moles = 1 – 0.20 + 0.20 + 0.20 =
(Molecular mass of Na2SO4 = 142) 1 + 0.20 = 1.2
i = 2.63
Tf = 1.2 × 1.86 × 0.5 = 1.1160 1.12 K
37. (c) If compound dissociates in solvent i > 1
43. (c) During osmosis water flows through
and on association i < 1. semipermeable membrane from lower
concentration to higher concentration.
38. (d) Given = 30% i.e., 0.3 44. (a) Osmotic pressure of urea from the formula
HA H+ + A– nRT
V nRT
1– V
1 – 0.3 0.3 0.3 10
B l.0821l T [1 dm3 = 1 litre]
i = 1 – 0.3 + 0.3 + 0.3 60
5% solution means
i = 1.3 100 ml f 5gm
Tf = 1.3 × 1.86 × 0.1 = 0.2418 1000 ml f 50 g/L
Tf = 0 – 0.2418 = – 0.2418 °C Osmotic pressure of solution having non-
39. (d) = CRT (Osmotic pressure) 50
volatile solute 2ã × .0821 × T
wt 1000 M
= Molecular mass V RT For isotonic solution, 1= 2

3 10 50
= 2.57 10 .0821 T .0821 T
60 M
1.26 1000 M = 300 gm mol–1
0.083 300
Mol.mass 200 1 1000
45. (c) T Kf m 5.12 0.4K
1.26 0.083 300 1000 250 51.2
Molecular mass =
200 2.57 10 3 46. (b) As Tf = Kf. m
= 61038 g mol–1 Tb = Kb. m
Tf Tb
1000W2 Hence, we have m
40. (a) Tf Kf Kf Kb
M 2W1 K
or Tf Tb f
1.86×1000×68.5 Kb
= = 0.372°C [ Tb 100.18 100 0.18 C]
342×1000
T f = – 0.372°C (0 – Tf ) 1.86
0.18 0.654 C
0.512
EBD_7324
ïìè ÝØÛÓ×ÍÌÎÇ
As the Freezing Point of pure water is 0°C, 51. (a) Conc. of particles in CaCl2 sol. will be max.
Tf = 0 –Tf as i = 3 is max. So it exhibit highest osmotic
0.654 = 0 – Tf pressure.
Tf = – 0.654 Note : Glucose and Urea do not dissociate into
Thus the freezing point of solution will be ions, as they are nonelectrolytes.
– 0.654°C.
47. (b) Solvent having high cryoscopic constant 52. (a) K 4 [Fe(CN) 6 ] and Al 2 (SO 4 )3 both
can be used in determination of molecular mass dissociates to give 5 ions or i = 5
by cryoscopic method.
K 4 [Fe(CN )6 ] 4K [ Fe(CN )6 ]
2 RT
48. (b) V RT RT or M 2 and
M V
Where M2 = molecular mass of solute Al 2 (SO 4 )3 2Al3õ õ 3SO 24ó
and m2 = mass of solute
49. (a) Molecular masses of polymers are best 53. (c) Osmotic pressure is a colligative property.
determined by osmotic pressure method . Firstly 54. (a) As both the solutions are isotonic hence
because other colligative properties give so low there will be no net movement of the solvent
values that they cannot be measured accurately through the semipermeable membrane between
and secondly, osmotic pressure measurements two solutions.
can be made at room temperature and do not
55. (a) Tf = i × Kf × m
require heating which may change the nature of
the polymer. Van't Hoff factor, i = 2 for NaCl,
50. (a) Depression in F.P. No. of particles. hence Tf = 0.02 Kf which is maximum in the
Al 2 (SO 4 )3 provides five ions on ionisation present case.
Hence Tf is maximum or freezing point is
Al 2 (SO 4 ) 3 2 Al 3 3SO 24 –
minimum.
while KCl provides two ions
56. (b) Blood cells neither swell nor shrink in
KCl K õ õ Cló isotonic solution. As isotonic solution have
equal concentration therefore there is no flow
C6H12O6 and C12H22O11 are not ionised so they of solvent is found and hence solvent neither
have single particle. enters nor flow out of the blood cells.
Hence, Al 2 (SO4 )3 have maximum value of
depression in F.P or lowest F.P
ݸ¿°¬»®

17 Electrochemistry

̱°·½ ïæ ݱ²¼«½¬¿²½» ¿²¼ ݱ²¼«½¬·ª·¬§ (a) 425.5 S cm2 mol–1 (b) 180.5 S cm2 mol–1
(c) 290.8 S cm2 mol–1 (d) 390.5 S cm2 mol–1
1. Ionic mobility of which of the following alkali 6. An increase in equivalent conductance of a
metal ions is lowest when aqueous solution of strong electrolyte with dilution is mainly due to:
their salts are put under an electric field ? [2017] [2010]
(a) K (b) Rb (a) increase in ionic mobility of ions
(c) Li (d) Na (b) 100% ionisation of electrolyte at normal
2. Aqueous solution of which of the following dilution
compounds is the best conductor of electric (c) increase in both i.e. number of ions and
current ? [2015 RS] ionic mobility of ions
(a) Acetic acid, C2H4O2 (d) increase in number of ions
(b) Hydrochloric acid, HCl 7. Which of the following expressions correctly
(c) Ammonia, NH3 represents the equivalent conductance at infinite
(d) Fructose, C6H12O6 dilution of Al2 (SO4)3, Given that Al3 and
3. At 25°C molar conductance of 0.1 molar aqueous SO24 are the equivalent conductances at
solution of ammonium hydroxide is 9.54 infinite dilution of the respective ions? [2010]
ohm-1 cm2mol-1 and at infinite dilution its molar
conductance is 238 ohm-1 cm2 mol-1. The degree (a) 1 Al3 1 SO24
or ionisation of ammonium hydroxide at the same 3 2
concentration and temperature is : [NEET 2013]
(b) 2 3
(a) 20.800% (b) 4.008% Al3 SO 24
(c) 40.800% (d) 2.080%
4. Limiting molar conductivity of NH4OH (c) Al3 SO24
o
i.e., m(NH 4OH is equal to : [2012] (d) 6
Al3 SO 24
o o o
(a) M
m NH 4 Cl m NaCl m NaOH 8. The equivalent conductance of solution of
o o o
32
(b) m NaOH m NaCl m NH 4Cl a weak monobasic acid is 8.0 mho cm2 and at
o o o infinite dilution is 400 mho cm2. The dissociation
(c) m NH 4OH m NH 4Cl m HCl constant of this acid is: [2009]
o o o (a) 1.25 × 10–6 (b) 6.25 × 10–4
(d) m NH 4 Cl m NaOH m NaCl (c) 1.25 × 10–4 (d) 1.25 × 10–5
5. Molar conductivities ( m ) at infinite dilution 9. Kohlrausch’s law states that at : [2008]
(a) finite dilution, each ion makes definite
of NaCl, HCl and CH3COONa are 126.4, 425.9
contribution to equivalent conductance of
and 91.0 S cm2 mol–1 respectively. m for an electrolyte, whatever be the nature of
CH3COOH will be : [2012 M] the other ion of the electrolyte.
EBD_7324
ïëð ÝØÛÓ×ÍÌÎÇ
(b) infinite dilution each ion makes definite 16. Which of the following is an insulator ?[1992]
contribution to equivalent conductance of (a) Graphite (b) Aluminium
an electrolyte depending on the nature of (c) Diamond (d) Silicon.
the other ion of the electrolyte.
̱°·½ îæ Û´»½¬®±´§­·­ ¿²¼ ̧°»­ ±º Û´»½¬®±´§­·­
(c) infinite dilution, each ion makes definite
contribution to conductance of an 17. When 0.1 mol MnO42– is oxidised the quantity
electrolyte whatever be the nature of the of electricity required to completely oxidise
other ion of the electrolyte. MnO42– to MnO4– is: [2014]
(d) infinite dilution, each ion makes definite (a) 96500 C (b) 2 × 96500 C
contriubtion to equivalent conductance of (c) 9650 C (d) 96.50 C
an electrolyte, whatever be the nature of 18. The weight of silver (at wt. = 108) displaced by a
the other ion of the electrolyte. quantity of electricity which displaces 5600 mL
10. The ionic conductance of Ba 2+ and Cl – are of O2 at STP will be :- [2014]
respectively 127 and 76 ohm–1 cm2 at infinite (a) 5.4 g (b) 10.8 g
dilution. The equivalent conductance (c) 54.9 g (d) 108.0 g
(in ohm–1 cm2) of BaCl2 at infinite dilution will be : 19. How many grams of cobalt metal will be
(a) 139.5 (b) 203 [2000] deposited when a solution of cobalt (II) chloride
(c) 279 (d) 101.5 is electrolyzed with a current of 10 amperes for
11. Specific conductance of a 0.1 N KCl solution at 109 minutes (1 Faraday = 96,500 C; Atomic mass
23ºC is 0.012 ohm–1 cm–1. Resistance of cell of Co = 59 u) [NEET Kar. 2013]
containing the solution at same temperature was (a) 0.66 (b) 4.0
found to be 55 ohm. The cell constant is [2000] (c) 20.0 (d) 40.0
(a) 0.918 cm–1 (b) 0.66 cm–1
–1 20. Al2O3 is reduced by electrolysis at low potentials
(c) 1.142 cm (d) 1.12 cm–1
and high currents. If 4.0 × 104 amperes of current
12. Equivalent conductances of NaCl, HCl and
is passed thr ough molten Al 2 O 3 for
CH3COONa at infinite dilution are 126.45, 426.16
6 hours, what mass of aluminium is produced?
and 91 ohm–1 cm2 respectively. The equivalent
(Assume 100% current efficiency and At. mass
conductance of CH3COOH at infinite dilution
of Al = 27 g mol –1) [2009]
would be [1997]
(a) 8.1 × 104 g (b) 2.4 × 105 g
(a) 101.38 ohm–1 cm2 (b) 253.62 ohm–1 cm2
(c) 1.3 × 104 g (d) 9.0 × 103 g
(c) 390.71 ohm–1 cm2 (d) 678.90 ohm–1 cm2
21. 4.5 g of aluminium (at. mass 27 amu) is
13. If 0.01 M solution of an electrolyte has a
deposited at cathode from Al 3+ solution by a
resistance of 40 ohms in a cell having a cell
certain quan tity of electric charge. The
constant of 0.4 cm–1, then its molar conductance
volume of hydrogen produced at STP from
in ohm–1 cm2 mol–1 is [1997]
H+ ions in solution by the same quantity of
(a) 102 (b) 104
electric charge will be [2005]
(c) 10 (d) 103
(a) 44.8 L (b) 22.4 L
14. On heating one end of a piece of a metal, the
(c) 11.2 L (d) 5.6 L
other end becomes hot because of [1995]
22. In electrolysis of NaCl when Pt electrode is taken
(a) resistance of the metal
then H2 is liberated at cathode while with Hg
(b) mobility of atoms in the metal
cathode it forms sodium amalgam. This is
(c) energised electrons moving to the other end
because [2002]
(d) minor perturbation in the energy of atoms
(a) Hg is more inert than Pt
15. If 0.5 A current is passed through acidified silver
(b) More voltage is required to reduce H+ at
nitrate solution for 100 minutes. The mass of
Hg than at Pt
silver deposited on cathode, is (eq.wt.of silver
(c) Na is dissolved in Hg while it does not
nitrate = 108) [1995]
dissolve in Pt
(a) 2.3523 g (b) 3.3575 g
(d) Conc. of H+ ions is larger when Pt electrode
(c) 5.3578 g (d) 6.3575 g
is taken
Û´»½¬®±½¸»³·­¬®§ ïëï
23. Standard electrode potentials are : Fe+2/Fe (a) 0.59 V (b) 0.118 V
[ Eº = –0.44]; Fe+3/Fe+2 Eº = + 0.77 ; If Fe+2, Fe+3 (c) 1.18 V (d) 0.059 V
and Fe blocks are kept together, then [2001] 30. Consider the half-cell reduction reaction :
(a) Fe+2 increases Mn2+ + 2e– Mn, E° = –1.18 V
(b) Fe+3 decreases Mn2+ Mn3+ + e–, E° = –1.51 V
Fe 2 The E° for the reaction 3Mn 2+ Mn0 + 2Mn3+,
(c) remains unchanged
Fe 3 and possibility of the forward reaction are,
+2
(d) Fe decreases respectively [NEET Kar. 2013]
24. On passing a current of 1.0 ampere for 16 min and (a) – 2.69 V and no (b) – 4.18 V and yes
5 sec through one litre solution of CuCl2, all (c) + 0.33 V and yes (d) + 2.69 V and no
copper of the solution was deposited at cathode. 31. Standard electrode potential of three metals X,
The strength of CuCl 2 solution was (Molar mass Y and Z are – 1.2 V, + 0.5 V and – 3.0 V,
of Cu= 63.5; Faraday constant = 96,500 Cmol–1) respectively. The reducing power of these metals
[1996] will be : [2011]
(a) 0.01 N (b) 0.01 M
(a) Y > Z > X (b) X > Y > Z
(c) 0.02 M (d) 0.2 N
(c) Z > X > Y (d) X > Y > Z
25. On electrolysis of dilute sulphuric acid using
32. The electrode potentials for [2011]
platinum electrodes, the product obtained at the
anode will be [1992] Cu2+(aq) + e– Cu+(aq)
(a) hydrogen (b) oxygen and Cu+(aq) + e– Cu(s)
(c) hydrogen sulphide (d) sulphur dioxide. are + 0.15 V and + 0.50, respectively. The value
of E 2 will be :
̱°·½ íæ Ý»´´­ ¿²¼ Û´»½¬®±¼» ᬻ²¬·¿´ô Cu / Cu
Ò»®²­¬ Û¯«¿¬·±² (a) 0.500 V (b) 0.325 V
(c) 0.650 V (d) 0.150 V
26. In the electrochemical cell :- [2017] 33. Standard electrode potential for Sn4+ / Sn2+ couple
Zn | ZnSO4 (0.01M) | | CuSO4 (1.0 M) | Cu, the emf is + 0.15 V and that for the Cr3+ / Cr couple is
of this Daniel cell is E1. When the concentration – 0.74 V. These two couples in their standard state
of ZnSO4 is changed to 1.0M and that of CuSO4 are connected to make a cell. The cell potential
changed to 0.01M, the emf changes to E 2. From will be : [2011]
the followings, which one is the relationship (a) + 1.19 V (b) + 0.89 V
RT (c) + 0.18 V (d) + 1.83 V
between E1 and E4? (Given, = 0.059)
F 34. If the E°cell for a given reaction has a negative
(a) E1 < E2 (b) E1 > E2 value, then which of the following gives the
(c) E2 = 0 E1 (d) E1 = E2 correct relationships for the values of G° and
27. The pressure of H2 required to make the potential Keq ? [2011]
of H2-electrode zero in pure water at 298 K is : (a) G° > 0 ; Keq > 1 (b) G° < 0 ; Keq > 1
[2016] (c) G° < 0 ; Keq < 1 (d) G° > 0 ; Keq < 1
(a) 10–14 atm (b) 10–12 atm 35. A solution contains Fe2+, Fe3+ and I– ions. This
(c) 10–10 atm (d) 10–4 atm solution was treated with iodine at 35°C. E° for
28. A device that converts energy of combustion of Fe3+ / Fe2+ is + 0.77 V and E° for I2/2I– = 0.536 V.
fuels like hydrogen and methane, directly into The favourable redox reaction is : [2011 M]
electrical energy is known as : [2015] (a) I2 will be reduced to I–
(a) Electrolytic cell (b) Dynamo (b) There will be no redox reaction
(c) Ni-Cd cell (d) Fuel Cell (c) I– will be oxidised to I2
29. A hydrogen gas electrode is made by dipping (d) Fe2+ will be oxidised to Fe3+
platinum wire in a solution of HCl of pH = 10 and 36. For the reduction of silver ions with copper metal,
by passing hydrogen gas around the platinum the standard cell potential was found to be
wire at one atm pressure. The oxidation potential + 0.46 V at 25°C. The value of standard
of electrode would be ? [NEET 2013] Gibbs energy, G0 will be (F = 96500 C mol–1)
EBD_7324
ïëî ÝØÛÓ×ÍÌÎÇ
(a) – 89.0 kJ (b) – 89.0 J [2010] R = 8.314JK–1mol–1) [2004]
(c) – 44.5 kJ (d) – 98.0 kJ (a) 2.0 × 1011 (b) 4.0 × 1012
37. Consider the following relations for emf of a (c) 1.0 × 102 (d) 1.0 × 1010
electrochemical cell: [2010] 44. The e.m.f. of a Daniell cell at 298 K is E1.
(i) emf of cell = (Oxidation potential of anode) ZnSO 4 CuSO 4
– (Reduction potential of cathode) Zn Cu
(ii) emf of cell = (Oxidation potential of anode) (0.01 M) (1.0 M)
+ (Reduction potential of cathode) When the concentration of ZnSO4 is 1.0 M and
(iii) emf of cell = (Reduction potential of anode) that of CuSO4 is 0.01 M, the e.m.f. changed to
+ (Reduction potential of cathode) E2. What is the relationship between E 1 and E2?
(iv) emf of cell = (Oxidation potential of anode) (a) E 2 0 E1 (b) E1 E 2 [2003]
– (Oxidation potential of cathode)
(c) E1 E 2 (d) E1 E 2
Which of the above relations are correct?
(a) (ii) and (iv) (b) (iii) and (i) 45. On the basis of the information available from
(c) (i) and (ii) (d) (iii) and (iv) the reaction
38. Given: [2009] 4 2
Al O 2 Al 2 O 3 , G = – 827 kJ mol–1 of
(i) Cu2+ + 2e– Cu, Eo = 0.337 V 3 3
(ii) Cu2+ + e– Cu+, Eo = 0.153 V O2 the minimum e.m.f required to carry out an
Electrode potential, Eo for the reaction, electrolysis of Al2O3 is (F = 96500 C mol–1)
Cu + + e– Cu, will be : (a) 8.56 V (b) 2.14 V [2003]
(a) 0.90 V (b) 0.30 V (c) 4.28 V (d) 6.42 V
(c) 0.38 V (d) 0.52 V 46. Which reaction is not feasible? [2002]
39. On the basis of the following E° values, the
(a) 2KI Br2 2KBr I2
strongest oxidizing agent is : [2008]
[Fe(CN)6]4– [Fe(CN)6]3– + e–; E° = – 0.35 V (b) 2KBr I 2 2KI Br2
Fe2+ Fe3+ + –e– E° = – 0.77 V (c) 2KBr Cl 2 2KCl Br2
(a) [Fe(CN)6]4– (b) Fe2+ (d) 2H 2O 2F2 4HF O 2
(c) Fe3+ (d) [Fe(CN)6]3– 47. Cuaq+ is unstable in solution and undergoes
40. The equilibrium constant of the reaction: simultaneous oxidation and reduction according
Cu (s) 2Ag (aq) Cu 2 (aq) 2Ag(s) ; to the reaction : [2000]
2Cu (aq ) Cu 2 (aq) Cu (s)
E° = 0.46 V at 298 K is [2007] choose correct Eº for above reaction if Eº
(a) 2.0 × 1010 (b) 4.0 × 1010 Cu2+/Cu = 0.34 V and Eº Cu2+/Cu+ = 0.15 V
(c) 4.0 × 1015 (d) 2.4 × 1010 (a) –0.38 V (b) +0.49 V
p (c) +0.38 V (d) –0.19 V
41. If EpFe2õ / Fe ã ó0.441V and E Fe3õ / Fe2õ = +
48. What is the Eºcell for the reaction
0.771V, the standard EMF of the reaction Fe + 2õ
Cu (aq) 2õ
õ Sn (aq) õ 4õ
Cu (s) õ Sn (aq)
2Fe3+ 3Fe2+ will be [2006]
(a) 1.653 V (b) 1.212 V at 25ºC if the equilibrium constant for the reaction
(c) 0.111 V (d) 0.330 V is 1 × 106? [1999]
42. A hypothetical electrochemical cell is shown below (a) 0.5328 V (b) 0.3552 V
(c) 0.1773 V (d) 0.7104 V
A|A (xM)||B (yM)|B [2006]
49. For the cell reaction, [1998]
The emf measured is +0.20 V. The cell reaction is Cu2+ (C1, aq) + Zn(s) = Zn2+ (C2, aq) + Cu(s) of an
(a) A+ + e– A; B++ e– B electrochemical cell, the change in free energy,
(b) The cell reaction cannot be predicted G, at a given temperature is a function of
(c) A + B+ A+ + B (a) ln (C1) (b) ln (C2/C1)
(d) A+ + B A + B+ (c) ln (C2) (d) ln (C1 + C2)
43. The standard e.m.f. of a galvanic cell involving 50. Without losing its concentration ZnCl2 solution
cell reaction with n = 2 is found to be 0.295 V cannot be kept in contact with [1998]
at 25°C. The equilibrium constant of the (a) Au (b) Al
reaction would be (Given F = 96500 C mol–1; (c) Pb (d) Ag
Û´»½¬®±½¸»³·­¬®§ ïëí

51. Eº for the cell, Zn Zn 2 (aq) Cu 2 (aq) Cu is ̱°·½ ìæ ݱ³³»®½·¿´ Ý»´´­ ¿²¼ ݱ®®±­·±²
1.10 V at 25ºC. The equilibrium constant for the
56. A button cell used in watches functions as
cell reaction:
2+ 2+ following
Zn + Cu (aq) Cu + Zn (aq) , Zn(s) + Ag2O(s) + H2O(l)
is of the order of [1997] 2Ag(s) + Zn2+(aq) + 2OH–(aq)
(a) 10–18 (b) 10–37 If half cell potentials are :
(c) 1018 (d) 1037 Zn2+(aq) + 2e– Zn(s); Eo = – 0.76 V
52. Standard potentials (Eº) for some half-reactions Ag2O(s) + H2O (l) + 2e– 2Ag(s) +
are given below : 2OH–(aq); Eo = 0.34 V
(1) Sn 4 2e Sn 2 ; Eº 0.15 V The cell potential will be : [NEET 2013]
2
(2) 2 Hg 2e Hg 2 2 ; Eº 0.92 V (a) 0.42 V (b) 0.84 V
(3) PbO 2 4H 2e Pb 2 2H 2O ; (c) 1.34 V (d) 1.10 V
Eº 1.45 V 57. Standard free energies of formation (in kJ/mol)
at 298 K are – 237.2, – 394.4 and – 8.2 for H2O(l),
Based on the above, which one of the following
CO2(g) and pentane (g), respectively. The value
statements is correct ? [1997]
of E°cell for the pentane-oxygen fuel cell is :
(a) Sn4+ is a stronger oxidising agent than Pb4+
(a) 1.968 V (b) 2.0968 V [2008]
(b) Sn2+ is a stronger reducing agent than
(c) 1.0968 V (d) 0.0968 V
Hg22+
58. The efficiency of a fuel cell is given by [2007]
(c) Hg2+ is a stronger oxidising agent than Pb4+
(d) Pb2+ is a stronger reducing agent than Sn2+ G G
(a) (b)
53. The correct relationship between free energy and S H
equilibrium constant K of a reaction is [1996] S H
(a) G RT ln K (b) G RT lnK
(c) (d)
G G
(c) G RT lnK (d) G RT lnK 59. In the silver plating of copper, K[Ag(CN)2] is
54. An electrochemical cell is set up as: Pt; H2 used instead of AgNO3. The reason is [2002]
(1atm)|HCl(0.1 M) || CH3COOH (0.1 M)| H2 (1atm); (a) A thin layer of Ag is formed on Cu
Pt. The e.m.f of this cell will not be zero, because (b) More voltage is required
[1995] (c) Ag+ ions are completely removed from
(a) the temperature is constant solution
(b) e.m.f depends on molarities of acids used (d) Less availability of Ag + ions, as Cu cannot
(c) acids used in two compartments are displace Ag from [Ag(CN)2]– ion
different 60. The most convenient method to protect the
(d) pH of 0.1 M HCl and 0.1 M CH3COOH is bottom of ship made of iron is [2001]
not same (a) Coating it with red lead oxide
55. The standard reduction potentials at 25°C of Li+ (b) White tin plating
/ Li, Ba2+ / Ba, Na+ / Na and (c) Connecting it with Mg block
Mg2+ /Mg are – 3.03, – 2.73, – 2.71 and – 2.37 (d) Connecting it with Pb block
respectively. Which one of the following is the 61. The most durable metal plating on iron to protect
strongest oxidising agent? [1994] against corrosion is [1994]
(a) Na+ (b) Li+ (a) nickel plating (b) copper plating
(c) Ba2+ (d) Mg2+ (c) tin plating (d) zinc plating.
ANSWER KEY
1 (c) 7 (c) 13 (d) 19 (c) 25 (b) 31 (c) 37 (a) 43 (d) 49 (b) 55 (d) 61 (d)
2 (b) 8 (d) 14 (c) 20 (a) 26 (b) 32 (b) 38 (d) 44 (b) 50 (b) 56 (d)
3 (b) 9 (d) 15 (b) 21 (d) 27 (a) 33 (b) 39 (c) 45 (b) 51 (d) 57 (c)
4 (d) 10 (a) 16 (c) 22 (b) 28 (d) 34 (d) 40 (c) 46 (b) 52 (b) 58 (b)
5 (d) 11 (b) 17 (c) 23 (b) 29 (a) 35 (c) 41 (b) 47 (c) 53 (a) 59 (d)
6 (a) 12 (c) 18 (d) 24 (a) 30 (a) 36 (a) 42 (c) 48 (c) 54 (d) 60 (c)
EBD_7324
ïëì ÝØÛÓ×ÍÌÎÇ

Hints & Solutions


1. (c) Li+ being smallest, has maximum charge 8. (d) Degree of dissociation,
density.
8.0 2
Li+ is most heavily hydrated among all alkali 2 10
400
metal ions. Effective size of Li + in aqueous
solution is therefore, largest. So, moves slowest
C 2 1 2
under electric field. Ka C 2 2 10 2
(1 ) 32
2. (b) HCl completely dissociates to give H and
Cl ions, hence act as very good electrolyte. 1.25 10 5
While others are non- electrolytes. 9. (d) Kohlrausch’s Law states that at infinite
dilution, each ion migrates independently of its
m 9.54 co-ion and contributes to the total equivalent
3. (b) = = = 0.04008 = 4.008 %. conductance of an electrolyte a definite share
m 238
which depends only on its own nature.
o o o From this definition we can see that option (d) is
4. (d) m NH 4 Cl mNH 4 mCl
the correct answer.
o o o 10. (a) The equivalent conductance of BaCl2 at
m NaOH mNa mOH infinite dilution
1
o o o of BaCl 2 of Ba of Cl –
NaCl mNa mCl 2
o o 127
76 139.5 ohm –1 cm 2
m NH 4 m OH 2
11. (b) Given specific conductance of the solution
o o o ( ) = 0.012 ohm –1 cm –1 and resistance (R)
m NH 4 m Cl m Na = 55 ohm. We know that cell constant = Specific
conductance × Observed resistance = 0.012 ×
o o o 55= 0.66cm–1.
m OH m Na m Cl
12. (c) By Kohlraush's law, eq NaCl 126.45

o o o 126.45 ....(1)
Na Cl
m NH 4 OH m NH 4Cl m NaOH
426.16 ...(2)
o H Cl
m NaCl
91 ....(3)
CH3OO – Na
o o o o
5. (d) CH 3COOH CH 3COONa HCl NaCl on adding (2) and (3) then subtract (1) from it
= 91 + 425.9 – 126.4 = 390.5 517.16 126.45
6. (a) Dilution of strong electrolytes increases CH3COO Na
ionisation, hence ionic mobility of ions 1
increases which in turn increases equivalent Gp(CH3COOH) 390.71ohm cm 2
conductance of the solution. 13. (d) Given molarity = 0.01 M
7. (c) Equivalent conductance of an electrolyte Resistance = 40 ohm;
at infinite dilution is given by the sum of
equivalent conductances of the respective ions l 1
Cell constant 0.4cm
at infinite dilution. A
Û´»½¬®±½¸»³·­¬®§ ïëë

Specific conductivity ( ) 21. (d) No. of gram equivalent of H+ = No. of gm eq. of


4.5
cell constant 0.4 Al3+ = 0.5
= = 0.01 ohm 1 cm 1 9
resistance 40
27
1000 Eq. wt of Al3õ ã ã9
Molar conductance ( m) 3
Molarity
No. of gm eq. of H+ = no. of mole of H+
1000 .01 Hence Mass of H+ = 0.5 × 1g = 0.5 g
103 ohm 1cm 2 mol 1
.01 We know that, 2g H2 at STP = 22.4 L
14. (c) When one end of a metal is heated, the free
22.4
electrons are energised and move to the other 0.5 g H2 at STP = ã l 0.5 = 5.6 L
end. It heats up the other end of the metal. 2
15. (b) Given current (i) = 0.5 A; 22. (b) In electrolysis of NaCl when Pt electrode is
Time (t) = 100 minutes × 60 = 6000 sec taken then H2 liberated at cathode while with Hg
Equivalent weight of silver nitrate (E) = 108. cathode it forms sodium amalgam because more
According to Faraday's first law of electrolysis voltage is required to reduce H+ at Hg than Pt
2
Eit 108 0.5 6000 23. (b) Fe / Fe ; Eº = –0.44
W 3.3575 g.
96500 96500 3 2
Fe / Fe ; Eº 0.77
16. (c) Diamond is an insulator.
The metals having higher negative electrode
( 6) ( 7) potential values can displace metals having
17. (c) Mn O 24 MnO 4 e lower values of negative electrode potential from
0.1 mole their salt solutions.
Quantity of electricity required = 0.1F W q
= 0.1 × 96500 = 9650 C 24. (a) By Faraday's Ist Law,
E 96500
18. (d) wO = nO × 32
2 2 (where q it = charge of ion)
5600 we know that no of equivalent
wO = 32 8g = 1 equivalent of O2
2 22400
W it 1 965 1
= 1 equivalent of Ag = 108 E 96500 96500 100
19. (c) Applying, (where i = 1 A, t = 16×60+5 = 965 sec.)
Eit Since, we know that
w = Zit
96500 1
Equivalent weight of cobalt (II) = 59/2 no. of equivalent 100
Normality
I = 10 A Volume (in litre) 1
Time (t) = 109 min = 109 × 60 sec = 0.01 N
Substituting these values we get, 25. (b) At anode :
59 10 109 60 1
w = 20.0 2OH H 2O O2
2 96500 2
20. (a) As Q = i × t 26. (b) For cell,
Q = 4.0 × 104 × 6 × 60 × 60 C Zn|ZnSO4(0.01 M) || CuSO4(1.0 M)|Cu
= 8.64 × 10 8 C
Now since 96500 C liberates 9 g of Al 2.303RT log Zn 2
E cell E cell
8.64 × 108 C liberates 9
8.64 108 g Al
nF Cu 2
96500
= 8.1 × 104 g of Al 2.303RT 0.01
E1 E cell log
2 F 1
EBD_7324
ïëê ÝØÛÓ×ÍÌÎÇ
When concentrations are changed for ZnSO 4 G G1 G 2
and CuSO4, we can write
– nE° F = –1 n1 E1o F + (–1) n2 E2o F
2.303RT 1
E2 = E cell
2F
log
0.01
n1E1o n 2 E 2o 0.15 1 0.50 1
E
E1 > E2 n 2
27. (a) 2H+(aq) + 2e– H2(g) 0.325 V
33. (b) Given ESn 4 2 = + 0.15 V
0.0591 PH2 Sn
E = E0 – log
2
2 E Cr = – 0.74 V
H 3
Cr

PH 2 E ocell o
E cathode o
– E anode
0 = 0 – 0.0295 log
7 2 = 0.15 – (– 0.74) = + 0.89 V
10
34. (d) Standard Gibbs free energy is given as
PH 2 G° = – nE° F
=1 If E°cell < 0 i.e. – ve
7 2
10 G° > 0
Further G° = – RT ln Keq
PH2 = 10–14 atm
G° > 0 when Keq < 1
28. (d) A device th at converts en ergy of
35. (c) Given Fe+3/Fe2+ = + 0.77 V
combustion of fuels, directly into electrical
and I2 / 2I– = 0.536V
energy is known as fuel cell.
2 (e– + Fe+3 Fe+2) E° = 0.77 V
29. (a) H2 2H+ + 2e– – –
2I I2 + 2e E° = – 0.536 V
1 atm 10-10
2 2Fe+3 + 2I– 2Fe+2 + I2 E° = E°ox + E°red
10
E .059 10 Reduction potential of Fe3+ / Fe2+ > I2 / I–, so
H 2 /H =0– log Fe3+ will be reduced and I– will be oxidised.
2 1
+
E 36. (a) Cu + 2 Ag (aq) Cu 2+
(aq) + 2 Ag(s)
H 2 /H = +0.59 V
Here, n = 2 , E0cell = + 0.46 V
30. (a) E E red E oxd
1.81 1.51 2.69 G o = – nE o F
Since E° is negative – 2×0.46×96500
= kJ å – 89 kJ
G = –nFE°, G will have positive value so, 1000
forward reaction is not possible. 37. (a)
38. (d) Cu2+ + 2e– Cu; Go = – nEo F
31. (c) As the value of standard reduction potential = – 2 × F × 0.337
decreases the reducing power increases i.e., = – 0.674 F ....(i)
Z X Y Cu+ Cu2+ + e– ; Go = – nEo F
( 3.0) ( 1.2) ( 0.5) = – 1 × F × – 0.153
= 0.153 F ....(ii)
32. (b) Cu 2 1e Cu
On adding eqn (i) & (ii)
E1o 0.15V; G1o n1E1o F Cu+ + e– Cu ;
Go = – 0.521 F = – nEoF;
Cu 1e Cu Here n = 1 Eo = + 0.52 V
39. (c) From the given data we find Fe3+ is
E o2 0.50V; G o2 n 2 E 2o F
strongest oxidizing agent. More the positive
value of reduction potential, more is the
Cu 2 2e Cu tendency to get reduced (better oxidising agent).
Û´»½¬®±½¸»³·­¬®§ ïëé

o 0.0591 46. (b) 2KBr I 2 2KI Br2


40. (c) As E cell log K c
n reaction is not possible because Br – ion is not
0.0591 oxidised in Br2 with I2 due to higher electrode
0.46 log K c (oxidation) potential of I2 than bromine.
2
2 0.46
47. (c) 2 Cu Cu 2 Cu
log K c 15.57
0.0591 2e Cu 2 Cu ; E1º 0.34V; ...(i)
or Kc = Antilog 15.57 = 3.7 × 1015 4 × 1015
41. (b) e Cu 2 Cu ; E º2 0.15V; ...(ii)

Fe Fe 2 2e – [Anode] E –0.441V Cu e Cu; E3º ? ... (iii)

[Fe 3 e– Fe 2 ] 2 [Cathode] E +0.771V Now, G1º nFE1º 2 0.34F

Fe 2Fe 3 3Fe 2 G º2 1 0.15F , G 3º 1 E 3º F

E ocell o
Ecathode o
EAnode Again, G1º G º2 G 3º
= .771 – (–.441) = .771 + .441 = 1.212 V
0.68F 0.15F E3º F
42. (c) The cell reacton is as follows :
+ 0 E3º 0.68 0.15 0.53V
A A + e–
+
B+e B E ºcell E cathode
º
(Cu / Cu)
+ º
+
Adding A + B A + B E anode (Cu 2 / Cu )
= 0.53 – 0.15 = 0.38 V.
0.0591
43. (d) E log10 K 2.303 RT .0591
n 48. (c) E cell log K eq log K eq
nF n
Here, n 2, E 0.295
.0591
2 0.295 = log 10 6 = .0591 × 3 = 0.1773V
log 10 K = 10 or K 1 1010 2
0.0591
49. (b) G nE F
44. (b) Cell reaction is, Zn Cu 2 Zn 2 Cu
RT C 2
RT [ Zn 2 ] For concentration cell, E ln
E cell E º cell In nF C1
nF [Cu 2 ] In it R, T, n and F are constant
So E is based upon ln C2 / C1
( Zn 2 )
Greater the factor , less is the EMF RT
(Cu 2 ) Now G nEF = nF ln C2 / C1
nF
Hence E1 > E2 = –RTlnC2/C1
45. (b) G = –nEF At constant temperature G is based upon
For 1 mol of Al, n = 3 lnC2/C1.
4 4 50. (b) Without losing its concentration ZnCl 2
for mol of Al, n 3 4 solution cannot be kept in contact with Al
3 3
because Al is more reactive than Zn due to
According to question,
electrode (reduction) potential.
827 1000 4 E 96500 51. (d) Given for the reaction
827 l1000 Zn s Cu 2aq Cu s Zn 2aq , ,
Eã = 2.14V
4 l 96500
EBD_7324
ïëè ÝØÛÓ×ÍÌÎÇ
Eº = +1.10 V. C 5 H12 10H 2 O 5CO 2 32H 32e
At equilibrium (pentane)

0.0591 At Cathode:
E cell log10 K eq
n
8O 2 32H 32e 16H 2 O
here (n number of exchange of electrons)
Over all :C5 H12 8O 2 5CO 2 6H 2 O
0.0591 2.20
or 1.10 log10 K eq log10 K eq Calculation of G° for the above reaction
2 0.059
= 37.22 or Keq = 1.66 × 1037 G° = [5×(–394.4) + 6× (–237.2)] – [–8.2]
52. (b) In electrochemical series, Sn is above = – 1972.0 – 1423.2 + 8.2 = – 3387.0 kJ
hydrogen and Hg is below hydrogen. Elements = – 3387000 Joules.
above hyrogen are reducing in nature while From the overall equation we find n = 32
below hydrogen are oxidising in nature.
53. (a) The relation between free energy change Usin g the relation, G° = – nFE0cell and
and equilibrium constant is given by Nernst substituting various values, we get
equation
– 3387000 = –32×96500× E 0cell (F = 96500C)
RT
E cell Eº ln Q 3387000 3387000
nF or E 0cell =
32 96500 3088000
At equilibrium, E cell = 0 and Q = KC
3387
RT or V = 1.0968 V
Eº ln K … (i) 3088
nF
G
Again Gº = nFEº … (ii) 58. (b) Efficiency of a fuel cell ( )
put in (i) H
59. (d) In the silver plating of copper, K[Ag(CN) 2 ]
G º RT
lnK ; G º RT lnK is used instead of AgNO3. Copper being more
nF nF
electropositive readily precipitate silver from their
54. (d) For a concentration cell having different
salt solution
concentrations of ions.
0.0591 C Cu õ 2AgNO3 bb r Cu(NO 3 ) 2 õ Ag
E log 1
n C2 whereas in K[Ag (CN)2] solution a complex anion
[Ag(CN)2]– is formed and hence Ag + are less
If all the concentrations are identical then
available in the solution and therefore copper
obviously the cell voltage is zero. But as the pH
cannot displace Ag from its complex ion.
of 0.1 M HCl (strong acid) & pH of 0.1M
60. (c) For bottom of ship to be protected it is
CH3COOH(weak acid) is not same, therefore the
cell voltage will not be zero. connected with more reactive metal than iron like
55. (d) Higher the reduction potential, stronger is magnesium. This technique is called cathodic
the oxidising agent. protection.
61. (d) This is because zinc has higher oxidation
56. (d) E°Cell = E°OP + E°RP
potential than Ni, Cu and Sn. The process of
= 0.76 + 0.314 = 1.10 V coating of iron surface with zinc is known as
57. (c) Writing the equation for pentane-oxygen galvanization. Galvanized iron sheets maintain
fuel cell at respective electrodes and over all their lustrue due to the formation of protective
reaction, we get layer of basic zinc carbonate.
At Anode:
ݸ¿°¬»®

18 Chemical Kinetics

̱°·½ ïæ כּ ±º λ¿½¬·±²ô כּ Ô¿©­ ¿²¼ (c) 6.25 × 10–3 mol L–1s–1 and 3.125 × 10–3
כּ ݱ²­¬¿²¬ mol L–1s–1
(d) 1.25 × 10–2 mol L–1s–1 and 6.25 × 10–3
1. The rate constant of the reaction A B is 0.6 × 10–3 mol L–1s–1
mole per second. If the concentration of A is 5 5. The rate of the reaction
M then concentration of B after 20 minutes is : 2NO Cl2 2NOCl is given by the rate
[2015 RS] equation rate = k [NO]2 [Cl2] [2010]
(a) 1.08 M (b) 3.60 M The value of the rate constant can be increased by:
(c) 0.36 M (d) 0.72 M (a) increasing the concentration of NO.
2. In a reaction, A + B Product, rate is doubled (b) increasing the temperature.
when the concentration of B is doubled, and (c) increasing the concentration of the Cl 2
rate increases by a factor of 8 when the (d) doing all of these
concentrations of both the reactants (A and B) 6. For the reaction, N2 + 3H2— 2NH3, [2009]
are doubled rate, law for the reaction can be
written as : [2012] d NH3
= 2 × 10–4 mol L–1 s–1 , the value of
(a) Rate = k[A][B]2 (b) Rate = k[A]2 [B]2 dt
(c) Rate = k[A] [B] (d) Rate = k[A]2 B] – d H2
would be :
3. The rate of the reaction 2N2O5 4NO2 + O2 can dt
be written in three ways : [2011 M] (a) 4 × 10–4 mol L–1 s–1
d[N 2O5 ] (b) 6 × 10–4 mol L–1 s–1
k [N 2O5 ]
dt (c) 1 × 10–4 mol L–1 s–1
d[NO 2 ] (d) 3 × 10–4 mol L–1 s–1
k [N 2O5 ]
dt 7. In the reaction [2009]
d[O 2 ]
k [N 2O 5 ] BrO3– (aq) + 5Br(aq)

+ 6H + 3Br2(l) + 3H 2O(l)
dt
The relationship between k and k' and between The rate of appearance of bromine (Br 2) is related
k and k are : to rate of disappearance of bromide ions as
(a) k = 2k ; k = k (b) k = 2k ; k = k / 2
following:
(c) k = 2k ; k = 2k (d) k = k ; k = k
4. For the reaction [N 2 O 5(g) 2NO 2(g) +
d[Br2 ] 5 d [Br – ]
(a) –
1/2 O2(g)] the value of rate of disappearance of dt 3 dt
N2O5 is given as 6.25 × 10–3 mol L–1s–1. The rate d[Br2 ] 5 d[Br – ]
(b)
of formation of NO2 and O2 is given respectively dt 3 dt
as : [2010] d[Br2 ] 3 d[Br – ]
(c)
(a) 6.25 × 10–3 mol L–1s–1 and 6.25 × 10–3 dt 5 dt
mol L–1s–1 d [Br2 ] 3 d[Br – ]
(b) 1.25 × 10–2 mol L–1s–1 and 3.125 × 10–3 (d)
dt 5 dt
mol L–1s–1
EBD_7324
ïêð ÝØÛÓ×ÍÌÎÇ
8. For the reaction 2 A + B 3C + D 13. The rate of reaction depends upon the [1995]
which of the following does not express the (a) volume (b) force
reaction rate ? [2006] (c) pressure (d) conc. of reactants
d[ B] d[ D]
(a) (b) ̱°·½ îæ Ñ®¼»® ±º λ¿½¬·±² ¿²¼ Ø¿´º Ô·º»
dt dt Ð »® ·± ¼
1 d[A] 1 d[C]
(c) – (d) ó 14. Mechanism of a hypothetical reaction [2017]
2 dt 3 dt
9. Consider the reaction [2006] X2 + Y2 2XY is given below :
N2 (g) + 3H2 (g) 2 NH3 (g) (i) X2 X + X(fast)
d[ NH 3 ] (ii) X + Y2 XY + Y (slow)
The equality relationship between and (iii) X + Y XY (fast)
dt
d[H 2 ] The overall order of the reaction will be :
is (a) 2 (b) 0
dt
d[ NH 3 ] (c) 1.5 (d) 1
2 d[ H 2 ]
(a) 15. A first order reaction has a specific reaction rate
dt 3 dt of 10–2 sec–1. How much time will it take for 20g
d[ NH 3 ] 3 d[ H 2 ] of the reactant to reduce to 5 g ? [2017]
(b)
dt 2 dt (a) 138.6 sec (b) 346.5 sec
d[ NH 3 ] d[ H 2 ] (c) 693.0 sec (d) 238.6 sec
(c) 16. The rate of a first -order reaction is 0.04
dt dt
d[ NH 3 ] 1 d[ H 2 ] mol –1s–1 at 10 seconds and 0.03 mol –1s–1 at
(d) 20 seconds after initiation of the reaction. The
dt 3 dt half-life period of the reaction is [2016]
d[B]
10. 3A 2B , rate of reaction is equal to (a) 24.1 s (b) 34.1 s
dt [2002] (c) 44.1 s (d) 54.1 s
2 d[A]
(a) ó 3 d[A] (b) ó 17. When initial concentration of a reactant is
2 dt 3 dt doubled in a reaction, its half-life period is not
1 d[A] d[A]
(c) ó (d) õ2 affected. The order of the reaction is : [2015]
3 dt dt (a) First
11. For the reaction 2 N 2O 5 4NO 2 O 2 , rate (b) Second
and rate constant are 1.02 × 10–4 mol lit–1 sec–1 (c) More than zero but less than first
an d 3.4×10 –5 sec –1 r espectively then (d) Zero
concentration of N2O5 at that time will be [2001] 18. A reaction is 50% completed in 2 hours and 75%
(a) 1.732M (b) 3M completed in 4 hours. The order of reaction is
(c) 3.4 × 105M (d) 1.02 × 10–4M [NEET Kar. 2013]
12. In the following reaction, how is the rate of (a) 0 (b) 1
appearance of the underlined product related to (c) 2 (d) 3
the rate of disappearance of the underlined 19. For a reaction between A and B the order with
reactant ? [2000] respect to A is 2 and the order with respect to B
is 3. The concentrations of both A and B are
BrO 3( aq ) 5Br ( aq ) 6 H (aq ) doubled, the rate will increase by a factor of:
3Br2( l) 3H 2 O ( l) [NEET Kar. 2013]
(a) 10 (b) 12
d[Br2 ] 5 d[Br ] (c) 16 (d) 32
(a)
dt 3 dt 20. In a zero-order reaction for every 10° rise of
d[Br2 ] d[Br ] temperature, the rate is doubled. If the
(b)
dt dt temperature is increased from 10°C to 100°C, the
d[Br2 ] 3 d[Br ] rate of the reaction will become : [2012]
(c)
dt 5 dt (a) 256 times (b) 512 times
d[Br2 ] 3 d[Br ] (c) 64 times (d) 128 times
(d)
dt 5 dt
ݸ»³·½¿´ Õ·²»¬·½­ ïêï

21. Which one of the following statements for the 26. The bromination of acetone that occurs in acid
order of a reaction is incorrect ? [2011] solution is represented by this equation. [2008]
(a) Order can be determin ed only CH3COCH3 (aq) + Br2 (aq)
experimentally. CH3COCH2Br (aq) + H+ (aq) + Br– (aq)
(b) Order is not influenced by stoichiometric These kinetic data were obtained for given
coefficient of the reactants. reaction concentrations.
(c) Order of reaction is sum of power to the Initial Concentrations, M
concentration terms of reactants to express
[CH3 COCH3] [Br2] [H+]
the rate of reaction.
(d) Order of reaction is always whole number. 0.30 0.05 0.05
22. The unit of rate constant for a zero order reaction 0.30 0.10 0.05
is [2011 M] 0.30 0.10 0.10
(a) mol L–1 s–1 (b) L mol–1 s–1 0.40 0.05 0.20
(c) L2 mol–2 s–1 (d) s–1 Initial rate, disappearance of Br 2, Ms–1
23. During the kinetic study of the reaction, 5.7×10–5
2A + B C + D, following results were obtained: 5.7 × 10–5
Run [A]/mol L–1 [B]/mol L–1 Initial rate of 1.2 × 10–4
formation of 3.1 × 10–4
D/mol L–1min–1 Base on these data, the rate equations is:
I 0.1 0.1 6.0 × 10–1 (a) Rate = k[CH3COCH3][H+]
II 0.3 0.2 7.2 × 10–1 (b) Rate = k [CH = COCH3][Br2]
III 0.3 0.4 2.88 × 10–1 (c) Rate = k [CH3 COCH3] [Br2] [H+]2
IV 0.4 0.1 2.40 × 10–1 (d) Rate = k [CH3COCH3][Br2] [H+]
27. The reaction of hydrogen and iodine
Based on the above data which one of the monochloride is given as: [2007]
following is correct? [2010] H 2(g) 2ICl(g) 2HCl(g) I2(g)
2 The reaction is of first order with respect to H2(g)
(a) rate = k [A] [B] (b) rate = k[A] [B]
and ICI(g), following mechanisms were proposed.
2 2
(c) rate = k [A] [B] (d) rate = k [A] [B]2 Mechanism A:
24. Half life period of a first-order reaction is 1386 H 2(g) 2ICl(g) 2HCl(g) I 2(g)
seconds. The specific rate constant of the Mechanism B:
reaction is: [2009] H 2(g) ICl(g) HI(g) ;slow
(a) 0.5 × 10–2 s–1 (b) 0.5 × 10–3 s–1
HI(g) ICl(g) HCl(g) I2(g) ; fast
(c) 5.0 × 10–2 s–1 (d) 5.0 × 10–3 s–1
Which of the above mechanism(s) can be
25. For the reaction A + B products, it is consistent with the given information about the
observed that: [2009] reaction?
(a) A and B both (b) neither A nor B
(1) On doubling the initial concentration of A
(c) A only (d) B only
only, the rate of reaction is also doubled
28. In a first-order reaction A B, if k is rate
and
constant and inital concentration of the reactant
(2) On doubling the initial concentrations of A is 0.5 M, then the half-life is [2007]
both A and B, there is a change by a factor log 2
log 2
of 8 in the rate of the reaction. (a) (b)
k k 0.5
The rate of this reaction is given by:
ln 2 0.693
(a) rate = k [A] [B]2 (b) rate = k [A]2 [B]2 (c) (d)
k 0.5k
(c) rate = k [A] [B] (d) rate = k [A]2 [B]
EBD_7324
ïêî ÝØÛÓ×ÍÌÎÇ
29. If 60% of a first order reaction was completed in (a) 0.0075 M (b) 0.06 M
60 minutes, 50% of the same reaction would be (c) 0.03 M (d) 0.015 M
completed in aproximately [2007] 37. The plot of concentration of the reactant vs.
time for a reaction is a straight line with a negative
(a) 45 minutes (b) 60 minutes
slope. The reaction follows a [1996]
(c) 40 minutes (d) 50 minutes (a) zero order rate equation
(log 4 = 0.60, log 5 = 0.69) (b) first order rate equation
30. For a first order reaction A B the reaction (c) second order rate equation
rate at reactant concentration of 0.01 M is (d) third order rate equation
found to be 2.0 10 5 mol L 1 s 1. The half 38. A substance 'A' decomposes by a first order
life period of the reaction is [2005] reaction starting initially with [A] = 2.00 m and
(a) 30 s (b) 220 s after 200 min, [A] becomes 0.15 m. For this
(c) 300 s (d) 347 s reaction t1/2 is [1995]
31. The rate of reaction between two reactants A (a) 53.72 min (b) 50.49 min
an d B decreases by a factor of 4 if the (c) 48.45 min (d) 46.45 min
concentration of reactant B is doubled. The 39. Select the rate law that corresponds to data
order of this reaction with respect to reactant shown for the following reaction [1994]
B is: [2005] A+B products.
(a) 2 (b) 2 Exp. [A] [B] Initial rate
(c) 1 (d) 1 1 0.012 0.035 0.1
32. The rate of a first order reaction is 1.5 × 10–2 2 0.024 0.070 0.8
mol L–1 min–1 at 0.5 M concentration of the
3 0.024 0.035 0.1
reactant. The half life of the reaction is [2004]
4 0.012 0.070 0.8
(a) 0.383 min (b) 23.1 min
(a) rate = k [B]3
(c) 8.73 min (d) 7.53 min
(b) rate = k [B]4
33. If the rate of the reaction is equal to the rate
(c) rate = k [A] [B]3
constant, the order of the reaction is [2003]
(a) 3 (b) 0 (d) rate = k [A]2 [B]2
(c) 1 (d) 2 ̱°·½ íæ ̸»±®·»­ ±º כּ ±º λ¿½¬·±²
34. The reaction A B follows first order kinetics.
40. The addition of a catalyst during a chemical
The time taken for 0.8 mole of A to produce 0.6
reaction alters which of the following quantities?
mole of B is 1 hour. What is the time taken for
[2016]
conversion of 0.9 mole of A to produce 0.675
(a) Entropy (b) Internal energy
mole of B? [2003]
(c) Enthalpy (d) Activation energy
(a) 2 hours (b) 1 hour
41. The activation energy of a reaction can be
(c) 0.5 hour (d) 0.25 hour
determined from the slope of which of the
35. 3A B + C, it would be a zero order reaction
following graphs ? [2015]
when [2002]
(a) the rate of reaction is proportional to ln K l
(a) vs.T (b) ln K vs.
square of concentration of A T T
(b) the rate of reaction remains same at any T l
concentration of A (c) ln K vs. T (d) ln K vs . T
(c) the rate remains unchanged at an y
concentration of B and C 42. What is the activation energy for a reaction if its
(d) the rate of reaction doubles if concentration rate doubles when the temperature is raised from
of B is increased to double 20°C to 35°C? (R = 8.314 J mol–1 K–1)
36. Half life of a first order reaction is 4 s and the [NEET 2013]
initial concentration of the reactants is 0.12 M. (a) 269 kJ mol–1 (b) 34.7 kJ mol–1
The concentration of the reactant left after 16 s (c) 15.1 kJ mol–1 (d) 342 kJ mol–1
is [1999]
ݸ»³·½¿´ Õ·²»¬·½­ ïêí

43. A reaction having equal energies of activation (a) Is always double of Ea


for forward and reverse reaction has : (b) Is negative of E a
[NEET 2013] (c) Is always less than Ea
(a) G= 0 (b) H = 0 (d) Can be less than or more than E a
49. When a biochemical reaction is carried out in
(c) H = G = S = 0 (d) S = 0
laboratory in the absence of enzyme then rate of
44. Activation energy (E a ) and rate constants reaction obtained is 10–6 times, then activation
(k1 and k2) of a chemical reaction at two different energy of reaction in the Presence of enzyme is
temperatures (T1 and T2) are related by :
6
[2012 M] (a) [2001]
RT
k Ea 1 1 (b) Different from Ea obtained in laboratory
(a) ln 2
k1 R T1 T2 (c) P is required
k Ea 1 1 (d) Can't say anything
(b) ln 2 50. Activation energy of a chemical reaction can be
k1 R T2 T1
determined by [1998]
k Ea 1 1 (a) evaluating rate constant at standard
(c) ln 2
k1 R T2 T1 temperature
k Ea 1 1 (b) evaluating velocities of reaction at two
(d) ln 2 different temperatures
k1 R T1 T 2
(c) evaluating rate constants at two different
45. For an endothermic reaction, energy of temperatures
activation is Ea and enthalpy of reaction of
(d) changing concentration of reactants
H (both of these in kJ/mol). Minimum value 51. In a reversible reaction the energy of activation
of Ea will be. [2010] of the forward reaction is 50 kcal. The energy of
(a) less than H (b) equal to H activation for the reverse reaction will be
(c) more than H (d) equal to zero (a) < 50 kcal [1996]
46. The rate constants k1 and k2 for two different (b) either greater than or less than 50 kcal
(c) 50 kcal
reactions are 1016 . e–2000/T and 1015 . e–1000/T,
(d) > 50 kcal
respectively. The temperature at which k1 = k2 is : 52. A chemical reaction is catalyzed by a catalyst X.
2000 [2008] Hence X [1995]
(a) 1000 K (b) K
2.303 (a) reduces enthalpy of the reaction
(b) decreases rate constant of the reaction
1000
(c) 2000 K (d) K (c) increases activation energy of the reaction
2.303 (d) does not affect equilibrium constant of the
47. The temperature dependence of rate constant reaction
(k) of a chemical reaction is written in terms of 53. For an exothermic reaction, the energy of
óE*a / RT activation of the reactants is [1994]
Arrhenius equation, k = Ae . Activation
(a) equal to the energy of activation of products
energy (E*a ) of the reaction can be calculated
(b) less than the energy of activation of
by plotting [2003]
products
1
(a) log k vs (b) k vs T (c) greater than the energy of activation of
log T products
1 1 (d) Sometimes greater and sometimes less than
(c) k vs (d) log k vs
log T T that of the products
48. The activation energy for a simple chemical
reaction A B is Ea in forward direction. The
activation energy for reverse reaction [2003]
EBD_7324
ïêì ÝØÛÓ×ÍÌÎÇ

ANSWER KEY
1 (d) 7 (d) 13 (d) 19 (d) 25 (a) 31 (b) 37 (b) 43 (b) 49 (b)
2 (d) 8 (d) 14 (c) 20 (b) 26 (a) 32 (b) 38 (a) 44 (b,d) 50 (c)
3 (b) 9 (a) 15 (a) 21 (d) 27 (d) 33 (b) 39 (a) 45 (c) 51 (b)
4 (b) 10 (b) 16 (a) 22 (a) 28 (c) 34 (b) 40 (d) 46 (d) 52 (d)
5 (b) 11 (b) 17 (a) 23 (d) 29 (a) 35 (b) 41 (b) 47 (d) 53 (b)
6 (d) 12 (d) 18 (b) 24 (b) 30 (d) 36 (a) 42 (b) 48 (d)

Hints & Solutions


1. (d) Rate constant k = 0.6 × 10–3 mole per second. 5. (b) 2 NO (g) + Cl2(g) 2 NOCl(g)
(unit mole per second shows zero order reaction) Rate = k [NO]2 [Cl2]
For a zero order reaction The value of rate constant can be increased by
[A] = [A]0 – kt increasing the temperature and is independent
and [A0] – [A] = [B] = kt of the initial concerntration of the reactants.
= 0.6 × 10–3 × 20 × 60 = 0.72 M 6. (d) Rate of disappearance of H 2 = rate of
2. (d) According to statement given in the formation of NH3.
question, it is clear that 1 d[H 2 ] 1 d[NH3 ]
r [A]2 and r [B] – =
3 dt 2 dt
That means order of reaction with respect to B
is 1 and w.r.t A is 2 – d[H 2 ] 3 d[NH3 ] 3 –4
= = × 2×10
Hence, Rate = k[A]2[B]1 dt 2 dt 2
3. (b) Rate of disappearance of reactants = Rate = 3×10 –4 mol L–1s –1
of appearance of products
7. (d) Rate of disappearance of Br –
1 d(N 2 O5 ) 1 d(NO 2 ) d(O 2 ) = rate of appearance of Br 2
2 dt 4 dt dt 1 d[Br – ] 1 d[Br2 ]

1 1 5 dt 3 dt
k(N 2 O5 ) k (N 2 O5 ) k (N 2 O5 )
2 4 d[Br2 ] 3 d[Br – ]
k k dt 5 dt
k
2 4 d[C]
k 8. (d) In the given options will not
k = 2k, k 3dt
2 represent the reaction rate. It should not have –
4. (b) N 2O5 (g) 2 NO 2 (g) + 1/2 O2 (g)
1 dC
ve sign as it is product since show the
d 1d d 3 dt
– N2 O5 NO2 2 O2
dt 2 dt dt rate of formation of product C, which will be
positive.
d 2 9. (a) If we write rate of reaction in terms of
NO2 1.25 10 mol L–1s–1 and
dt concentration of NH3 and H2,then
d 3 1 d[ NH 3 ] 1 d[ H 2 ]
O2 3.125 10 mol L–1s–1 Rate of reaction
dt 2 dt 3 dt
ݸ»³·½¿´ Õ·²»¬·½­ ïêë
15. (a) Half life for a first order reaction,
d[ NH 3 ] 2 d[ H 2 ]
So, 0.693
dt 3 dt
t1/2 =
10. (b) 3A 2B K
Rate of appearance of B is equal to rate of 0.693
disappearance of A. So, t1/2 = sec .
10 2
1 d[B] 1 d[A] d[B] 2 d[A]
ãó ãó Also, for the reduction of 20 g or reactant to 5 g,
2 dt 3 dt dt 3 dt
two half lives will be required.
11. (b) 2 N 2O 5 4 NO 2 O 2 For 20 g of the reactant to reduce to 5g, time
From the unit of rate constant it is clear that the taken,
reaction follow first order kinetics. Hence
0.693
by rate law equation, r = k [N2O5] t=2× sec = 138.6 sec.
where r = 1.02 × 10–4, k = 3.4 × 10–5 10 2
1.02 × 10–4 = 3.4 × 10–5 [N2O5] 16. (a) For a first order reaction
[N2O5] = 3M 2.303 a x1
12. (d) Rate of reaction K = t t log a x
2 1 2
1 d[Br ] 1 d[Br2 ]
– 2.303 0.04
5 dt 3 dt K= log
20 10 0.03
d[Br2 ] 3 d[Br – ]
– 2.303 0.1249
dt 5 dt K=
13. (d) The rate of a reaction is the speed at which 10
the reactants are converted into products. It 0.6932 2.303 0.1249
depends upon the concentration of reactants.
t1/2 = 10
e.g for the reaction
A õ B bb r Product ; r k[A][B] 0.6932 10
t1/2 = = 24.1 sec
14. (c) The overall reaction rate depends on the 2.303 0.1249
rate of the slowest step.
0.693
i.e., Overall rate = Rate of slowest step (ii) 17. (a) t1/ 2
= k[X][Y2] … (1) k
k = rate constant For first order t1/2 is independent of initial
Assuming step (i) to be reversible, its equilibrium concentration of reactant.
constant, 18. (b) For a first order reaction,
2 t75% = 2 × t50%
X 2
k eq X k eq X 2 ; 19. (d) Rate1 = k[A]2[B]3
X2
when concentrations of both A and B are
1 1
doubled then
X 2
k eq X2 2 … (2) Rate2 = k[2A]2[2B]3 = 32 k[A]2[B]3
rate will increase by a factor of 32.
From eq (1) and (2) 100 10
r
100 C
1 1 20. (b) 2 10
29 512 times
Rate = kk 2 r
eq X2 2 Y2 10 C
21. (d) order of reaction can be zero, whole number
1 3 or fractional.
Overall order = 1 1.5 22. (a) Rate = K[A]0
2 2
Unit of k = mol L–1 sec–1
EBD_7324
ïêê ÝØÛÓ×ÍÌÎÇ
23. (d) In case of (II) and (III) Keepin g and (Br2) remain constant but H+ increases from
concentration of [A] constant, when the 0.05 M to 0.10 i.e. doubled, the rate of reaction
concentration of [B] is doubled, the rate changes from 5.7×10 –5 to 1.2 × 10 –4
quadruples. Hence it is second order with (or 12 × 10–5), thus it also becomes almost
respect to B. In case of I & IV Keeping the doubled. It shows that rate of reaction is directly
concentration of [B] constant, when the proportional to [H+]. From (3) and (4), the rate
concentration of [A] is increased four times, rate should have doubled due to increase in conc of
also increases four times. Hence, the order with [H+] from 0.10 M to 0.20 M but the rate has
respect to A is one. hence changed from 1.2× 10–4 to 3.1×10–4. This is due
to change in concentration of CH3 CO CH3 from
Rate = k [A] [B]2
0.30 M to 0.40 M. Thus the rate is directly
proportional to [CH3 COCH3].
24. (b) For a first order reaction
rate = k [CH3COCH3]1[Br2]0[H+]1
0.693 0.693
t1/2 = ; k= = 0.5 × 10–3s–1 = k [CH3COCH3][H+].
k 1386
25. (a) When concentration of A is doubled, rate is 27. (d) As the slowest step is the rate determining
doubled. Hence order with respect to A is one. step thus the mechanism B will be more
When concentrations of both A and B are consistent with the given information also
doubled, rate increases by 8 times hence order because it involve one molecule of H2 and one
with respect to B is 2. molecule of ICl it can expressed as
rate = k [A]1 [B]2 r = k [H2][ICl]
26. (a) Rewriting the given data for the reaction Which shows that the reaction is of first order
w.r.t. both H2 & ICl.
H
CH 3 COCH3 (aq) Br2 (aq) 28. (c) For a first order reaction

CH 3COCH 2 Br(aq) H (aq) Br (aq) 2.303 a


k log10
t a x
S. Initialconcent Initial concentr Initial concentr Rate of
No. -ration of -ation of Br2 -ation of H disappearance
when t = t½
CH3 COCH 3 in M in M of Br2 in MS 1
2.303 a
in M
i.e.
d
[Br ]or
dx k log10
dt 2 dt t½ a a/2
1 0.30 0.05 0.05 5.7 10 5
2 0.30 0.10 0.05 5.7 10 5 2.303 ln 2
or t ½ log10 2
3 0.30 0.10 0.10 1.2 10 4 k k
4 0.40 0.05 0.20 3.1 10 4
29. (a) For a first order reaction
Actually this reaction is autocatalyzed and
2.303 a
involves complex calculation for concentration k log
t a x
terms.
We can look at the above results in a simple way when t = 60 and x = 60%
to find the dependence of reaction rate (i.e. rate 2.303 100 2.303 100
k log log = 0.0153
of disappearance of Br 2). 60 100 60 60 40
From data (1) and (2) in which concentration of Now,
CH3COCH3 and H+ remain unchanged and only
the concentration of Br 2 is doubled, there is no 2.303 100 2.303
t½ log log 2
change in rate of reaction. It means the rate of 0.0153 100 50 0.0153
reaction is independent of concentration of Br 2.
Again from (2) and (3) in which (CH3CO CH3) 2.303
0.3010 45.31 min .
0.0153
ݸ»³·½¿´ Õ·²»¬·½­ ïêé

30. (d) Given [A] = 0.01 M again a = 0.9, a – x = 0.9 – 0.675 = 0.225
Rate = 2.0 × 10–5 mol L–1 S–1
2.303 0. 9
For a first order reaction k log
t 0.225
Rate = k[A]
2.303
2.0 10 5 2.303 log 4 log 4
k= = 2 × 10–3 t
[0.01]
Hence t = 1 hour
0.693 35. (b) For reaction 3A B C
t1/2 = 3 = 347 sec.
2 10 If it is zero order reaction r = k [A] 0, i.e the rate
31. (b) Rate1= k [A]x[B]y ... (1) remains same at any concentration
of 'A'. i.e independent upon concentration of A.
Rate1
= k [A]x [2B]y ... (2) 36. (a) t1/2 = 4 s T = 16
4
or Rate1 = 4k[A]x[2B]y T 16
n 4 ( T = n × t½)
From (1) and (2) we get t1/ 2 4

k[A]x [B]y 1
n
1
4
0.12
= k[A]x[2B]y A Ao 0.12 0.0075 M
4 2 2 16
where Ao = initial concentration &
[B]y A = concentration left after time t.
= [2B] y
4
dx
37. (b) For a first reaction ã k(a ó x) on
1 2B
y 1 dt
or y –2 y
4 B 4 = 2 or (2) = 2 intergration
y = –2. dx
k dt
32. (b) For a first order reaction, A products (a x)
r i.e – n (a –x) = kt + c or kt = n a – n (a–x)
r k[A] or k
[ A] or kt = 2.303 [log a – log (a – x)]
Thus if we plot a graph between log a & t we get
2
1.5 10
k = 3 × 10–2
0.5
st
1 order
0.693 0.693 log[a]
Further, t1/ 2 2
23.1 min.
k 3 10

33. (b) r k[A]n time t

if n = 0 38. (a) Given initial concentration (a) = 2.00 m; Time


taken (t) = 200 min and final concentration
r k[A]0
(a – x) = 0.15 m. For a first order reaction, rate
or r = k thus for zero order reactions rate is equal constant,
to the rate constant.
2.303 a 2.303 2.00
34. (b) A B For a first order reaction k log log
t a x 200 0.15
given a = 0.8 mol, (a – x) = 0.8 – 0.6 = 0.2
2.303 0.8 2.303 2 1
k log (0.301 0.824) 1.29 10 min .
or k = 2.303 log 4 200
1 0. 2
EBD_7324
ïêè ÝØÛÓ×ÍÌÎÇ
Further
45. (c)
0.693 0.693
( t1 / 2 ) 53.49 min .
k 1.29 10 2
39. (a) From data 1 and 3, it is clear that keeping (B) Ea
const, When [A] is doubled, rate remains
H
unaffected. Hence rate is independent of [A].
from 1 and 4, keeping [A] constant, when [B] is
doubled, rate become 8 times. Hence Reaction coordinate
rate [ B]3 .
40. (d) A catalyst provides an alternative route for Ea > H
the reaction with a lower activation energy. 2000
41. (b) Arrhenius equation 46. (d) Given, k1 1016.e T

Ea 1000
K=A.e E a /RT ln K = ln A – and k2 1015.e T
RT
When k1 and k2 are equal at any temperature T,
–E a we have
slope =
R 2000 1000

so, activation energy of reaction can be 1016.e T 15


= 10 .e T

1 2000 1000
determined from the slope of ln K vs 15
or 10 10 .e T 15
= 10 .e T
T
2000 1000
k Ea 1 1 or 10.e T
=e T
42. (b) log 2 =
k1 2.303R T1 T2
2000 1000
or ln 10
Ea 1 1 T T
log 2 =
2.303 8.314 293 308 2000 1000
or ln 10
T T
Ea 15
0.3 = × 1000
2.303 8.314 293 308 or 2.303 log 10 =
T
0.3 2.303 8.314 293 308
Ea = . or 2.303 ×1×T=1000 [ log 10= 1]
15
1000
= 34673 J mol-1 = 34.7 kJ mol-1 or T K
2.303
43. (b) H = Ea f Ea b = 0
44. (b, d) According to Arrhenius equation 47. (d) k Ae Ea / Rt
k Ea 1 1
ln 2 or log k ã log A ó
Ea
k1 R T1 T2
2.303 RT

k2 E Comparing the above equation with


1 1
ln =– a y = mx + c
k1 R T2 T1
1
y = log k, x
T
ݸ»³·½¿´ Õ·²»¬·½­ ïêç

Thus A plot of log10k vs 1/T should be a straight Here ET is the threshold energy.
line, with slope equal to – Ea/2.303 RT and Ea and Ea is energy of activation of reaction in
1
intercept equal to log A absence and presence of catalyst respectively.

50. (c) We know that the activation energy of


Ea chemical reaction is given by formula =
Slope =
log k 2.303RT
k2 E a 76 T2 ó T1 '&
log ã
k1 2.303R 65 T1T2 &% , where k1 is the rate
constant at temperature T1 and k2 is the rate
1/T constant at temperature T 2 and E a is the
activation energy. Therefore activation energy
Ea of chemical reaction is determined by evaluating
Slope
2.303R rate constant at two different temperatures.
or E a –2.303R Slope 51. (b) H E a(f ) Ea(b)
Thus energy of activation for reverse reaction
48. (d) The activation energy of reverse reaction depend upon whether reaction is exothermic or
will depend upon whether the forward reaction endothermic
is exothermic or endothermic. If reaction is exothermic,
As H = Ea (forward reaction) – Ea(backward reaction)
H ve E a(b) E a(f )
For exothermic reaction
H = –ve If reaction is endothermic
– H = Ea(f) – Ea(b) H ve E a(b) E a(f )
or Ea(f) = Ea(b) – H 52. (d) A catalyst affects equally both forward and
Ea(f) < Ea(b) backward reactions, therefore it does not affect
for endothermic reaction equilibrium constant of reaction.
H = + ve 53. (b) Ea(Forward) + H = Ea(backword)
H = Ea(f) + Ea(b) or Ea(f) = H + Ea(b)
For Exothermic reaction, H = –ve and
Ea(f) > Ea(b).
activation energy of reactant is less than the
49. (b) The presence of enzyme (catalyst) increases
energy of activation of products.
the speed of reaction by lowering the energy
barrier, i.e. a new path is followed with lower
activation energy. Threshold energy

ET
E'T
Ea(FR) Ea’ (B.R.)
E
Ea A
Ea Products
1 H B

Reactants + catalyst
Progress of Reaction
Progress of reaction
EBD_7324
ïéð ÝØÛÓ×ÍÌÎÇ

ݸ¿°¬»®

19 Surface Chemistry

̱°·½ ïæ ß¼­±®°¬·±² 5. The Langmuir adsorption isotherm is deduced


using the assumption: [2007]
1. Which one of the following characteristics is
(a) the adsorption sites are equivalent in their
associated with adsorption ? [2016] ability to adsorb the particles
(a) G is negative but H and S are positive
(b) the heat of adsorption varies with coverage
(b) G, H and S all are negative
(c) the adsorbed molecules interact with each other
(c) G and H are negative but S is positive
(d) the adsorption takes place in multilayers.
(d) G and S are negative but H is positive
6. For adsorption of a gas on a solid, the plot of
2. Which of the following statements is correct for
log x/m vs log P is linear with slope equal to
the spontaneous adsorption of a gas ? [2014] (n being whole number) [1994, 2006]
(a) S is negative and, therefore, H should (a) k (b) log k
be highly positive
(b) S is negative and therefore, H should 1
(c) n (d)
be highly negative n
(c) S is positive and, therefore, H should 7. Accor ding to the adsorption theor y of
be negative catalysis, the speed of the reaction increases
(d) S is positive and, therefore, H should because [2003]
also be highly positive (a) adsorption lowers the activation energy of
3. In Freundlich Adsorption isotherm, the value of the reaction
1/n is [2012] (b) the concentration of reactant molecules at
(a) between 0 and 1 in all cases the active centres of the catalyst becomes
high due to strong adsorption
(b) between 2 and 4 in all cases
(c) in the process of adsorption, the activation
(c) 1 in case of physical adsorption energy of the molecules becomes large
(d) 1 in case of chemisorption (d) adsorption produces heat which increases
4. If x is amount of adsorbate and m is amount of the speed of the reaction
adsorbent, which of the following relations is 8. Which is not correct regarding the adsorption
not related to adsorption process ? [2011] of a gas on surface of solid? [2001]
(a) x / m = f (p) at constant T. (a) On increasing temperature, adsorption
increases continuously
(b) x / m = f (T) at constant p.
(b) Enthalpy and entropy changes are –ve
(c) p = f (T) at constant (x / m). (c) Adsorption is more for some specific
x substances
(d) p T
m (d) This phenomenon is reversible
Í«®º¿½» ݸ»³·­¬®§ ïéï

̱°·½ îæ Ý¿¬¿´§­·­ ¿²¼ ̸»±®·»­ ±º Ý¿¬¿´§­·­ 16. Position of non-polar and polar part in micelle
is [2002]
9. Which one of the following statements is not
(a) polar at outer surface and non-polar at inner
correct? [2017]
surface
(a) The value of equilibrium constant is
(b) polar at inner surface and non-polar at outer
changed in the presence of a catalyst in
surface
the reaction at equilibrium
(b) Enzymes catalyse mainly bio-chemical (c) distributed all over the surface
reactions (d) present in the surface only
(c) Coenzymes increase the catalytic activity 17. Which is used for ending charge on colloidal
of enzyme solution? [2000]
(d) Catalyst does not initiate any reaction (a) Electrons
(b) Electrolytes
̱°·½ íæ ݱ´´±·¼­ ¿²¼ Û³«´­·±²­
(c) Positively charged ions
10. Fog is colloidal solution of [2016] (d) Compounds
(a) Liquid in gas (b) Gas in liquid 18. Hardy-Schulze rule explains the effect of
(c) Solid in gas (d) Gas in gas electrolytes on the coagulation of colloidal
11. Which proper ty of colloidal soluti on is solution. According to this rule, coagulation
independent of charge on the colloidal particles:- power of cations follow the order [1999]
[2015] (a) Ba+2 > Na+ > Al+3
(a) Electrophoresis (b) Electro-osmosis (b) Al+3 > Na+ > Ba+2
(c) Tyndall effect (d) Coagulation (c) Al+3 > Ba+2 > Na+
12. Which propeherty of colloids is not dependent (d) Ba+2 > Al+3 > Na+
on the charge on colloidal particles ? 19. At the Critical Micelle Concentration (CMC) the
[2014] surfactant molecules [1998]
(a) Coagulation (b) Electrophoresis (a) decompose
(c) Electro - osmosis (d) Tyndall effect (b) dissociate
13. The protecting power of lyophilic colloidal sol (c) associate
is expressed in terms of [2012] (d) become completely soluble
(a) Coagulation value 20. The ability of an ion to bring about coagulation
(b) Gold number of a given colloid depends upon [1997]
(a) its size
(c) Critical miscelle concentration
(b) the magnitude of its charge
(d) Oxidation number
(c) the sign of the charge alone
14. Which one of the following forms micelles in (d) both magnitude and sign of its charge
aqueous solution above certain concentration?
21. During dialysis [1996]
[2005]
(a) only solvent molecules can diffuse
(a) Dodecyl trimethyl ammonium chloride
(b) solvent molecules, ions and colloidal
(b) Glucose
particles can diffuse
(c) Urea
(c) all kinds of particles can diffuse through
(d) Pyridinium chloride the semi-permeable membrane
15. Which of the following forms cationic micelles (d) solvent molecules and ions can diffuse
above certain concentration? [2004]
(a) Sodium dodecyl sulphate
(b) Sodium acetate
(c) Urea
(d) Cetyl trimethyl ammonium bromide
EBD_7324
ïéî ÝØÛÓ×ÍÌÎÇ

ANSWER KEY
1 (b) 4 (d) 7 (a) 10 (a) 13 (b) 16 (a) 19 (c)
2 (b) 5 (a) 8 (a) 11 (c) 14 (a) 17 (b) 20 (d)
3 (a) 6 (d) 9 (a) 12 (d) 15 (d) 18 (c) 21 (d)

Hints & Solutions


1. (b) Adsorption is sponta neous process, 6. (d) According to Freundli ch adsorption
therefore change in the free energy ( G) for the isotherm.
process is negative. At intermediate pressure, extent of adsorption
According to Gibbs's Helmholtz eqn. x x 1
G= H– T S ã kP1/ n or log ã log k õ log P;
m m n
S is negative because adhering of gas molecules
to the surface lowers the randomness.
G can be –ve only when H is –ve. x/m
2. (b) For adsorption S < 0 and for a Ps
spontaneous change G = – ve
P
hence H should be highly negative which is
x
clear from the equation plot of log vs log P is linear with slope
m
G= H – T S
= – H – T(– S) = – H + T S 1
=
So if H is highly negative G will also n
be (– ve) 7. (a) A catalyst lowers the activation energy of
the reaction.
3. (a) According to Freundlich Adsorption
isotherm 1
1
Rate of reaction activation energy
x
KP n 8. (a) On increasing temperature adsorption of a
m
1 gas on surface of solid decreases. Solid adsorb
at low pressure = 1 greater amount of substances at lower
n
x 1
temperature.
P
m 9. (a) A catalyst speeds up both forward and
1 backward reaction with the same rate.
at high pressure = 0
n So, equilibrium constant is not affected by the
x
P presence of a catalyst at any given temperature.
m
i.e. the value of n varies between 0 to 1 10. (a) Fog is a colloidal system having dispersed
4. (d) phase as liquid and dispersion medium as gas.
5. (a) Langmuir adsorption isotherm is based on 11. (c) Tyndall effect is an optical property, and it
the assumption that every adsorption site is is independent of charge on colloidal particles.
equivalent and the ability of a particle to bind
12. (d) Tyndall effect is optical property, whereas
there is independent of whether or not nearby
other properties are electrical properties. Hence
sites are occupied.
dependent on the charge on colloids.
Í«®º¿½» ݸ»³·­¬®§ ïéí

13. (b) The lyophobic sols are less stable than 19. (c) The critical micelle concentration is the
lyophilic sols. The lyophilic sols are thus used lowest concentration at which micelle formation
to protect the lyophobic sols. This property of appears. When surfectants are present above
lyophilic sols is known as protective action of that CMC, they can act as emulsifiers that will
lyophilic sols. solubilise a compound which is normally
Which can be represented by gold number. insoluble in the solvent being used.
14. (a) Micelle formation is shown by surfactants 20. (d) According to the Hardy schulze rule the
detergents (Dodecyl trimethyl ammonium coagulating effect of an ion on dispersed phase
chloride) in their aqueous solutions. of opposite charge increases with the valency
15. (d) Cetyl trimethyl ammonium bromide, of the ion. Therefore more the charge on
oppositely charged ion higher is the coagulation
[C16 H 33 (CH 3 )3 N Br ] is a cationic micelle
value
21. (d) Dialysis is a process of removing a dissolved
substance from a colloidal solution by means of
16. (a) o- Polarhead diffusion through suitable membrane. Colloidal
n- Non-polar tail particles cannot pass through animal membrane.
(micelle)
Only solvent molecules and ions (in case of
electrodialysis) can diffuse.
17. (b) Electrolyte is used for ending charge on
colloidal particles.
18. (c) According to this law the coagulating effect
of an ion on dispersed phase of opposite charge
increases with the valency of the ion. The
precipitating power of Al3+ , Ba++, Na+ ions is in
order Al3+ > Ba2+ > Na+.
EBD_7324
ïéì ÝØÛÓ×ÍÌÎÇ

ݸ¿°¬»®

20
General Principles and
Processes of Isolation of
Elements
̱°·½ ïæ ѽ½«®®»²½» ±º Ó»¬¿´­ Column-I Column-II
1. "Metals are usually not found as nitrates in their (A) Cyanide (i) Ultrapure Ge
ores". process
Out of the following two ((i) and (ii)) reasons (B) Froth flotation (ii) Dressing of ZnS
which is/are true for the above observation ? process
[2015] (C) Electrolytic (iii) Extraction of Al
(i) Metal nitrates are highly unstable. reduction
(ii) Metal nitrates are highly soluble in water. (D) Zone refining (iv) Extraction of Au
(a) (i) and (ii) are false (v) Purification of Ni
(b) (i) is false but (ii) is true Code :
(c) (i) is true but (ii) is false (1) (2) (3) (4)
(d) (i) and (ii) are true (a) (iv) (ii) (iii) (i)
2. Which one of the following is a mineral of iron? (b) (ii) (iii) (i) (v)
[2012] (c) (i) (ii) (iii) (iv)
(a) Malachite (b) Cassiterite (d) (iii) (iv) (v) (i)
(c) Pyrolusite (d) Magnetite 6. In the extraction of copper from its sulphide ore,
3. Identify the alloy containing a non-metal as a the metal finally obtained by the reduction of
constituent in it. [2012] cuprous oxide with : [2015 RS]
(a) Invar (b) Steel (a) iron (II) sulphide (b) carbon monoxide
(c) Bell metal (d) Bronze (c) copper (I) sulphide (d) sulphur dioxide
7. In the extraction of copper from its sulphide ore,
̱°·½ îæ Ó»¬¿´´«®¹·½¿´ Ю±½»­­»­
the metal is finally obtained by the reduction of
4. Extraction of gold and silver involves leaching cuprous oxide with : [2012]
with CN– ion. Silver is later recovered by (a) Copper (I) sulphide (Cu2S)
[2017] (b) Sulphur dioxide (SO2)
(a) distillation (c) Iron sulphide (FeS)
(b) zone refining (d) Carbon monoxide (CO)
(c) displacement with Zn 8. Which of the following elements is present as
(d) liquation the impurity to the maximum extent in the pig iron ?
[2011]
5. Match items of Column I with the items of
(a) Manganese (b) Carbon
Column II and asign the correct code : [2016]
(c) Silicon (d) Phosphorus
Ù»²»®¿´ Ю·²½·°´»­ ¿²¼ Ю±½»­­»­ ±º ×­±´¿¬·±² ±º Û´»³»²¬­ ïéë

9. The following reactions take place in the blast ̱°·½ íæ Ы®·º·½¿¬·±² ¿²¼ Ë­»­ ±º Ó»¬¿´­
furnace in the preparation of impure iron. Identify 12. The metal oxide which cannot be reduced to
the reaction pertaining to the formation of the metal by carbon is [NEET Kar. 2013]
slag. [2011 M]
(a) Fe2O3 (b) Al2O3
(a) Fe2O3(s) + 3 CO(g) 2 Fe (l) + 3 CO2 (g)
(c) PbO (d) ZnO
(b) CaCO3 (s) CaO (s) + CO2 (g)
13. Which of the following pairs of metals is purified
(c) CaO (s) + SiO2(s) CaSiO3 (s) by van Arkel method ? [2011]
(d) 2C(s) + O2 (g) 2 CO(g) (a) Ga and In (b) Zr and Ti
10. Which of the following statements, about the (c) Ag and Au (d) Ni and Fe
advantage of roasting of sulphide ore before
14. The method of zone refining of metals is based
reduction is not true? [2007]
on the principle of [2003]
(a) The G of of the sulphide is greater than (a) Greater solubility of the impurity in the
those for CS2 and H2S. molten state than in the solid
(b) Greater mobility of the pure metal than that
(b) The G of is negative for roasting of of the impurity
sulphide ore to oxide. (c) Higher melting point of the impurity than
(c) Roasting of the sulphide to the oxide is that of the pure metal
thermodynamically feasible. (d) Greater noble character of the solid metal
(d) Carbon and hydrogen are suitable reducing than that of the impurity
agents for metal sulphides. 15. Method used for obtaining highly pure silicon
11. Sulphide ores of metals are usually concentrated used as a semiconductor material, is [1994, 96]
by froth flotation process. Which one of the (a) Oxidation (b) Electrochemical
following sulphide ores offer an exception and (c) Crystallization (d) Zone refining
concentrated by chemical leaching? [2007]
(a) Galena (b) Copper pyrite
(c) Sphalerite (d) Argentite

ANSWER KEY
1 (b) 4 (c) 7 (a) 10 (d) 13 (b)
2 (d) 5 (a) 8 (b) 11 (d) 14 (a)
3 (b) 6 (c) 9 (c) 12 (b) 15 (d)
EBD_7324
ïéê ÝØÛÓ×ÍÌÎÇ

Hints & Solutions


1. (b) Metal nitrates are highly soluble in water give CaO which acts as a flux and combines with
and are very stable for e.g. NaNO3 and KNO3. SiO2 present as impurity (gangue) in the ore to
2. (d) [Fe3O4 Magnetite] form calcium silicate (fusible slag)
[CuCO3 Cu(OH)2 – Malachite] CaO(s) (basic flux) + SiO2 (s) (acidic flux)
Pyrolusite – MnO2 Cassiterite – SnO2. CaSiO3 (s) (slag)
3. (b) Invar is a nickel iron alloy, Bell metal is an
alloy of about 80% copper and 20% tin, Bronze 10. (d) The sulphide ore is roasted to oxide before
is also an alloy of copper and tin. reduction because the G of of most of the
Steel : It always have few % of carbon.
sulphides are greater than those of CS2 and H2S,
4. (c) Zn being more reactive than Ag and Au,
therefore neither C nor H can reduce metal
displaces them.
sulphide to metal. Further, the standard free
4Ag + 8NaCN + 2H2O + O2 Leaching energies of formation of oxide are much less than
4Na[Ag(CN)2] + 4NaOH those of SO2. Hence oxidation of metal sulphides
to metal oxide is thermodynamically favourable.
Soluble Sodium dicyanoargentate (I)
11. (d) Leaching is the selective dissolution of the
Soluble cyanide compound can be treated with desired mineral leaving behind the impurities in
Zn to give metal by displacement.
a suitable dissolving agent, e.g. Argentitie or
2Na[Ag(CN)2] + Zn Displacement Silver glance, Ag 2S is an ore of silver. Silver is
Na2[Zn(CN)4] + 2Ag extracted from argentite by the mac-Arthur and
5. (a) Highly electropositive metals like Al, K, Na Forest process (leaching process).
etc. are extracted by the electrolytic reduction. Ag 2S 4NaCN 2Na[Ag CN 2 ] Na 2S
• zone refining method is used for obtaining
metals of high purity e.g. Ge 4Au 8KCN 2H 2 O O2
• Froth flotation process is suitable for
4K[Au CN 2 ] 4KOH
sulphide ores
• Cyanide process is used for the extraction 12. (b) Al2O3 cannot be reduced by carbon.
of gold. 13. (b) Zr and Ti are purified by van Arkel method.
6. (c) 2 Cu2S (s) + 3O2 (g) 2Cu2O(s) + 2SO2(g)
Zr(s) 2I 2 (g) ZrI 4 (g)
The unchanged Cu 2S, mixed with Cu2O and
heated strongly in absence of air On the hot
ZrI4 (g) Zr(s) 2I 2 (g)
2 Cu2S + 2Cu2O 6Cu + SO2 filament

7. (a) Cuprous oxide formed during roasting of 14. (a) Zone refining is based on the difference in
cuprous sulphide is mixed with few amount of solubility of impurities in molten and solid state
cuprous sulphide and heated in a reverberatory of the metal. This method is used for obtaining
furnace to get metallic copper. metals of very high purity.
15. (d) Si obtained by reduction of SiCl 4 with H2 is
2Cu 2O Cu 2S 6Cu SO2 (g)
further purified by zone refining method to get
8. (b) Pig iron or cast iron contains 3 – 5% carbon Si of very high purity. Silicon is purified by zone-
and varying amounts of Mn, Si, P and S which refining process because the impurities present
makes the iron hard and brittle. in it are more soluble in the liquid phase than in
9. (c) In blast furnace at about 1270 K, calcium the solid phase.
carbonate is almost completely decomposed to
ݸ¿°¬»®

21 The p-Block Elements


(Group 15, 16, 17 and 18)

̱°·½ ïæ Ò·¬®±¹»² Ú¿³·´§ (a) + 3, + 5, + 4 (b) + 5, + 3, + 4


(c) + 5, + 4, + 3 (d) + 3, + 4, + 5
1. Which is the correct statement for the given
7. How many bridging oxygen atoms are present
acids? [2016] in P4O10? [2010]
(a) Phosphinic acid is a diprotic acid while (a) 5 (b) 6
phosphonic acid is a monoprotic acid (c) 4 (d) 2
(b) Phosphinic acid is a monoprotic acid while 8. Nitrogen forms N2, but phosphorus is converted
phosphonic acid is a diprotic acid into P4 from P, the reason is [2001]
(a) Triple bond is present between phosphorus atom
(c) Both are triprotic acids
(b) p – p bonding is strong
(d) Both are diprotic acids (c) p – p bonding is weak
2. The product obtained as a result of a reaction of (d) Multiple bond is formed easily
nitrogen with CaC2 is [2016] 9. Which of the following oxy-acids has the
(a) Ca(CN)2 (b) CaCN maximum number of hydrogens directly attached
(c) CaCN3 (d) Ca2CN to phosphorus? [1999]
3. Strong reducing behaviour of H3PO2 is due to (a) H4P2O7 (b) H3PO2
[2015 RS] (c) H3PO3 (d) H3PO4
(a) presence of one –OH group and two P–H 10. Repeated use of which one of the following
bonds fertilizers would increase the acidity of the soil?
[1998]
(b) high electron gain enthalpy of phosphorus
(a) Urea
(c) high oxidation state of phosphorus (b) Superphosphate of lime
(d) presence of two –OH groups and one P–H bond. (c) Ammonium sulphate
4. In which of the following compounds, nitrogen (d) Potassium nitrate
exhibits highest oxidation state ? [2012] 11. Which of the following species has the highest
(a) N2H4 (b) NH3 dipole moment ? [1997]
(c) N3H (d) NH2OH (a) NH3 (b) PH3
5. Which of the following statements is not valid (c) AsH3 (d) SbH3
12. The structural formula of hypophosphorous
for oxoacids of phosphorus? [2012]
acid is [1997]
(a) Orthophosphoric acid is used in the
manufacture of triple superphosphate. O O
(b) Hypophosphorous acid is a diprotic acid. P P
(c) All oxoacids contain tetrahedral four (a) H (b) H
OH OH
coordinated phosphorus. H OH
(d) All oxoacids contain atleast one P = O and O O
one P — OH group.
P P
6. Oxidation states of P in H4 P2O5 , H4 P2O6 , and (c) HO (d) H
OH OOH
H4 P2O7 , are respectively: [2010] OH OH
EBD_7324
ïéè ÝØÛÓ×ÍÌÎÇ
13. Brown ring test is used to detect [1994] 22. Basicity of orthophosphoric acid is [1991]
(a) Iodine (b) Nitrate (a) 2 (b) 3
(c) Iron (d) Bromide (c) 4 (d) 5
14. Which of the following fertilizers has the highest 23. PCl3 reacts with water to form [1991]
nitrogen percentage ? [1993]
(a) PH3 (b) H3PO3, HCl
(a) Ammonium sulphate
(b) Calcium cyanamide (c) POCl3 (d) H3PO4
(c) Urea 24. PH4I + NaOH forms [1991]
(d) Ammonium nitrate (a) PH3 (b) NH3
15. Which one of the following substance is used (c) P4O6 (d) P4O10
in the laboratory for fast drying of neutral 25. Pure nitrogen is prepared in the laboratory by
gases? [1992] heating a mixture of [1991]
(a) Phosphorus pentoxide (a) NH4OH + NaCl (b) NH4 NO3 + NaCl
(b) Active charcoal (c) NH4 Cl + NaOH (d) NH4 Cl + NaNO2.
(c) Anhydrous calcium chloride 26. Which of the following statements is not correct
(d) Na3PO4. for nitrogen ? [1990]
16. Number of electrons shared in the formation of (a) Its electronegativity is very high
nitrogen molecule is [1992] (b) d-orbitals are available for bonding
(a) 6 (b) 10 (c) It is a typical non-metal
(c) 2 (d) 8 (d) Its molecular size is small
17. Sugarcane on reaction with nitric acid gives 27. Of the following hybrides which one has the
[1992]
lowest boiling point ? [1989]
(a) CO2 and SO2
(b) (COOH)2 (a) AsH 3 (b) SbH 3
(c) 2 HCOOH (two moles) (c) PH 3 (d) NH 3
(d) No reaction.
28. Which of the following metal evolves hydrogen
18. Nitrogen is relatively inactive element
on reacting with cold dilute HNO3 ? [1989]
because [1992]
(a) Its atom has a stable electronic configuration (a) Mg (b) Al
(b) It has low atomic radius (c) Fe (d) Cu.
(c) Its electronegativity is fairly high 29. Which one of the following compounds does
(d) Dissociation energy of its molecule is fairly not exist ? [1989]
high (a) NCl5 (b) AsF5
19. H3PO2 is the molecular formula of an acid of (c) SbCl5 (d) PF5
phosphorus. Its name and basicity respectively 30. Each of the following is true about white and
are [1992] red phosphorus except that they [1989]
(a) Phosphorus acid and two (a) Are both soluble in CS2
(b) Hypophosphorous acid and two (b) Can be oxidised by heating in air
(c) Hypophosphorous acid and one (c) Consist of the same kind of atoms
(d) Hypophosphoric acid and two (d) Can be converted into one another
20. Aqueous solution of ammonia consists of 31. When orthophosphoric acid is heated to 600°C,
[1991] the product formed is [1989]
(a) H+ (b) OH–
(a) PH3 (b) P2O5
(c) NH4+ (d) NH4+and OH–
(c) H3PO3 (d) HPO3
21. P2O5 is heated with water to give [1991]
32. Which of the following is a nitric acid anhydride?
(a) Hypophosphorous acid
[1988]
(b) Phosphorous acid
(a) NO (b) NO2
(c) Hypophosphoric acid
(d) Orthophosphoric acid (c) N2O5 (d) N2O3.
̸» °óÞ´±½µ Û´»³»²¬­ øÙ®±«° ïëô ïêô ïé ¿²¼ ïè÷ ïéç

̱°·½ îæ Ѩ§¹»² Ú¿³·´§ List - I List - II


Substances Processes
33. In which pair of ions both the species contain 1. Sulphuric acid (i) Haber’s process
S – S bond? [2017] 2. Steel (ii) Bessemer’s
(a) S4 O62 ,S2 O32 (b) S2 O72 ,S2 O82 process
3. Sodium hydroxide (iii) Leblanc process
(c) S4 O62 ,S2 O72 (d) S2 O72 ,S2 O32 4. Ammonia (iv) Contact process
34. Nitrogen dioxide and sulphur dioxide have some Code :
properties in common. Which property is shown 1 2 3 4
by one of these compounds, but not by the (a) (iv) (ii) (iii) (i)
other? [2015] (b) (i) (iv) (ii) (iii)
(a) is a reducing agent (c) (i) (ii) (iii) (iv)
(b) is soluble in water (d) (iv) (iii) (ii) (i)
(c) is used as a food-preservative 41. Which of the following is the most basic oxide?
(d) forms 'acid-rain' [2006]
35. Which of the statements given below is (a) Sb2O3 (b) Bi2O3
incorrect? [2015 RS] (c) SeO2 (d) Al2O3
(a) Cl2O7 is an anhydride of perchloric acid 42. During its reactions, ozone [1999]
(b) O3 molecule is bent (a) can only combine with hydrogen atoms
(c) ONF is isoelectronic with O2N–. (b) accepts electrons
(d) OF2 is an oxide of fluorine (c) loses electrons
36. Acidity of diprotic acids in aqueous solutions (d) shows the role of electrons to be irrelevant
increases in the order : [2014] 43. Which of the following oxides will be the least
(a) H2S < H2Se < H2Te acidic? [1996]
(b) H2Se < H2S < H2Te (a) As4O 6 (b) As 4O10
(c) H2Te < H2S < H2Se (c) P4O10 (d) P4O6
(d) H2Se < H2Te < H2S 44. Oxidation of thiosulphate by iodine gives
37. Which of the following does not give oxygen [1996]
on heating? [NEET 2013] (a) tetrathionate ion (b) sulphide ion
(a) Zn(ClO3)2 (b) K2Cr2O7 (c) sulphate ion (d) sulphite ion
(c) (NH4)2Cr2O7 (d) KClO3 45. About 20 km above the earth, there is an ozone
38. Sulphur trioxide can be obtained by which of layer. Which one of the following statements
the following reaction : [2012] about ozone and ozone layer is true? [1995]
(a) ozone has a triatomic linear molecule
(a) CaSO 4 C (b) it is harmful as it stops useful radiation
(c) it is beneficial to us as it stops U.V radiation
(b) Fe 2 SO 4 3 (d) conversion of O3 to O2 is an endothermic
reaction
(c) S H 2SO 4
46. By passing H 2S gas in acidified KMnO 4
(d) H 2SO 4 PCI5 solution, we get [1995]
(a) S (b) K2S
39. Which one of the following compounds is a
(c) MnO2 (d) K2SO3
peroxide ? [2010]
47. Polyanion formation is maximum in [1994]
(a) KO 2 (b) BaO2 (a) Nitrogen (b) Oxygen
(c) MnO 2 (d) NO 2 (c) Sulphur (d) Boron
48. The acid which has a peroxy linkage is [1994]
40. Match List - I (substances) with List - II (processes)
(a) Sulphurous acid (b) Pyrosulphuric acid
employed in the manufacture of the substances
(c) Dithionic acid (d) Caro’s acid
and select the correct option. [2010]
EBD_7324
ïèð ÝØÛÓ×ÍÌÎÇ
49. Which would quickly absorb oxygen? (a) HClO3 < HClO4 < HClO2 < HClO
[1991, 92] (b) HClO < HClO2 < HClO3 < HClO4
(a) Alkaline solution of pyrogallol (c) HClO2 < HClO < HClO3 < HClO4
(b) Conc. H2SO4 (d) HClO4 < HClO2 < HClO < HClO3
(c) Lime water 57. Which one of the following orders is correct for
(d) Alkaline solution of CuSO4 the bond dissociation enthalpy of halogen
50. Oleum is [1991] molecules? [2016]
(a) Castor Oil (b) Oil of vitriol (a) I2 > Br2 > Cl2 > F2 (b) Cl2 > Br2 > F2 > I2
(c) Fuming H2SO4 (d) None of them (c) Br2 > I2 > F2 > Cl2 (d) F2 > Cl2 > Br2 > I2
51. Oxygen will directly react with each of the 58. The variation of the boiling point of the
following elements except [1989] hydrogen halides is in the order HF > HI > HBr >
(a) P (b) Cl HCl. [2015 RS]
(c) Na (d) S. What explains the higher boiling point of
52. The gases respectively absorbed by alkaline hydrogen fluoride?
pyrogallol and oil of cinnamon are [1989] (a) The electronegativity of fluorine is much
(a) O3, CH4 (b) O2, O3 higher than for other elements in the group.
(c) SO2,CH4 (d) N2O, O3. (b) There is strong hydrogen bonding between
53. It is possible to obtain oxygen from air by HF molecules
fractional distillation because [1989] (c) The bond energy of HF molecules is greater
(a) oxygen is in a different group of the periodic than in other hydrogen halides.
table from nitrogen (d) The effect of nuclear shielding is much
(b) oxygen is more reactive than nitrogen reduced in fluorine which polarises the HF
(c) oxygen has higher b.p. than nitrogen molecule.
(d) oxygen has a lower density than nitrogen. 59. Which is the strongest acid in the following :
54. Hypo is used in photography to [1988] [NEET 2013]
(a) reduce AgBr grains to metallic silver (a) HClO3 (b) HClO4
(b) convert metallic silver to silver salt (c) H2SO3 (d) H2SO4
(c) remove undecomposed silver bromide as a 60. In which of the following arrangements the given
soluble complex sequence is not strictly according to the property
(d) remove reduced silver indicated against it ? [2012 M]
(a) HF < HCl < HBr < HI : increasing acidic
̱°·½ íæ Ø¿´±¹»² Ú¿³·´§
strength
55. Match the interhalogen compounds of column-I (b) H2O < H2S < H2Se < H2Te : increasing
with the geometry in column II and assign the pKa values
correct code. [2017] (c) NH3 < PH3 < AsH3 < SbH3 : increasing
Column-I Column-II acidic character
1. XX' (i) T-shape
(d) CO2 < SiO2 < SnO2 < PbO2 : increasing
2. XX'3 (ii) Pentagonal bipyramidal
oxidising power
3. XX'5 (iii) Linear
61. The correct order of increasing bond angles in
4. XX'7 (iv) Square-pyramidal
the following species are : [2010]
(v) Tetrahedral
Code : (a) Cl 2 O < ClO 2 < ClO 2–
1 2 3 4
(a) (iii) (i) (iv) (ii) (b) ClO2 < Cl 2 O < ClO 2–
(b) (v) (iv) (iii) (ii)
(c) (iv) (iii) (ii) (i) (c) Cl 2 O < ClO 2– < ClO2
(d) (iii) (iv) (i) (ii)
(d) ClO 2– < Cl 2 O < ClO2
56 Among the following, the correct order of acidity
is [2016]
̸» °óÞ´±½µ Û´»³»²¬­ øÙ®±«° ïëô ïêô ïé ¿²¼ ïè÷ ïèï
62. In the case of alkali metals, the covalent character 70. Which one of the following arrangements does
decreases in the order: [2009] not truly represent the property indicated
(a) MF > MCl > MBr > MI against it? [2000]
(b) MF > MCl > MI > MBr (a) Br2 < Cl2 < F2 : Electronegativity
(c) MI > MBr > MCl > MF (b) Br2 < F2 < Cl2 : Electron affinity
(d) MCl > MI > MBr > MF (c) Br2 < Cl2 < F2 : Bond energy
63. Among the following which is the strongest (d) Br2 < Cl2 < F2 : Oxidising power
oxidising agent? [2009] 71. A one litre flask is full of brown bromine vapour.
(a) Br2 (b) I2 The intensity of brown colour of vapour will not
(c) Cl2 (d) F2 decrease appreciably on adding to the flask
64. Which one of the following arrangements does some [1998]
not give the correct picture of the trends (a) pieces of marble
indicated against it ? [2008] (b) animal charcoal powder
(a) F2 > Cl2 > Br2 > I2 : Oxidizing power (c) carbon tetrachloride
(b) F2 > Cl2 > Br2 > I2 : Electron gain enthalpy (d) carbon disulphide
(c) F2 > Cl2 > Br2 > I2 : Bond dissociation 72. Which one is the correct order of the size of
energy iodine species? [1997]
(d) F2 > Cl2 > Br2 > I2 : Electronegativity. (a) I > I+ > I– (b) I > I– > I+
65. Which one of the following orders correctly (c) I+ > I– > I (d) I– > I > I+
represents the increasing acid strengths of the 73. Regarding F– and Cl– which of the following
given acids? [2007] statements is/are correct? [1996]
(a) HOClO < HOCl < HOClO3 < HOClO2 (i) Cl– can give up an electron more easily than
(b) HOClO2 < HOClO3 < HOClO < HOCl F–
(c) HOClO3 < HOClO2 < HOClO < HOCl
(ii) Cl– is a better reducing agent than F–
(d) HOCl < HOClO < HOClO2 < HOClO3
(iii) Cl– is smaller in size than F–
66. Which one of the following orders is not in
accordance with the property stated against is ? (iv) F– can be oxidized more readily than Cl–
[2006] (a) (i) and (ii) (b) (i), (ii) and (iv)
(a) HI > HBr > HCl > HF : Acidic property in (c) (iii) and (iv) (d) only (i)
water 74. A certain compound (X) when treated with
(b) F2 > Cl2 > Br2 > I2 : Electronegativity copper sulphate solution yields a brown
(c) F2 > Cl2 > Br2 > I2 : Bond dissociation precipitate. On adding hypo solution, the
energy precipitate turns white. The compound is
(d) F2 > Cl2 > Br2 > I2 : Oxidising power [1994]
67. The correct order of acid strength is: [2005] (a) K2CO3 (b) KI
(a) HClO4 < HClO3 < HClO2 < HClO (c) KBr (d) K3PO4
(b) HClO < HClO2 < HClO3 < HClO4 75. HI can be prepared by all the following methods,
(c) HClO4 < HClO < HClO2 < HClO3 except [1994]
(d) HClO2 < HClO3 < HClO4 < HClO (a) PI 3 H 2 O (b) KI H 2SO 4
68. Which is the best description of the behaviour
of bromine in the reaction given below? [2004] (c) Pt (d) I 2 H 2S
H2 I2
H 2 O Br2 HOBr HBr 76. Which among the following is paramagnetic?
(a) Proton acceptor only [1994]
(b) Both oxidized and reduced
(a) Cl 2 O (b) ClO2
(c) Oxidized only
(d) Reduced only (c) Cl 2O 7 (d) Cl 2O 6
69. Which of the following statements is not true ?
[2003] 77. Which one of the following oxides of chlorine is
(a) HF is a stronger acid than HCl obtained by passing dry chlorine over silver
(b) Among halide ions, iodide is the most chlorate at 90°C ? [1994]
powerful reducing agent (a) Cl2 O (b) ClO3
(c) Fluorine is the only halogen that does not
show a variable oxidation state (c) ClO2 (d) ClO4
(d) HOCl is a stronger acid than HOBr
EBD_7324
ïèî ÝØÛÓ×ÍÌÎÇ
78. The formula for calcium chlorite is [1994] ̱°·½ ìæ Ò±¾´» Ù¿­»­
(a) Ca(ClO 4 ) 2 (b) Ca(ClO3 )2 85. Match the compounds given in column I with
(c) CaClO2 (d) Ca(ClO 2 )2 the hybridisation and shape given in column II
and mark the correct option. [2016]
79. A solution of potassium bromide is treated with Column-I Column-II
each of the following. Which one would liberate 1. XeF6 (i) Distorted octahedral
bromine ? [1993] 2. XeO3 (ii) Square planar
(a) Hydrogen iodide (b) Sulphur dioxide 3. XeOF4 (iii) Pyramidal
(c) chlorine (d) Iodine 4. XeF4 (iv) Square pyramidal
80. When chlorine is passed over dry slaked lime at Code :
room temperature, the main reaction product is 1 2 3 4
[1992] (a) (i) (iii) (iv) (ii)
(b) (i) (ii) (iv) (iii)
(a) Ca (ClO2 ) 2 (b) CaCl 2
(c) (iv) (iii) (i) (ii)
(c) CaOCl 2 (d) Ca (OCl) 2 (d) (iv) (i) (ii) (iii)
81. In the manufacture of bromine from sea water, 86. Identify the incorrect statement, regarding the
the mother liquor containing bromides is treated molecule XeO4: [NEET Kar. 2013]
(a) XeO4 molecule is tetrahedral
with [1992]
(b) XeO4 molecule is square planar
(a) Carbon dioxide (b) Chlorine (c) There are four p – d bonds
(c) Iodine (d) Sulphur dioxide (d) There are four sp3 – p, bonds
82. The bleaching action of chlorine is due to [1991] 87. Noble gases do not react with other elements
(a) Reduction (b) Hydrogenation because [1994]
(c) Chlorination (d) Oxidation (a) They are mono atomic
83. Bleaching powder reacts with a few drops of (b) They are found in abundance
dilute HCl to give [1989] (c) The size of their atoms is very small
(a) chlorine (d) They are completely paired up and stable
(b) hypochlorous acid electron shells
(c) calcium oxide 88. Which of the following statements is false ?
(d) oxygen [1994]
84. Bleaching powder is obtained by the action of (a) Radon is obtained from the decay of radium
chlorine gas and [1988] (b) Helium is inert gas
(c) Xenon is the most reactive among the rare
(a) dilute solution of Ca(OH)2
gases
(b) concentrated solution of Ca(OH)2
(d) The most abundant rare gas found in the
(c) dry CaO
atmosphere is helium
(d) dry slaked lime

ANSWER KEY
1 (b) 10 (c) 19 (c) 28 (a) 37 (c) 46 (a) 55 (a) 64 (b,c) 73 (d) 82 (d)
2 (a) 11 (a) 20 (d ) 29 (a) 38 (b) 4 7 (c) 56 (b) 65 (d) 74 (b) 83 (a)
3 (a) 12 (a) 21 (d ) 30 (a) 39 (b) 4 8 (d) 57 (b) 66 (c) 75 (b) 84 (d)
4 (c) 13 (b) 22 (b ) 31 (d) 40 (a) 49 (a) 58 (b) 67 (b) 76 (b) 85 (a)
5 (b) 14 (c) 23 (b ) 32 (c) 41 (b) 5 0 (c) 59 (b) 68 (b) 77 (c) 86 (b)
6 (a) 15 (a) 24 (a) 33 (a) 42 (a) 51 (b) 60 (b) 69 (a) 78 (d) 87 (d)
7 (b) 16 (a) 25 (d ) 34 (c) 43 (a) 52 (b) 61 (c) 70 (c) 79 (a) 88 (d)
8 (c) 17 (b) 26 (b ) 35 (d) 44 (a) 53 (c) 62 (c) 71 (a) 80 (c)
9 (b) 18 (d) 27 (c) 36 (a) 45 (c) 54 (c) 63 (d) 72 (d) 81 (b)
̸» °óÞ´±½µ Û´»³»²¬­ øÙ®±«° ïëô ïêô ïé ¿²¼ ïè÷ ïèí

Hints & Solutions


1. (b) Phosphinic acid as shown in structure below O
has one P—OH bend thus it is monobasic or
monoprotic present H P OH

O H
3 3
P
6. (a) HO P O P OH H 4 P2 O5
H OH (Monoprotic) | |
H OH OH
Phosphonic acid as shown in structure has two
P–OH bonds thus it is dibasic or diprotic O O
|| ||
O HO P P OH H 4 P2O6
4 | | 4
P OH OH
H (Diprotic acid)
OH
OH O O
|| ||
2. (a) (Bonus) HO P O P OH H 4 P2 O7
5 | | 5
CaC2 + N2 Ca(CN)2 + C OH OH

3. (a) The acids which contain P-H bond have 7. (b) O


strong reducing properties. Thus H3PO2
acid is good reducing agent as it contains bridging P bridging
two P–H bonds and reduces, for example, O
AgNO3 to metallic silver. O
O
4 AgNO3 + 2H2O + H3PO2 —
P O
4Ag + 4HNO3 + H3PO4
O P
4. (c) Compound Oxidation number of O
nitrogen O P O
N2H4 = –2 bridging
NH3 = –3 O bridging
oxygen
N3H = –1/3 i.e. 6-bridging oxygen.
NH2OH = –1 8. (c) Nitrogen form N 2 (i.e. N N) but
phosphorus form P4, because in P2, p — p
O bonding is present which is a weaker bonding.
9. (b)
5. (b) H P H Hypophosphorous acid
OH OH
O (a) H4P2O7 O P O P O
H
OH OH
(H3PO2) is a monobasic acid. i.e., it has only Pyrophosphoric acid
one ionisable hydrogen atom or one OH is O
P
(b) H3PO2 OH H
H
EBD_7324
ïèì ÝØÛÓ×ÍÌÎÇ
Hypophosphorous acid
(2HNO 3 H 2 O 2NO 2 O)18
O
C12H 22O11 18[O] 6 ( COOH ) 2
(c) H3PO3 HO – P – OH Cane sugar From HNO3 Oxalic acid
H
Phosphorous acid 5H 2O.
O C12H22O11 + 36HNO3
(d) H3PO4 HO – P – OH 6(COOH)2 + 36NO2 + 23H2O
OH 18. (d) N2 molecule contains triple bond between
orthophosphoric acid N atoms having very high dissociation energy
10. (c) Ammonium sulphate is a salt of weak base (946 kJ mol–1) due to which it is relatively
and strong acid, so it produces acidity. Hence inactive.
aqueous solution of ammonium sulphate 19. (c) H3PO2 is named as hypophosphorous acid.
increases the acidity of soil. As it contains only one P – OH group, its basicity
11. (a) Order of dipole moment decreases as is one.
NH3 > PH3 > AsH3 > SbH3 20. (d) Aqueous solution of ammonia is obtained
(Based upon electronegativity) by passing NH3 in H2O which gives NH4+ and
12. (a) We know that empirical formula of OH– ions.
hypophosphorus acid is H3PO2. In this only NH 3 H 2 O NH 4 OH
one ionisable hydrogen atom is present i.e. it is
monobasic. Therefore option (a) is correct 21. (d) P2O5 have great affinity for water. The final
structural formula of it. product is orthophosphoric acid.
13. (b) Brown ring test is done for the confirmation 2H2O
P4O10 4HPO3
of NO 3 ions.
Metaphosphoric
NaNO3 (aq) H 2SO 4 (aq) acid
2H2O
NaHSO 4 (aq) HNO3 (aq) 2H 2O
6FeSO 4 2HNO3 +3H 2SO 4 4H3PO4 2H4P2O7
orthophosphoric Pyrophosphoric
3Fe2 (SO 4 )3 +2NO+4H 2O acid acid

FeSO4 NO [Fe(NO)]SO 4 22. (b) It is a tribasic acid all the three hydrogen
Ferrous nitroso-sulphate (Brown ring) atoms are ionisable. It forms three series of salts.
14. (c) Urea (46.6%N). % of N in other compound + –
H3PO4 H + H2PO4
are : ( NH 4 ) 2 SO 4 21.2%;
CaCN 2 35.0% and NH 4 NO 3 35.0%
+ 3– + 2–
15. (a) Phosphorus pentoxide has great affinity for 3H + PO4 2H + HPO4
water. It forms ortho phosphoric acid on
absorbing water 23. (b) PCl3 3H 2 O H 3PO 3 3HCl
i.e. P4 O10 6H 2 O 4H 3 PO 4 24. (a) PH 4 I NaOH NaI PH 3 H 2 O
It is thus used as a powerful dehydrating or 25. (d) Pure nitrogen in the lab can be obtained by
drying agent. heating ammonium nitrate. Ammonium nitrate is
16. (a) Nitrogen molecule is diatomic containing a not a stable compound it dissociate to give
triple bond between two N atoms, N N . nitrogen.
Therefore, nitrogen molecule is formed by Heat
sharing six electrons. NH 4 Cl NaNO 2 NH 4 NO 2
17. (b) Cane sugar is oxidised to oxalic acid Heat
N2 2 H 2 O.
̸» °óÞ´±½µ Û´»³»²¬­ øÙ®±«° ïëô ïêô ïé ¿²¼ ïè÷ ïèë

26. (b) In case of nitrogen, d-orbitals are not


available. 2 KClO3 2KCl + 3O2
27. (c) NH3 undergoes H-bonding and hence has 4K2Cr2O7
the highest b.p. Among the remaining hydrides
4K2CrO4 + 2Cr2O3 + 3O2
i.e. PH3, AsH3 and SbH3 the b.p. increases as
the size of the element increases and hence the
magnitude of the van der Waal’s forces of 38. (b) Fe 2 (SO 4 )3 Fe2 O3 SO3
attraction increases. Thus, PH3 has the lowest
b.p. O
28. (a) Magnesium and manganese are the metals
that produce hydrogen with dilute nitric acid 39. (b) Ba
Mg + 2HNO3 Mg (NO3)2 + H2
29. (a) As Nitrogen does not have d-orbital in its O
valence shell, its maximum covalency is 3 while
in the case of other elements the maximum 40. (1) Sulphuric acid (iv) Contact process
covalency is 5. (2) Steel (ii) Bessemer’s
30. (a) Red phosphorus is not soluble in CS2 only process
white P is soluble. (3) Sodium hydroxide (iii) Leblanc process
600 C (4) Ammonia (i) Haber’s process
31. (d) 2H 3 PO 4 2HPO 3
orthophosphoric 2H 2O metaphosphoric
41. (b) More the oxidation state of the central atom
acid acid (metal) more is its acidity. Hence SeO2 (O. S. of
Se = +4) is acidic. Further for a given O.S., the
32. (c) N 2 O 5 H 2O 2 HNO 3
basic character of the oxides increases with the
O O O increasing size of the central atom. Thus Al2O3
– and Sb2O3 are amphoteric and Bi2O3 is basic.
33. (a) O – S – S – S – S – O S
42. (a) Since ozone can easily lose oxygen atom
O O S O
O (nascent oxygen), it acts as a powerful oxidising
S4 O6 2 S2 O32 agent, and hence reacts with hydrogen atoms.
34. (c) SO2 is widely used in food and drinks 43. (a) As the O.N of the central atom of the
industries for its property as a preservative and compounds increases acidic strength of that
compound also increases and on moving from
antioxidant while NO2 is not used as food
top to bottom in groups acidic strength of oxides
preservative.
also decrease due to decreasing
35. (d) OF2; among the following O and F, F is more electronegativity in groups.
electronegative than oxygen. +5 +3 +5 +3
So OF2 cannot be called oxide because in that P4O10 > P4O6 > As 4O10 > As 4O6
case fluorine is in +1 oxidation state which is
not possible, so OF2 is called oxygen difluoride. 44. (a) 2 S2 O 3 2 I2 S 4O 6 2 2I
Tetrathion ate
36. (a) The weaking of M—H bond with increase
45. (c) Ozone layer is beneficial to us, because it
in size of M (where M = S, Se, Te) explains the
stops harmful ultraviolet radiations to reach the
acid character of hydrides. Since on moving
earth.
down the group atomic size increases hence
bond length increases and hence removal 46. (a) 2KMnO 4 5H 2 S 3H 2 SO 4
tendency of H also increases. K 2 SO 4 2MnSO 4 5S 8H 2 O.
Thus in this reaction sulphur (S) is produced.
37. (c) (NH4)2Cr2O7 N2 + Cr 2O3 + 4H2O 47. (c) Due to greater tendency for catenation,
sulphur shows property of polyanion formation
Zn(ClO3)2 ZnCl2 + 3O2
to a greater extent. For example, in polysulphides
EBD_7324
ïèê ÝØÛÓ×ÍÌÎÇ
the bond distance increases from F 2 to I2. This
such as S32 , S24 , S52
is due to increase in the size of the atom, on
48. (d) Caro’s acid is H 2SO 5 which contains one moving from F to I.
S – O – O – H peroxy linkage. It is also known as F – F bond dissociation enthalpy is smaller then
permonosulphuric acids. Cl – Cl and even smaller than Br – Br. This is
O because F atom is very small and hence the three
|| lone pairs of electrons on each F atom repel the
H – O – O – S – OH bond pair holding the F-atoms in F 2
||
molecules.
O
Caro's acid The increasing order of bond dissociation
49. (a) Alkaline solution of pyrogallol absorbs enthalphy is
oxygen quickly. I2 < F2 < Br2 < Cl2
50. (c) Oleum is H 2S2 O 7 ( H 2SO 4 SO 3 ) which 58. (b) The H-bonding is present in HF due to high
electronegativity of fluorine atom. While H-
is obtained by dissolving SO 3 in H2SO4 and is
bonding is not present in HI, HBr and HCl.
called fuming sulphuric acid. 59. (b) HClO4 is the strongest acid amongst all
51. (b) S O 2 SO 2 (burns with blue light) because the oxidation state or Cl is maximum
4Na O2 2NaO (+7).
(burns with yellow light) 60. (b) If acidic nature is high, Ka is high and pKa
P4 3O 2 P4 O 6 is low
P4 5O 2 P4 O10 H2O H2S
Ka 1.8 × 10– 6 1.3 × 10–7
Cl O 2 No reaction
Chlorine does not react directly with oxygen. H2Se H2Te
52. (b) Alkaline pyrogallol absorbs O 2 and oil of Ka 1.3 × 10–4 2.3 × 10–3
cinnamon absorbs O3. since pKa = – log Ka
53. (c) Air is liquified by making use of the joule-
Hence the order of pKa will be
Thomson effect (cooling by expansion of the
gas). Water vapour and CO2 are removed by H2O > H2S > H2Se > H2Te
solidification. The remaining constituents of 61. (c) The correct order of increasing bond angle
liquid air i.e., liquid oxygen and liquid nitrogen is Cl 2 O ClO 2 ClO 2
are separated by means of fractional distillation Cl2O
2

ClO2
(b.p. of O2 = –183°C : b. p. of N2 = – 195.8°C).
2

54. (c) Undecomposed AgBr forms a soluble O Cl


complex with hypo
AgBr 2 Na 2S 2 O 3 Na 3[Ag (S 2 O 3 ) 2 ] NaBr 112° O
Cl Cl O
soluble complex
55. (a) XX' Linear (e.g. ClF, BrF) ClO 2
XX3' T-Shape (e.g. ClF3, BrF3)
2

XX5' Square pyramidal (e.g. BrF5 IF5) Cl


XX7' Pentagonal bipyramidal (e.g. IF 7)
56. (b) Acidic strength increases as the oxidation
number of central atom increases. O O
HClO < HClO2 < HClO3 < HClO4 118°

+1 +3 +5 +7
* In ClO2– there are 2 lone pairs of electrons
57. (b) Bond dissociation enthalpy decreases as
present on the central chlorine atom. Therefore
̸» °óÞ´±½µ Û´»³»²¬­ øÙ®±«° ïëô ïêô ïé ¿²¼ ïè÷ ïèé

the bond angle in ClO2– is less than 118° which (2p) of F2 in comparision to non- bonding pair
(3p) repulsion in Cl2, the bond energy of F2 <
is the bond angle in ClO2 which has less number
Cl2.
of electrons on chlorine.
Bond energy (kJ mol–1) :
62. (c) MI > MBr > MCl > MF. As the size of the
F – F Cl – Cl Br – Br I – I
anion decreases covalency decreases. 158.8 242.6 192.8 151.1
63. (d) Standard reduction potential of halogens 71. (a) There is no possibility of reaction between
are positive and decreases from fluorine to marble and bromine
iodine. Therefore halogens act as strong 72. (d) We know that positive ion is always smaller
oxidising agent and their oxidising power and negative ion is always larger than the
decreases from fluorine to iodine. corresponding atom. Therefore the correct order
64. (b and c) of the size is I I I
(a) The oxidising power of halogen follow the 73. (d) The halide ions act as reducing agents . F–
order F2 > Cl2 > Br2 > I2. ion does not show any reducing nature but Cl–
(b) The correct order of electron gain enthalpy , Br– & I– ion act as reducing agents and their
of halogens is Cl2 > F2 > Br2 > I2. The low value reducing nature is in increasing order
of F2 than Cl2 is due to its small size.
(c) The correct order of bond dissociation Cl – Br – I –
energies of halogens is Reducing nature increases
Cl2 > Br2 > F2 > I2. 74. (b) KI reacts with CuSO4 solution to produce
It is the correct order of electronegativity values cuprous iodide (white precipitate) and I2 (which
of halogens. gives brown colour). Iodine reacts with hypo
F2 > Cl2 > Br2 > I2 (Na2S2O3.5H2O) solution. Decolourisation of
solution shows the appearance of white
65. (d) HOCl HO Cl O HOClO 2 HOCl O3
1 3 5 7 precipitate.
As the oxidation number of the central atom 2CuSO 4 4KI 2K 2SO 4 2CuI I2
Cuprous iodide (Brown colour
increases, strength of acid also increases. (White ppt.) in solution)
66. (c) Bond dissociation energy of fluorine is less
because of its small size and repulsion between 2Na 2S2 O3 I2 Na 2S4 O6 2NaI
electrons of two atoms. So option (c) is wrong Sod. tetra
thionate
order. The correct order is (colourless)
[Cl2 > Br2 > F2 > I2]
75. (b) HI cannot be prepared by heating iodides
67. (b) As oxidation number of central atom in oxy- with concentrated H2SO4. The reaction between
acid increases strength increases. Hence the KI & H2SO4 is as follows:
correct order of acidic strength is
1 3 5 7 8KI 5H 2 SO 4 4K 2 SO 4 4I 2 4H 2 O H 2 S
H Cl O H Cl O 2 H Cl O3 H Cl O 4
0 +1 -1 76. (b) ClO2 contains 7 + 12 i.e. 19 electrons
68. (b) H 2O + B r2 HOBr+ HBr (valence) which is an odd number, i.e. there is
Thus here oxidation number of Br increases from (are) free electron(s). Hence it is paramagnetic
0 to +1 and also decreases from 0 to –1. Thus it in nature.
is oxidised as well as reduced. 77. (c) Pure ClO2 is obtained by passing dry Cl2
69. (a) F is more electronegative than Cl therefore over AgClO3 at 90°C.
HF bond is stronger than HCl and hence proton 90 C
2AgClO 3 Cl 2 (dry )
is not given off easily and hence HF is a weakest
acid. 2AgCl 2ClO 2 O 2
70. (c) The bond energy of F2 < Cl2 due to more 78. (d) Calcium chlorite is Ca (ClO2 ) 2
repulsion in between non-bonding electrons
EBD_7324
ïèè ÝØÛÓ×ÍÌÎÇ
79. (a) A stronger oxidising agent (Cl 2 ) displaces O
a weaker oxidising agent (Br2 ) from its salt F F F F

solution. Xe Xe
F F
2KBr Cl 2 2KCl Br2 F F

80. (c) Ca (OH) 2 Cl 2 CaOCl 2 H 2O 2 p

81. (b) Bromide in the mother liquor is oxidised to Square pyramidal Square planar
Br2 by Cl 2 which is a stronger oxidising agent.

2Br Cl2 Br2 2Cl 86. (b)


(sea water)
82. (d) Bleaching action of chlorine is due to
oxidation in presence of moisture. It is permanent (Tetrahedral sp 3)
87. (d) On account of highly stable ns 2 np 6
H 2O+Cl2 2HCl+[O]
configuration in the valence shell. These
Colouring matter + [O ] colourless matter
elements have no tendency either to lose gain
83. (a) CaOCl 2 2HCl CaCl 2 H 2 O Cl 2 or share electrons with atoms of other elements
Bleaching powder i.e., their combining capacity or valency is zero.
84. (d) Cl 2 gas reacts with dry slaked lime, Further all the orbitals in the atoms of these
elements are doubly occupied i.e electrons are
Ca (OH) 2 at 40°C to give bleaching powder not available for sharing.
88. (d) The most abundant rare gas found in the
Ca (OH) 2 Cl 2 CaOCl 2 H 2O atmosphere is argon and not helium.
85. (a) XeF6 XeO3
F
F
Xe
F Xe F
O O
O
F F
1p
distorted octahedral Pyramidal
XeOF4 XeF4
ݸ¿°¬»®

22 The d-and f-Block Elements

̱°·½ ïæ ݸ¿®¿½¬»®·­¬·½­ ±º ¼óÞ´±½µ Û´»³»²¬­ (At nos. Ce = 58, Sm = 62, Eu = 63, Yb = 70)
[NEET 2013]
1. Magnetic moment 2.84 B.M. is given by :- (a) Sm2+ (b) Eu2+
(At. nos, Ni = 28, Ti = 22, Cr = 24, Co = 27) [2015] (c) Yb2+ (d) Ce2+
(a) Ti3+ (b) Cr2+ 8. Sc (Z = 21) is a transition element but Zn (Z = 30)
(c) Co 2+ (d) Ni2+ is not because [NEET Kar. 2013]
2. The number of d-electrons in Fe2+ (Z = 26) is not (a) both Sc and Zn do not exhibit variable
equal to the number of electrons in which one of oxidation states
the following? [2015] (b) both Sc3+ and Zn 2+ ions are colourless and
(a) p-electrons in Cl (Z = 17) form white compounds
(b) d-electrons in Fe (Z = 26) (c) in case of Sc, 3d orbitals are partially filled
(c) p-electrons in Ne (Z = 10) but in Zn these are completely filled
(d) s-electrons in Mg (Z = 12) (d) last electron is assumed to be added to 4s
3. The angular momentum of electron in 'd' orbital level in case of Zn
is equal to : [2015] 9. Which one of the following does not correctly
represent the correct order of the property
(a) 2 (b) 2 3 indicated against it? [2012 M]
(a) Ti < V < Cr < Mn : increasing number of
(c) 0 (d) 6 oxidation states
4. Which is the correct order of increasing energy (b) Ti3+ < V3+ < Cr3+ < Mn3+ : increasing
of the listed orbitals in the atom of titanium ? magnetic moment
[2015 RS] (c) Ti < V < Cr < Mn : increasing melting
points
(a) 3s 4s 3p 3d (b) 4s 3s 3p 3d
(d) Ti < V < Mn < Cr : increasing 2nd ionization
(c) 3s 3p 3d 4s (d) 3s 3p 4s 3d enthalpy
5. Magnetic moment 2.83 BM is given by which of 10. Four successive members of the first series of
the following ions ? the transition metals are listed below. For which
(At. nos. Ti = 22, Cr = 24, Mn = 25, Ni = 28):-
[2014] one of them the standard potential E M2 /M
(a) Ti3+ (b) Ni2+ value has a positive sign? [2012 M]
(c) Cr3+ (d) Mn 2+ (a) Co (Z = 27) (b) Ni (Z = 28)
6. Reason of lanthanoid contraction is:- [2014] (c) Cu (Z = 29) (d) Fe (Z = 26)
(a) Negligible screening effect of ‘f ’ orbitals 11. The catalytic activity of transition metals and
(b) Increasing nuclear charge their compounds is ascribed mainly to : [2012 M]
(c) Decreasing nuclear charge (a) their magnetic behaviour
(d) Decreasing screening effect (b) their unfilled d-orbitals
7. Which of the following lanthanoid ions is (c) their ability to adopt variable oxidation state
diamagnetic ? (d) their chemical reactivity
EBD_7324
ïçð ÝØÛÓ×ÍÌÎÇ
12. For the four successive transition elements (Cr, (a) Cr > Mn > V > Ti
Mn, Fe and Co), the stability of +2 oxidation (b) V > Mn > Cr > Ti
state will be there in which of the following order? (c) Mn > Cr > Ti > V
[2011] (d) Ti > V > Cr > Mn
(a) Mn > Fe > Cr > Co 20. Which of the following oxidation states are the
(b) Fe > Mn > Co > Cr most characteristic for lead and tin respectively?
(c) Co > Mn > Fe > Cr [2007]
(d) Cr > Mn > Co > Fe (a) + 2, + 4 (b) + 4, + 4
13. Which of the following ions will exhibit colour (c) + 2, + 2 (d) + 4, + 2
in aqueous solutions? [2010] 21. Which one of the following ions is the most
(a) La 3+
(Z = 57) (b) Ti 3+
(Z = 22) stable in aqueous solution? [2007]
3+
(a) V3+ (b) Ti3+
(c) Lu 3+ (Z = 71) (d) Sc (Z = 21) (c) Mn 3+ (d) Cr3+
14. Which one of the following ions has electronic
(At.No. Ti = 22, V = 23, Cr = 24, Mn = 25)
configuration [Ar] 3d6 ? [2010]
22. In which of the following pairs are both the ions
(a) Ni 3+ (b) Mn 3+ coloured in aqueous solutions ? [2006]
3+
(a) Sc , Ti 3+ (b) Sc , Co2+
3+
(c) Fe3+ (d) Co3+
2+
(c) Ni , Cu + (d) Ni2+, Ti3+
(At. Nos. Mn = 25, Fe = 26, Co = 27, Ni = 28)
15. Which of the following pairs has the same size? (At. no. : Sc = 21, Ti = 22, Ni = 28, Cu = 29, Co = 27)
[2010] 23. The aqueous solution containing which one
of the following ions will be colourless?
(a) Fe 2+ , Ni 2+ (b) Zr 4+ , Ti 4+ (Atomic number: Sc = 21, Fe = 26, Ti = 22,
Mn = 25) [2005]
(c) Zr 4+ , Hf 4+ (d) Zn 2+ , Hf 4+ (a) Sc3+ (b) Fe2+
16. Which of the following oxidation states is the (c) Ti 3+ (d) Mn 2+
most common among the lanthanoids? [2010] 24. Four successive members of the first row
(a) 3 (b) 4 transition elements are listed below with their
(c) 2 (d) 5 atomic numbers. Which one of them is
17. Which one of the elements with the following expected to have the highest third ionization
outer orbital configurations may exhibit the enthalpy? [2005]
(a) Vanadium (Z = 23)
largest number of oxidation states? [2009]
(b) Chromium (Z = 24)
(a) 3d54s1 (b) 3d54s2 (c) Manganese (Z = 25)
(c) 3d24s2 (d) 3d34s2 (d) Iron (Z = 26)
25. Among the following series of transition metal
18. Out of TiF62– , CoF63– , Cu2Cl 2 and NiCl 2–4 ions, the one where all metal ions have 3d2
(Z of Ti = 22, Co = 27, Cu = 29, Ni = 28), the electronic configuration is(At. nos. Ti = 22;
colourless species are: [2009] V = 23; Cr = 24; Mn = 25) [2004]
(a) Cu2Cl2 and NiCl 2– (a) Ti 3 , V 2 , Cr 3 , Mn 4
4

(b) TiF62– ,and Cu2Cl2


(b) Ti , V 4 , Cr 6 , Mn 7

(c) CoF63– ,and NiCl 2–


(c) Ti 4 , V 3 , Cr 2 , Mn 3
4
(d) Ti 2 , V 3 , Cr 4 , Mn 5
(d) TiF62– ,and CoF63– ,
26. The basic character of the transition metal
19. The correct order of decreasing second monoxides follows the order
ionisation enthalpy of Ti (22), V(23), Cr(24) and (Atomic No.,Ti = 22, V = 23, Cr = 24, Fe = 26)
Mn (25) is : [2008] [2003]
̸» ¼ó¿²¼ ºóÞ´±½µ Û´»³»²¬­ ïçï

(a) TiO > VO > CrO > FeO


(b) VO > CrO > TiO > FeO (a) [Xe]4f 14 5d10 6s 2 (b) [ Kr]4d10 5s 2
(c) CrO > VO > FeO > TiO 2
(c) [ Ne] 3s 3p
5
(d) [Ar] 3d 4s
10 2
(d) TiO > FeO > VO > CrO
36. Which one of the following is an ore of silver ?
27. Which one of the following characteristics of
[1988]
the transition metals is associated with their
(a) Argentite (b) Stibnite
catalytic activity? [2003]
(c) Haematite (d) Bauxite
(a) Variable oxidation states 37. Which of the following metals corrodes readily
(b) High enthalpy of atomization in moist air ? [1988]
(c) Parmagnetic behaviour (a) Gold (b) Silver
(d) Colour of hydrated ions (c) Nickel (d) Iron
28. Which of the following shows maximum number
of oxidation states? [2002] ̱°·½ îæ ݱ³°±«²¼­ ±º Ì®¿²­·¬·±² Ó»¬¿´­
(a) Cr (b) Fe 38. Name the gas that can readily decolourise
(c) Mn (d) V acidified KMnO4 solution : [2017]
29. Of the following transition metals, the maximum (a) SO2 (b) NO2
numbers of oxidation states are exhibited by: (c) P2O5 (d) CO2
[2000] 39. Which one of the following statements is correct
(a) Chromiun (Z = 24) when SO2 is passed through acidified K2Cr2O7
(b) Manganese (Z = 25) solution ? [2016]
(c) Iron (Z = 26) (a) The solution turns blue
(d) Titanium (Z=22) (b) The solution is decolourized
30. Which of the following forms colourless (c) SO2 is reduced
compound? [2000] (d) Green Cr 2(SO4)3 is formed
(a) Sc3+ (b) V3+ 40. When copper is heated with conc. HNO3 it
(c) Ti3+ (d) Cr3+ produces [2016]
31. Which one of the following elements shows (a) Cu(NO3)2 and NO2
maximum number of different oxidation states in (b) Cu (NO3)2 and NO
its compounds? [1998] (c) Cu(NO3)2, NO and NO2
(a) Eu (b) La (d) Cu(NO3)2 and N2O
(c) Ge (d) Am 41. Which of the following processes does not
32. Which one of the following ionic species will involve oxidation of iron ? [2015]
impart colour to an aqueous solution? [1998] (a) Decolourization of blue CuSO 4 solutution
(a) Ti4+ (b) Cu+ by iron
(c) Zn 2+ (d) Cr3+ (b) Formation of Fe(CO)5 from Fe
33. The common oxidation states of Ti are [1994]
(c) Liberation of H2 from steam by iron at high
(a) + 2, + 3 (b) + 3, + 4
temperature
(c) – 3, – 4 (d) + 2, + 3, + 4
(d) Rusting of iron sheets
34. The electronic configuration of Cu (atomic
number 29) is [1991] 42. Assuming complete ionization, same moles of
which of the following compounds will require
(a) 1s2 , 2s2 2p6 ,3s2 3p6 , 4s2 3d 9 the least amount of acidified KMnO 4 for
(b) 1s2 , 2s2 2p6 , 3s2 3p6 3d10 , 4s1 complete oxidation [2015 RS]
(a) FeSO4 (b) FeSO3
(c) 1s2 , 2s2 2p6 , 3p2 3p6 , 4s2 4p6 ,5s 2 5p1
(c) FeC2O4 (d) Fe(NO2)2
(d) 1s 2 , 2s2 2p6 ,3s 2 3p 6 , 4s2 4p 6 3d 3 43. The pair of compounds that can exist together is:
35. The electronic configurations of four elements [2014]
are given below. Which element does not belong (a) FeCl3, SnCl2 (b) HgCl2, SnCl2
to the same family as others ? [1989] (c) FeCl2, SnCl2 (d) FeCl3, KI
EBD_7324
ïçî ÝØÛÓ×ÍÌÎÇ
44. In acidic medium, H2O2 changes Cr2O7–2 to 52. On heating chromite (FeCr 2O4) with Na2CO3 in
CrO5 which has two (–O–O) bonds. Oxidation air, which of the following product is obtained?
state of Cr in CrO5 is:- [2014] [1999]
(a) + 5 (b) + 3 (a) Na2Cr2O7 (b) FeO
(c) + 6 (d) – 10 (c) Fe3O4 (d) Na2CrO4
45. The reaction of aqueous KMnO 4 with H2O2 in 53. The addition of excess of aqueous HNO3 to a
acidic conditions gives: [2014] solution containing [Cu(NH3)4]2+ produces
(a) Mn4+ and O2 [1999]
(b) Mn2+ and O2 (a) Cu+ (b) [Cu(H2O)4]2+
(c) Mn2+ and O3 (c) Cu(OH)2 (d) Cu(NO3)2
(d) Mn4+ and MnO2 54. An acidic solution of 'X' does not give precipitate
46. KMnO4 can be prepared from K2MnO4 as per on passing H2 S through it. 'X' gives white
the reaction: precipitate when NH4OH is added to it. The
white precipitate dissolves in excess of NaOH
3MnO 24 2H 2 O solution. Pure 'X' fumes in air and dense white
fumes are obtained when a glass rod dipped in
2MnO24 MnO2 + 4OH NH4OH is put in the fumes. Compound 'X' can
The reaction can go to completion by removing be [1999]
OH– ions by adding. [NEET 2013] (a) ZnCl2 (b) FeCl3
(a) KOH (b) CO2 (c) AlCl3 (d) SnCl2
(c) SO2 (d) HCl 55. Which one of the following elements constitutes
a major impurity in pig iron ? [1998]
47. Which of the statements is not true? [2012]
(a) Silicon (b) Oxygen
(a) On passing H2S through acidified K2Cr2O7
(c) Sulphur (d) Graphite
solution, a milky colour is observed.
56. K2Cr2O7 on heating with aqueous NaOH gives
(b) Na2Cr2O7 is preferred over K2Cr2O7 in [1997]
volumetric analysis.
(c) K2 Cr 2 O7 solution in acidic medium is (a) CrO 24 (b) Cr(OH)3
orange.
(c) Cr2 O 72 (d) Cr(OH)2
(d) K2 Cr 2 O7 solution becomes yellow on
increasing the pH beyond 7. 57. CrO3 dissolves in aqueous NaOH to give [1997]
(a) Cr2O72– (b) CrO42–
48. Acidified K2Cr2O7 solution turns green when
Na 2 SO3 is added to it. This is due to the (c) Cr(OH)3 (d) Cr(OH)2
formation of : [2011] 58. Cuprous compounds such as CuCl, CuCN and
CuSCN are the only salts stable in water due to
(a) Cr2(SO4)3 (b) CrO42–
[1996]
(c) Cr2(SO3)3 (d) CrSO4
(a) high hydration energy of Cu+ ions
49. Copper sulphate dissolves in excess of KCN to
(b) their inherent tendency do not
give [2006]
3–
disproportionate
(a) [Cu(CN)4] (b) [Cu(CN)4]2– (c) diamagnetic nature
(c) Cu(CN)2 (d) CuCN (d) insolubility in water
50. German silver is an alloy of [2000] 59. Stainless steel contains iron and [1995]
(a) Fe, Cr, Ni (b) Cu, Zn, Ag (a) Cr + Ni (b) Cr + Zn
(c) Cu, Zn, Ni (d) Cu, Sn, Al (c) Zn + Pb (d) Fe +Cr + Ni
51. Which of the following combines with Fe (II) 60. The most durable metal plating on iron to protect
ions to form a brown complex? [2000] against corrosion is [1994]
(a) NO (b) N2O (a) Nickel plating (b) Tin plating
(c) N2O3 (d) N2O5 (c) Copper plating (d) Zinc plating
̸» ¼ó¿²¼ ºóÞ´±½µ Û´»³»²¬­ ïçí

61. When ( NH 4 ) 2 Cr2O 7 is heated, th e gas ̱°·½ íæ Ô¿²¬¸¿²±·¼­ ¿²¼ ß½¬·²±·¼­


evolved is [1994] 70. The reason for greater range of oxidation states
(a) N 2 (b) NO 2 in actinoids is attributed to :- [2017]
(c) O 2 (d) N 2 O (a) actinoid contraction
62. When CuSO 4 is electrolysed using platinum (b) 5f, 6d and 7s levels having comparable energies
electrodes, [1993] (c) 4f and 5d levels being close in energies
(a) Copper is liberated at cathode, sulphur at (d) the redioactive nature of actinoids
anode 71. The electronic configurations of Eu(Atomic No.
(b) Copper is liberated at cathode, oxygen at 63), Gd(Atomic No. 64) and Tb (Atomic No. 65)
anode are [2016]
(c) Sulphur is liberated at cathode, oxygen at
anode (a) [Xe]4f76s2, [Xe]4f8 6s2 and [Xe]4f 85d16s2
(d) Oxygen is liberated at cathode, copper at (b) [Xe]4f7 5d 1 6s 2 , [Xe]4f7 5d 1 6s 2 and
anode [Xe]4f96s2
63. Cinnabar is an ore of [1991]
(a) Hg (b) Cu (c) [Xe]4f 6 5d 1 6s 2 , [Xe]4f 7 5d 1 6s 2 and
(c) Pb (d) Zn [Xe]4f85d16s2
64. The composition of ‘golden spangles’ is (d) [Xe]4f76s2, [Xe]4f75d16s2 and [Xe]4f96s2
[1990] 72. Because of lanthanoid contraction, which of the
(a) PbCrO 4 (b) PbI 2 following pairs of elements have nearly same
(c) As 2S3 (d) BaCrO4 atomic radii ? (Numbers in the parenthesis are
65. Prussian blue is formed when [1989] atomic numbers). [2015]
(a) Ferrous sulphate reacts with FeCl3 (a) Zr (40) and Nb (41)
(b) Ferric sulphate reacts with K 4 [Fe(CN) 6 ] (b) Zr (40) and Hf (72)
(c) Ferrous ammonium sulphate reacts with (c) Zr (40) and Ta (73)
FeCl 3 (d) Ti (22) and Zr (40)
(d) Ammonium sulphate reacts with FeCl 3 73. Gadolinium belongs to 4f series. It's atomic
66. Photographic films and plates have an essential number is 64. Which of the following is the
ingredient of [1989] correct electronic configuration of gadolinium ?
(a) Silver nitrate [1997, NEET Kar. 2013, 2015 RS]
(b) Silver bromide
(c) Sodium chloride (a) [Xe]4f 86d2
(d) Oleic acid
67. Nitriding is the process of surface hardening of (b) [Xe]4f 95s1
steel by treating it in an atmosphere of [1989]
(a) NH 3 (b) O 3 (c) [Xe] 4f 75d16s2
(c) N 2 (d) H 2S (d) [Xe] 4f 65d26s2
68. While extracting an element from its ore, the ore 74. Which of the following exhibit only + 3 oxidation
is ground and leached with dil. potassium
cyanide solution to form the soluble product state ? [2012 M]
potassium argento cyanide. The element is (a) U (b) Th
[1989] (c) Ac (d) Pa
(a) Lead (b) Chromium 75. Identify the incorrect statement among the
(c) Manganese (d) Silver following: [2007]
69. A blue colouration is not obtained when
[1989] (a) Lanthanoid contraction is the accumulation
(a) Ammonium hydroxide dissolves in copper of successive shrinkages.
sulphate (b) As a result of lanthanoid contraction, the
(b) Copper sulphate solution reacts with properties of 4d series of the transition
K 4 [Fe(CN) 6 ] elements have no similarities with the 5d
(c) Ferric chloride reacts with sod. ferrocyanide series of elements.
(d) Anhydrous CuSO4 is dissolved in water
EBD_7324
ïçì ÝØÛÓ×ÍÌÎÇ
(c) Shielding power of 4f electrons is quite (c) Y3 Lu 3 Eu 3 La 3
weak.
(d) There is a decrease in the radii of the atoms (d) Lu 3 Eu 3 La 3 Y3
or ions as one proceeds from La to Lu. (Atomic nos. Y =39, La = 57, Eu = 63, Lu = 71)
76. The main reason for l arger number of 79. General electronic configuration of lanthanides
oxidation states exhibited by the actinoids is [1991, 2002]
than the corresponding lanthanoids, is (a) (n – 2) f1 –14 (n –1) s2p6d0 – 1 ns2
[2005, 2006] (b) (n – 2) f10 –14 (n –1) d0 – 1 ns2
(a) more energy difference between 5f and (c) (n – 2) f0 –14 (n –1) d10 ns2
6d orbitals than between 4f and 5d (d) (n – 2) d0 –1 (n –1) f1 – 14 ns2
orbitals. 80. Which of the following statement is not correct?
(b) lesser energy difference between 5f and [2001]
6d orbitals than between 4f and 5d (a) La (OH)3 is less basic than Li(OH) 3
orbitals. (b) La is actually an element of transition series
(c) larger atomic size of actinoids than the rather lanthanides
lanthanoids. (c) Atomic radius of Zr and Hf are same
(d) greater reactive nature of the actinoids because of lanthanide contraction
than the lanthanoids. (d) In lanthanide series ionic radius of Ln +3
77. Lanthanoids are [2004] ions decreases
(a) 14 elements in the sixth period 81. The lanthanide contraction is responsible for
(atomic no. = 90 to 103) that are filling 4f the fact that [1997]
sublevel (a) Zr and Y have about the same radius
(b) 14 elements in the seventh period (b) Zr and Nb have similar oxidation state
(atomic no. = 90 to 103) that are filling 5f (c) Zr and Hf have about the same radius
sublevel (d) Zr and Zn have the same oxidation states
(c) 14 elements in the sixth period (Atomic numbers : Zr = 40, Y = 39, Nb = 41,
(atomic no. = 58 to 71) that are filling 4f Hf = 72, Zn = 30)
sublevel 82. Actinides [1994]
(d) 14 elements in the seventh period (a) Are all synthetic elements
(atomic no. = 58 to 71) that are filling 4f sub (b) Include element 104
level (c) Have any short lived isotopes
78. The correct order of ionic radii of Y3+, La3+, Eu3+ (d) Have variable valency
and Lu3+ is [2003] 83. Among the lanthanides the one obtained by
3 3 3 3
synthetic method is [1994]
(a) La Eu Lu Y (a) Lu (b) Pm
(c) Pr (d) Gd
(b) Y3 La 3 Eu 3 Lu 3

ANSWER KEY
1 (d) 10 (c) 19 (a) 28 (c) 37 (d) 46 (b) 55 (a) 64 (b) 73 (c) 82 (d)
2 (a) 11 (c) 20 (a) 29 (b) 38 (a) 47 (b) 56 (a) 65 (b) 74 (c) 83 (b)
3 (d) 12 (a) 21 (d) 30 (a) 39 (d) 48 (a) 57 (b) 66 (b) 75 (b)
4 (d) 13 (b) 22 (d) 31 (d) 40 (a) 49 (a) 58 (a) 67 (a) 76 (b)
5 (b) 14 (d) 23 (a) 32 (d) 41 (b) 50 (c) 59 (d) 68 (d) 77 (c)
6 (a) 15 (c) 24 (c) 33 (d) 42 (a) 51 (a) 60 (d) 69 (b) 78 (c)
7 (c) 16 (a) 25 (d) 34 (b) 43 (c) 52 (d) 61 (a) 70 (b) 79 (a)
8 (c) 17 (b) 26 (a) 35 (c) 44 (c) 53 (b) 62 (b) 71 (d) 80 (a)
9 (c) 18 (b) 27 (a) 36 (a) 45 (b) 54 (a) 63 (a) 72 (b) 81 (c)
̸» ¼ó¿²¼ ºóÞ´±½µ Û´»³»²¬­ ïçë

Hints & Solutions


1. (d) Magnetic moment = 2.84 B.M. This 7. (c) Sm2+(Z = 62)
indicates that 2 unpaired electrons are present. [Xe]4f6 6s2 – 6 unpaired e–
Eu2+(Z = 63)
For Ni+2 = 4s0 3d8
[Xe]4f7 6s2 – 7 unpaired e–
Number of unpaired electrons (n) = 2 Yb2+(Z = 70)
Hence Ni2+ gives magnetic moment 2.84 B.M. [Xe]4f14 6s2 – 0 unpaired e–
2. (a) Fe+2 = 3d6 (number of ‘d’ electrons = 6) Ce2+(Z = 58)
In Cl = 1s2 2s2 2p6 3s2 3p5 [Xe]4f1 5d1 6s2 – 2 unpaired e–
total p electrons = 11, which are not equal to Only Yb2+ is diamagnetic.
number of ‘d’ electrons in Fe+2 8. (c) A transition element must have incomplete
p-electrons in Ne = 1s2 2s2 2p6 = 6 d-subshell. Zinc have completely filled d
subshell having 3d10 configuration. Hence do
s-electrons in Mg = 1s2 2s2 2p6 3s2 = 6
not show properties of transition elements to
3. (d) Orbital angular momentum = ( 1). any appreciable extent except for their ability to
form complexes.
For d-orbital =2 9. (c) The melting points of the transition
angular momentum = elements first rise to a maximum and then fall as
2(2 1) 6
the atomic number increases, manganese have
4. (d) (n + l) rule can be used. Titanium is a multi abnormally low melting points.
electron system
10. (c) E Cu 2 /Cu = 0.34 volt,
(n + l) 3s < 3p < 4s < 3d
other has – ve E R.P.
(3 + 0) (3 + 1) (4 + 0) (3 + 2)
|| || || || E
3 4 4 5 Co /Co = – 0.28

If (n + l) values are same, than value of "n" has E


to be considered. Ni / Ni = – 0.25
5. (b) Magnetic moment
E
Fe /Fe = – 0.44
= n(n 2)
11. (c) The transition metals and their compounds
are used as catalysts because of the variable
2.83 = n(n 2) oxidation states. Due to this, they easily absorb
On solving n = 2 and re-emit wide range of energy to provide the
necessary activation energy.
Ni2+ have two unpaired electron.
12. (a)
6. (a) The shape of f-orbitals is very much 3+
diffused and they have poor shielding effect. 13. (b) La : 54 e– = [Xe]
The effective nuclear charge increases which Ti3+ : 19 e– = [Ar] 3 d 1 (Colour)
causes the contractiion in the size of electron
charge cloud. This contraction in size is quite Lu 3+ : 68 e– = [Xe] 4 f 14
regular and known as lanthanoid contraction.
Sc3+ : 18 e– = [Ar]
EBD_7324
ïçê ÝØÛÓ×ÍÌÎÇ
Ti3+ : 1s2, 2s2p6, 2s2p6d1, 4s0; 1 unpaired electron
14. (d) Ni 3+ : [Ar] 3d 7 Mn3+ : [Ar] 3d 4
present
Co2+ : 1s2, 2s2p6, 3s2p6d7, 4s0; 3 unpaired electron
Fe3+ : [Ar] 3d 5 Co3+ : [Ar] 3d 6 present
15. (c) Due to lanthanide contraction, the size of So from the given option the only correct
Zr and Hf (atom and ions) become nearly similar combination is Ni2+ and Ti3+
16. (a) 23. (a) Sc3+ 3d0 4s0
Fe2+ 3d6 4s0
3d 4s
17. (b) Mn - 3d5 4s2 4 unpaired e–
The no. of various oxidation states possible Ti3+ 3d14s0
are + 2, + 3, + 4, (+ 5), + 6, + 7. 1 unpaired e–
18. (b) The colour exhibited by transition metal ions
Mn2+ 3d5 4s0
is due to the presence of unpaired electrons in
d-orbitals which permits the d - d excitation of 5 unpaired e–
electrons. In Sc3+ there is no unpaired electron. So the
In TiF62– ,– Ti is in + 4 O.S. ; 3d0 = colourless aqueous solution of Sc3+ will be colourless.
24. (c) For third ionization enthalpy electronic
In CoF63– –, Co is in + 3 O.S ; 3d 5 = coloured configuration of
3d 4s
In Cu2Cl2– Cu is in +1 O.S. ; 3d10 – colourless V – 4s 0 3d3
23
In NiCl 2– 8
4 – Ni is in + 2 O.S ; 3d – coloured
24Cr – 4s0 3d4
19. (a) Ti ; Z (22) is 1s 2s 2p 3s 3p64s23d2
2 2 6 2

V ; Z (23) is 1s22s22p63s23p64s23d3 25Mn – 4s0 3d5


Cr ; Z (24) is 1s22s22p63s23p63d54s1 26Fe – 4s0 3d6
Mn ; Z (25) is 1s22s22p63s23d54s2 Mn has most stable configuration due to half
The second electron in all the cases (except Cr) filled d-orbital. Hence 3rd ionization energy will
is taken out from 4s-orbital and for Cr it is taken be highest for Mn.
from completely half filled 3d-orbital. The force 25. (d) The electronic configuration of different
required for removal of second electron will be species given in the option (d) are :
more for Mn than others (except for Cr) due to
Ti 2+ :1s 2 2s 2 2p 6 3s 2 3p 6 3d 2
having more positive charge. Based on this we
find the correct order Mn > V > Ti. V 3+ :1s 2 2s 2 2p 6 3s 2 3p 6 3d 2
i.e. Cr > Mn > V > Ti.
20. (a) The last three elements of group 4 i.e. Ge, Cr 4+ :1s 2 2s 2 2p 6 3s 2 3p 6 3d 2
Sn and Pb, have a tendency to form divalent as
well as tetravalent ions. Since the inert pair effect Mn 5+ :1s 2 2s 2 2p 6 3s 2 3p 6 3d 2
increases from Ge to Pb, the stability of 26. (a) The basic character of the transition metal
tetravalent ion decreases and that of divalent monoxide is TiO > VO > CrO > FeO because
ion increases that is why Pb++ is more stable basic character of oxides decrease with increase
than Pb4+ and Sn4+ ions is more stable than Sn 2+. in atomic number. Oxides of transitional metals
21. (d) For chromium ion + 3oxidation state is most in low oxidation state i.e., + 2 and + 3 are generally
stable. basic except Cr2O3.
22. (d) Sc3+ : 1s2, 2s2p6, 3s2p6d0, 4s0; no unpaired 27. (a) Transition metals have the ability to utilize
electron. (n –1)d -orbitals. Thus variable oxidation states
Cu+ : 1s2, 2s2p6, 3s2p6d10 , 4s0; no unpaired enables the transition element to associate with
electron. the reactants in different forms.
Ni2+: 1s2, 2s2p6, 3s2p6d8, 4s0; 2 unpaired electron 28. (c) Mn : [Ar] 3d5 4s2
present. Shows +2, +3, +4, +5, +6 & +7 oxidation states
̸» ¼ó¿²¼ ºóÞ´±½µ Û´»³»²¬­ ïçé
29. (b) Manganese shows max. no. of oxidation 39. (d) K2Cr2O7 + SO2 + H2SO4
states, K2SO4 + Cr 2(SO4)3 + H2O
+2, +3, + 4, + 5, + 6, + 7
green colour
Other metals shows the following oxidation states
Cr = + 2, +3, + 4, + 5, + 6 40. (a) Less active metals like Cu, Ag etc react with
Fe = + 2, + 3 conc acid and form nitrate and nitrogen dioxide.
Ti = + 2, + 3, + 4 41. (b) Formation of Fe(CO) 5 from Fe involves no
3 change in oxidation state of iron.
30. (a) Sc [Ar]18 3d 0 ; V 3
[Ar]18 3d 2
42. (a) Considering the same moles of compounds
3 18 1; 3 18 3
Ti [Ar] 3d Cr [Ar] 3d Fe2 SO2–
4 only Fe
2 is oxidised by KMnO .
4
Scandium ion doesn’t contain any unpaired ( 6)
electron in d orbitals hence it form colourless SO2–
4 in which sulphur is in highest oxidation
compound. state cannot be oxidised.
31. (d) Eu – + 2, + 3
La – +3 Fe 2 SO32– Fe 2
Gd – + 3
Am = +2, +3, +4, +5, +6 (+4) (+6)
2– 2– –
32. (d) We know that chromium (III) salts dissolve SO3 — SO4 + 2e
in water to give violet solution. The violet colour
is due to the hydrated chromium (III) in Sulphur can be oxidised to (+6) i.e to SO 2–
4

[Cr(H 2 O) 6 ]3 . In FeC2 O 4 Fe 2
33. (d) The minimum oxidation state of transition
Carbon can be oxidised to (+IV) i.e to CO2
metals is equal to the number of electrons in 4s
shell and the maximum oxidation state is equal Fe(NO 2 )2 Fe 2
to the sum of the 4s and 3d electrons.
Ti (22) = [Ar] 3d2 4s2 (+3)
hence minimum oxidation state is +2 and maximum Nitrogen can be
+ 2NO2–
oxidation state is +4. Thus the common oxidation oxidised to (+V)
states of Ti are +2, +3 & +4 – – state
34. (b) Electronic configuration of Cu (29) is 1s2 2s2 2NO3 + 2e
2p6 3s2 3p6 4s1 3d10 and not 1s2, 2s2 2p6 3s2 3p6
4s2, 3d9 due to extra stability of fully filled (+5)
orbitals. 43. (c) Both are reducing agents.
2 5
35. (c) [ Ne]3s 3p is the electronic configuration 44. (c) The structure of CrO5 is
of a p-block element whereas oth er O O
configurations are those of d-block elements
36. (a) Argentite or silver glance (Ag 2S) is an ore Cr
of Ag. O O
37. (d) In moist air, corrosion of iron takes place in O
the form of rust (Fe 2O 3 Fe(OH)3 ) . As iron is Hence CrO5 has two proxy linkage.
most reactive among all. Now suppose the oxi. no. of Cr is x then
38. (a) Potassium permanganate has a purple
x + (– 1 × 4) + (– 2) = 0
colour. When sulphur dioxide reacts with
potassium permanganate the solution x = +6
decolourises. 45. (b) 2 KMnO4 + 3H2SO4 + 5H2O2
SSO2 + 2KMnO4 + 2H2O 2H2SO4 + 2MnSO4 K2SO4 + 2MnSO4 + 8H2O + 5O2
+ K2SO4
i.e. Mn++ ion and O2.
EBD_7324
ïçè ÝØÛÓ×ÍÌÎÇ
46. (b) HCl and SO2 are reducing agents and can 2 NaOH
reduce MnO4–. CO2 which is neither oxidising Zn (OH ) 2 Na 2 ZnO 2 2H 2 O
Excess
and nor reducing will provide only acidic 55. (a) Pig iron is the most impure form of iron
medium. It can shift reaction in forward and contain highest proportion of carbon
direction and reaction can go to completion. (2.5 – 4%).
47. (b) Na 2Cr 2 O7 is hygroscopic in nature and
therefore accurate weighing is not possible in 56. (a) K 2 Cr2 O 7 2 NaOH
normal atmospheric conditions. A hygroscopic K 2 CrO4 Na 2 CrO 4 H 2O
substance absorbs moisture from atmosphere
and this could lead to inaccuracies in weight. Hence CrO 24 ion is obtained.
48. (a) The green colour appears due to the 57. (b) CrO3 2NaOH Na 2CrO 4 H 2 O
formation of Cr3+ion 58. (a) The smaller is the size of cation, the more is
Cr2 O72– 3SO32– 8H 3SO 2– 2Cr 3 4H 2 O heat of hydration. Since hydration is exothermic
4
process, thus heat evolves and hence system
49. (a) Copper sulphate on treatment with excess
having lower energy are stable.
of KCN forms complex K 3 [Cu(CN) 4 ] or
59. (d) Stainless steel contains 73% Fe, 18% Cr and
[Cu(CN)4]3–
8% Ni.
CuSO 4 2KCN Cu ( CN ) 2 K 2SO 4 60. (d) Galvanisation is the process of deposition of
2 Cu ( CN ) 2 Cu 2 (CN ) 2 (CN ) 2 zinc metal on the surface of Fe to prevent it from
rusting. Zinc forms a protective layer of basic
Cu 2 (CN ) 2 3KCN K 3 [ Cu ( CN ) 4 ] CuCN
carbonate (ZnCO3. Zn(OH)2) on it.
50. (c) German Silver Composition : Cu = 56%,
Zn = 24%, Ni = 20% Heat
51. (a) We know that when nitrogen oxide (NO) 61. (a) ( NH 4 ) 2 Cr2 O 7 Cr2 O 3 N2 4H 2 O
combines with Fe (II) ions, a brown complex is 62. (b) CuSO 4 Cu 2
SO 24 ;
formed. This reaction is called brown ring test
52. (d) By heating chromite with Na2 CO3, Na2CrO4 H 2O H OH
is obtained
At cathode : Cu 2 2e Cu
4FeCr2 O 4 8 Na 2 CO 3 7O 2
At anode : 4OH 2H 2 O O 2 4e
8Na 2 CrO 4 2Fe 2 O 3 8CO 2
53. (b) [Cu(NH3) 4]2+ on addition of excess of 63. (a) Cinnabar (HgS) is an ore of Hg.
aqueous HNO3 gives [Cu (H2O)4]2+.
64. (b) PbI 2 is yellow and is called golden spangles.
NH 4 OH
54. (a) X White ppt 65. (b) Potassium ferrocyanide solution is added to
excess Fe3+ ions in solution to give deep blue solution
Acidic solution (soluble ) or precipitate.
NaOH ( No. ppt with H 2S)
Given reactions (white precipitate with H2S in 6 FeSo4 3 K 4 [ Fe(CN ) 6 ]
presence of NH4OH) indicate that 'X' should be
ZnCl2 which explains all given reactions. Fe4 [ Fe(CN )6 ]3 6K 2 So4 Fe2
ZnCl2 2H 2 O Zn(OH) 2 HCl Prussian blue
White fumes 66. (b) AgBr is highly photosensitive and is used
in photographic films and plates.
NH 4 OH HCl
white fumes H 2O 67. (a) When steel is heated in presence of NH 3 , iron
nitride on the surface of steel is formed which imparts
NH 4 Cl a hard coating. This process is called nitriding.
Dense white fumes

ZnCl 2 2 NaOH Zn(OH) 2 2NaCl 68. (d) Cyanide process is used in the metallurgy of
Ag
̸» ¼ó¿²¼ ºóÞ´±½µ Û´»³»²¬­ ïçç
lanthanide series is very small, thus their chemical
2Ag 2S 8 NaCN O 2 2 H 2 O properties are similar.
4 Na[Ag(CN ) 2 ] 4 NaOH 2S 76. (b) Lesser energy difference between 5f and 6d
orbitals than between 4f and 5d orbitals result in
2 Na[ Ag (CN ) 2 ] Zn larger no. of oxidation state.
Na 2 [ Zn (CN ) 4 ] 2 Ag 77. (c) Lanthanides are 4 f-series elements starting
from cerium (Z= 58) to lutetium ( Z = 71). These
69. (b) 2CuSO 4 K 4 [ Fe (CN ) 6 ] are placed in the sixth period and in third group.
78. (c) In lanthanide series there is a regular
Cu 2 [Fe(CN) 6 ] 2K 2SO 4 decrease in the atomic as well as ionic radii of
Chocolate ppt. trivalent ions (M3+) as the atomic number
70. (b) Minimum or comparable energy gap increases. Although the atomic radii do show
between 5f, 6d and 7s subshell makes electron some irregularities but ionic radii decreases from
excitation easier, hence there is a greater range La(103 pm) to Lu (86pm).
79. (a) The Lanthanides are transition metals from
of oxidation states in actinoids.
atomic numbers 58 (Ce) to 71(Lu).
71. (d) Eu (63) = [Xe] 4f7 6s2
Hence the electron configulation becomes :
Gd (64) = [Xe] 4f7 5d1 6s2 (n –2) f 1– 14 (n – 1) s2p6 d0 – 1 ns2.
Tb (65) = [Xe] 4f9 6s2
72. (b) Due to lanthanoid contraction atomic radii 80. (a) La (OH)3 is more basic than Li (OH)3. In
of Zr and Hf is almost similar. lanthanides the basic character of hydroxides
73. (c) Gd (64) = [Xe]4f7 5d1 6s2 decreases as the ionic radius decreases.
74. (c) Ac (89) = [Rn] [6d1] [7s2] 81. (c) We know that regular decrease in the size of
75. (b) There is a steady decrease in the radii as the atoms and ions is called lanthanide
the atomic number of the lanthanide elements contraction. In vertical column of transition
increases. For every additional proton added in elements there is a very small change in size and
some times size is found same from second
nucleus the corresponding electron goes to 4f
member to third member.The similarity in size of
subshell.
the atoms of Zr and Hf is evident due to the
The shape of f -orbitals is very much diffused
object of lanthanide contraction. Therefore Zr
and they have poor shielding effect. The and Hf both have same radius 160 pm.
effective nuclear charge increases which causes 82. (d) Actinides have variable valency due to very
the contraction in the size of electron charge small difference in energies of 5f, 6d and 7s
cloud. This contraction in size is quite regular orbitals. Actinides are the elements from atomic
and known as Lanthanoid contraction. number 89 to 103.
Since the change in the ionic radii in the 83. (b) Pm is obtained by synthetic method.
EBD_7324
îðð ÝØÛÓ×ÍÌÎÇ

ݸ¿°¬»®

23 Coordination Compounds

̱°·½ ïæ ݱ±®¼·²¿¬·±² Ò«³¾»®ô (d) Difluorobis-(ethylene diamine) chromium


Ò±³»²½´¿¬«®» ¿²¼ ×­±³»®·­³ ±º (III) chloride
ݱ±®¼·²¿¬·±² ݱ³°±«²¼­ 7. In a particular isomer of [Co(NH3)4Cl2]0, the
Cl-Co-Cl angle is 90°, the isomer is known as
1. Cobalt (III) chloride forms several octahedral
[NEET Kar. 2013]
complexes with ammonia. Which of the
(a) Linkage isomer (b) Optical isomer
following will not give test of chloride ions with
silver nitrate at 25ºC ? [2015] (c) cis-isomer (d) Position isomer
(a) CoCl3·4NH3 (b) CoCl3·5NH3 8. The complexes [Co(NH3)6 ] [Cr(CN)6 ] and
(c) CoCl3·6NH3 (d) CoCl3·3NH3 [Cr(NH3)6] [Co(CN)6] are the examples of which
2. The sum of coordination number and oxidation type of isomerism? [2011]
number of the metal M in the complex (a) Linkage isomerism
[M(en)2(C2O4)]Cl (where en is ethylenediamine) is:
(b) Ionization isomerism
(a) 9 (b) 6 [2015 RS]
(c) 7 (d) 8 (c) Coordination isomerism
3. The name of complex ion, [Fe(CN)6]3– is : (d) Geometrical isomerism
(a) Hexacyanoiron (III) ion [2015 RS] 9. The complex, [Pt(Py)(NH3)BrCl] will have how
(b) Hexacyanitoferrate (III) ion many geometrical isomers ? [2011]
(c) Tricyanoferrate (III) ion
(a) 3 (b) 4
(d) Hexacyanidoferrate (III) ion
4. Number of possible isomers for the complex (c) 0 (d) 2
[Co(en)2Cl2]Cl will be (en = ethylenediamine) 10. The existence of two different coloured
(a) 2 (b) 1 [2015 RS] complexes with the composition of
(c) 3 (d) 4 [Co(NH3 )4 Cl2 ]+ is due to : [2010]
5. An excess of AgNO3 is added to 100 mL of a
(a) linkage isomerism
0.01 M solution of dichlorotetraaquachromium
(b) geometrical isomerism
(III) chloride. The number of moles of AgCl (c) coordination isomerism
precipitated would be : [NEET 2013] (d) ionization isomerism
(a) 0.002 (b) 0.003 11. Which one of the following complexes is not
(c) 0.01 (d) 0.001 expected to exhibit isomerism? [2010]
6. The correct IUPAC name for [CrF2(en)2]Cl is: 2
[NEET Kar. 2013] (a) Ni(en)3
(a) Chlorodifluoridobis (ethylene diamine) 2
chromium (III) (b) Ni NH3 4
H2O 2
(b) Chlorodifluoridoethylenediaminech-
romium (III) chloride (c) Pt NH3 2 Cl 2
(c) Difluoridobis (ethylene diamine) chromium
(III) chloride (d) Ni NH 3 2 Cl 2
ݱ±®¼·²¿¬·±² ݱ³°±«²¼­ îðï

12. Which of the following does not show optical (a) [MA5B] (b) [MA2B4]
isomerism? [2009] (c) [MA3B3] (d) [MA4B2]
(a) [Co(NH3)3Cl3]0 19. According to IUPAC nomenclature sodium
(b) [Co (en) Cl2 (NH3)2]+ nitroprusside is named as [2003]
(c) [Co (en)3]3+ (a) Sodium pentacyanonitrosyl ferrate (III)
(d) [Co (en)2Cl2]+ (b) Sodium nitroferrocyanide
(en = ethylenediamine) (c) Sodium nitroferrocyanide
13. Which of the following will give a pair of (d) Sodium pentacyanonitrosyl ferrate (II)
enantiomorphs? [2007] 20. The hypothetical complex chlorodiaquatriammine
(a) [Cr(NH3)6][Co(CN)6] cobalt (III) chloride can be represented as [2002]
(b) [Co(en)2Cl2]Cl (a) [CoCl(NH3)3 (H2O)2]Cl2
(c) [Pt(NH3)4] [PtCl6] (b) [Co(NH3)3 (H2O)Cl3]
(d) [Co(NH3)4Cl2]NO2. (en =NH2CH2CH2NH2) (c) [Co(NH2)3 (H2O)2 Cl]
14. [Co(NH3)4 (NO2)2] Cl exhibits [2006] (d) [Co(NH3)3 (H2O)3]Cl3
(a) linkage isomerism, ionization isomerism 21. Which of the following will give maximum
and geometrical isomerism number of isomers? [2001]
(b) ionization isomerism, geometrical isomerism (a) [Ni (C2O4) (en)2]2–
and optical isomerism (b) [Ni (en) (NH3)4]2+
(c) linkage isomerism, geometrical isomerism (c) [Cr (SCN)2 (NH3)4]+
and optical isomerism (d) [Co (NH3)4 Cl2]
(d) linkage isomerism, ionization isomerism 22. Which of the following will exhibit maximum
and optical isomerism ionic conductivity? [2001]
15. Which one of the following is expected to (a) K4[Fe(CN)6] (b) [Co(NH3)6] Cl3
exhibit optical isomerism? (c) [Cu(CN3)4]Cl2 (d) [Ni(CO)4]
(en = ethylenediamine) [2005] 23. Oxidation number of Ni in [Ni(C 2O4)3]4– is
(a) cis [Pt(NH3)2 Cl2] (a) 3 (b) 4 [2001]
(b) trans [Pt(NH3)2Cl 2] (c) 2 (d) 6
(c) cis [Co(en)2Cl 2] 24. Which one of the following complexes will have
(d) trans [Co(en)2Cl 2] four different isomers ? [2000]
16. Which of the following is considered to be an
anticancer species ? [2004] (a) [Co(en ) 2 Cl 2 ]Cl
(b) [Co(en )( NH 3 )Cl 2 ]Cl
Cl CH2 Cl Cl
(a) CH2 (b) (c) [Co( PPH 3 ) 2 Cl 2 ]Cl
Pt Pt
Cl Cl Cl Cl (d) [Co(en )3 ]Cl 3
25. In which of the following compounds does iron
H3N Cl H3N Cl exhibit zero oxidation state? [1999]
(c) Pt (d) Pt (a) [Fe(H2O)6] (NO3)3 (b) K3[Fe(CN)6]
H3N Cl Cl NH3 (c) K4[Fe(CN)6] (d) [Fe(CO)5]
17. Which of the following coordination compounds 26. The total number of possible isomers for the
would exhibit optical isomerism? [2004] complex compound [CuII (NH3)4] [PtII Cl4]
(a) pentamminenitrocobalt(III) iodide (a) 3 (b) 6 [1998]
(b) diamminedichloroplatinum(II) (c) 5 (d) 4
(c) trans-dicyanobis (ethylenediamine) 27. IUPAC name of [Pt(NH3)3 (Br) (NO2) Cl] Cl is
chromium (III) chloride [1998]
(d) tris-(ethylendiamine) cobalt (III) bromide (a) Triamminechlorobromonitroplatinum (IV)
18. Which one of the following octahedral chloride
complexes will not show geometric isomerism? (b) Triamminebromonitrochloroplatinum (IV)
( A and B are monodentate ligands) [2003] chloride
EBD_7324
îðî ÝØÛÓ×ÍÌÎÇ
(c) Triamminebromochloronitroplatinum (IV) 35. An example of double salt is [1989]
chloride
(a) Bleaching powder (b) K 4 [Fe(CN) 6 ]
(d) Triamminenitrochlorobromoplatinum (IV)
chloride (c) Hypo (d) Potash alum
28. A coordination complex compound of cobalt has ̱°·½ îæ Ó¿¹²»¬·½ Ó±³»²¬ô Ê¿´»²½» Þ±²¼
the molecular formula containing five ammonia
̸»±®§ ¿²¼ Ý®§­¬¿´ Ú·»´¼ ̸»±®§
molecules, one nitro group and two chlorine
atoms for one cobalt atom. One mole of this 36. HgCl2 and I2 both when dissolved in water
compound produces three mole ions in an containing I ions the pair of species formed is :
aqueous solution. On reacting this solution with (a) HgI2, I– (b) HgI24 , I3 [2017]
excess of AgNO3 solution, we get two moles of
AgCl precipitate. The ionic formula for this (c) Hg2I2, I– (d) HgI2 , I3
complex would be [1998] 37. The correct order of the stoichiometries of AgCl
(a) [Co(NH3)4 (NO2) Cl] [(NH3) Cl] formed when AgNO3 in excess is treated with
(b) [Co (NH3)5 Cl] [Cl (NO2)] the complexes : CoCl 3 .6NH3, CoCl 3.5NH3 ,
(c) [Co (NH3)5 (NO2)] Cl2 CoCl3.4NH3 respectively is :- [2017]
(d) [Co (NH3)5] [(NO2)2Cl2] (a) 3 AgCl, 1 AgCl, 2 AgCl
29. The formula for the complex, dichlorobis (urea) (b) 3 AgCl, 2 AgCl, 1 AgCl
copper (II) is [1997] (c) 2 AgCl, 3 AgCl, 1 AgCl
(a) [Cu{O = C (NH2)2}] Cl2
(d) 1 AgCl, 3 AgCl, 2 AgCl
(b) [Cu{O = C (NH2)2}Cl]Cl
38. Correct increasing order for the wavelengths of
(c) [CuCl2 {O = C(NH2)2}2]
(d) [CuCl2] [{O = C (NH2)2}]2 absorption in the visible region the complexes
30. The number of geometrical isomers of the of Co3+ is :- [2017]
complex [Co(NO2)3 (NH3)3] is [1997] (a) [Co(H2O)6]3+, [Co(en)3]3+, [Co(NH3)6]3+
(a) 2 (b) 3 (b) [Co(H2O)6]3+, [Co(NH3)6]3+, [Co(en)3]3+
(c) 4 (d) zero (c) [Co(NH3)6]3+, [Co(en)3]3+, [Co(H2O)6]3+
31. Among the following, the compound that is both (d) [Co(en)3]3+, [Co(NH3)6]3+, [Co(H2O)6]3+
paramagnetic and coloured, is [1996] 39. Pick out the correct statement with respect to
(a) KMnO4 (b) CuF2 [Mn(CN)6]3- [2017]
(c) K4[Fe(CN)6] (d) K2Cr2O7 (a) It is sp3d2 hybridised and tetrahedral
32. The number of geometrical isomers for (b) It is d2sp3 hybridised and octahedral
[Pt (NH3)2 Cl2] is [1995] (c) It is dsp2 hybridised and square planar
(a) 2 (b) 1 (d) It is sp3d2 hybridised and octahedral
(c) 3 (d) 4 40. Which of these statements about [Co(CN)6]3–
33. K 3[Al(C 2 O 4 ) 3 ] is called [1994] is true ? [2015]
(a) [Co(CN)6]3– has four unpaired electrons
(a) Potassium alumino oxalate
and will be in a low-spin configuration.
(b) Potassium trioxalateoaluminate (III)
(b) [Co(CN)6]3– has four unpaired electrons
(c) Potassium aluminium (III) oxalate
and will be in a high spin configuration.
(d) Potassium trioxalato aluminate (VI) (c) [Co(CN)6]3– has no unpaired electrons and
34. Among the following complexes, optical activity will be in a high-spin configuration.
is possible in [1994] (d) [Co(CN)6]3– has no unpaired electrons and
(a) [Co( NH 3 ) 6 ]3 will be in a low-spin configuration.
(b) [ Co ( H 2 O ) 2 ( NH 3 ) 2 Cl 2 ] 41. Among the following complexes the one which
shows zero crystal field stabilization energy
(c) [Cr(H 2 O) 2 Cl 2 ] (CFSE): [2014]
(d) [Co(CN) 5 NC] (a) [Mn(H2O)6]3+ (b) [Fe(H2O)6]3+
(c) [Co(H2O)6]2+ (d) [Co(H2O)6]3+
ݱ±®¼·²¿¬·±² ݱ³°±«²¼­ îðí

42. Which of the following complexes is used as an 2 2


anti-cancer agent: [2014] (c) o 2P (d) o P
5 5
(a) mer-[Co(NH3)3Cl3] (b) cis-[PtCl2(NH3)2] ( o= Crystal Field Splitting Energy in an
(c) cis-K2[PtCl2Br2] (d) Na2CoCl4 octahedral field, P = Electron pairing energy)
43. A magnetic moment of 1.73 BM will be shown
51. Of the following complex ions, which is
by one among the following : [NEET 2013]
diamagnetic in nature ? [2011]
(a) [Ni(CN)4]2– (b) TiCl4
(c) [CoCl6]4– (d) [Cu(NH3)4]2+ (a) [NiCl4]2– (b) [Ni(CN)4]2–
44. Crystal field splitting energy for high spin d4 (c) [CuCl4]2– (d) [CoF6]3–
octahedral complex is: [NEET Kar. 2013] 52. The d-electron configurations of Cr 2+, Mn2+,
(a) – 1.6 0 (b) – 1.2 0 Fe2+ and Co2+ are d4, d5, d6 and d7, respectively.
(c) – 0.6 0 (d) – 0.8 0 Which one of the following will exhibit minimum
45. Which is diamagnetic? [NEET Kar. 2013] paramagnetic behaviour? [2011]
(a) [Fe(CN)6]3– (b) [Co(F6)]3– (a) [Mn(H2O)6]2+ (b) [Fe(H2O)6]2+
(c) [Ni(CN)4]2– (d) [NiCl4]2– (c) [Co(H2O)6]2+ (d) [Cr(H2O)6]2+
46. The anion of acetylacetone (acac) forms (At, nos. Cr = 24, Mn = 25, Fe = 26, Co = 27)
Co(acac)3 chelate with Co3+. The rings of the 53. Which of the following complex compounds will
chelate are [NEET Kar. 2013] exhibit highest paramagnetic behaviour?
(a) three membered (b) five membered [2011M]
(c) four membered (d) six membered (At. No. : Ti = 22, Cr = 24, Co = 27, Zn = 30)
47. Which among the following is a paramagnetic (a) [Ti (NH3)6]3+ (b) [Cr (NH3)6]3+
complex? [NEET Kar. 2013] (c) [Co (NH3)6] 3+ (d) [Zn (NH3)6]2+
(a) Mo(CO)6 (b) [Co(NH3)6]3+ 54. Which of the following complex ion is not
(c) [Pt(en)Cl2] (d) [CoBr4]2– expected to absorb visible light ? [2010]
(At. No. of Mo = 42, Pt = 78) 2 3
48. Which one of the following is an outer orbital (a) Ni(CN)4 (b) Cr(NH3 )6
complex and exhibits paramagnetic behaviour ?
(c) Fe(H 2 O)6 2 (d) Ni(H 2O)6 2
[2012]
(a) [Ni(NH3)6]2+ (b) [Zn(NH3)6)]2+ 55. Crystal field stabilization energy for high spin
(c) [Cr(NH3)6]3+ (d) [Co(NH3)6]3+ d 4 octahedral complex is: [2010]
49. Red precipitate is obtained when ethanol
(a) – 1.8 0 (b) – 1.6 0 + P
solution of dimethylglyoxime is added to
ammoniacal Ni(II). Which of the following (c) – 1.2 0 (d) – 0.6 0
statements is not true ? [2012 M] 56. Which of the following complex ions is expected
(a) Red complex has a square planar geometry. to absorb visible light? [2009]
(b) Complex has symmetrical H-bonding (a) [Ti (en)2(NH3)2]4 +
(c) Red complex has a tetrahedral geometry. (b) [Cr (NH3)6]3 +
(d) Dimethylglyoxime functions as bidentate (c) [Zn (NH3)6]2 +
ligand. (d) [Sc (H2O)3 (NH3)3]3+
(At. no. Zn = 30, Sc = 21, Ti = 22, Cr = 24)
OH 57. Which of the following complexes exhibits the
H3C C N
dimethylglyoxime = highest paramagnetic behaviour ? [2008]
H3C C N (a) [V(gly)2(OH)2(NH3)2]+
OH (b) [Fe(en)(bpy)(NH3)2]2+
50. Low spin complex of d6-cation in an octahedral (c) [Co(ox)2(OH)2]–
field will have the following energy : [2012 M] (d) [Ti(NH3)6]3+
12 12 where gly = glycine, en = ethylenediamine and
(a) o P (b) o 3P bpy = bipyridyl moities)
5 5
(At.nosTi = 22, V = 23, Fe = 26, Co = 27)
EBD_7324
îðì ÝØÛÓ×ÍÌÎÇ
58. In which of the following coordination entities 65. The number of unpaired electrons in the complex
the magnitude o (CFSE in octahedral field) will ion [CoF6]3– is (Atomic no.: Co = 27) [2003]
be maximum? [2008] (a) Zero (b) 2
(a) [Co(H2O)6]3+ (b) [Co(NH3)6]3+ (c) 3 (d) 4
(c) [Co(CN)6]3– (d) [Co (C2O4)3]3– 66. Atomic number of Cr and Fe are respectively 25
(At. No. Co = 27) and 26, which of the following is paramagnetic?
59. The d electron configurations of Cr 2+, Mn2+, [2002]
Fe 2+ and Ni 2+ are 3d 4 , 3d 5 , 3d 6 and 3d 8 (a) [Cr(CO)6] (b) [Fe(CO)5]
respectively. Which one of the following aqua (c) [Fe(CN)6]–4 (d) [Cr(NH3)6]+3
complexes will exhibit the minimum paramagnetic 67. CuSO4 when reacts with KCN forms CuCN,
behaviour? [2007] which is insoluble in water. It is soluble in excess
(a) [Fe(H2O)6]2+ (b) [Ni(H2O)6]2+ of KCN due to formation of the following complex
(c) [Cr(H2O)6]2+ (d) [Mn(H2O)6]2+ [2002]
(At. No. Cr = 24, Mn = 25, Fe = 26, Ni = 28) (a) K2[Cu(CN)4] (b) K3[Cu(CN)4]
60. [Cr(H2O)6]Cl3 (at no. of Cr = 24) has a magnetic (c) CuCN2 (d) Cu[K Cu(CN)4]
moment of 3.83 B. M. The correct distribution of 68. Which statement is incorrect? [2001]
3d electrons in the chromium of the complex is (a) Ni(CO)4 – Tetrahedral, paramagnetic
1 [2006] (b) [Ni(CN)4]2– – Square planar, diamagnetic
(a) 3d xy1, 3d x 2 y 2 , 3dyz1 (c) Ni(CO)4 – Tetrahedral, diamagnetic
(d) [NiCl4]2– – Tetrahedral, paramagnetic
(b) 3dxy1, 3dyz1, 3dxz1 69. Which one of the following will show
(c) 3dxy1, 3dyz1, 3d1z 2 paramagnetism corresponding to 2 unpaired
electrons?(Atomic numbers : Ni = 28, Fe = 26)
(d) (3dx2 – y2)1, 3d z 2 , 3dxz1 (a) [Fe F6]3– (b) [Ni Cl4]2– [1999]
61. Which one of the following is an inner orbital (c) [Fe (CN)6]3– (d) [Ni (CN)4]2–
complex as well as diamagnetic in behaviour? 70. The number of unpaired electrons in the complex
(Atomic number: Zn = 30, Cr = 24, Co = 27, [Cr(NH3)6]Br3 is (Atomic number Cr = 24)
Ni = 28) [2005] (a) 4 (b) 1 [1999]
(a) [Zn(NH3)6]2+ (b) [Cr(NH3)6]3+ (c) 2 (d) 3
(c) [Co(NH3)6]3+ (d) [Ni(NH3)6]2+ 71. Which of the following statements is correct ?
(Atomic number of Ni = 28) [1997]
62. Among [ Ni(CO) 4 ], [ Ni(CN) 4 ]2 , [ NiCl 4 ]2 (a) Ni(CO)4 is diamagnetic and [NiCl4]2– and
species, the hybridization states of the Ni atom [Ni(CN)4]2– are paramagnetic
are, respectively(At. No. of Ni = 28) [2004] (b) Ni(CO)4and [Ni(CN)4]2– are diamagnetic
and [NiCl4]2– is paramagnetic
(a) sp 3 , dsp 2 , dsp 2 (b) sp 3 , dsp 2 , sp 3
(c) Ni(CO)4and [NiCl4]2–are diamagnetic and
(c) sp 3 , sp 3 , dsp 2 (d) dsp 2 , sp 3 , sp 3 [Ni(CN)4]2– is paramagnetic
(d) [NiCl4]2– and [Ni(CN)4]2– are diamagnetic
63. CN– is a strong field ligand. This is due to the
and Ni(CO)4 is paramagnetic
fact that [2004]
72. Which of the following is common donor atom
(a) it carries negative charge
in ligands? [1995]
(b) it is a pseudohalide
(a) arsenic (b) nitrogen
(c) it can accept electrons from metal species
(c) oxygen (d) both 'b' and 'c'
(d) it forms high spin complexes with metal
73. The complex ion [Co(NH3)6]3+ is formed by
species sp3 d2 hybridisation. Hence the ion should
64. Considering H 2 O as a weak field ligand, the possess
number of unpaired electrons in [ Mn ( H 2 O ) 6 ]2 (a) Octahedral geometry [1990]
will be (At. no. of Mn = 25) [2004] (b) Tetrahedral geometry
(a) three (b) five (c) Square planar geometry
(c) two (d) four (d) Tetragonal geometry.
ݱ±®¼·²¿¬·±² ݱ³°±«²¼­ îðë

̱°·½ íæ Ñ®¹¿²±³»¬¿´´·½ ݱ³°±«²¼­ 78. Among the following, which is not the -bonded
organometallic compound? [2003]
74. An example of a sigma bonded organometallic (a) (CH 3 ) 4 Sn
compound is : [2017] (b) K [PtCl 3 ( 2 C 2 H 4 )]
(a) Grignard's reagent (b) Ferrocene 5
(c) Fe ( C5 H 5 ) 2
(c) Cobaltocene (d) Ruthenocene
(d) Cr ( 6
75. Which of the following has longest C–O bond C6H 6 )2
length? (Free C–O bond length in CO is 1.128Å) 79. Which of the following organometallic
[2016] compound is and bonded? [2001]
(a) Ni(CO)4 (b) [Co(CO)4]– (a) [Fe ( 5 – C5H5)2]
(c) [Fe(CO)4]2– (d) [Mn(CO)6]+ (b) Fe (CH3)3
76. Which of the following carbonyls will have the (c) K [PtCl3( 2 – C2H4)]
strongest C – O bond ? [2011 M] (d) [Co(CO)5 NH3]2+
(a) Mn (CO)6+ (b) Cr (CO)6 80. Which of the following may be considered to be
(c) V (CO)6– (d) Fe (CO)5 an organometallic compound? [1996]
77. Which of the following does not have a metal- (a) Nickel tetracarbonyl
carbon bond? [2004] (b) Chlorophyll
(c) K3 [Fe (C2O4)3]
(a) Al(OC 2 H 5 ) 3 (b) C 2 H 5MgBr (d) [Co (en)3] Cl3
(c) K[Pt (C 2 H 4 )Cl 3 ] (d) Ni(CO) 4

ANSWER KEY
1 (d) 9 (a) 17 (d) 25 (d) 33 (b) 41 (b) 49 (c) 57 (c) 65 (d) 73 (a)
2 (a) 10 (b) 18 (a) 26 (d) 34 (b) 42 (b) 50 (b) 58 (c) 66 (d) 74 (a)
3 (d) 11 (d) 19 (a) 27 (c) 35 (d) 43 (d) 51 (b) 59 (b) 67 (b) 75 (c)
4 (c) 12 (a) 20 (a) 28 (c) 36 (b) 44 (c) 52 (c) 60 (b) 68 (a) 76 (a)
5 (d) 13 (b) 21 (c) 29 (c) 37 (b) 45 (c) 53 (b) 61 (c) 69 (b) 77 (a)
6 (c) 14 (a) 22 (a) 30 (a) 38 (d) 46 (d) 54 (a) 62 (b)
70 (d) 78 (a)
7 (c) 15 (c) 23 (c) 31 (b) 39 (b) 47 (d) 55 (d) 63 (b) 71 (b) 79 (d)
8 (c) 16 (c) 24 (b) 32 (a) 40 (d) 48 (a) 56 (b) 64 (b ) 72 (d) 80 (b)
EBD_7324
îðê ÝØÛÓ×ÍÌÎÇ

Hints & Solutions


1. (d) CoCl3 . 3NH3 will not give test for chloride 5. (d) [Cr(H2O)4Cl2] Cl + AgNO3 AgCl +
ions with silver nitrate due to absence of [Cr(H2O)4Cl2]NO3
ionisable chloride atoms.
wt 1000
CoCl3 . 3NH3 [Co(NH3)3 Cl3] Molarity =
mol. mass vol.
[Co(NH3)3 Cl3] AgNO3 no ppt wt vol. 0.01 100
molarity
2. (a) [M (en)2 (C2O4)] Cl mol. mass 1000 1000
C2O4 = bidentate ligand, carry – 2 charge = 0.001
en = bidentate ligand, carry 0 charge 6. (c) IUPAC name of [CrF2 (en)2]Cl is
M carry+ 3 charge; coordination number = 6 Difluoridobis(ethylenediamine) chromium (III)
Sum = + 3 + 6 = 9 chloride.
3. (d) Hexacyanidoterrate (III) ion. Cl
90°
4. (c) [Co (en)2 Cl2] Cl Cl NH3
Cl N 7. (c) Co
NH3
N N N N NH3
Co and Co NH3
N N N Cl cis-isomer
Cl Cl 8. (c) Coordination isomerism occurs when
trans cis cationic and anionic complexes of different metal
(I) (II) ions are present in a salt. Interchange of ligand
Mirror image of (II) between the complexes give isomers e.g.
N [Co (NH3)6] [Cr (CN)6] is an isomer of [Co
N
(CN)6] [Cr (NH3)6]
N
Co
Cl N
Cl

9. (a) Complexes of the type MABCD may exist in three isomeric forms.
A B A C A C

M M M

D C D B B D
(I) (II) (III)
Similarly, [Pt (Py) (NH3) BrCl] may exist in three isomeric form in which
M = Pt, A = Py, B = NH3, C = Br, D = Cl.

10. (b)
ݱ±®¼·²¿¬·±² ݱ³°±«²¼­ îðé

trans (violet) NO2 NO2


Cl +
H3N Cl H3 N NH3 H3N NO2
CO CO
Co
NH3 NH3 NH3 NH3

H3N NH3 NO2 NH3


NH3
cis (green) Trans-form cis-form
Geometrical isomers
11. (d) In Ni NH 3 2 Cl 2 , Ni 2 is in sp 3 15. (c)
hybridisation, thus tetrahedral in shape. Hence en + en +
the four ligands are not different to exhibit optical Cl Cl
isomerism.
Co Co
Correct choice : (d)
12. (a) The octahedral coordination compounds of
Cl Cl
the type MA3B3 exhibit fac-mer isomerism. en
en
NH3 NH3
Cis-d-isomer Cis-l-isomer
Cl NH3 NH3 Cl
Mirror
Co Co Transform of [M(AA) 2 a 2]n± does not shows
optical isomerism.
Cl NH3 NH3 Cl
16. (c) Diaminodichloroplatinum (II) commonly
Cl Cl known as cis - platin is found to have anticancer
fac. mer.
13. (b) Non superimposable mirror images are property.
called optical isomers and may be described as 17. (d) The optical isomers are pair of molecules
“chiral’. They are also called enantiomers and which are non super imposable mirror images of
rotate plan e polarised light in opposite each other.
directions. 3+ N en 3+
en – N
N N
N N Co
N N N en
N Co en
N N
N Co Cl Cl Co N N N
N en
N N en
Cl Cl N
d form l form
14. (a) The given compound may have linkage Enantiomers
isomerism due to presence of NO2 group which The two optically active isomers are collectively
may be in the form –NO2 or –ONO. called enantiomers.
It may have ionisation isomerism due to presence 18. (a) MA3 B3 – 2 geometrical isomers
of two ionisable group –NO2 & –Cl. It may have MA2 B4 – 2 geometrical isomers
geometrical isomerism in the form of cis-trans MA4 B2 – 2 geometrical isomers
form as follows : The complexes of general formula Ma 6 and Ma5b
[Co(NH3)4Cl(NO2)]NO2 & octahedral geometry do not show geometrical
isomerism.
[Co(NH3) (NO2)2]Cl ––– ionisation isomers.
19. (a) IUPAC name of sodium nitroprusside
[Co(NH3)5(NO2)2]Cl & [Co(NH3)5(ONO)2Cl Na 2 [Fe(CN) 5 NO] is sodium pentacyanoni-
––– Linkage isomers trosoyl ferrate (III) because in it NO is neutral
EBD_7324
îðè ÝØÛÓ×ÍÌÎÇ
ligand. Hence [Cu ( NH 3 )Cl 3 ] [Pt ( NH 3 ) 3 Cl] ,
2×O.N. of Na + O.N. of Fe + 5×O.N. of CN
1×O.N. of NO = 0 [CuCl 4 ][Pt ( NH 3 ) 4 ]
2×(+1) + O.N. of Fe + 5 ×(–1) +1×0 = 0
and Cu(NH3 )4 PtCl4 .
O.N. of Fe = 5 – 2 = +3, Hence ferrate (III)
20. (a) Chlorodiaquatriammine cobalt (III) chloride The isomer [Cu (NH3)2 Cl2][Pt (NH3)2 Cl2] does
not exist as both the parts are neutral.
is [CoCl( NH 3 )3 (H 2O) 2 ]Cl 2 27. (c) We know that IUPAC name of
[Pt(NH3)3 (Br) (NO2)Cl]Cl is triammine-
21. (c) [Cr(SCN ) 2 ( NH 3 ) 4 ] shows linkage, bromochloronitroplatinum (IV) chloride.
geometrical and optical isomerism. Hence 28. (c) As it forms two moles of silver chloride thus
produces maximum no. of isomers. it has two moles of ionisable Cl.
22. (a) The complex ions which give maximum ion [Co(NH3 )5 NO 2 ]Cl 2 [Co(NH 3 )5 NO 2 ]2 2Cl
in solution exhibit maximum ionic conductivity
2Cl 2AgNO3 2AgCl 2NO 3
K 4 [ Fe(CN) 6 ] [ Fe (CN) 6 ] 4
4K 5 ions 29. (c) [CuCl2{(O = C(NH2)2}2]
30. (a) [Co( NO 2 )3 ( NH 3 )3 ]
[Co( NH 3 ) 6 ]Cl 3 [Co( NH 3 ) 6 ]3 3 Cl 4 ions Possible geometrical isomers
H3N NH3 O2N NH3
[Cu ( NH 3 ) 4 ]Cl 2 [Cu ( NH 3 ) 4 ]2 2Cl 3 ions
Co Co
[ Ni(CO) 4 ] No ions
O2N NO2 NH3 NO2
23. (c) O.N. of Ni in N i(C 2 O 4 ) 3 4
cis trans
Total geometrical isomers = 2
x 3( 2) 4 x 4 6 2 31. (b) CuF2 is both paramagnetic and coloured
due to unpaired electron in d-orbital (3d94s0).
24. (b) Complex Co(en)(NH3)2Cl2]Cl will have four
32. (a) Pt (NH3)2 Cl2 is a disubstituted complex and
different isomers.
shows only cis-& trans-isomers
(i) Geometrical isomers NH3 Cl
Cl NH 3
+ +
NH3 Cl
Cl NH3 Pt Pt
en Co en Co
Cl NH3 Cl NH3
Cl NH3
NH3 Cl cis trans
cis-form trans-form 33. (b) IUPAC name is potassium trioxalato-
(ii) Optical isomers aluminate (III).
+ + 34. (b) It is optically active when two Cl atoms are
NH3
NH3
NH3
Cl in cis position.
en Co en Co 35. (d) Potash alum K 2SO 4 .Al 2 (SO 4 )3 24H 2O
Cl NH3 is a double salt.
Cl Cl
cis-form trans-form 36 (b) In a solution containing HgCl2, I2 and I–,
25. (d) In [Fe(CO)5] iron exist in zero oxidation state. both HgCl2 and I2 compete for I–.
26. (d) The total number of isomers for the complex Since formation constant of [HgI 4]2– is very
compound large (1.9 × 10 30) as compared with I 3
(Kf = 700)
[Cu II ( NH 3 ) 4 ][Pt II Cl 4 ] is four.. I– will preferentially combine with HgCl2.
These four isomers are HgCl2 + 2I– HgI2 + 2Cl–
Red ppt
[Cu ( NH 3 )3 Cl] [Pt ( NH 3 )Cl 3 ],
ݱ±®¼·²¿¬·±² ݱ³°±«²¼­ îðç

HgI2 2I HgI 4
2 41. (b) Due to d5 configuration CFSE is zero.
soluble 42. (b)
AgNO3 43. (d) [Cu(NH3)4]2+ hybridisation dsp2
37. (b) Co NH3 6 Cl3 3mol of AgCl
Cu+2 – 3d9 has one unpaired e-
AgNO3
Co NH 3 5
Cl Cl2 2mol of AgCl

AgNO3
Co NH 3 4 Cl2 Cl 1mol of AgCl So magnetic moment
Complexes are respectively [Co(NH 3 )6 Cl 3 , = n n 2 = 11 2
[Co(NH3)5Cl]Cl2 and [Co(NH3)4Cl2]Cl
38. (d) The order of the ligand in the spectro- = 3 = 1.73
chemical series is : 44. (c) CFSE = (– 0.4x + 0.6y) 0
H2O < NH3 < en where
Hence, the wavelength of the light observed will x e–s in t2g orbital
be in the order y e–s in eg orbital
For high spin d4, complex x = 3 & y = 1
[Co(H2O)6]3+ < [Co(NH3)6]3+ < [Co(en)3]3+
– 0.4 × 3 + 0.6 × 1 = – 0.6 0
Thus, wavelength absorbed will be in the
opposite order 45. (c) Ni+2 3d 8 =
i.e., [Co(en)3]3+, [Co(NH3)6]3+, [Co(H2O)6]3+
CN– is a strong ligand and causes pairing of 3d
39. (b) In the complex [Mn(CN)6]3–, O.S. of Mn is + 3 electrons of Ni2+.
E.C. of Mn+3 3d4 It is diamagnetic.
3d 4s 4p 46. (d) Acetylacetone forms six membered stable
ring complexes.
The presence of a strong field ligand CN– causes 47. (d) Co2+ [Ar]3d74s0, here, Br– is a weak field
pairing of electrons. ligand so will not cause pairing of d-electrons
in Co2+.
[CoBr 4 ] 2– will exhibit paramagnetic
3d 4s 4p behaviour due to unpaired electrons.
d 2 sp3 48. (a) [Ni(NH3)6]2+
As, coordination number of Mn = 6, so it will
6 2
form an octahedral complex. Ni2+ = 3d8, according to CFT = t 2g eg therefore,
[Mn(CN)6]3– = hybridisation is sp 3 d 2 & complex is
paramagnetic and outer orbital.
[Ar] 49. (c) Nickel ions are frequently detected by the
3d 4s 4p formation of red precipitate of the complex of
d 2 sp3 nickel dimethylglyoxime, when heated with
40. (d) In [Co(CN)6]–3, O.N. of Co is +3 dimethylglyoxime.
Co+3 = 3d 6 4s0 CH3 C NOH

CN is a strong field ligand + Ni2+
CH3 C NOH
Pairing of electrons occurs so in this complex
no unpaired electron is present and it is low spin Dimethylglyoxime
complex.
Sum = + 3 + 6 = 9
EBD_7324
îïð ÝØÛÓ×ÍÌÎÇ
OH O 51. (b) Ni2+ = 3d8 4sº

CH3 C N N C CH3
Ni
CH3 C N N C CH3
Since, the coordination number of Ni in this
O OH complex is 4, the configuration of Ni2+ at first
sight shows that the complex is paramagnetic
Nickel dimethylglyoxime with two unpair ed electron. However,
50. (b) for d t2g6 eg0 (in low spin)
6, experiments show that th e complex is
diamagnetic. This is possible when the 3d
C.F.S.E = – 0.4 × 6 0 + 3P
electrons rearrange against the Hund’s rule as
12 shown below. This is in accordance with the
= o + 3P fact that the ligand involved here is strong i.e.,
5
CN– ion.

3d 4s 4p
Ni2+ (after rearrangement)

Hence, now dsp2 hybridization involving one 3d, one 4s and two 4p orbitals, takes place leading to
four dsp2 hybrid orbitals, each of which accepts four electron pairs from CN– ion forming
[Ni (CN)4]2– ion.
3d 4s 4p

[Ni (CN)4]2– ×× ×× ×× ×× ××

four dsp2 hybrid bonds


Thus, the complex is diamagnetic as it has no unpaired electron.

52. (c) Cr2+, d4 4 unpaired e s

Mn2+, d5 5 unpaired e s

Fe2+, d6 4 unpaired e s

Co2+, d7 3 unpaired e s

Minimum paramagnetic behaviour = [Co (H2O)6]2+


53. (b) Thus, the complex is paramagnetic.
(a) [Ti(NH3)6]3+ : 3d1 configuration and thus has (c) [Co(NH3)6]3+ : In this complex are cobalt
one unpaired electron. ion is in + 3 oxidation state with 3d 6
(b) [Cr(NH3)6]3+ : In this complex Cr is in +3 configuration.
oxidation state. Co3+,[Ar]3d6
Cr3+ : 3d3 4so 4s 4p

Cr3+ ion [Cr (NH3)6]3+ [Co(NH3)6]3+

CN CN CN CN CN CN NH3 NH3 NH3 NH3 NH3 NH3


ݱ±®¼·²¿¬·±² ݱ³°±«²¼­ îïï

(inner orbital or d2sp3 hybrid orbital low spin 58. (c) In octahedral field the crystal field splitting
complex) diamagnetic of d- orbitals of a metal ion depends upon the
(d) In this complex Zn exists as field produced by the ligands. In general ligands
Zn2+ ion can be arranged in a series in the order of
Zn2+ ion : 3d10 4s0 increasing fields and splittings which they
produce around a central metal ion. A portion of
the series is given below.
Zn2+ ion in [Zn(NH3)4]2+ cyanide > ethylene - diamine > ammonia >
pyridine>thiocyanate > water> fluoride > oxalate
> hydroxide> chloride> bromide > iodide.
Out of the given ligands water, ammonia, cyanide
NH3 NH3 NH3 NH3 and oxalate, we can find from the above series of
Due to presence of paired electrons complex is ligands that the maximum splitting will occur in
diamagnetic in nature. case of cyanide (CN–) i.e. the magnitude of o will
2 be maximum in case of [Co(CN)6]3+.
54. (a) Ni(CN)4 : Number of unpaired electrons 59. (b) Lesser is the number of unpaired electrons
=0 smaller will be the paramagnetic behaviour. As
3
Cr(NH3 )6 : Number of unpaired electrons = 3 Cr2+, Mn2+, Fe2+ & Ni2+ contains.
2
Fe(H 2 O)6 : Number of unpaired electrons = 4 Cr2+ (3d4) =
2
Ni(H 2O)6 : Number of unpaired electrons = 2 = 4 unpaired e–.
55. (d) d 4 in high spin octahedral complex
Mn2+ (3d5) =
eg –
= 5 unpaired e–.
t 2g
CFSE = [0.6 × 1] + [–0.4 × 3] = – 0.6 0 Fe2+ (3d6)=
56. (b) Since Cr 3+ in the complex has unpaired = 4 unpaired e–.
electrons in the d orbital, hence will be coloured
Ti = [Ar]3d2 4 s2 ; Ti4 + = 3d0 Ni2+ (3d8) =
Cr = [Ar] 3d5 4s1; Cr3+ = 3d3
Zn= [Ar] 3d10 4s2; Zn2+= 3d10 = 2 unpaired e–.
Sc = [Ar] 3d1 4s2; Sc3+ = 3d0 As Ni2+ has minimum no. of unpaired e– thus
57. (c) More the number of unpaired electrons this is least paramagnetic.
present in a complex more is its paramagnetic
behaviour. 60. (b) n(n 2)
To find unpaired electrons let us calculate the
oxidation states of elements in each complex and 3.83 n(n 2)
then write the electronic configuration for that on solving n = 3
oxidation state to find the number of unpaired As per magnetic moment, it has three
electrons in it. unpaired electron.
We find that in their complexes V, Fe, Co and Ti Cr3+ will have configuration as =
are in + 3, + 2, + 5, and + 3 oxidation states 1s2 2s2 2p6 3s2 3p6 3d3
respectively. In these state the number of
unpaired electrons present in them are 2, 4, 4
and 1 respectively. Since the maximum number
of unpaired electrons are in cobalt complex so it dxy dyz dzx
has highest paramagnetic behaviour i.e. choice
(c) is correct answer. So 3dxy1 3dyz1 3dxz1
EBD_7324
îïî ÝØÛÓ×ÍÌÎÇ
61. (c) [Co(NH3)6]3+ Co3+(27 –3 = 24)
3d 4s 4p
3
sp Hybridisation
2 3
d sp (inner octahedral Cl– is a weak ligand, hence no pairing of electrons
complex & diamagnetic) takes place
[Cr(NH3)6]3+ Cr3+(24 –3 = 21) 63. (b) CN– is a strong field ligand as it is a
psuedohalide ion. These ions are strong
3d 4s 4p coordinating ligands and hence have the
tendency to form -bond (from the pseudo
2 3 halide to the metal) and -bond (from the metal
d sp (inner octahedral to pseudo halide).
complex & paramagnetic)
2+ 2+
[Ni(NH3)6] Ni (28 – 2 = 26) 64. (b) Since H 2 O is a weak ligand, it will not cause
pairing of electrons in the metal ion Mn 2+. Thus
3d 4s 4p electronic configuration of the metal (Mn2+) in
the complex will be
3 2
sp d (outer octahedral Mn 2+ :1s2 2s2 p6 3s 2 3p6 3d5
complex & paramagnetic) i.e 5 unpaired electrons.
65. (d) Co here is in +3 oxidation state
[Zn(NH3)6]2+ Zn 2+(30 – 2 = 28)
3d 4s 4p
Co
3 2
sp d (outer octahedral Co3+
complex &diamagnetic)
62. (b) 28Ni :
Unpaired electrons = 4 and sp3d2 hybridisation
3d 4s 4p and octahedral shape.
66. (d) Cr3+ has 4sº 3d3 electronic configuration
with 3 unpaired electrons, hence paramagnetic.
Ni in Ni (CO)4 : In other cases pairing of d-electrons take place
in presence of strong field ligands such as CO
[Ne]
or CN–.
sp3 Hybridisation In Cr(CO)6 molecule 12 electrons are contributed
Remember CO is a strong ligand by CO group and it contain no odd electron
[Cr(CO)6]
Ni 2 :
3d 4s 4p
3d 4s 4p
2 2 ––– diamagnetic
Ni in [Ni(CN) 4 ]
[Fe(CO)5]

dsp2 Hybridisation 3d 4s 4p
Remember CN– is also a strong ligand
––– diamagnetic
Ni 2 in [ NiCl 4 ]2 [Fe(CN)6]–4

3d 4s 4p
––– diamagnetic
ݱ±®¼·²¿¬·±² ݱ³°±«²¼­ îïí

[Cr(NH3)6]+3 72. (d) In the formation of a coordinate bond, the


ligands donate a pair of electrons to the metal
atom. Futher nitrogen and oxygen has great
3d 4s 4p tendency to donate the pair of electrons in most
of the compounds. Therefore both nitrogen
––– Paramagnetic
and oxygen are common donor atoms in ligands.
67. (b) Copper sulphate react with KCN to give
73. (a) According to VSEPR theory, a molecule with
white ppt of Cu(CN)2 and cyanogen gas. The
six bond pairs must be octahedral.
insoluble copper cyanide dissolve in excess of 74. (a) Grignard's reagent (RMgX) is a -bonded
KCN and give soluble potassium cuprocyanide organometallic compound.
CuSO 4 2 KCN K 2SO 4 Cu (CN ) 2 75. (c) [Fe(CO)4]2–
2Cu(CN) 2 2CuCN CN CN
inso lub le cyanogen Since metal atom is carrying maximum –ve
CuCN 3KCN K 3 [Cu ( CN ) 4 ] charge therefore it would show maximum
So lub le synergic bonding as a resultant C—O bond
68. (a) (CO) carbonyl group being a strong ligand length would be maximum.
paired all electrons present in d orbital of Ni. 76. (a) As + ve charge on the central metal atom
Hence form dsp2 hybrid orbitals and hence the increases, the less readily the metal can donate
shape of molecule is square planar. electron density into the * orbitals of CO ligand
In Ni(CO) 4 complex, Ni will have 3d 10 to weaken the C – O bond. Hence, the C – O
configuration. It has tetrahedral geometry but bond would be strongest in Mn(CO)6+.
diamagnetic as there are no unpaired electrons. 77. (a) Triethoxyaluminium has no Al – C linkage
69. (b) As in [NiCl4]–2, chloride ion being a weak O CH 2 CH 3
ligand is not able to pair up the d–e–s. Al O CH 2 CH 3
70. (d) In [Cr (NH3)6]Br3, Cr is in +3 oxidation state etc.
O CH 2 CH 3
3d 4s 4p
78. (a) The number of carbon atom found in
Cr
24 bonded organometallic compounds is indicated
by the greek latter ' ' with a number. The prefixes
d2sp 3 hybridized 2, 5 and 6 indicate that 2, 5 and 6 carbon

3d 4s 4p atom are bound to the metal in the


Cr
+3
compound.(CH3)4Sn does not involve any pi ( )
21 bond formation. (CH3 )4 Sn is a bonded
Its ion is octahedral in nature. Due to the organometallic compound
presence of three unpaired electrons it is
paramagnetic. 79. (d) [Co(CO) 5 NH 3 ]2 . In this complex. Co-
atom attached with NH3 through bonding
71. (b) while with CO it is attached with dative -bond.
Atom/Ion Configuration
No. of
unpaired Magnetic
80. (b) Organometallic compounds are those
Complex nature
electrons compounds in which there is a bond which
3d 4s 4p
2+ 8 involve metal-carbon bond. In chlorophyll there
Ni (d ) 2
[NiCl4]
2– is bond involving metal Mg
2

sp 3
2–
[Ni(CN)4] 0
Rearrangement
dsp 2
8 2
Ni (d s ) 2
[Ni(CO)4]
0

Rearrangement 3
sp
EBD_7324
îïì ÝØÛÓ×ÍÌÎÇ

ݸ¿°¬»®

24 Haloalkanes and
Haloarenes
̱°·½ ïæ Ю»°¿®¿¬·±² ¿²¼ Ю±°»®¬·»­ ±º CH3
Ø¿´±¿´µ¿²»­ |
(c) CH3 C CH 2 Br
1. In an SN1 reaction on chiral centres there is : |
[2015 RS] CH3
(a) 100 % racemization
(d) CH3CH2Br
(b) inversion more than retention leading to 5. Reactivityorder of halides for dehydrohalogenation
partial racemization is [2002]
(c) 100 % retention (a) R – F > R – Cl > R – Br > R –I
(d) 100 % inversion (b) R –I > R – Br > R – Cl > R – F
2. Consider the reactions : [2011 M] (c) R –I > R – Cl > R – Br > R – F
C2 H 5OH (d) R – F > R –I > R – Br > R – Cl
(i) (CH 3 ) 2 CH CH 2 Br (CH 3 ) 2
NaCN Ni / H 2
6. CH 3 CH 2 Cl X Y
(CH3)2 CH CH2OC2H5 HBr
Acetic
Y Z
C 2 H 5O anhydride
(ii) (CH3 )2 CH CH 2 Br (CH 3 )2
Z in the above reaction sequence is [2002]
(CH3)2 CH CH 2OC 2H 5 Br (a) CH3CH2CH2NHCOCH3
The mechanisms of reactions (i) and (ii) are (b) CH3CH2CH2NH2
respectively : (c) CH3CH2CH2CONHCH3
(a) SN1 and SN2 (b) SN1 and SN1 (d) CH3CH2CH2CONHCOCH3
(c) SN2 and SN2 (d) SN2 and SN1 7. When CH3CH2CHCl2 is treated with NaNH2,
the product formed is [2002]
3. Which of the following reactions is an example
(a) CH3 — CH = CH2
of nucleophilic substitution reaction? [2009]
(a) 2 RX + 2 Na R – R + 2 NaX (b) CH 3 — C CH
(b) RX + H2 RH + HX NH2
(c) RX + Mg RMgX (c) CH3CH2 CH
(d) RX + KOH ROH + KX NH2
4. In a SN2 substitution reaction of the type Cl
(d) CH3CH2CH
R Br Cl
DMF
R Cl Br [2008]
NH2
which one of the following has the highest
8. An organic compound A (C4H9Cl) on reaction
relative rate ?
with Na/diethyl ether gives a hydrocarbon which
(a) CH3 – CH2 – CH2Br
on monochlorination gives only one chloro
(b) CH 3 CH CH 2 Br derivative, then A is [2001]
|
(a) tert-butyl chloride
CH 3
(b) sec-butyl chloride
Ø¿´±¿´µ¿²»­ ¿²¼ Ø¿´±¿®»²»­ îïë

(c) isobutyl chloride H


(d) n-butyl chloride CH — Cl
9. CH 3 CH 2 C H CH 3 obtained by H3C
| C — Cl
Cl (a) H3C (b)
chlorination of n-butane, will be [2001] CH3
(a) l-form (b) d-form H H
H 3C
(c) Meso form (d) Racemic mixture (c) C — Cl (d) C — Cl
O 2N H 2C H H 3C
10. 2-Bromopentane is heated with potassium
ethoxide in ethanol. The major product obtained 15. What products are formed when the following
is [1998] compounds is treated with Br2 in the presence
(a) 2-ethoxypentane (b) pentene-1 of FeBr 3? [2014]
(c) trans-2-pentene (d) cis-pentene-2 CH3
11. Zerevitinov’s determination of active hydrogen
in a compound is based upon its reaction
with [1994]
CH3
(a) Na (b) CH3MgI
(c) Zn (d) Al. CH3 CH3
12. When chlorine is passed through propene at Br
400°C, which of the following is formed ? [1993] (a) and
(a) PVC (b) Allyl chloride CH3 CH3
(c) Alkyl chloride (d) 1, 2-Dhloroethane Br
̱°·½ îæ Ю»°¿®¿¬·±² ¿²¼ Ю±°»®¬·»­ ±º CH3 CH3
Ø¿´±¿®»²»­ Br Br
(b) and
13. For the following reactions :
[2016] CH3 CH3
(1) CH3CH2CH2Br + KOH CH3 CH3
CH3CH=CH2+KBr + H2O
Br
(2) H3C CH3 H3C CH3 (c) and
+ KOH + KBr
CH3 CH3
Br OH
Br
Br
+ Br2 CH3 CH3
(3)
Br
Which of the following statements is correct ? (d) and
(a) (1) and (2) are elimination reaction and (3) CH3 Br CH3
is addition reaction Br
(b) (1) is elimination, (2) is substitution and (3)
16. The reaction of toluene with Cl2in presence of
is addition reaction
(c) (1) is elimin ation, (2) an d (3) a re FeCl3 gives 'X' and reaction in presence of light
substitution reactions gives ‘Y’. Thus, ‘X’ and ‘Y’ are : [2010]
(d) (1) is substitution, (2) and (3) are addition (a) X = Benzal chloride, Y = o – Chlorotoluene
reaction (b) X = m – Chlorotoluene, Y = p –Chlorotoluene
14. In which of the following compounds, the C - Cl (c) X = o –and p – Chlorotoluene, Y =
bond ionisation shall give most stable carbonium Trichloromethyl – benzene
ion? [2015]
(d) X = Benzyl chloride, Y = m – Chlorotoluene
EBD_7324
îïê ÝØÛÓ×ÍÌÎÇ
17. Which one is most reactive towards SN1 reaction ? ̱°·½ íæ ͱ³» ׳°±®¬¿²¬ б´§¸¿´±¹»²
(a) C6 H5CH(C6 H5 )Br [2010] ݱ³°± «²¼­
(b) C6 H5CH(CH3 )Br 23. Trichloroacetaldehyde, CCl 3CHO reacts with
(c) C6 H5C(CH3 )(C6 H5 )Br chlorobenzene in presence of sulphuric acid
and produces: Cl [2009]
(d) C6 H5CH 2 Br
18. Benzene reacts with CH3Cl in the presence of
anhydrous AlCl3 to form: [2009]
(a) chlorobenzene (b) benzylchloride
(c) xylene (d) toluene
19. The replacement of chlorine of chlorobenzene (a) Cl C Cl
to give phenol requires drastic conditions, but H
the chlorine of 2,4-dinitrochlorobenzene is
readily replaced since, [1997] OH
(a) nitro groups make the aromatic ring electron
rich at ortho/para positions (b) Cl C Cl
(b) nitro groups withdraw electrons from the
meta position of the aromatic ring Cl
(c) nitro groups donate electrons at meta
position (c) Cl CH Cl
(d) nitro groups withdraw electrons from
ortho/para positions of the aromatic ring CCl3
20. Benzene reacts with n-propyl chloride in the
presence of anhydrous AlCl3 to give [1993] Cl
(a) 3 – Propyl – 1 – chlorobenzene
(b) n-Propylbenzene (d) Cl C Cl
(c) No reaction
CH2Cl
(d) Isopropylbenzene.
21. Chlorobenzene reacts with Mg in dry ether to 24. Which of the following is responsible for
give a compound (A) which further reacts with depletion of the ozone layer in the upper strata
ethanol to yield [1993] of the atmosphere? [2004]
(a) Phenol (b) Benzene (a) Polyhalogens (b) Ferrocene
(c) Ethylbenzene (d) Phenyl ether. (c) Fullerenes (d) Freons
22. Which chloro derivative of benzene among the
25. Chloropicrin is obtained by the reaction of
following would undergo hydrolysis most
readily with aqueous sodium hydroxide to [2004]
furnish the corresponding hydroxy derivative? (a) steam on carbon tetrachloride
[1989] (b) nitric acid on chlorobenzene
(c) chlorine on picric acid
NO2
(d) nitric acid on chloroform
26. Industrial preparation of chloroform employs
(a) O2 N Cl acetone and [1993]
(a) Phosgene
(b) Calcium hypochlorite
NO2 (c) Chlorine gas
(d) Sodium chloride.
(b) O2 N Cl 27. Phosgene is a common name for [1988]
(a) phosphoryl chloride
(b) Thionyl chloride
(c) Me2 N Cl (c) Carbon dioxide and phosphine
(d) Carbonyl chloride.
(d) C6H5Cl.
Ø¿´±¿´µ¿²»­ ¿²¼ Ø¿´±¿®»²»­ îïé

ANSWER KEY
1 (b) 4 (d) 7 (b) 10 (c) 13 (b) 16 (c) 19 (d) 22 (a) 25 (d)
2 (a) 5 (b) 8 (a) 11 (b) 14 (a) 17 (c) 20 (d) 23 (c) 26 (b)
3 (d) 6 (a) 9 (d) 12 (b) 15 (c) 18 (d) 21 (b) 24 (d) 27 (d)

Hints & Solutions


1. (b) In SN1 reaction, carbocation a planar 5. (b) The order of atomic size of halogens
species as intermediate is formed. decrease in the order I > Br > Cl > F
R1 i.e on moving down a group atomic size increases.
Nu
Further the bond length of C-X bond decreases
in the order
C R3
C – I > C – Br > C – Cl > C – F
Nu
R2 and h ence the bond dissociation energy
So attack from below or above the plane can decreases in the order
take place. R – F > R – Cl > R – Br > R – I
If 50% attack below and above the plane of
hence R – I being a weakest bond break most
carbocation take place than 100% racemization easily. Hence R – I is most reactive.
occurs but it may not be highly probable.
i.e. if inversion occurs more than retention leading NaCN Ni/H 2
6. (a) CH3CH 2 Cl CH3CH 2 CN
to partial racemization. (X)
2. (a) A strong nucleophile favours the SN2
CH 3CH 2 CH 2 NH 2
reaction and a weak nucleophile favours the SN1 (Y )
( CH 3CO ) 2 O
reaction.
First reaction is SN1 reaction because C 2H5OH CH 3CH 2 CH 2 NHCOCH 3
is used as solvent which is a weak nucleophile. (Z)
Second reaction is SN2 reaction because C2H5O– NaNH 2
is strong nucleophile. 7. (b) CH 3 — CH 2 — CHCl 2
3. (d) In nucleophilic substitution, a nucleophile
provides an electron pair to the substrate and CH 3 — CH CHCl
NaNH2 CH 3 — C CH
Final Product
the leaving group departs with an electron pair.
CH3 CH3
8. (a) CH3 – C – Cl + 2Na + Cl – C – CH3
4. (d) SN2 mechanism is followed in case of CH3 CH3
primary and secondary halides i.e., S N2 reaction t-Butyl chloride

is favoured by small groups on the carbon atom


attached to halogens so
CH3 CH3 CH3 CH3
CH3 CH2 Br > CH3 CH2 CH2 Br > CH3 – C – C –CH 2Cl CH3– C – C – CH3
Mono
CH3 CH3CH3 Clorination CH CH3
3
| 9. (d) Chlorination of n-butane taken place via free
CH3– CH – CH2Br > CH3 – C – Br
| | radical formation i.e., Cl2 h • •
CH3 CH3 Cl Cl

i.e. option (d) is correct.


EBD_7324
îïè ÝØÛÓ×ÍÌÎÇ
Cl 2 / h Br
CH3 — CH2 — CH2 — CH3 (3) + Br2
CH3 CH3 Br
Above reaction involves addition of Br 2 across
H — C — Cl + Cl — C — H double bond. Thus it is called addition reaction.
C2H5 C2H5 14. (a) Tertiary butyl chloride will give the most
d l stable tertiary carbonium ion among the other
Racemic mixture given compounds
50% d form + 50% l form
Cl• may attack on either side and give a racemic H3C – H3C
mixture of 2 chloro butane which contain 50% d- Cl +
C Cl C
form and 50% l-form. H3C H3C
10. (c) Potassium ethoxide is a strong base, and 2- CH3 CH3
t-butyl carbonium ion
bromopentane is a 2º bromide, so elimination
raction predominates 15. (c) Methyl group is ortho para directing but
OC 2 H 5 due to steric hindrance effect, generated by two
CH 3 CH (Br )CH 2 CH 2 CH 3
CH3 groups substitution will not take place on
CH 3 CH CHCH 2 CH 3 CH 2 CHCH 2 CH 2 CH 3 position (I). Hence only two products are possible.
Pentene - 2(major) trans Pentene 1(min or ) cis

Since trans- alkene is more stable than cis.thus CH3


trans-pentene -2 is the main product.
III I
11. (b) Number of active hydrogen in a compound
corresponds to the number of moles of CH4
evolved per mole of the compound. II CH3
CH 3MgI
O , N 2, S , O or C C
i.e. CH3 CH3
CH 4 ( 2CH 4 from NH 2 ) Br2
12. (b) At high temp. i.e., 400°C substitution occurs FeBr2
in preference to addition. CH3 CH3
Cl2 , 400 C
Br
CH 3 CH CH 2
HCl
CH3
ClCH 2 CH CH 2 Br
13. (b) (1) CH3CH2CH2–Br + KOH +
CH3CH=CH2 + KBr + H2O CH3
This is dehydrohalogenation reaction which is
an example of elimination reaction. CH3 CH3 CH3
Cl2 Cl
(2) H3C CH3 H3C CH3 16. (c) +
FeCl3
C + KOH C + KBr
Br OH Cl
CH3
Replacement of Br – by OH– is substitution
Cl2 ,hv
reaction thus it is a nucleophilic substitution
reaction.
Ø¿´±¿´µ¿²»­ ¿²¼ Ø¿´±¿®»²»­ îïç
CH2Cl CHCl2 CCl3 23. (c)
Cl2 ,hv Cl2 ,hv

Cl 3C–C=O + conc. H2 SO4


Cl2C–CH
17. (c) SN1 reactions involve the formation of H Cl
carbocations, hence higher the stability of DDT
intermediate carbocation, more will be reactivity
of the parent alkyl halide. The tertiary carbocation
formed from (c) is stabilized by two phenyl groups 24. (d) Chlorofluorocarbons, e.g. CF2Cl2, CHF2Cl2,
and one methyl group, hence most stable. HCF 2 CHCl 2 . These are non-inflammable
colourless and stable upto 550ºC. These are
CH3 emitted as propellants in aerosol spray, cans
refrigerators, fire fighting reagents etc. They are
18. (d) + CH3Cl
Anhyd.
+HCl chemically inert and hence do not react with any
AlCl3
substance with which they come in contact and
Toluene therefore float through the atmosphere and as a
Friedel Craft's reaction result enter the stratosphere. There they absorb
UV-rays and due to this they produce free atomic
19. (d) —NO2 group is electron attracting group, chlorine which results decomposition of ozone
so it is able to deactivate the benzene ring. which cause depletion of ozone layer.
Cl 25. (d) Chloropicrin is nitrochloroform. It is obtained
NO2 by the nitration of chloroform with HNO3.
HNO3
HCCl 3 O 2 NCCl3
Chloroform Chloropicrin
NO2 Chloropicrin is poisonous and used as an
hence withdrawl of electrons from ortho & para insecticide and a war gas.
position cause easy removal of –Cl atom due to 26. (b) By distilling ethanol or acetone with a paste
development of +ve charge on o- and p – positions. of bleach in g powder (laboratory and
commercial method).
Anhyd .
20. (d) C 6 H 6 CH 3 CH 2 CH 2 CI
AlCl 3 CaOCl 2 H 2O Ca (OH ) 2 Cl 2
Cl2, so obtained acts as a mild oxidising as well
CH 3
| as chlorinating agent
C6 H 5 CH CH 3 (i) CH 3COCH 3 3Cl 2
Isopropyl benzene
CCl 3 .CO.CH 3 3HCl
Mg (Chlorination)
21. (b) C6 H5Cl C6 H5 MgCl
dryether (A) (ii) 2CCl3 .CO.CH 3 Ca (OH) 2

CH 3CH 2OH 2CHCl 3 (CH 3COO) 2 Ca


C6 H 6 CH3CH 2OMgCl
(Hydrolysis)
27. (d) Carbonyl chloride (COCl2).
22. (a) Cl in 2, 4, 6-trinitrochlorobenzene is activated
by three NO2 groups at o- and p-positions and
hence undergoes hydrolysis most readily.
EBD_7324
ݸ¿°¬»®

25 Alcohols, Phenols and


Ethers

̱°·½ ïæ Ю»°¿®¿¬·±² ¿²¼ Ю±°»®¬·»­ (a) Acetone (b) Methane


±º ß´½±¸± ´­ (c) Acetaldehyde (d) Ethyl alcohol
1. Which of the following compounds can be used 6. When glycerol is treated with excess of HI, it
as antifreeze in automobile radiators ? produces: [2010]
[2012 M] (a) glycerol triiodide (b) 2–iodopropane
(a) Methyl alcohol (b) Glycol (c) allyl iodide (d) propene
(c) Nitrophenol (d) Ethyl alcohol 7. Consider the following reaction,
2. How many isomers of C5H11 OH will be primary [2009]
alcohols ? [1992]
(a) 5 (b) 4 PBr3 alc.KOH
ethanol X Y
(c) 2 (d) 3.
(i) H 2SO4 room temperature
3. Which of the following reaction (s) can be used Z;
(ii) H2O, heat
for the preparation of alkyl halides ? [2015 RS]
anh.ZnCl 2
the product Z is:
(I) CH3CH2OH + HCl
(a) CH3CH2 – O – CH2 – CH3
(II) CH3CH2OH + HCl
(b) CH3 – CH2 – O – SO3H
(III) (CH3)3COH + HCl
(c) CH3CH2OH
(IV) (CH3)2CHOH + HCl anh.ZnCl 2
(d) CH2 = CH2
(a) (I), (III) and (IV) only
8. H2COH · CH2OH on heating with periodic acid
(b) (I) and (II) only gives: [2009]
(c) (IV) only CHO
(d) (III) and (IV) only (a) 2 HCOOH (b) |
4. Number of isomeric alcohols of molecular CHO
formula C6H14O which give positive iodoform H
(c) 2 C=O (d) 2 CO2
test is [NEET Kar. 2013]
H
(a) two (b) three
9. Ethylene oxide when treated with Grignard
(c) four (d) five reagent yields [2006]
5. In the following sequence of reactions [2012] (a) Tertiary alcohol
KCN H3O (b) Cyclopropyl alcohol
CH3 Br A
(c) Primary alcohol
LiAlH 4
B C, the end product (C) is : (d) Secondary alcohol
ether
ß´½±¸±´­ô и»²±´­ ¿²¼ Û¬¸»®­ îîï

10. Which of the following will not form a yellow (c) Tertiary alcohol
precipitate on heating with an alkaline solution (d) All of these
of iodine? [2004] 17. What is formed when a primary alcohol
(a) CH 3CH (OH)CH 3 undergoes catalytic dehydrogenation ? [1993]
(b) CH 3CH 2 CH(OH )CH 3 (a) Aldehyde (b) Ketone
(c) CH 3OH (c) Alkene (d) Acid
18. Methanol is industrially prepared by [1992]
(d) CH 3CH 2OH
(a) Oxidation of CH4 by steam at 900°C
11. n-Propyl alcohol and isopropyl alcohol can be (b) Reduction of HCHO using LiAIH4
chemically distinguished by which reagent? (c) Reaction HCHO with a solution of NaOH
(a) PCl5 [2002] (d) Reduction of CO using H2 and
(b) Reduction ZnO – Cr2O3.
(c) Oxidation with potassium dichromate 19. HBr reacts fastest with [1992]
(d) Ozonolysis (a) 2-Methylpropan-1-ol
12. Which of the following is correct ? [2001] (b) 2-Methylpropene-2-ol
(a) On reduction of any aldehyde, secondary (c) propan-2-ol
alcohol is formed (d) propan-1-ol.
(b) Reaction of vegetable oil with H2SO4 gives 20. The compound which reacts fastest with Lucas
glycerine reagent at room temperature is [1989]
(c) Sucrose on reaction with NaCl gives invert (a) Butan-1-ol
sugar (b) Butan-2-ol
(d) Alcoholic iodine gives iodoform with (c) 2-Methyl propan-1-ol
NaOH (d) 2-Methylpropan-2-ol
13. Propan - 1- ol may be prepared by the reaction
21. Propene, CH 3CH CH 2 can be converted into
of propene with [2000]
1-propanol by oxidation. Indicate which set of
(a) H3BO3
reagents amongst the following is ideal to effect
(b) H2SO4/H2O the above conversion ? [1989]
(c) B2H6,NaOH–H2O2 (a) KMnO4 (alkaline)
O (b) Osmium tetraoxide (OsO4/CH2Cl2)
|| (c) B2H6 and alk. H2O2
(d) CH 3 C O O H (d) O3/Zn
14. Reaction of CH — CH with RMgX leads to 22. Lucas reagent is [1988]
2 2
O (a) Conc. HCl and anhydrous ZnCl2
formation of [1998] (b) Conc. HNO3 and hydrous ZnCl2
(a) RCHOHR (b) RCHOHCH3 (c) Conc. HCl and hydrous ZnCl2
R (d) Conc. HNO3 and anhydrous ZnCl2
(c) RCH2CH2OH (d) CHCH2OH ̱°·½ îæ Ю»°¿®¿¬·±² ¿²¼
R Ю±°»®¬·»­ ±º и»²±´­
15. The stablest among the following is [1994]
23. Which one is the most acidic compound ?
(a) CH3CH(OH)2 (b) ClCH2CH(OH)2
[2017]
(c) (CH3)2 C (OH)2 (d) CCl3 CH (OH)2.
16. Which one of the following on oxidation gives a OH
OH
ketone ? [1993]
(a) Primary alcohol (a) (b)
(b) Secondary alcohol
NO2
EBD_7324
îîî ÝØÛÓ×ÍÌÎÇ
OH 29. Which of the following will not be soluble in
OH sodium hydrogen carbonate? [2014]
(c) O2N NO2 (d) (a) 2, 4, 6-trinitrophenol
(b) Benzoic acid
CH3 (c) o-Nitrophenol
NO2 (d) Benzenesulphonic acid
24. Which of the following reagents would
30. Phenol is distilled with Zn dust followed by
distinguish cis-cyclopenta-1,2-diol from the
Friedel Crafts alkylation with propyl chloride in
trans-isomer? [2016]
the presence of AlCl3 to give a compound (B).
(a) Acetone (B) is oxidised in the presence of air to form the
(b) Ozone compound (C). The structural formula of (C) is
(c) MnO2 [NEET Kar. 2013]
(d) Aluminium isopropxide
COOH
25. The reaction COOH
Me
OH
NaOH Me–1
OH

O Na
+
O (a) (b)

Can be classified as :- [2016]


H3C
(a) Williamson ether synthesis reaction C O O H
(b) Alcohol formation reaction (c) H3C
(c) Dehydration reaction
(d) Williamson alcohol synthesis reaction
26. Reaction of phenol with chloroform in presence
of dilute sodium hydroxide finally introduces CH3
which one of the following functional group ? H3C C O O H
[2015 RS] (d)
OH
(a) –CH2Cl (b) –COOH
(c) –CHCl2 (d) –CHO
27. Which of the following is not the product of
31. Which one of the following compounds has the
most acidic nature? [2010]
dehydration of OH ? [2015 RS]
CH2OH
(a)
(a) (b)
OH
(b)

(c) (d)
OH
(c)
28. Among the following sets of reactants which
one produces anisole? [2014] OH
(a) CH3CHO; RMgX
CH
(b) C6H5OH; NaOH ; CH3I (d)
(c) C6H5OH; neutral FeCl3
(d) C6H5–CH3; CH3COCl; AlCl3
ß´½±¸±´­ô и»²±´­ ¿²¼ Û¬¸»®­ îîí

32. Among the following four compounds [2010] 38. When phenol is treated with excess bromine
water. It gives [1992]
(i) phenol (ii) methylphenol
(a) m-Bromophenol
(iii) meta-nitrophenol (iv) para-nitrophenol
(b) o-and p-Bromophenols
the acidity order is : (c) 2,4-Dibromophenol
(a) ii > i > iii > iv (b) iv > iii > i > ii (d) 2,4, 6-Tribomophenol.
(c) iii > iv > i > ii (d) i > iv > iii > ii 39. Which one of the following compounds will be
most readily attacked by an electrophile ? [1989]
33. Consider the following reaction: [2009]
CH3Cl
(a) Chlorobenzene (b) Benzene
Zn dust
Phenol X Y (c) Phenol (d) Toluene
Anhydrous AlCl3
40. When phenol is heated with CHCl3 and alcoholic
Alkaline KMnO4
Z KOH when salicyladehyde is produced. This
The product Z is: reaction is known as [1988, 89]
(a) benzaldehyde (b) benzoic acid (a) Rosenmund’s reaction
(c) benzene (d) toluene (b) Reimer-Tiemann reaction
34. When phenol is treated with CHCl3 and NaOH, (c) Friedel-Crafts reaction
the product formed is [2002] (d) Sommelet reaction
(a) Benzaldehyde (b) Salicylaldehyde
̱°·½ íæ Ю»°¿®¿¬·±² ¿²¼ Ю±°»®¬·»­
(c) Salicylic acid (d) Benzoic acid ±º Û¬¸»®­
35. The ionization constant of phenol is higher than
41. The heating of phenyl-methyl ethers with HI
that of ethanol because : [2000]
produces [2017]
(a) Phenoxide ion is bulkier than ethoxide (a) Iodobenzene (b) Phenol
(b) Phenoxide ion is stronger base than (c) Benzene (d) Ethyl chlorides
ethoxide 42. Identify A and predict the type of reaction
(c) Phenoxide ion is stabilized through [2017]
delocalization
OCH3
(d) Phenoxide ion is less stable than ethoxide
36. 1-Phenylethanol can be prepared by the reaction
of benzaldehyde with [1997] NaNH2
A
(a) Ethyl iodide and magnesium
Br
(b) Methyl iodide and magnesium
(c) Methyl bromide and aluminium bromide OCH3
(d) Methyl bromide NH2
37. Increasing order of acid strength among (a) and elimination addition reaction
p-methoxyphenol, p-methylph enol and
p-nitrophenol is [1993]
OCH3
(a) p-Nitrophenol, p-Methoxyphenol,
Br
p-Methylphenol
(b) and cine substitution reaction
(b) p-Methylphenol, p-Methoxyphenol,
p-Nitrophenol
(c) p-Nitrophenol, p-Methylphenol, OCH3

p-Methoxyphenol.
(d) p-Methoxyphenol, p-Methylphenol, (c) and cine substitution reaction
p-Nitrophenol
EBD_7324
îîì ÝØÛÓ×ÍÌÎÇ

OCH3 Which of the following compounds will be


formed?

(d) and substitution reaction (a) CH 3 CH CH 3 CH 3CH 2 OH


NH2 |
CH 3
43. The reaction
(b) CH3 CH CH 2 OH CH3CH3
CH3 CH3
|
CH3–C–ONa + CH3CH2Cl–— CH3–C–O–CH2–CH3 CH3
–NaCl
CH3 CH3 CH3
|
is called : [2015] (c) CH3 CH CH 2 OH CH3 CH 2 I
(a) Williamson continuous etherification process
(b) Etard reaction CH3
(c) Gatterman - Koch reaction |
(d) CH3 CH CH 2 I CH3CH 2OH
(d) Williamson Synthesis
44. Identify Z in the sequence of reactions: 47. The major organic product in the reaction,
[2014] CH3 — O — CH(CH3)2 + HI Product is
HBr / H 2 O 2 C 2 H 5 ONa [2006]
CH 3 CH 2 CH=CH 2 Y Z
(a) ICH2OCH(CH3)2
(a) CH3—(CH2)3 —O—CH2CH3
(b) (CH3)2 CH2—O—CH2CH3 (b) CH 3O C(CH 3 ) 2
|
(c) CH3(CH2)4—O—CH3 I
(d) CH3CH2—CH(CH3)—O—CH2CH3
(c) CH3I + (CH3)2CHOH
45. Among the following ethers, which one will
(d) CH3OH + (CH3)2CHI
produce methyl alcohol on treatment with hot
48. Ethanol and dimethyl ether form a pair of
concentrated HI? [NEET 2013]
functional isomers. The boiling point of ethanol
(a) CH 3 CH 2 CH O CH 3
| is higher than that of dimethyl ether, due to the
CH 3 presence of [1993]
CH3 (a) H-bonding in ethanol
| (b) H-bonding in dimethyl ether
(b) CH3 C O CH3
(c) CH3 group in ethanol
CH3 (d) CH3 group in dimethyl ether
(c) CH3 CH CH2 O CH3 49. Which one is formed when sodium phenoxide is
|
CH3 heated with ethyl iodide ? [1988]
(d) CH3 – CH2 – CH2 – CH2 – O – CH3 (a) Phenetole
46. In the reaction: [2007] (b) Ethyl phenyl alcohol
(c) Phenol
CH3
| (d) None of these
Heated
CH3 CH CH 2 O CH 2 CH 3 HI

ANSWER KEY
1 (b) 6 (b) 11 (c) 16 (b) 21 (c) 26 (d) 31 (b) 36 (b) 41 (b) 46 (c)
2 (b) 7 (c) 12 (d) 17 (a) 22 (a) 27 (b) 32 (b) 37 (d) 42 (d) 47 (c)
3 (a) 8 (c) 13 (c) 18 (d) 23 (c) 28 (b) 33 (b) 38 (d) 43 (d) 48 (a)
4 (c) 9 (c) 14 (c) 19 (b) 24 (a) 29 (c) 34 (b) 39 (c) 44 (a) 49 (a)
5 (d) 10 (c) 15 (d) 20 (d) 25 (a) 30 (c) 35 (c) 40 (b) 45 (b)
ß´½±¸±´­ô и»²±´­ ¿²¼ Û¬¸»®­ îîë

Hints & Solutions


1. (b) Glycol is used as an antifreeze in automo- (iv) In the presence of ZnCl 2, 2° carbocation is
biles. formed from (CH3 ) 2 — C— OH
2. (b) Four primary alcohols of C 5H11 OH are |
possible. These are: H
(i) CH 3CH 2 CH 2 CH 2 CH 2 OH i.e., CH3 — C H— CH3
(ii) CH 3CH 2 CH CH 2 OH 4. (c) Compound containing CH3 CH(OH) or
| CH3CO–group give positive iodoform test.
CH 3
OH
(iii) CH 3 CH CH 2CH 2 OH |
| CH 3 CH— CH 2 CH 2 CH 2 CH 3
CH 3
OH
CH 3 |
| CH 3 — CH — CH CH 2 CH 3 ,
(iv) CH 3 C CH 2 OH |
| CH 3
CH 3
OH
3. (a) ZnCl2 is a lewis acid and interact with |
alcohol. CH 3 — CH — CH 2 — CH — CH3 ,
|
CH 3
CH3 — CH 2OH ZnCl 2 R — O— ZnCl 2
|
OH CH 3
H | |
(R CH3 — CH 2 —) (I) CH 3 — CH — C — CH 3
|
ó CH 3
R — O— ZnCl2 R [HOZnCl 2 ]
|
KCN H 3O
H 5. (d) CH3 Br CH3 CN
(I) (A)
LiAlH 4
1 mole of carbon = 12gm of carbon CH3 COOH CH3 CH 2 OH
(B) (C)
= 6.023 × 1023 C – atoms. Ethyl alcohol
Carbocation is formed as intermediate in the SN1
mechanism which these reaction undergoes. 6. (b) Glycerol when treated with excess HI
produces 2–iodopropane
In the absence of ZnCl 2 formation of
primary carbocation is difficult which is the case CH 2 OH CH 2 I CH 2
with (ii) while (i) undergoes reaction.
I2
(iii) Tertiary carbocation casily formed due to CHOH 3HI CHI CH
the stability.
CH 2 OH CH 2 I CH 2 I
OH
| CH3 CH3 CH3
CH3 — C — CH3 | | |
| +HI +I 2 +HI
CHI CH CHI
CH3 | || |
CH 2I CH 2 CH2
CH3 — C — CH3 H2O Correct choice : (b)
|
CH3
EBD_7324
îîê ÝØÛÓ×ÍÌÎÇ
7. (c) H2C – CH2+RMgX CH2 – CH2
CH3CH2OH
PBr3
CH3CH2 Br
alc.KOH
CH2 CH 2 O OMgX R
H2SO4 H2O CH2 – CH2
–Mg(OH)X
CH3 CH 2OH
H22O
HO
CH3 – CH 2 – HSO4 OH R
heat

8. (c) 1, 2 – Diols, when treated with an aqueous 15. (d) Due to –I-effect of the three C–Cl-bonding
solution of periodic acid give aldehyde or between Cl and C-atom of the OH group, CCl3
ketones CH (OH)2 is most stable.
16. (b) Secondary alcohols on oxidation give
CH2OH HIO4 ketones.
| CH2O +CH2O
CH2 OH Note : – Primary alcohols from aldehydes.
Note that a 1° alcohol gives CH2O. Since in R R
glycol both the OH groups, are primary hence [O]
CHOH C=O
give 2 molecules of CH2O is formed as by R R
product. Isopropyl Ketone
9. (c) Ethylene oxide when treated with Grignard alcohol
Reagent gives primary alcohol.
17. (a) 1° Alcohols on catalytic dehydrogenation
CH2 CH2 – OMgX give aldehydes.
| O + R–MgX | + H2O
CH2 CH2 – R Cu
RCH2OH RCHO + H2
300°C
X 1° alcohol Aldehyde
R – CH2 – CH 2 – OH + Mg 18. (d) Water gas is mixed with half of its volume of
OH hydrogen.The mixture is compressed to
10. (c) CH 3OH does not have CH(OH )CH 3 approximately 200 – 300 atmospheres. It is then
group hence it will not form yellow precipitate passed over a catalyst (ZnO + Cr 2O3) at 300°C.
with an alkaline solution of iodine (haloform Methyl alcohol vapours are formed which are
reaction). condensed
ZnO Cr2O3
11. (c) Primary alcohol on oxidation give aldehyde CO 2H 2 CH3OH
300 C
which on further oxidation give carboxylic acid Compressed gas Methyl alcohol
whereas secondary alcohols give ketone. 19. (b) Greater the stability of the intermediate
carbocation, more reactive is the alcohol. Since
[ O]
CH 3CH 2 CH 2 OH 2-methylpropan-2-ol generates 3° carbocation,
n propyl alcohal therefore, it reacts fastest with HBr.
[O]
20. (d) The rates of reaction of different alcohols
CH 3 CH 2 CHO CH 3 CH 2 COOH with lucas reagent follows the order.
H3C 3° alcohol > 2° alcohol > 1° alcohol
[O] H3C
CH – OH C=O since carbocations are formed as
isopropyl alcohal Ketone
intermediate, more stable the carbocation, higher
12. (d) C 2 H 5 OH 4 I 2 6 NaOH will be the reactivity of the parent compound
(alcohol). 2-Methylpropan-2-ol generates a 3º
CHI 3 HCOONa 5 NaI 5H 2O
Iodoform carbocation, so it will react fastest; other three
generates either 1º or 2º carbocations.
13. (c) CH3 CH3 CH3
B2H 6 | | |
CH3– CH=CH2 CH3–CH2–CH2–OH CH3 C OH
H
CH3 C
Br
CH3 C Br
NaOH / H 2O 2 Propanol | | |
14. (c) We know that CH3 CH3 CH3
2-Methylpropan-2-ol
ß´½±¸±´­ô и»²±´­ ¿²¼ Û¬¸»®­ îîé

21. (c) KMnO4 (alkaline) and OsO4 / CH2Cl2 are 28. (b) Phenols react with alkyl halides in alkaline
used for hydroxylation of double bond while O3 medium to form ethers. Therefore,
/Zn is used for ozonolysis. Therefore, the right
option is (c), i.e., OH OCH3
BH3 in THF
3CH 3CH CH 2 (CH 3CH 2CH 2 )3 B
(i) NaOH
3H 2O 2
3CH 3CH 2CH 2OH + H3BO3 (ii) CH3I
NaOH
1-propanol
22. (a) Lucas reagent is conc. HCl + anhyd. ZnCl 2. 29. (c) o-nitrophenol will not be soluble in
23. (c) Electron withdrawing – NO2 group has very NaHCO3 . Due to intramolecular hydrogen
strong –I and –R effects so, compound 3 will be bonding hydrogen on OH is strongly bound.
most acidic. So it cannot be have as an acid and can not
24. (a) react with sodium bicarbonate.
25. (a) This is an example of Williamson ether
30. (c)
synthesis reaction in which sodium alkoxide
reacts with alkyl halide and gives ether. OH

26. (d) CHCl3 NaOH : CCl3 H 2 O Zn AlCl3


CH3CH2CH2Cl
: CCl3 : CCl2 Cl CH3
C O O H CH3 CH CH3
CH3
– – O2
O O Cl
Cl C
– Cl + H O
+ :C 2 –
Cl – OH

– – 31. (b) Phenol is most acidic because its conju-


O O gate base is stabilised due to resonance, while
C=O – CCl2
OH the rest three compounds are alcohols, hence,
H – 2HCl H their corrosponding conjugate bases do not ex-
hibit resonance
Therefore functional group – CHO is OH OH
introduced.
27. (b) 32. (b) >
NO2
OH +H O—H NO2
(iv) (iii)
H (– I and – M effects, (only – I effect)
both increase acidity)
H
OH OH

> > CH3


H
(i) (ii)
(+ I effect of CH3 group decreases
acidity)
Correct choice : (b)
EBD_7324
îîè ÝØÛÓ×ÍÌÎÇ
C6H5
OH
CH3MgI + C O
H
Zn CH3Cl
33. (b) C6H5
anhy
AlCl3
CH O MgI
Phenol X CH3

CH3 COOH C6H5


H 2O
CHOH
CH3 Mg ( OH ) I
alc.
KMnO4 (1-Phenyl ethanol)

Y Z 37. (d) Electron-donating groups (– OCH3, – CH3


etc.) tend to decrease and electron withdrawing
OH OH groups (– NO2, – OCH3 etc.) tend to increase
CHO the acidic character of phenols. Since – OCH 3
CHCl3 is a more powerful electron-donating group
34. (b) NaOH than – CH3 group, therefore, p-methylphenol
(Riemer Tiemann reaction) Salicylaldehyde is slightly more acidic than p-methoxyphenol
while p-nitrophenol is the strongest acid.
35. (c) The acidic nature of phenol is due to the
Thus, option (d), i.e. p-methoxyphenol, p-
formation of stable phenoxide ion in solution
methylphenol, p-nitrophenol is correct.
C6 H 5 OH H 2 O C6 H5O H 3O 38. (d) Wit h Br 2 wa ter, phen ol gi ves 2, 4,
Phenoxide ion 6- tribromophenol.
Th e phenoxide ion is stable due to OH
resonance.
+ 3Br2 (excess)

O O O
OH
•• Br Br
H2O
+ 3HBr
••
Br
OH O– O 2, 4, 6 Tribromphenol
– –
39. (c) Due to strong electron-donating effect of the
••
OH group, the electron density in phenol is much

higher than that in toluene, benzene and
chlorobenzene and hence phenol is readily
attacked by the electrophile.
The negative charge is delocalized in the benzene
ring which is a stabilizing factor in the phenoxide OH OH
ion and increase acidity of phenol. wheras no CHCl2
resonance is possible in alkoxide ions (RO– 40. (b) + CHCl3
)derived from alcohol. The negative charge is
localized on oxygen atom. Thus, alcohols are
OH OH
not acidic. CH(OH)2 CHO
36. (b) CH 3 I Mg Dry ether 2KOH
CH 3 MgI
–H2O

Reimer-Tiemann reaction.
ß´½±¸±´­ô и»²±´­ ¿²¼ Û¬¸»®­ îîç

41. (b) When Ar – O – R ethers are reacted with HI, 45. (b) The reaction will proceed via SN1 mechanism
they are cleaved at weaker O – R bond to give
because alkyl group attached is 3°.
phenol and alkyl iodide.
CH3 CH3
O – CH3 OH | | H
CH3 C O CH3 HI CH3 C O
S1N + CH3
HI
+ CH3I CH3 CH 3

CH3 CH3
42. (d) |+
I
– |
CH3 C CH3 C I
OCH 3 OCH3 CH3 CH3
(Highly stable)
H
NH 2
46. (c) In the cleavage of mixed ethers with two
Br Br different alkyl groups, the alcohol and alkyl
iodide that form depend on the nature of alkyl
OCH3
group. When primary or secondary alkyl groups
are present, it is the lower alkyl group that forms
alkyl iodide therefore

Benzyne CH3 C H CH 2 O CH 2 CH 3 HI
|
CH3
OCH3
NH 2
CH3
OCH3
a |
NH 2
(Less stable) CH3 CH CH 2 OH CH3CH 2 I
OCH3 OCH3
47. (c) In case of unsymmetrical ethers, the site of
b
H — NH 2 cleavage depends on the nature of alkyl group
NH 2 NH 2
e.g.,
(More stable)
373K
More stable as –ve charge is close to electron CH3O CH(CH 3 )2 HI CH 3I (CH 3 )2 CHOH
withdrawing group. Methyl
iodide
Isopropyl
alcohol
Also, incoming nucleophile gets attached on
same 'C' on which 'Br' (Leaving group) was The alkyl halide is always formed from the
present. smaller alkyl group.
not a cine substitution reaction 48. (a) Due to H-bonding, the boiling point of
43. (d) Williamson synthesis is one of the best ethanol is much higher than that of the isomeric
methods for the preparation of symmetrical and diethyl ether.
unsymmetrical ethers. In this method, an alkyl 49. (a) C 6 H 5 ONa C 2 H 5 I C6 H 5OC 2 H 5
halide is allowed to react with sodium alkoxide. Phenetole
44. (a) NaI
HBr/H2O2
CH3 CH2 CH CH2 (Peroxide effect)

CH3 CH2 CH2 CH2


(Y)
Br

CH3 (CH2)3 O CH2 CH3


(Z)
EBD_7324
ݸ¿°¬»®

26 Aldehydes, Ketones, and


Carboxylic Acids
̱°·½ ïæ Ó»¬¸±¼­ ±º Ю»°¿®¿¬·±² ±º (b) Ortho-nitrophenol
Ý¿®¾±²§´ ݱ³°±«²¼­ (c) Phenol
(d) 2-methyl-2 hydroxypropane
1. Reaction by which Benzaldehyde cannot be 5. In the following reaction, product 'P' is [2002]
prepared : [NEET 2013] H2
R C Cl P
COCI || Pd BaSO 4
O
(a) + H2 in presence of (a) RCH2OH (b) RCOOH
(c) RCHO (d) RCH3
Pd - BaSO4 6. The catalyst used in Rosenmund's reduction is
[2000]
(b) + CO + HCl in presence of (a) HgSO4 (b) Pd/BaSO4
(c) anhydrous AlCl 3 (d) anhydrous ZnCl2
anhydrous AlCl3 7. Ketones
COOH
[ R — C — R1 , where R = R1 = alkyl groups]
(c) + Zn/Hg and conc. HCl ||
O
CH3 can be obtained in one step by [1998]
(d) + CrO2Cl2 in CS2 followed (a) oxidation of primary alcohols
(b) hydrolysis of esters
by H3O+ (c) oxidation of tertiary alcohols
2. Consider the following reaction : (d) reaction of acid halides with alcohols
COCl 8. Pinacolone is [1994]
H2 (a) 2, 3-Dimethyl-2, 3-butanediol
'A'
Pd-BaSO4 (b) 3, 3-Dimethyl-2 butanone
The product ‘A’ is : [2012 M] (c) I-Phenyl-2Propanone
(a) C6H5CHO (d) 1,1-Diphenyl-2-ethandiol.
(b) C6H5OH
̱°·½ îæ Ю±°»®¬·»­ ±º Ý¿®¾±²§´
(c) C6H5COCH3
ݱ³°± «²¼­
(d) C6H5Cl
3. Which of the following reactions will not result
9. Consider the reactions :- [2017]
in the formation of carbon-carbon bonds? +
(a) Reimer-Tieman reaction [2010] Cu [Ag(NH3)2]
X A Silver mirror
(b) Cannizaro reaction
(C2H6O) 573K –OH observed
(c) Wurtz reaction –OH
(d) Friedel-Crafts acylation O Y
4. Which one of the following can be oxidised to
the corresponding carbonyl compound? [2004] NH2–NH– C –NH2
Z
(a) 2-hydroxypropane Identify A, X, Y and Z
ß´¼»¸§¼»­ô Õ»¬±²»­ô ¿²¼ Ý¿®¾±¨§´·½ ¿½·¼­ îíï

(a) A-Methoxymethane, X-Ethanol, equilibrates with its corresponding enol


Y-Ethanoic acid, Z-Semicarbazide. and this process is known as keto-enol
(b) A - Ethanal, X-Ethanol, Y - But - 2-enal, tautomerism.
Z-Semicarbazone 12. The product formed by the reaction of an
(c) A-Ethanol, X-Acetaldehyde, Y - Butanone, aldehyde with a primary amine is [2016]
Z-Hydrazone (a) Schiff base (b) Ketone
(d) A-Methoxymethane, X-Ethanoic acid, (c) Carboxylic acid (d) Aromatic acid
Y-Acetate ion, Z-hydrazine. 13. An organic compound 'X' having molecular
10. Of the following, which is the product formed
formula C5H10O yields phenyl hydrazone and
when cyclohexanone undergoes aldol
gives negative response to the Iodoform test
condensation followed by heating ? [2017]
and Tollen's test. It produces n-pentane on
reduction. 'X' could be :- [2015]
(a)
(a) 2-pentanone (b) 3-pentanone
(c) n-amyl alcohol (d) pentanal
O

14. Treatment of cyclopentanone =O with


(b)
methyl lithium gives which of the following
OH species? [2015]
(a) Cyclopentanonyl cation
(c) (b) Cyclopentanonyl radical
(c) Cyclopentanonyl biradical
O O (d) Cyclopentanonyl anion
O 15. The enolic form of ethyl acetoacetate as below
has: [2015]
(d) H H2
H3C C O H3C C O
C C C C
OH
11. The correct statement regarding a carbonyl OH OC2H5 O OC2H5
compoun d with a hydrogen atom on its
alphacarbon, is : [2016] (a) 16 sigma bonds and 1 pi - bond
(a) a carbonyl compound with a hydrogen (b) 9 sigma bonds and 2 pi - bonds
atom on its alpha-carbon never equilibrates (c) 9 sigma bonds and 1 pi - bond
with its corresponding enol. (d) 18 sigma bonds and 2 pi - bonds
(b) a carbonyl compound with a hydrgen atom 16. Reaction of a carbonyl compound with one of
on its alpha-carbon rapidly equilibrates the following reagents involves nucleophilic
with its corresponding enol and this addition followed by elimination of water. The
process is known as aldehyde-ketone reagent is : [2015 RS]
equilibration. (a) a Grignard reagent
(c) a carbonyl compound with a hydrogen
(b) hydrazine in presence of feebly acidic
atom on its alpha-carbon rapidly
solution
equilibrates with its corresponding enol
and this process is known as (c) hydrocyanic acid
carbonylation. (d) sodium hydrogen sulphite
(d) a carbonyl compound with a hydrogen 17. Which one is most reactive towards Nucleophilic
atom on its alpha-carbon rapidly addition reaction? [2014]
EBD_7324
îíî ÝØÛÓ×ÍÌÎÇ

CHO COCH3 CH2OH COO
(a) (b) (d) õ

OH OH
CHO CHO
20. Acetone is treated with excess of ethanol in the
presence of hydrochloric acid. The product
(c) (d)
obtained is : [2012]

CH3 NO2 O

18. The order of stability of the following tautomeric (a) CH3 CH2CH2 C CH3
compounds is :
O
OH O
CH 2 C CH 2 C CH 3 (b) CH3 CH2CH2 C CH2CH 2CH3

O O OH
(c) (CH3)2C
CH3 C CH 2 C CH 3 OC2H5
II
OH OC2H5
O
(d) (CH3)2C
CH 3 C CH C CH 3 OC2H5
[NEET 2013] 21. CH3 CHO and C6H5CH2CHO can be distin-
(a) III > II > I (b) II > I > III guished chemically by : [2012]
(c) II > III > I (d) I > II > III (a) Benedict test
19. Predict the product in the given reaction. [2012] (b) Iodoform test
CHO (c) Tollen’s reagent test
(d) Fehling solution test
50 û KOH
22. Consider the reaction :
Cl RCHO + NH2NH2 RCH = N – NH2
What sort of reaction is it ? [2012 M]
– (a) Electrophilic addition – elimination
CH2OH CH2 COO
(a) õ reaction
(b) Free radical addition – elimination reaction
Cl Cl (c) Electrophilic substitution – elimination
reaction
(d) Nucleophilic addition – elimination
CH2OH OH reaction
(b) õ
23. Which of the following compounds will give a
OH OH yellow precipitate with iodine and alkali ?
[2012 M]
– (a) Acetophenone
CH2 OH COO
(c) õ (c) Methyl acetate
(b) Acetamide
Cl Cl (d) 2-Hydroxypropane
ß´¼»¸§¼»­ô Õ»¬±²»­ô ¿²¼ Ý¿®¾±¨§´·½ ¿½·¼­ îíí

24. Clemmensen reduction of a ketone is carried out in 28. Acetophenone when reacted with a base,
the presence of which of the following? [2011] C2H5ONa, yields a stable compound which has
(a) Glycol with KOH the structure. [2008]
(b) Zn-Hg with HCl
(c) Li Al H4 – C = CH – C –
(d) H2 and Pt as catalyst (a)
CH3 O
25. The order of reactivity of phenyl magnesium
bromide (PhMgBr) with the following compounds
– CH – CH2C –
[2011 M]
(b)
CH3 CH3 Ph CH3 O
C=O, C=O and C=O
H CH 3 Ph CH3CH3
I II III –C–C–
(c)
(a) III > II > I (b) II > I > III OH OH
(c) I > III > II (d) I > II > III
26. Following compounds are given: – CH – CH –
(1) CH3CH2OH (2) CH3COCH3 (d)
OH OH
CH 3 CHO H 29. Reduction of aldehydes and ketones into
(3) (4) CH3OH hydrocarbons using zinc amalgam and conc. HCl
CH 3 is called [2007]
Which of the above compound(s), on being (a) Cope reduction
warmed with iodine solution and NaOH, will (b) Dow reduction
give iodoform? [2010] (c) Wolff-Kishner reduction
(a) (1) and (2) (b) (1), (3) and (4) (d) Clemmensen reduction.
(c) only (2) (d) (1), (2) and (3) 30. Which one of the following on treatment with
27. Which one of the following compounds will 50% aqueous sodium hydroxide yields the
be most readily dehydrated? [2010] corresponding alcohol and acid? [2007]
OH (a) C6H5CHO (b) CH3CH2CH2CHO
O
(a) ||
CH3 (c) CH 3 C CH 3 (d) C6H5CH2CHO
O O
31. The product formed in Aldol condensation is
[2007]
(b) CH
3 (a) A beta-hydroxy aldehyde or a beta-hydroxy
O OH ketone
(b) An alpha-hydroxy aldehyde or ketone
(c) (c) An alpha, beta unsaturated ester
H3 C (d) A beta-hydroxy acid
OH
32. A carbonyl compound reacts with hydrogen
O OH
cyan ide to form cyan ohydrin which on
hydrolysis forms a racemic mixture of -hydroxy
(d)
CH3 acid. The carbonyl compound is [2006]
(a) Acetone (b) Diethyl ketone
(c) Formaldehyde (d) Acetaldehyde
EBD_7324
îíì ÝØÛÓ×ÍÌÎÇ
33. Nucleophilic addition reaction will be most 36. In the reaction
favoured in [2006] CH 3CHO HCN CH 3CH (OH) CN
(a) (CH3)2C = O
H.OH
(b) CH3CH2CHO CH 3 CH (OH )COOH
(c) CH3CHO an asymmetric centre is generated. The acid
O obtained would be [2003]
||
(d) CH 3 — CH 2 — CH 2C — CH 3 (a) 20 % D + 80 % L-isomer
(b) D-isomer
34. A and B in the following reactions are [2003]
(c) L-isomer
OH (d) 50% D + 50% L-isomer
B R– C
R–C–R' HCN/ A 37. Which of the following is correct? [2001]
KCN CH2NH2
O R' (a) Diastase is an enzyme
CN (b) Acetophenone is an ether
(a) A = RR'C , B = LiAlH4 (c) Cycloheptane is an aromatic compound
OH (d) All the above
38. Which of the following is incorrect? [2001]
OH (a) NaHSO3 is used in detection of carbonyl
(b) A = RR'C , B = NH3 compound
COOH
(b) FeCl3 is used in detection of phenolic group
CN (c) Tollens’ reagent is used in detection of
(c) A = RR'C , B H3O unsaturation
OH (d) Fehling solution is used in detection of
glucose
(d) A = RR'CH2CN, B = NaOH 39. Polarization of electrons in acrolein may be written
35. When m-chlorobenzaldehyde is treated with as: [2000]
50% KOH solution, the product(s) obtained is – +
(are) [2003] (a) CH2 =CH — CH= O
+ –
OH OH
(b) CH 2 = CH—CH=O
CH CH
(a)
(c) CH 2 = CH — CH= O
OH OH
(d) CH 2 C H — CH O
COO CH2OH
+ 40. During reduction of aldehydes with hydrazine
(b) and potassium hydroxide, the first is the formation
of : [2000]
OH OH

(a) R — CH — N — NH 2
COO CH2OH
(c) + (b) R — C N
(c) R — C — NH 2
Cl Cl ||
O
OH OH (d) R — CH—
— NH
CH CH
(d) 41. Iodoform test is not given by [1999]
(a) 2-Pentanone (b) Ethanol
Cl Cl (c) Ethanal (d) 3-Pentanone
ß´¼»¸§¼»­ô Õ»¬±²»­ô ¿²¼ Ý¿®¾±¨§´·½ ¿½·¼­ îíë

42. Reaction of phenylacetylene with dil. H2SO4 and (c) Free radicals only
HgSO4 gives [1999] (d) Both electrophiles and nucleophiles.
(a) Acetophenone 51. The reagent (s) which can be used to distinguish
(b) 2-Phenylethanol acetophenone from benzophenone is (are)
(c) Phenylacetaldehyde [1990]
(d) Phenylacetic acid (a) 2,4- Dinitrophenylhydrazine
43. The cyanohydrin of a compound on hydrolysis (b) Aqueous solution of NaHSO3
gives an optically active -hydroxy acid. The (c) Benedict reagent
compound is [1999] (d) I2and NaOH.
(a) Diethyl ketone (b) Formaldehyde
CH 2 O
(c) Acetaldehyde (d) Acetone
44. Phenylmethyl ketone can be converted into 52. O CH 2
ethylbezene in one step by which of the following CH 2 O
reagents? [1999] The above shown polymer is obtained when a
(a) LiAlH4 (b) Zn-Hg/HCl carbonyl compound is allowed to stand. It is a
(c) NaBH4 (d) CH3MgI white solid. The polymer is [1989]
45. (CH 3 ) 3 C—CHO does not undergo Aldol (a) Trioxane (b) Formose
condensation due to [1996] (c) Paraformaldehyde (d) Metaldehyde.
(a) three electron donating methyl groups
(b) cleavage taking place between —C— CHO 53. CH 3
bond
(c) absence of alpha hydrogen atom in the
molecule
(d) bulky (CH3)3 C—group CH 3 CH 3
46. Acetone reacts with iodine (I 2) to form iodoform
in the presence of [1995] The above compound describes a condensation
(a) CaCO3 (b) NaOH polymer which can be obtained in two ways :
(c) KOH (d) MgCO3 either treating 3 molecul es of acetone
47. Benzaldehyde reacts with ethanoic KCN to give (CH3 COCH3 ) with conc. H2SO 4 or passing
[1994] propyne (CH3 C CH) through a red hot tube.
(a) C6H5CHOHCN The polymer is [1989]
(b) C6H5CHOHCOC6H5 (a) Phorone
(c) C6H5CHOHCOOH (b) Mesityl oxide
(d) C6H5CHOHCHOHC6H5 (c) Deacetonyl alcohol
48. Aldehydes and ketones will not form crystalline (d) Mesitylene.
derivatives with [1994] HCl
(a) Sodium bisulphite 54. 3CH 3COCH 3
3H 2O
(b) Phenylhydrazine
(c) Semicarbazide hydrochloride (A)
(d) Dihydrogen sodium phosphate. (CH 3 )2 C CH CO CH C(CH 3) 2
49. Which of the following compound will undergo (B)
self aldol condensation in the presence of cold This polymer (B) is obtained when acetone is
dilute alkali ? [1994] saturated with hydrogen chloride gas, B can be
(a) CH 2 CH CHO (b) CH C CHO [1989]
(a) phorone
(c) C 6 H 5CHO (d) CH 3CH 2CHO.
(b) formose
50. Acetaldehyde reacts with [1991] (c) diacetone alcohol
(a) Electrophiles only (d) mesityl oxide.
(b) Nucleophiles only
EBD_7324
îíê ÝØÛÓ×ÍÌÎÇ
55. If formaldehyde and KOH are heated, then we Code :
get [1988] (A) (B) (C) (D)
(a) Methane (b) Methyl alcohol (a) (iv) (i) (iii) (ii)
(c) Ethyl formate (d) Acetylene. (b) (iv) (ii) (iii) (i)
56. Formalin is an aqueous solution of [1988] (c) (ii) (iii) (iv) (i)
(a) Fluorescein (b) Formic acid (d) (ii) (i) (iv) (iii)
(c) Formaldehyde (d) Furfuraldehyde. 60. An organic compound ‘A’ on treatment with NH3
gives ‘B’ which on heating gives ‘C’, ‘C’ when
̱°·½ íæ Ю»°¿®¿¬·±² ¿²¼ Ю±°»®¬·»­ ±º treated with Br 2 in the presence of KOH
Ý¿®¾±¨§´·½ ß½·¼­ produces ethylamine. Compound ‘A’ is:
[2011 M]
57. Which one of the following esters gets (a) CH3COOH
hydrolysed most easily under alkaline (b) CH3 CH2 CH2 COOH
conditions? [2015 RS] (c) CH3 – CHCOOH
OCOCH3 CH3
(a) (d) CH3CH2COOH
ON 61. In a set of reactions, ethylbenzene yielded a
2
product D. [2010]
OCOCH3
CH2CH3 KMnO 4 Br2
KOH B
(b) FeCl3
H3CO
C2H 5OH
C D
OCOCH3 H
(c) CH 2 – CH – COOC 2H 5
(a)
Br
OCOCH3
Br
(d)
Cl
(b)
58. The correct order of decreasing acid strength of Br
trichloroacetic acid (A), trifluoroacetic acid (B),
CH2 COOC2H5
acetic acid (C) and formic acid (D) is : [2012]
(a) B > A > D > C (b) B > D > C > A COOH
(c) A > B > C > D (d) A > C > B > D
59. Match the compounds given in List-I with (c)
List-II and select the suitable option using the OCH2CH3
code given below : [2011 M]
List I List-II COOC2H5
(A) Benzaldehyde (i) Phenolphthalein
(B) Phthalic anhydride (ii) Benzoin
(d)
condensation
Br
(C) Phenyl benzoate (iii) Oil of wintergreen
(D) Methyl salicylate (iv) Fries 62. Propionic acid with Br 2|P yields a dibromo
rearrangement product. Its structure would be: [2009]
ß´¼»¸§¼»­ô Õ»¬±²»­ô ¿²¼ Ý¿®¾±¨§´·½ ¿½·¼­ îíé

Br 67. In a set of reactions acetic acid yielded a


| product D.
(a) H– C – CH2COOH
| SOCl2 Benzene
CH3COOH A B
Anhy.AlCl3
Br
HCN H.OH
(b) CH2Br – CH2 – COBr C D
Br The structure of D would be: [2005]
|
(c) CH3– C – COOH COOH
| CH2 – C – CH3
Br (a)
OH
(d) CH2 Br – CHBr – COOH
63. The relative reactivities of acyl compounds CN
towards nucleophilic substitution are in the order
C – CH3
of : [2008] (b)
(a) Acyl chloride > Acid anhydride > Ester > OH
Amide
OH
(b) Ester > Acyl chloride > Amide > Acid
anhydride CH2 – C – CH3
(c)
(c) Acid anhydride > Amide > Ester > Acyl CN
chloride
(d) Acyl chloride > Ester > Acid anhydride > OH
Amide C – COOH
(d)
64. Which of the following represents the correct CH3
order of the acidity in the given compounds?
68. The OH group of an alcohol or the –COOH
[2007]
group of a carboxylic acid can be replaced by
(a) FCH2COOH > CH3COOH > BrCH2COOH > –Cl using [2004]
ClCH2COOH (a) Phosphorus pentachloride
(b) BrCH2COOH > ClCH2COOH > FCH2COOH (b) Hypochlorous acid
> CH3COOH (c) Chlorine
(c) FCH2COOH > ClCH2COOH > BrCH2COOH (d) Hydrochloric acid
> CH3COOH 69. In a set of the given reactions, acetic acid yielded
(d) CH3 COOH > BrCH2COOH > ClCH2COOH > a product C.
FCH2COOH C6 H6
CH 3COOH PCl5 A B
65. Self condensation of two moles of ethyl acetate Anh.AlCl3
in presence of sodium ethoxide yields [2006] C 2H 5MgBr
C
(a) acetoacetic ester Ether
(b) methyl acetoacetate Product C would be - [2003]
(c) ethyl propionate C2 H 5
(d) ethyl butyrate |
(a) CH 3 C ( OH )C 6 H 5
66. In a set of reactions propionic acid yielded a
compound D. [2006] (b) CH 3CH(OH)C 2 H 5
SOCl2 NH3 KOH
CH 3CH 2 COOH B C D
Br2 (c) CH 3COC 6 H 5
The structure of D would be
(d) CH 3CH(OH)C6 H 5
(a) CH3CH2CONH2 (b) CH3CH2NHCH3
(c) CH3CH2NH2 (d) CH3CH2CH2NH2
EBD_7324
îíè ÝØÛÓ×ÍÌÎÇ
70. 75. Which one of the following esters cannot
MgBr
undergo Claisen self-condensation? [1998]
(a) CH 3 CH 2 CH 2 CH 2 COOC 2 H 5
( i ) CO 2
P (b) C6H5COOC2H5
( ii ) H 3O (c) C6H5CH2COOC2H5
In the above reaction product 'P' is [2002] (d) C6H11CH2COOC2H5
76. Consider the following transformations :
CHO COOH
CaCO 3 heat I2
CH3COOH A B C
NaOH
(a) (b) The molecular formula of C is [1996]

OH
|
OH (a) CH 3 C CH 3 (b) ICH2 — COCH3
O |
|| I
(c) (d) C6 H5 C C 6 H5 (c) CHI3 (d) CH3I
77. Formic acid is obtained when [1994]
71. Benzoic acid may be converted to ethyl benzoate (a) Calcium acetate is heated with conc. H2SO4
by reaction with : [2000] (b) Calcium formate is heated with calcium
(a) Sodium ethoxide acetate
(b) Ethyl chloride (c) Glycerol is heated with oxalic acid at 373 K
(c) Dry HCl—C2H5OH (d) Acetaldehyde is oxidised with K 2Cr2O7 and
H2SO4.
(d) Ethanol
72. Acetaldehyde reacts with semicarbazide and 78. The preparation of ethyl acetoacetate involves
forms semicarbazone. Its structure is [1999] [1994]
(a) CH3CH = NNHCON = CHCH3 (a) Wittig reaction
(b) CH3CH = NNHCONH2 (b) Cannizzaro’s reaction
(c) Reformatsky reaction
(c) CH 3 CH N — N — CONH 2 (d) Claisen condensation.
|
OH 79. Schotten-Baumann reaction is a reaction of
(d) CH3CH = N—CONHNH2 phenols with [1994]
73. Aspirin is an acetylation product of [1998] (a) Benzoyl chloride and sodium hydroxide
(b) Acetyl chloride and sodium hydroxide
(a) o-Hydroxybenzoic acid
(c) Salicylic acid and conc. H2SO4
(b) o-Dihydroxybenzene
(d) Acetyl chloride and conc H2SO4
(c) m-Hydroxybenzoic acid
80. An ester is boiled with KOH. The product is
(d) p-Dihydroxybenzene
cooled and acidified with concentrated HCl. A
74. An ester (A) with molecular fomula, C9H10O2 was
white crystalline acid separates. The ester is
treated with excess of CH3MgBr and the complex
[1994]
so formed was treated with H2SO4 to give an
(a) Methyl acetate (b) Ethyl acetate
olefin (B). Ozonolysis of (B) gave a ketone with
(c) Ethyl formate (d) Ethyl benzoate
molecular formula C8H8O which shows +ve
81. Sodium formate on heating yields [1993]
iodoform test. The structure of (A) is [1998]
(a) Oxalic acid and H2
(a) C6H5COOC2H5
(b) Sodium oxalate and H2
(b) C2H5COOC6H5
(c) CO2 and NaOH
(c) H3COCH2COC6H5
(d) Sodium oxalate.
(d) p — H 3CO — C 6 H 4 — COCH 3
ß´¼»¸§¼»­ô Õ»¬±²»­ô ¿²¼ Ý¿®¾±¨§´·½ ¿½·¼­ îíç

82. Among acetic acid, phenol and n-hexanol, which (a) Soda-lime and copper
of the following compounds will react with (b) Zn dust and NaOH
NaHCO3 solution to give sodium salt and carbon (c) Zn dust and soda-lime
dioxide ? [1993] (d) Soda-lime and zinc dust.
(a) Acetic acid 86. The compound formed when malonic ester is
(b) n-Hexanol heated with urea is [1989]
(c) acetic acid and phenol (a) Cinnamic acid (b) Butyric acid
(d) Phenol. (c) Barbituric acid (d) Crotonic acid.
83. (CH3)2 C = CHCOCH3 can be oxidized to 87. Which of the following is the correct decreasing
(CH3)2C = CHCOOH by [1993] order of acidic strength of [1988]
(a) Chromic acid (b) NaOI (i) methanoic acid (ii) ethanoic acid
(c) Cu at 300°C (d) KMnO4.
(iii) propanoic acid (iv) butanoic acid.
84. In which of the following, the number of carbon
(a) (i) > (ii) > (iii) > (iv) (b) (ii) > (iii) > (ii) > (i)
atoms does not remain same when carboxylic acid
is obtained by oxidation [1992] (c) (i) > (iv) > (iii) > (ii) (d) (iv) > (i) > (iii) > (ii)
88. Among the following the strongest acid is [1988]
(a) CH 3COCH 3 (b) CCl 3CH 2 CHO
(a) CH 3 COOH
(c) CH 3CH 2 CH 2OH (d) CH 3CH 2CHO.
(b) CH 2 ClCH 2COOH
85. Benzoic acid gives benzene on being heated with
X and phenol gives benzene on being heated (c) CH 2 ClCOOH
with Y. Therefore X and Y are respectively [1992]
(d) CH 3CH 2 COOH.

ANS W ER KEY
1 (c) 10 (a) 19 (c) 28 (a) 37 (a) 46 (b) 55 (b) 64 (c) 73 (a) 82 (a)
2 (a) 11 (d) 20 (d) 29 (d) 38 (c) 47 (b) 56 (c) 65 (a) 74 (a) 83 (b)
3 (b) 12 (a) 21 (b) 30 (a) 39 (c) 48 (d) 57 (a) 66 (c) 75 (b) 84 (a)
4 (a) 13 (b) 22 (d) 31 (a) 40 (a) 49 (d) 58 (a) 67 (d) 76 (c) 85 (d)
5 (c) 14 (d) 23 (a, d) 32 (d) 41 (d) 50 (b) 59 (d) 68 (a) 77 (c) 86 (c)
6 (b) 15 (d) 24 (b) 33 (c) 42 (a) 51 (d) 60 (d) 69 (a) 78 (d) 87 (a)
7 (c) 16 (b) 25 (d) 34 (a) 43 (c) 52 (a) 61 (d) 70 (b) 79 (a) 88 (c)
8 (b) 17 (d) 26 (d) 35 (c) 44 (b) 53 (d) 62 (c) 71 (c) 80 (d)
9 (b) 18 (a) 27 (d) 36 (d) 45 (c) 54 (a) 63 (a) 72 (b) 81 (b)
EBD_7324
îìð ÝØÛÓ×ÍÌÎÇ

Hints & Solutions


1. (c) Zn/Hg and conc. HCl reduce carboxyl 6. (b) Catalyst used in Rosenmund reduction is
group to meth ylene group (Clemmensen Pd/BaSO4. Rosenmund Reduction is used for
reduction). reduction of acid chloride.
O O O
O
Pd / BaSO4
C Cl CH R C Cl R C H
2. (a) H2 7. (c) By oxidation of tertiary alcohol with stronger
Pd–BaSO 4 oxidising agents ketones may be formed along
with carboxylic acid.
It is Rosenmund reaction.
3. (b) 4[O]
CH3COCH3+CO2+2H2O
OH OH
(CH3)3 COH
CHO 8[O]
CHCl3 CH3COOH+2CO 2+3H2O
(a) NaOH
8. (b) Pinacolone is 3,3-dimethyl-2 butanone.
Riemer-Tieman in CH3
reaction |
CH3 C C CH 3
O | ||
||
NaOH CH3 O
(b) HCH CH3OH + HCOONa
Cannizzaro reaction 9. (b) Since 'A' gives positive silver mirror test
Na therefore, it must be an aldehyde of
(c) 2CH3Cl CH3 – CH3 -Hydroxyketone. Also, reaction with OH– i.e.,
dry ether
Wurtz reaction aldol condensation (by assuming alkali to be
dilute) indicates that A is aldehyde as aldol
COCH3 reaction of ketones is reversible and carried out
in special apparatus. It indicates that A is an
CH3COCl
(d) anh.AlCl3
aldehyde
Cu
CH 3 CH 2 OH CH3 CHO
Friedel-Craft acylation 573K
(X)
Note that no new C–C bond is formed in
Cannizzaro reaction. Ag NH3 OH
2
silver mirror observed
4. (a) Carbonyl compounds (aldehydes and
ketones) are obtained by the oxidation of 1° and (A)
ethanal
2° alcohols respectively. Among the given
options, only (a) is 2° alcohol hence it can be
O
oxidized to ketone. –
OH
H2 N NH C NH 2 OH, |
OH O CH3 CH CH 2 CHO
3-Hydroxybutanal
| oxidation ||
H3C – CH – CH3 H3C – C – CH3 O

2-hydroxypropane Acetone CH3 CH N NH C NH 2 CH3 CH CH CHO


(Z) (Y)
Semicarbazone But-2-enal
H2
5. (c) R — C — Cl R — C — H + HCl
Pd – BaSO4
O O
‘P’
ß´¼»¸§¼»­ô Õ»¬±²»­ô ¿²¼ Ý¿®¾±¨§´·½ ¿½·¼­ îìï
10. (a) H H
O O
H H C C C C O
(i) OH H H
+H H O H
(ii)
O C C H
O
11. (d) Keto-enol tautomerism is possible only in H H
those aldehydes and ketones which have at least O
one -hydrogen atom, which can convert the ||
H
ketonic group to the enolic group. e.g. 16. (b) R — C— R1 NH 2 — NH 2
O O NH2
|| || OH N
CH3 — C — CH2 — C CH3 1 H2O
Ketonic form R—C—R R—C—R
Elimination
OH O
| || NH—NH2
CH 3 — C CH — C — CH 3 (Addition)
enolic form While in all other case no elimination take place.
12. (a) Schiff base is formed when 1° amine reacts 17. (d) Any substituent in the carbonyl
with aldehydes. compound that increases the positive charge
R R on the carbonyl carbon will increase reactivity
H
C = O + R' — NH2 C = N – R' towards nucleophilic addition.
H H —NO2 shows –M effect hence
Aldehyde + primary amine Schiff base
CHO
13. (b) H 3C— CH 2— C— CH 2— CH 3
O is most reactive towards nucleophilic
does not give iodoform test due to absence of addition reaction.
O NO 2
(CH3—C—) group. It also does, not give 18. (a) Enolic form predominates in compounds
Tollen’s test. containing two carbonyl groups separated by a
– CH2 group. This is due to following two
On reduction it gives n-pentane factors.
reduction (i) Presence of conjugation which increases
H3C—CH2—C—CH2—CH3 stability.
Zn-Hg/HCl
O or NH2-NH2/OH

(ii) Formation of intramolecular hydrogen
bond between enolic hydroxyl group and
CH3—CH2—CH2—CH2—CH3 second carbonyl group which leads to
n-pentane stablisation of the molecule. Hence the cor-
rect answer is III > II > I.
14. (d) – 19. (c) Cannizzaro reaction - when an aldehyde
O O
CH3 containing no – H undergo reaction in presence
of 50% KOH. It disproportionates to form a molecule

+ CH3 Li of carboxylic acid and a molecule of alcohol.
O
Cylopentanoyl anion
CHO CH2–OH
C–O
15. (d) Enolic form of ethyl acetoacetate has 18
sigma and 2 pi-bonds as shown below: 50 % KOH õ

Cl Cl
Cl
EBD_7324
îìî ÝØÛÓ×ÍÌÎÇ
20. (d) Anhydrous alcohols add to the carbonyl carboxyl carbon atom in the transition state
group of aldehydes in the presence of anhydrous (steric factor). Both these factors predict the
hydrogen chloride to form acetals via hemiacetals. following order
OC2H5 CH3 CH3 Ph
C=O> C=O> C=O
C2H 5OH
CH3 C CH3 CH3 C CH3 H CH3 Ph
(due to steric crowding).
O OH 26. (d) Among th e given compounds only
Hemiacetal CH3OH does not give iodoform reaction.
27. (d) The intermediate is carbocation which is
OC2H5 destabilised by C = O group (present on -
C2H5OH carbon to the –OH group) in the first three cases.
CH3 C CH3 In (d), –hydrogen is more acidic which can be
removed as water. Moreover, the positive charge
OC2H5
on the intermediate carbocation is relatively away
Acetal from the C = O group.
21. (b) CH 3 CHO gives Iodoform t est but O O
C6H5CH2CHO does not give Iodoform test due –
to absence of methyl group OH
28. (a) C6H5 – C + H.CH2– C
22. (d) R – CH = O + H2N – NH2 CH3 C6H5
R – CH = N – NH2
Such reactions take place in slightly acidic OH O
medium and involve nucleophilic addition of the +
ammonia derivative. H /Heat
C6H5 – C – CH2 – C
–H2O
23. (a, d) The compounds with CH3 – C – group
|| CH3 C6H5
O O
or CH3 — CH — group give iodoform test.
| C6H5 – C = CH – C– C6H5
OH
Hence Acetophenone CH3
Thus two molecules of acetophenone
condense to form a -hydroxy ketone which gets
CH3 C dehydrated in the presence of acid upon heating
and 2-Hydroxypropane to form -unsaturated compound.
O i.e., option (a) is correct.
[Note: It is aldol condensation].
CH3 CH CH3
Zn-Hg / conc. HCl
both give a yellow precipitate 29. (d) C=O CH2 + H2O
OH
of CHI3 (iodoform) with iodine and alkali. Clemmensen reduction
24. (b) Clemmensen reduction is e.g.
Zn–Hg/HCl CH3 Zn-Hg / CH3
C=O CH2 conc. HCl
C=O CH2 + H2O
25. (d) The reactivity of the carbonyl group CH3 CH3
towards the nucleophilic addition reactions 30. (a) Aldehydes containing no -hydrogen
depend upon the magnitude of the positive atom on warming with 50% NaOH or KOH
charge on the carbonyl carbon atom (electronic undergo disproportionation i.e. self oxidation -
factor) and also on the crowding around the reduction known as cannizzaro’s reaction.
ß´¼»¸§¼»­ô Õ»¬±²»­ô ¿²¼ Ý¿®¾±¨§´·½ ¿½·¼­ îìí

50% NaOH 35. (c) It is a simple Cannizzaro reaction.


2 C6 H5 CHO NaOH –
CHO COO CH2OH
C6 H5COONa C6 H5CH 2 OH
31. (a) Aldehydes and ketones having at least one 2 50% KOH +
-hydrogen atom in presence of dilute alkali give Cl Cl Cl
-hydroxy aldehyde or -hydroxy ketone.
H H
O | |
|| 36. (d) CH 3 C O HCN CH 3 C OH
CH3 C HCH 2 CHO |
| CN
H
Acetaldehyde
H H
| |
OH H / H 2O
OH C CH 3 CH 3 C OH
| | |
dil.NaOH
CH3 C CH 2 CHO COOH COOH
| 50% L Lactic 50% D Lactic
H acid acid
Aldol
37. (a) Acetophenone is a ketone, cyclopentane
CH3 CH CH.CHO doesn’t contain (4n + 2) electron hence is not
H 2O Crotonaldehyde aromatic. Diastase is an enzyme used in the
32. (d) Out of given compound only acetaldehyde preparation of Maltose (Malt sugar,
can form optical active hydroxy acid as it C12H22O11) through hydrolysis of starch.
contains one asymmetric carbon atom as marked 38. (c) Tollen's reagent is used to detect aldehydic
below : group. Tollen's reagent is an ammonical solution
of silver nitrate. When aldehyde is added to
O OH Tollen's reagent, silver oxide is reduced to metallic
|| |
CH3 C H
HCN
CH3 C CN silver which deposits as mirror.
|
CH RCHO + Ag2O RCOOH + 2Ag
39. (c) In CH 2 CH — CHO due to – M effect of
O
hydrolysis
| — CHO group polarization of electron takes
CH3 C* H place as follows
|
COOH + –
CH 2 CH — C O CH 2 — CH C — O
33. (c) Aldehydes are more reactive than ketones | |
due to +I effect of –CH3 group. There are two – H H
CH3 group in acetone which reduces +ve charge Hence partial polarization is represented as
density on carbon atom of carbonyl group. More
hindered carbonyl group too becomes less C H2 CH — CH O
reactive. So in the given case CH3CHO is the 40. (a)
right choice.
O
O OH
|| | R C H + NH2 NH2 R C N NH2
HCN
34. (a) R– C –R R — C — CN H
KCN | Aldehyde Hydrazine Aldehyde hydrazone
R (A)
41. (d) Iodoform test is exhibited by ethyl alcohol
OH acetaldehyde, acetone, methyl ketones and those
|
Reduction by
R– C – CH 2 NH 2 alcohols which possess CH3CH(OH)-group. As
LiAlH 4 (B) | 3-pentanone does not contain CH3CO-group
R therefore it does not give iodoform test.
EBD_7324
îìì ÝØÛÓ×ÍÌÎÇ
42. (a) 48. (d) Dihydrogen sodium phosphate (NaH2PO4)
OH O does not have a lone pair of electrons on the P
C CH atom hence can not act as a nucleophile and
C CH 2 C CH3
hence does not react with aldehydes and
H 2SO 4 /HgSO 4 ketones.
49. (d) Aldehydes which contain a -hydrogen on a
enolform Ketoform saturated carbon, e.g. undergo aldol condensation.

O OH H CH3
| |
43. (c) CH3 C H + HCN CH3 C H CH3CH 2 – C O H H C CHO
CN propanol
Hydrolysis
CH3 CH COOH
CH3 CH3
OH OH
| |
2-Hydroxy propanol CH3CH 2 C CHCHO
|
(As it has a chiral C-atom thus it is optically OH
active) 3.hydroxy,2 methyl pentanal

O 50. (b) Acetaldehyde reacts only with nucleophiles.


C–CH3 CH2–CH3 Since the mobile electrons of carbon–oxygen
double bond are strongly pulled towards oxygen,
Zn-Hg/HCl carbonyl carbon is electron-deficient and carbonyl
44. (b) oxygen is electron-rich. Thus the electron deficient
Phenyl methyl ethyl (acidic) carbonyl carbon is most susceptible to
ketone benzene attack by electron rich nucleophilic reagents, that
is, by base. Hence the typical reaction of aldehydes
This reaction is known as clemmensen's
and ketones is nucleophilic addition.
reduction.
45. (c) Aldol condensation is given by the O OH

compounds which contain -hydrogen atom. As C = O + Nu C C
the given compound does not contain Nu Nu
hydrogen atom. Hence it does not undergo aldol 51. (d) I2 and NaOH react with acetophenone
condensation. (C6H5COCH3) to give yellow ppt. of CHI3 but
benzophenone (C6H5 COC6H5) does not and
46. (b) CH 3 COCH 3 3I 2 4NaOH
Acetone hence can be used to distinguish between them.
52. (a)
CHI 3 3NaI CH 3COONa 3H 2 O
Iodoform
CH2
O O
Thus acetone reacts with iodine to form On keeping
iodoform in the presence of NaOH. 3 HCHO CH2
CH2
47. (b) When benzaldehyde is refluxed with aqueous aq. solution O
alcoholic potassium cyanide, two molecules of Trioxane
benzaldehyde condense together to form benzoin (meta formaldehyde)
53. (d) Acetone forms mesitylene (1,3,5-trimethyl
H O
KCN (alc) benzene) on distillation with conc. H2SO4.
—C +C— 54. (a) When 3 molecules are combined in presence
O H of dry HCl gas they condense to form phorone.
H O CH3 CH3
—C—C— C = O + H2 CHCOCH H2 + O = C
OH CH3 CH3
ß´¼»¸§¼»­ô Õ»¬±²»­ô ¿²¼ Ý¿®¾±¨§´·½ ¿½·¼­ îìë

59. (d)
CH 3 CH 3
HCl KCN
C CHCOCH C (a) 2C6H5CHO
–2H 2O H2 O, C2H5 OH
CH 3 CH 3 OH
Phorone
C6H5–CH–C–C6H5
Note : Two molecules of acetone condense
to form mesityl oxide. O
(Benzoin)
CH3 O O
C = O + H2 CHCOCH3 (b) O H+
O
CH3
H O H
CH3
C = CH.CO.CH3
CH3
OH OH OH OH
55. (b) Aldehydes containing no -hydrogen atom Phenolphthalein
on warming with 50% NaOH or KOH undergo (c) The fries rearrangement enables the
disproportionation i.e. self oxidation - reduction preparation of acylphenols. The reaction is
known as Cannizzaro’s reaction. catalysed by Bronsted or lewis acid such as AlCl3
O
50%NaOH
2 HCHO HCOONa CH 3OH
O R AlCl3
56. (c) Formalin is an aqueous solution (40%) of
formaldehyde.
57. (a) Among the substituent attached to the
benzene ring, –NO2 group is the most electron Fries rearrangement
withdrawing, thus withdraws electron density
from carbonyl carbon thus facilitate the attack
– +
of OH ion.
O O
. . || ||
O – C – CH3 O – C – CH3 COOCH3
OH
(d) (Oil of wintergreen)

60. (d) Since, C when heated with Br2 in presence


N N of KOH produces ethylamine, hence it must be
– – –
O O O O propanamide and hence the organic compound
(–R effect of –NO2 group) (A) will be propanoic acid. The reactions follows.
58. (a) CF3 COOH > CCl3 COOH > HCOOH > CH3 CH 2 COOH
NH3
CH3COOH (Ka order) (A)
The halogenated fatty acids are much stronger
acids than the parent fatty acid and more over CH3 CH 2 CO O N H 4
the acidity among the halogenated fatty acid (B)
increases almost proportionely with the increase KOH Br2
in electronegativity of the halogen present. CH 3 CH 2 CONH 2
Hoffmann
Further formic acid having no alky group is more (C) bromamide
reaction
acidic than acetic acid.
CH 3 CH 2 NH 2
(Ethylamine )
EBD_7324
îìê ÝØÛÓ×ÍÌÎÇ
61. (d)
O O
COOH || ||
CH2 – CH3
CH 3 C CH 2 C OC 2 H 5
(ethyl acetoacetate)
KMnO 4
KOH SOCl 2
66. (c) CH 3 CH 2 COOH
(B)
NH 3
CH 3 CH 2 COCl
COOH COOC2H5
KOH
Br2 CH3 CH 2 CONH2 CH3 CH 2 NH 2
C2 H 5OH Br2
FeCl3 H
Br Br SOCl 2
(C) (D) 67. (d) CH 3COOH CH 3 COCl

62. (c) This reaction is an example of Hell - Volhard Friedel Craft


reaction C6 H6 / AlCl3
Zelinsky reaction. In this reaction acids
containing – H on treatment with X2 /P give
di-halo substituted acid.
Br2 /P
CH3 – CH 2 COOH CH 3 CBr2 COOH

63. (a) The more the basic character of the leaving


group, the lesser is the reactivity the basic
character follows the order
NH2– > OR– > RCOO – > Cl–
Hence, the relative reactivities of acyl
compounds towards nucleophilic substitution
follow the order Acyl halides > Acid anhydride
> Ester > Amide.
64. (c) Electron withdrawing substituents (like
halogen, – NO2, – C6H5 etc.) would disperse the
negative charge and hence stabilise the 68. (a) Among the given options, only PCl 5 can
carboxylate ion and thus increase acidity of the convert an alcoholic group as well as a carboxyl
parent acid. On the other hand, electron- group to chloride.
releasing substituents would intensify the RCH 2OH
PCl5
RCH 2Cl
negative charge, destabilise the carboxylate ion
and thus decrease acidity of the parent acid. PCl5
RCOOH RCOCl
Electronegativity decreases in order
F > Cl > Br 69. (a)
and hence –I effect also decreases in the COCH3
same order, therefore the correct option is C6 H6
[FCH2COOH > ClCH2COOH > CH 3COOH +PCl5 CH3COCl AnhydAlCl
3
BrCH2COOH > CH3COOH] [A]
Friedel Craft
65. (a) It is an example of Claisen condensation. The reaction
product is acetoacetic ester. OH OMgBr
H+ MgBrC2H 5
O O C2H5 – C – CH3 C2H5 – C – CH3
|| || ether
CH 3 C OC 2 H 5 H CH 2 C OC 2 H 5 hydrolysis

(C)
ß´¼»¸§¼»­ô Õ»¬±²»­ô ¿²¼ Ý¿®¾±¨§´·½ ¿½·¼­ îìé

70. (b) Grignard reagent forms addition product with


O
bubbled carbondioxide which on hydrolysis with ||
Mg(OC2 H5 )Br Excess
HCl yields benzoic acid. C6 H5 C CH3
CH3MgBr
O
OMgBr CH3
MgBr C OMgBr H 2O
+
C6 H5 C CH3 C6 H5 C CH3
(i) CO2 H3 O
CH3 OH
CH3
|
Conc. H 2SO 4 Ozonolysis
C6 H 5 — C CH 2
O
'B'
C O H C 6 H 5COCH 3 HCHO

2 3I 4 NaOH
+ Mg(OH)Br C 6 H 5 COCH 3 CHI 3
75. (b) The ester having -hydrogen atom show
'P' Claisen condensation reaction. We know that
Benzoic acid
ethyl benzoate (C6H5COOC2H5) does not contain
-hydrogen. Therefore C6H5COOC2H5 does not
COOH COOC2H5 undergo Claisen self condensation.
HCl 76. (c) CH 3COOH CaCO 3 (CH 3COO ) 2 Ca
71. (c) + C2H5OH + H2O
Ethanol
dry Heat CH 3
CO
CH 3
This process is known as esterification.
I 2 NaOH
CHI3 NaI CH 3 COONa 3H 2 O
O 77. (c) When glycerol is heated with oxalic acid
( H 2O )
72. (b) CH3–C=O + H2N–NH–C–NH2 following reaction occurs.
O
H CH2OH HOOC CH2OC.COOH
CH3–CH=N–NH–C–NH2
| | |
acetaldehyde semicarbazone 100 110 C
CHOH HOOC CHOH
| oxalic H2O |
CH2OH acid CH2OH
73. (a)
OH OCOCH3 CH2 CHO CH2OH
COOH COOH | |
CO 2 H2O
+ ClCOCH3
Pyridine CHOH CHOH HCOOH
| | Formic acid
O-hydroxy benzoic acid acetyl chloride CH2OH CH2OH
Aspirin
(Salicylic acid)
78. (d) In Claisen condensation Intermolecular
74. (a) condensation of esters containing -hydrogen
atom in presence of strong base form -keto ester
OMgBr CH3COO C2H5 + H.CH2.CO.OC2H5
| ethyl acetate
CH3MgBr
C6 H5COOC2 H5 C 6 H5 C OC2 H5
| O
A CH3 C2H5ONa
CH3C. CH2COOC2 H5+ C2H5OH
Ethyl acetoacetate
( -ketoester)
EBD_7324
îìè ÝØÛÓ×ÍÌÎÇ
79. (a) 84. (a) Ketones on oxidation give carboxylic acids
OH O.COC6H5
with lesser number of carbon atoms, i.e.,
[O]
aq. NaOH
CH 3 COCH 3 CH 3 COOH
+ C6H5COCl
CO 2 H 2O
phenyl benzoate 85. (d)
The function of NaOH is C 6 H 5 COOH
Soda lim e (X)
C6 H 6 Na 2 CO3 H2O
(i) To convert phen ol to more stronger
Zn dust (Y)
nucleophile PhO– C 6 H 5 OH C6 H 6 ZnO
(ii) To neutralize the acid formed
80. (d) Methyl acetate and ethyl acetate on
hydrolysis give CH3COOH which is a liquid. COOC2H5 H2N
Similarly ethyl formate on hydrolysis will give 86. (c) CH 2 + CO
formic acid which is also a liquid. Only ethyl COOC2H5 H2 N
benzoate on hydrolysis will give benzoic acid Malonic ester
which is a solid.
CO NH
COONa CH 2
81. (b) 2HCOONa | + H2 CO + C2H5OH
360
COONa CO NH
sod.oxalate Barbituric acid
82. (a) Among acetic acid, phenol and n-hexanol 87. (a) An electron releasing substituent (+I effect)
only CH3 COOH reacts with NaHCO3 to evolve intensify the negative charge on the anion
CO2 gas. resulting in the decrease of stability and thus
CH3COOH + NaHCO3 decreases the acidity of the acid. Hence acidic
character decreases as the + I-effect of the alkyl
CH3COONa + CO2 + H2O group increases as
83. (b) Chromic acid and KMnO4 will cleave the
CH 3 < CH 3CH 2 < CH 3CH 2CH 2
molecule at the site of double bond while Cu at
300°C cannot oxidise COCH3 COOH. The < CH3CH2CH2CH2
only reagent suitable for this conversion is NaOI Hence the order of acidic strength becomes:
or NaOH + I2 (iodoform test): (i) > (ii) > (iii) > (iv)
88. (c) Chlorine is electron withdrawing group.
NaOI
(CH 3 ) 2 C CHCOCH 3 Further Inductive effect is stronger at position
than -position. i.e.,
(CH 3 ) 2 C CHCOOH CHI 3 .

CH 2 ClCOOH CH 2 ClCH 2COOH


ß³·²»­ îìç

ݸ¿°¬»®

27 Amines

̱°·½ ïæ ß´·°¸¿¬·½ ¿²¼ ß®±³¿¬·½ ß³·²»­ alkylamines, beca use th e n i tr on gen


atom in arylamines is sp-hybridized.
1. The correct increasing order of basic strength
4. The electrolytic reduction of nitrobenzene in
for the following compounds is : [2017]
strongly acidic medium produces :- [2015]
NH2
(a) Azoxybenzene (b) Azobenzene
NH2
(c) Aniline (d) p-Aminophenol
(I) (II) 5. The number of structural isomers possible from
NO2 the molecular formula C3H9N is : [2015 RS]
NH2 (a) 4 (b) 5
(c) 2 (d) 3
6. Method by which Aniline cannot be prepared
(III)
is: [2015 RS]
CH3 (a) hydrolysis of phenylisocyanide with acidic
(a) III < I < II (b) III < II < I solution
(c) II < I < III (d) II < III < I (b) degradation of benzamide with bromine in
2. Which of the following reactions is appropriate alkaline solution
for converting acetamide to methanamine ?
(c) reduction of nitrobenzene with H2/Pd in
[2017]
ethanol
(a) Hoffmann hypobromamide reaction
(b) Stephens reaction (d) potassium salt of phthalimide treated with
(c) Gabriels phthalimide synthesis chlorobenzene followed by hydrolysis with
(d) Carbylamine reaction aqueous NaOH solution.
3. The correct statement regarding the basicity of 7. Some reactions of amines are given. Which one
arylamines is [2016] is not correct ? [NEET Kar. 2013]
(a) (CH3)2NH + NaNO2 + HCl
(a) Arylamines are generally less basic than
alkylamines because the nitrogen lone-pair (CH3)2 N – N = O
electrons are delocalized by interaction (b) (CH3)2N – + NaNO2 + HCl
with the aromatic ring electron system.
(b) Arylamines are generally more basic than (CH3)2N – – N = NCl
alkylamines because the nitrogen lone-pair
electrons are not delocalized by interaction (c) CH3CH2NH2 + HNO2 CH3CH2OH + N2
with the aromatic ring electron system. (d) CH3NH2 + C6H5SO2Cl
(c) Arylamines are generally more basic than CH3NHSO2C6H5.
alkylamines because of aryl group. 8. In a set of reactions m-bromobenzoic acid gave
(d) Arylamines are generally more basic than a product D. Identify the product D. [2011]
EBD_7324
îëð ÝØÛÓ×ÍÌÎÇ
COOH Options :
(1) (2) (3) (4)
SOCl2 NH3 NaOH
B C D (a) (iv) (ii) (iii) (i)
Br2
(b) (ii) (i) (iv) (iii)
Br (c) (iii) (ii) (i) (iv)
SO2NH2 COOH
(a) (b) (d) (ii) (iii) (i) (iv)
12. Predict the product: [2009]
Br NH2
NHCH 3+NaNO 2 + HCl Product
NH2 CONH2
(c) (d)
CH3
Br Br
9. Which of the following compounds is most basic? (a) N NO2
[2011 M]
NHCH3 NHCH3
(a) O2N NH2
NO
(b) +
(b) CH2NH2

NO
(c) N – COCH3 OH
H
(c) N CH3
(d) NH 2
10. Which of the following statements about primary CH3
amines is ‘False’ ? [2010] N N=O
(d)
(a) Alkyl amines are stronger bases than aryl
amines 13. Which of the following is more basic than
(b) Alkyl amines react with nitrous acid to aniline? [2006]
produce alcohols (a) Triphenylamine (b) p-Nitroaniline
(c) Aryl amines react with nitrous acid to (c) Benzylamine (d) Diphenylamine
produce phenols 14. The major organic product formed from the
(d) Alkyl amines are stronger bases than following reaction : [2005]
ammonia O
(i) CH3NH2
11. Match the compounds given in List - I with their ......
characteristic reactions given in List - II. Select (ii) LiAlH4, (iii) H2O
the correct option. [2010] is
List - I List - II O – NHCH3
(a)
Compounds Reactions
(1) CH3CH2CH2CH2NH2 (i) alkaline hydrolysis
(2) CH3C CH (ii) with KOH (alcohol) (b) NCH3
H
and CHCl3 produces H
bad smell NCH3
(3) CH3CH2COOCH3 (iii) gives white ppt. with (c)
ammonical AgNO3 OH
H
(4) CH3CH(OH)CH3 (iv) with Lucas reagent NCH3
cloudiness appears (d)
after 5 minutes OH
ß³·²»­ îëï

15. Electrolytic reduction of nitrobenzene in weakly 22. Mark the correct statement [1988]
acidic medium gives [2005] (a) Methylamine is slightly acidic
(a) NóPhenylhydroxylamine (b) Methylamine is less basic than ammonia
(b) Nitrosobenzene (c) Methylamine is a stronger base than ammonia
(c) Aniline (d) Methylamine forms salts with alkalies.
(d) póHydroxyaniline
16. The consituent of the powerful explosive RDX ̱°·½ îæ ß³·¼»­ô ݧ¿²·¼»­ ¿²¼ ×­±½§¿²·¼»­
is formed during the nitration of [2000] 23. The following reaction
(a) Toluene (b) Phenol
(c) Glycerol (d) Urotropine NH2
17. Which of the following is most basic in nature? + Cl
(a) NH3 (b) CH3NH2 [2000]
(c) (CH3)2NH (d) C6H5NHCH3 O
18. Aniline is reacted with bromine water and the H
resulting product is treated with an aqueous N
NaOH
solution of sodium nitrite in presence of dilute
O
hydrochloric acid. The compound so formed is
converted into a tetrafluoroborate which is
subsequently heated dry. The final product is is known by the name : [2015 RS]
(a) 1,3, 5-Tribromobenzene [1998] (a) Friedel-Craft's reaction
(b) p-Bromofluorobenzene
(b) Perkin's reaction
(c) p-Bromoaniline
(d) 2,4, 6-Tribromofluorobenzene (c) Acetylation reaction
19. The compound obtained by heating a mixture (d) Schotten-Baumen reaction
of a primary amine and chloroform with ethanolic 24. On h ydrolysis of a “compound”, two
potassium hydroxide (KOH) is [1997] compounds are obtained. One of which on
(a) an alkyl cyanide (b) a nitro compound treatment with sodium nitrite and hydrochloric
(c) an alkyl isocyanide (d) an amide acid gives a product which does not respond to
20. When aniline reacts with oil of bitter almonds
iodoform test. The second one reduces Tollen’s
(C 6 H 5CHO ) condensation takes place and reagent and Fehling’s solution. The
benzal derivative is formed. This is known as “compound” is [NEET Kar. 2013]
[1995] (a) CH3 CH2 CH2 CON(CH3)2
(a) Million's base (b) Schiff's reagent (b) CH3 CH2 CH2 NC
(c) Schiff's base (d) Benedict's reagent (c) CH3 CH2 CH2 CN
21. What is the decreasing order of basicity of (d) CH3 CH2 CH2 ON = O
primary, secondary and tertiary ethylamines and
NH3 ? [1994] 25. An organic compound (C3 H9 N) (A), when
treated with nitrous acid, gave an alcohol and
(a) NH 3 C 2 H 5 NH 2 (C 2 H 5 ) 2 NH N2 gas was evolved. (A) on warming with CHCl3
(C 2 H 5 )3 N and caustic potash gave (C) which on reduction
(b) (C2 H 5 )3 N (C2 H 5 )2 NH gave isopropylmethylamine. Predict the
structure of (A). [2012 M]
C2 H 5NH 2 NH 3
CH3
(c) (C2 H 5 ) 2 NH C2 H5 NH 2 (a) CH NH2
(C 2 H 5 )3 N NH 3 CH3
(b) CH3CH2 NH CH3
(d) (C 2 H 5 ) 2 NH (C 2 H 5 ) 3 N
(c) CH3 N CH3
C 2 H 5 NH 2 NH 3 .
CH3
(d) CH3CH2 CH2 NH2
EBD_7324
îëî ÝØÛÓ×ÍÌÎÇ
26. Acetamide is treated with the following reagents 30. Intermediates formed during reaction of
separately. Which one of these would yield R C NH 2 with Br2 and KOH are [2001]
||
methylamine? [2010] O
(a) NaOH – Br2 (b) Sodalime
(a) RNHBr and RCONHBr
(c) Hot conc. H2SO4 (d) PCl5
(b) RNHCOBr and RNCO
27. Which one of the following on reduction with
(c) RCONHBr and RNCO
lithium aluminium hydride yields a secondary
(d) RCONBr2
amine? [2007]
31. An isocyanide is prepared by [1999]
(a) Methyl isocyanide
(a) Friedel-Crafts reaction
(b) Acetamide
(b) Perkin reaction
(c) Methyl cyanide (c) Carbylamine reaction
(d) Nitroethane. (d) Wurtz reaction
28. The final product C, obtained in this reaction 32. Consider the following sequence of reactions :
NH2 Reduction HNO 2
Compound[A] [B] CH 3CH 2 OH
Ac2O Br2 H2O The compound [A] is [1996]
A B C
CH3COOH H
+
(a) CH3CH2CN (b) CH3NO2
(c) CH3NC (d) CH3CN
CH3
33. Aniline is an activated system for electrophilic
would be [2003] substitution. The compound formed on heating
NHCOCH3 NH2 aniline with acetic anhydride is [1996]
Br COCH3 NH2 NH2
(a) (b)
(a) (b)
CH3 CH3
COCH3
COCH3 NH2 COCH3
NH2 NHCOCH3
Br Br
(c) (d) (c) (d)
COCH3
CH3 CH3 34. Which is formed, when acetonitrile is hydrolysed
C N partially with cold concentrated HCl? [1995]
(a) Acetic acid (b) Acetamide
H 3O (c) Methyl cyanide (d) Acetic anhydrides
29. + CH3MgBr P
35. Acetamide and ethylamine can be distinguished
OCH3 by reacting with [1994]
Product 'P' in the above reaction is [2002] (a) Aqueous HCl and heat
(b) Aqueous NaOH and heat
OH O (c) Acidified KMnO4
CH – CH3 C – CH3 (d) Bromine water.
36. For carbylamine reaction, we need hot alcoholic
(a) (b) KOH and [1992]
(a) Any primary amine and chloroform
OCH3 OCH3 (b) Chloroform and silver powder
(c) A primary amine and an alkyl halide
CHO COOH (d) A monoalkylamine and trichloromethane.
37. Indicate which nitrogen compound amongst the
(c) (d) following would undergo Hoffmann reaction
OCH3
(i.e.., reaction with Br2 and strong KOH) to
OCH3
furnish the primary amine (R – NH2) [1989]
ß³·²»­ îëí

(a) RCONHCH 3 (b) RCOONH 4 42. What is the product obtained in the following
(c) RCONH2 (d) R – CO – NHOH. reaction : [2011]
NO2 Zn
̱°·½ íæ Ò·¬®±½±³°±«²¼­ô ß´µ§´ Ò·¬®·¬»­ ¿²¼ ........... ?
NH4Cl
Ü·¿¦±²·«³ Í¿´¬­

38. In the following reaction, the product (A) [2014] NHOH


+ –
(a)
N NCl NH2
N
+ H is :
A (b) N
Yellow dye


N=N–NH O
(a)
(c) N=N
NH2 +

(b) N=N NH2


(d)
NH2
43. Aniline in a set of the following reactions yielded
(c) N=N a coloured product ‘Y’ [2002, 2010]
NH2
NaNO 2 /HC1 N,N-dimethylaniline
(d) N=N NH2 (273-278K) X Y

39. Which of the following will be most stable The structure of ‘Y’ would be ;
diazonium salt RN2+X– ? [2014]
CH3
(a) CH3 N2+X– (b) C6H5N2+X–
(c) CH3CH2N2+X– (d) C6H5CH2N2+X– (a) N=N N
40. In the reaction CH3
NO2 NO2
CH3 CH3

A (b) HN NH NH
Br Br

N2C I (c) H3C N=N NH2
A is : [NEET 2013]
(a) Cu2Cl2 (b) H3PO2 and H2O
(c) H+/H2O (d) HgSO4/H2SO4 CH3 CH3
41. Nitrobenzene on reaction with conc. (d) HN N=N NH
HNO3/H2SO4 at 80 - 100°C forms which one of
the following products ? [NEET 2013]
44. Nitrobenzene can be prepared from benzene by
(a) 1, 3 - Dinitrobenzene using a mixture of conc. HNO3 and conc. H2SO4
(b) 1, 4 - Dinitrobenzene in the mixture, nitric acid acts as a/an: [2009]
(c) 1, 2, 4 - Trinitrobenzene (a) acid (b) base
(d) 1, 2 - Dinitrobenzene (c) catalyst (d) reducing agent
EBD_7324
îëì ÝØÛÓ×ÍÌÎÇ
45. Aniline in a set of reactions yielded a product D.
47. [A] reduction
[B]
CHCl +KOH
3
[2000]
reduction
NH2 [C] N-Methylaniline, A is
NaNO2 CuCN
A B
HCl (a) Formaldehyde (b) Trichloromethane
H2 HNO2 (c) Nitrobenzene (d) Toluene
C D
Ni 48. Which of the following reagents will convert p-
The structure of the product D would be: [2005] methylbenzenediazonium chloride into
(a) C6H5NHOH (b) C6H5NHCH2CH3 p-cresol? [1999]
(c) C6H5CH2NH2 (d) C6H5CH2OH (a) Cu powder (b) H2O
46. Aniline when diazotized in cold and when (c) H3PO2 (d) C6H5OH
treated with dimethyl aniline gives a coloured 49. Diazo coupling is useful to prepare some
product. Its structure would be [2004] [1994]
(a) CH3NH N=N NHCH3 (a) Pesticides (b) dyes
(c) proteins (d) vitamins
(b) CH3 N=N NH2

(c) (CH3)2N N=N

(d) (CH3)2N NH

ANSWER KEY
1 (c) 6 (d) 11 (d) 16 (d) 21 (d) 26 (a) 31 (c) 36 (a) 41 (a) 46 (c)
2 (a) 7 (b) 12 (d) 17 (c) 22 (c) 27 (a) 32 (d) 37 (c) 42 (a) 47 (c)
3 (a) 8 (c) 13 (c) 18 (d) 23 (d) 28 (d) 33 (d) 38 (d) 43 (a) 48 (b)
4 (d) 9 (b) 14 (b) 19 (c) 24 (b) 29 (b) 34 (b) 39 (b) 44 (b) 49 (b)
5 (a) 10 (c) 15 (c) 20 (c) 25 (a) 30 (c) 35 (b) 40 (b) 45 (d)
ß³·²»­ îëë

Hints & Solutions


1. (c) – NO2 group has strong – R effect and – O
CH3 shows +R effect.
COOH C – Cl
Order of basic strength is
SOCl2 NH3
NH2 NH2 NH2 8. (c)
Br Br
O NH2
< <
C NH2
NO2 CH3 NaOH
+Br2
O Br Br
2. (a) CH3 C NH 2 Br2 4NaOH 9. (b) CH2–NH2 compound is most
acetamide

CH3 – NH2 + 2NaBr + Na2CO3 + 3H2O basic due to localized lone pair of electron on
methanamine nitrogen atom while other compounds have
It is called Hoffmann Bromamide reaction. delocalized lone pair of electron.
3. (a) Arylamines are generally less basic than 10. (c) Aryl amines react with nitrous acid to
produce diazonium salt and not phenol
alkylamines due to following factors
11. (d) (1) CH3 CH2 – CH2 – CH2 – NH2
(a) Due to resonance in aromatic amines.
(b) Lower stability of anilinium ion (ii) with KOH (alcohol) and CHCl3 produces
bad smell
4. (d)
(2) CH3C CH
NO2 HN—OH NH2
Electrolytic (iii) gives white ppt with ammonical AgNO3
rearrangement
reduction
(Strongly acidic
(3) CH3 CH2 COOCH3
medium)
nitrobenzene Phenyl (i) alkaline hydrolysis
hydroxylamine OH
(4) CH3 CHOH – CH3
5. (a) C3H9N
CH3—CH2—CH2—NH2 (iv) with Lucas reagent cloudiness appears after
5 minutes
CH3 – NH – CH2 – CH3.
CH 3 — CH — CH 3 , CH 3 —N — CH 3 12. (d) Secondary amine with (NaNO2 + HCl) gives
| | a nitroso product
NH 2 CH3
CH3
6. (d) Because arylhali des do not undergo |
NHCH3 NaNO/2 N–N=O
nucleophilic substitution reaction with
HCl
potassium phthalimide easily.
7. (b) Secondary amine react with nitrous acid to N– Nitroso – N– methylaniline
give N-Nitrosoamines. 13. (c) Benzylamine is more basic than aniline. The
reason is that in aniline, the lone pair of nitrogen
(CH3)2 N + NaNO2 + HCl is conjugated with benzene ring so it is not
CH3 available readily for others. On the other hand
in Benzylamine, nitrogen is not directly attached
N — N = O + NaCl + CH3OH with ring so lone pairs are not conjugated with ring.
EBD_7324
îëê ÝØÛÓ×ÍÌÎÇ
14. (b) F
Br Br
N2
BF3
Br
2,4,6 tribromofluorobenzene
19. (c) CH 3CH 2 NH 2 CHCl 3 3KOH
CH 3CH 2 NC 3KCl 3H 2O
15. ø½÷ Electrolytic reduction of Nitroalkane in In this reaction, bad smelling compound ethyl
weakly acidic medium give aniline
NO2 NH2 isocyanide (CH 3CH 2 NC) is produced. This
Electrolytic reduction equation is known as carbyl amine reaction.
(weakly acidic medium) 20. (c) Benzaldehyde reacts with primary aromatic
Aniline amine to form schiff's base
Whereas in strongly acidic medium it gives
p-hydroxyaniline C6 H 5CH O C 6 H 5 NH 2
NO2 NHOH Benzaldehyde Aniline
electrolytic C 6 H 5C H NC 6 H 5 H 2O
reduction in presence Benzyliden e aniline
of conc. H 2 SO4
21. (d) All aliphatic amines are stronger bases than
øͬ®±²¹´§ ¿½·¼·½ ³»¼·«³÷ NH3 and among different ethylamines order of
NH2
basictity is 2° > 3° > 1°. Thus, the correct order
rearrangement is (d) i.e.,
(C 2 H 5 ) 2 NH (C 2 H 5 )3 N
OH
p-Hydroxyaniline C 2 H 5 NH 2 NH 3 .
16. (d) RDX is prepared by treating urotropine with This anomalous behaviour of tertiary ethyl amine
fuming nitric acid. When the inner bridge system is due to steric factors i.e., crowding of alkyl
is destroyed by oxidation and the peripheral groups cover nitrogen atom from all sides and
nitrogen atom are nitrated, it forms cyclonitrite thus makes the approach and bonding by a lewis
(or RDX). acid relatively difficult which results the
17. (c) (CH3)2NH is most basic because two electron maximum steric strain in tertiary amines. The
electrons are there but the path is blocked
releasing groups are present on Nitrogen. Also
resulting the reduction in its basicity.
aromatic amines are less basic then aliphatic amines.
22. (c) Methyl amine is a stronger base than
The basic character of amines follow the order
ammonia due to +I effect. The alkyl groups which
R2NH > RNH2 > C6H5NHCH3 > NH3
are electron releasing groups increase the
NH2 NH2
Br Br electron density around the nitrogen thereby
NaNO2 increasing the availability of the lone pair of
18. (d) +3Br2
& dil HCl electrons to proton or lewis acid and making the
amine more basic
Br
2, 4, 6 tribromoaniline
NH3 CH3NH2
+ – Kb = 1.8 × 10–5 44 × 10–5
N2Cl N2BF 4
23. (d)
Br Br Br Br
NaBF4
CH3
( NaCl ) AlCl3

Br CH3Cl
Br
Diazonium Diazonium (Friedel Craft’s
salt tetra fluro borate Reaction)
ß³·²»­ îëé

H NaOH
O O 26. (a) CH3CONH2 CH3 NH 2
C=O Br2
O Base
+ (Hofmann bromamide reaction)
R R
Perkin Reaction 27. (a) Reduction of alkyl isocyanides in presence
O O of LiAlH4 yields secondary amines containing
OH + HO
methyl as one of the alkyl group.
Ar
R R LiAlH 4
R N C 4[H] R NH CH3
O 2°amine
OH O
O – C – CH3
LiAlH4
+ CH3 – C – Cl e.g., CH3 N C 4[H] CH 3 NH CH 3
dimethyl amine
Acetylation Reaction

NH2
whereas, alkyl cyanides give 1° amine on reduction.
Cl
+
NaOH
O 28. (d) NH2 NHCOCH3
Schotten-Baumen Reaction
(CH 3CO)2 O Br2
NH – C –
CH3 COOH
O
CH3 CH3
24. (b) Hydrolysis of propyl isocyanide (A)
(CH3CH2CH2NH2) gives CH3CH2CH2NH2 + NHCOCH3 NH2
HCOOH. +
Br H /H2 O Br
On treatment with NaNO2 and HCl I gives
CH3CH2CH2OH which does not give iodoform
test. II (HCOOH) reduces Tollen’s reagent and CH3 CH3
(B) (C)
Fehling’s solution.
H2O
CH3CH 2 CH 2 NC
Propyl isocyanide C N
CH3CH 2 CH 2 NH 2 HCOOH
I II 29. (b) + CH3MgBr
HNO2
25. (a) CH3 CH NH2 OCH3

CH3 (A)
isopropyl amine
CH3 CH OH + N2 H3C –C = NMgBr H3C –C = NH
+
H3 O
CH3
isopropyl alcohol OCH3
OCH3
CHCl3 / KOH
CH3 CH N C
COCH3
CH3 +
H3 O
isopropyl isocyanide
(C)
reduction OCH3
CH3 CH NH CH3
30. (c) The mechanism of Hoffmann bromamide
CH3 reaction is
Isopropylmethylamine
EBD_7324
îëè ÝØÛÓ×ÍÌÎÇ
(i) RCONH 2 Br2 RCONHBr HBr
RCONH2 + Br2 + 4KOH
(ii) RCONHBr HBr OH
R — CO N Br H 2 O R NH 2 2KBr K 2CO3 2H 2O
K
+ (Hofmann's bromamide reaction)
(iii) R — C — N —Br R — C — N + KBr
O O
R
(iv) O = C N•• R—N=C=O NH 2
(rearrangement) 38. (d) N 2Cl +
(v) R N C O 2 KOH RNH 2 K 2 CO 3 H
31. (c) Carbylamine reaction :
R–NH2+3KOH+CHCl3 N=N NH 2
R–N C+3KCl+3H2O
39. (b) Arene diazonium salts are most stable
Reduction among the given options because of the
32. (d) CH3CN CH3 CH2 NH2
dispersal of +ve charge on the benzene ring
HONO due to resonance.
CH3 CH2 OH + N2+ H2O
NO2 NO2
33. (d) Aniline when treated with acetic anhydride
forms acetanilide (nucleophilic substitution) 40. (b)
O Br H 3 PO 2 H 2O Br
NH2 NH–C–CH3 – +H3 PO 3 HCl
N 2C l
O O
O NO 2
CH 3–C–O –C–CH 3
+ CH3–C–OH NO 2

34. (b) Acetonitrile (Methyl cyanide) on treatment 41. (a) HNO3 / H2 SO4
80 100 C
with conc. HCl give acetamide. NO 2
Conc .HCl Nitrobenzene 1,3 - Dinitrobenzene
CH 3C N CH 3CONH 2.
Acetonitri le Acetamide 42. (a) When nitro compound is reduced with a
35. (b) Acetamide and ethylamine can be neutral reducing agent (Zn dust + NH4Cl) the
distinguished by heating with NaOH solution. corresponding hydroxyl amine is formed
Acetamide evolves NH3 but ethylamine does Zn dust NH 4Cl
not. C6 H5 NO2 4[H] C6 H5 NHOH
Phenyl hydroxylamine
CH 3CONH 2 NaOH 43. (a) Refer ans 29
CH 3COONa NH 3 44. (b) HONO2 + H2SO4
CH 3 CH 2 NH 2 NaOH No reaction. Base Acid
36. (a) Any primary amine means both aliphatic as
well as aromatic but monoalkylamines means NO 2 H 2O HSO 4
only 1° aliphatic amines. Therefore, option (a) is Nitric acid acts as a base by accepting a proton.
correct while (d) is wrong. + –

RNH2 + CHCl3 + 3KOH


45. (d)
RNC 3KCl 3H 2 O
alkyl
isocyanide
37. (c) Only 1° amides (i.e. RCONH2) in the present
case undergo Hofmann bromamide reaction.
ß³·²»­ îëç

N2+ Cl OH

H 2O
+ N2+HCl
48. (b)
CH3 CH3
p- cresol

49. (b) Azo dyes can be prepared by diazotizing an


Benzyl alcohol aromatic amine and subsequent coupling
NaNO 2 , HCl with a suitable aromatic phenol or amine e.g.,
46. (c) N H2
0 5 C
N =NCl + H N(CH3)2
N(CH3)2
Diazonium chloride N,N-dimethylaniline
N 2 Cl +
H

N=N N(CH3)2
N=N N(CH3)2
p-Dimethyl amino azobenzene

47. (c)
NO 2 N C

Sn HCl CHCl3 KOH


reduction

Nitrobenzene (B) NH–CH3 (C)


(A)
Re duction
Na/C2 H5OH

N-methylaniline
EBD_7324
îêð ÝØÛÓ×ÍÌÎÇ

ݸ¿°¬»®

28 Biomolecules

̱°·½ ïæ Ý¿®¾±¸§¼®¿¬»­ ¿²¼ Ô·°·¼­ 4. Which one of the following statements is not
true regarding (+) Lactose ? [2011]
1. Which one given below is a non-reducing sugar? (a) On hydrolysis (+) Lactose gives equal
(a) Maltose (b) Lactose [2016] amount of D(+) glucose and D(+)
(c) Glucose (d) Sucrose galactose.
2. D (+) glucose reacts with hydroxylamine and (b) (+) Lactose is a -glycoside formed by the
yields an oxime. The structure of the oxime would union of a molecule of D(+) glucose and a
be : [2014] molecule of D(+) galactose.
(a) CH = NOH (b) CH = NOH (c) (+) Lactose is a reducing sugar and does
H – C – OH HO – C – H not exhibit mutarotation.
(d) (+) Lactose, C12H22 O11 contains 8-OH
HO – C – H HO – C – H
groups.
HO – C – H H – C – OH 5. Which one of the following does not exhibit the
H – C – OH H – C – OH phenomenon of mutarotation ? [2010]
(a) (+) – Sucrose (b) (+) – Lactose
CH2OH CH2OH (c) (+) – Maltose (d) (–) – Fructose
(c) CH = NOH (d) CH = NOH 6. Fructose reduces Tollen’s reagent due to [2010]
HO – C – H H – C – OH (a) enolisation of fructose followed by
conversion to aldehyde by base
H – C – OH HO – C – H
(b) asymmetric carbons
HO – C – H H – C – OH (c) primary alcoholic group
H – C – OH H – C – OH
(d) secondary alcoholic group
7. The cell membranes are mainly composed of
CH2OH CH2OH (a) fats (b) proteins [2005]
3. Which one of the following sets of monosac- (c) phospholipids (d) carbohydrates
charides forms sucrose? [2012] 8. Glycolysis is [2003]
(a) –D-Galactopyranose and (a) conversion of glucose to haem
(b) oxidation of glucose to glutamate
–D-Glucopyranose
(c) conversion of pyruvate to citrate
(b) –D-Glucopyranose and (d) oxidation of glucose to pyruvate
–D-Fructofuranose 9. Phospholipids are esters of glycerol with [2003]
(a) Three phosphate groups
(c) –D-Glucopyranose and
(b) Three carboxylic acid residues
–D- Fructofuranose (c) Two carboxylic acid residues and one
(d) –D-Glucopyranose and phosphate group
(d) One carboxylic acid residue and two
–D-Fructopyranose phosphate groups
Þ·±³±´»½«´»­ îêï

10. Cellulose is a polymer of [2002] (a) Conformation


(a) Glucose (b) Fructose (b) Configuration
(c) Ribose (d) Sucrose (c) Number of OH groups
11. Which of the following gives positive Fehling (d) Size of hemiacetal ring
solution test ? [2001] 20. On hydrolysis of starch, we finally get [1991]
(a) Protein (b) Sucrose (a) Glucose (b) Fructose
(c) Glucose (d) Fats (c) Both (a) and (b) (d) Sucrose
12. Which is correct statement? [2001]
(a) Starch is a polymer of -glucose ̱°·½ îæ ß³·²± ß½·¼­ ¿²¼ Ю±¬»·²­
(b) In cyclic structure of fructose, there are four 21. Which of the following statements is not
carbons and one oxygen atom correct : [2017]
(c) Amylose is a component of cellulose (a) Ovalbumin is a simple food reserve in egg-
(d) Proteins are composed of only one type of white
amino acids (b) Blood proteins thrombin and fibrinogen are
13. Mg is present in which of the following : [2000] involved in blood clotting
(a) Starch (b) Chlorophyll (c) Denaturation makes the proteins more active
(c) Both (d) None (d) Insulin maintains sugar level in the blood
14. Which of the following is the sweetest sugar? of a human body
(a) Sucrose (b) Glucose [1999] 22. In a protein molecule various amino acids are
(c) Fructose (d) Maltose linked together by [2016]
15. In cells the net production of ATP molecules (a) -glycosidic bond
generated from one glucose molecule is [1999] (b) -glycosidic bond
(c) peptide bond
(a) 46 (b) 32
(d) dative bond
(c) 36 (d) 40
23. Which of the statements about "Denaturation"
16. The number of molecules of ATP produced in given below are correct ? [2011 M]
the lipid metabolism of a molecule of palmitic (a) Denaturation of proteins causes loss of
acid is [1998] secondary and tertiary structures of the protein.
(a) 130 (b) 36 (b) Denturation leads to the conversion of
(c) 56 (d) 86 double strand of DNA into single strand
17. Glucose molecule reacts with 'X' number of (c) Denaturation affects primary strucrture
molecules of phenylhydrazine to yield osazone. which gets distorted
The value of 'X' is [1998] (a) (b) and (c) (b) (a) and (c)
(a) four (b) one (c) (a) and (b) (d) (a), (b) and (c)
(c) two (d) three 24. Which functional group participates in
18. Sucrose in water is dextro-rotatory, [ ]D= + 66.4º. disulphide bond formation in proteins? [2005]
When boiled with dilute HCl, the solution (a) Thioester (b) Thioether
becomes leavo-rotatory, [ ]D= –20º. In this (c) Thiol (d) Thiolactone
process the sucrose molecule breaks into[1996] 25. Which one of th e following structures
represents the peptide chain? [2004]
(a) L-glucose + D-fructose
H O
(b) L-glucose + L-fructose | | ||
(c) D-glucose + D-fructose (a) N C N C NH C NH
|| | |
(d) D-glucose + L-fructose O H
19. The -D glucose and -D glucose differ from H H
each other due to difference in carbon atom with | | | | | | | |
respect to its [1995] (b) N C C C C N C C C
|| | | | | | |
O
EBD_7324
îêî ÝØÛÓ×ÍÌÎÇ
H H H O (a) only oxygen-haeme complex
| | | | | | || (b) only carbon monoxide-haeme complex
(c) N C C N C C N C C (c) both oxygen-haeme and carbon monoxide-
| || | || |
O O haeme complexes but oxygen-haeme
H O H complex is more stable
| | | || | | | | | (d) both oxygen-haeme and carbon monoxide-
(d) N C C C N C C N C C C haeme complexes but carbon monoxide-
| | | | | || | |
H O haeme complex is more stable
26. The correct statement in respect of protein 33. Identify an element amongst the following which
haemoglobin is that it [2004] is responsible for oxidation of water to O 2 in
(a) functions as a catalyst for biological biological processes [1997]
reactions (a) Fe (b) Mn
(b) maintains blood sugar level (c) Mo (d) Cu
(c) acts as an oxygen carrier in the blood 34. In reference to biological role, Ca 2+ ions are
(d) forms antibodies and offers resistance to important in [1996]
dieases (a) triggering the contraction of muscles
27. The helical structure of protein is stabilized by (b) generating right electrical potential across
[2004] cell membrane
(a) dipeptide bonds (b) hydrogen bonds (c) hydrolysis of ATP
(c) ether bonds (d) peptide bonds (d) defence mechanism
28. Which is not a true statement? [2002] 35. A reagent suitable for the determination of
(a) -Carbon of -amino acid is asymmetric N-terminal residue of a peptide is [1996]
(b) All proteins are found in L-form (a) p-Toluenesulphonyl chloride
(c) Human body can synthesize all proteins (b) 2, 4-Dinitrophenylhydrazine
they need (c) Carboxypeptidase
(d) At pH = 7 both amino and carboxylic (d) 2, 4-Dinitrofluorobenzene
groups exist in ionised form 36. Which of the following protein destroy the
antigen when it enters in body cell? [1995]
O
|| (a) Antibodies (b) Insulin
29. For C N H (peptide bond) [2001] (c) Chromoprotein (d) Phosphoprotein
Which statement is incorrect about peptide bond?
(a) C — N bond length in proteins is longer ̱°·½ íæ Ò«½´»·½ ß½·¼ ¿²¼ Û²¦§³»­
than usual bond length of the C — N bond 37. The correct statement regarding RNA and DNA,
(b) Spectroscopic analysis shows planar respectively is [2016]
structure of the — C— NH — group (a) The sugar component in RNA is arabinose
|| and the sugar component in DNA is
O
2'-deoxyribose.
(c) C — N bond length in proteins is smaller (b) The sugar component in RNA is ribose and
than usual bond length of the C—N bond the sugar component in DNA is
(d) None of the above 2'-deoxyribose.
30. The number of essential amino acids in man is (c) The sugar component in RNA is arabinose
(a) 8 (b) 10 [2000] (d) The sugar component in RNA is
(c) 18 (d) 20 2'-deoxyribose and the sugar component
31. The dominant cation in the blood plasma in DNA is arabinose
(extracellular fluid) is [1999] 38. In DNA the linkages between different
(a) potassium (b) calcium nitrogenous bases are: [NEET Kar. 2013]
(c) magnesium (d) sodium (a) peptide linkage
32. The reactions of (a) oxygen and (b) carbon (b) phosphate linkage
monoxide with haeme (the prosthetic group of (c) H-bonding
haemoglobin) give [1997] (d) glycosidic linkage
Þ·±³±´»½«´»­ îêí

39. Which one of the following, statements is 46. Chargaff's rule states that in an organism [2003]
incorrect about enzyme catalysis? [2012] (a) Amounts of all bases are equal
(a) Enzymes are mostly proteinous in nature. (b) Amount of adenine (A) is equal to that of
(b) Enzyme action is specific. thymine (T) and the amount of guanine (G)
(c) Enzymes are denaturated by ultraviolet is equal to that of cytosine (C)
rays and at high temperature. (c) Amount of adenine (A) is equal to that of
(d) Enzymes are least reactive at optimum guanine (G) and the amount of thymine (T)
temperature. is equal to that of cytosine (C)
40. The segment of DNA which acts as the (d) Amount of adenine (A) is equal to that of
instrumental manual for the synthesis of the cytosine (C) and the amount of thymine
protein is: [2009] (T) is equal to that of guanine (G)
(a) ribose (b) gene 47. Enzymes are made up of [2002]
(c) nucleoside (d) nucleotide (a) Edible proteins
41. In DNA, the complimentary bases are: (b) Proteins with specific structure
[1998, 2008] (c) Nitrogen containing carbohydrates
(a) Adenine and thymine; guanine and (d) Carbohydrates
cytosine 48. Which of the following is correct about H-
(b) Adenine and thymine ; guanine and uracil bonding in nucleotide? [2001]
(c) Adenine and guan ine; th ymine and (a) A --- A and T --- T (b) G --- T and A --- C
cytosine (c) A --- G and T --- C (d) A --- T and G --- C
(d) Uracil and adenine; cytosine and guanine 49. Which one of the following chemical units is
42. RNA and DNA are chiral molecules, their certainly to be found in an enzyme? [1997]
chirality is due to [2007] OH H
(a) chiral bases O
O N—C
(b) chiral phosphate ester units (a) (b)
HO HO
(c) D-sugar component O O
(d) L-sugar component. O
43. During the process of digestion, the proteins R
present in food materials are hydrolysed to O O
amino acids. The two enzymes involved in the N O R
process (c) (d)
O R
Proteins Enzyme( A) Polypeptides N
Enzyme(B) O
Amino acids 50. Chemically considering digestion is basically
are respectively [2006] (a) Anabolism [1994]
(a) Diastase and Lipase (b) Hydrogenation
(b) Pepsin and Trypsin (c) Hydrolysis
(c) Invertase and Zymase (d) Dehydrogenation.
(d) Amylase and Maltase 51. Enzymes take part in a reaction and [1993]
44. A sequence of how many nucleotides in (a) decrease the rate of a chemical reaction
messenger RNA makes a codon for an amino (b) increase the rate of a chemical reaction
acid? [2004] (c) both (a) and (b)
(a) Three (b) Four (d) none of these
(c) One (d) Two 52. The couplings between base units of DNA is
45. The enzyme which hydrolyses triglycerides to through : [1992]
fatty acids and glycerol is called [2004] (a) Hydrogen bonding
(a) Maltase (b) Lipase (b) Electrostatic bonding
(c) Zymase (d) Pepsin (c) Covalent bonding
(d) Vander Waals forces
EBD_7324
îêì ÝØÛÓ×ÍÌÎÇ
̱°·½ ìæ Ê·¬¿³·²­ ¿²¼ ر®³±²»­ 58. Which one of the followin g is a peptide
hormone ? [2006]
53. Which of the following hormones is produced (a) Testosterone (b) Thyroxin
under the condition of stress which stimulates (c) Adrenaline (d) Glucagon
glycogenolysis in the liver of human beings? 59. The human body does not produce [2006]
(a) Thyroxin (b) Insulin [2014] (a) Vitamins (b) Hormones
(c) Adrenaline (d) Estradiol (c) Enzymes (d) DNA
54. Deficiency of vitamin B1 causes the disease 60. The hormone that helps in the conversion of
(a) Convulsions (b) Beri-Beri [2012] glucose to glycogen is [2004]
(c) Cheilosis (d) Sterility (a) Cortisone (b) Bile acids
55. Which of the following hormones contains (c) Adrenaline (d) Insulin
iodine? [2009] 61. Vitamin B12 contains [2003]
(a) Testosterone (b) Adrenaline (a) Ca(II) (b) Fe(II)
(c) Thyroxine (d) Insulin (c) Co(III) (d) Zn(II)
56. Which one of the following is an amine hormone ? 62. Which of the following is a steroid hormone?
(a) Thyroxine (b) Oxypurin [2008] [1999]
(c) Insulin (d) Progesterone (a) Cholesterol (b) Adrenaline
57. Which of the following is water-soluble? [2007] (c) Thyroxine (d) Progesterone
(a) Vitamin E (b) Vitamin K
(c) Vitamin A (d) Vitamin B

ANSWER KEY
1 (d) 8 (d) 15 (c) 22 (c) 29 (a) 36 (a) 43 (b) 50 (c) 57 (d)
2 (d) 9 (c) 16 (a) 23 (c) 30 (b) 37 (b) 44 (a) 51 (b) 58 (d)
3 (b) 10 (a) 17 (d) 24 (c) 31 (d) 38 (c) 45 (b) 52 (a) 59 (a)
4 (c) 11 (c) 18 (c) 25 (c) 32 (d) 39 (d) 46 (b) 53 (c) 60 (d)
5 (a) 12 (a) 19 (b) 26 (c) 33 (a) 40 (b) 47 (b) 54 (b) 61 (c)
6 (a) 13 (b) 20 (a) 27 (b) 34 (b) 41 (a) 48 (d) 55 (c) 62 (d)
7 (c) 14 (c) 21 (c) 28 (b) 35 (d) 42 (c) 49 (b) 56 (a)
Þ·±³±´»½«´»­ îêë

Hints & Solutions


1. (d) Sucrose is non-reducing disaccharide as the 9. (c) Phospholipids are derivatives of glycerol
two monosaccharide units are linked through in which two of the hydroxyl groups are
their respective carbonyl groups. esterified with fatty acids while the third is
2. (d) Glucose reacts with hydroxyl amine to form esterified with some derivative of phosphoric
an oxime. acid with some alcohol such as choline,
ethanolamine, serine or inositol.
R
O
O = P – O – CH2 O
OH CH – O – C –R
CH2O – C –R
HO O
3. (b) CH2 OH
10. (a) We know that cellulose (C 6 H12 O 6 ) n is
O CH2 the chief constituent of cell walls of plants. It is
O
OH + OH the most abundant organic substance found in
OH HO
OH H2C nature. It is a polymer of glucose with 3500
OH OH OH repeating units in a chain.
-D-Glucopyranose -D-Fructofuranose 11. (c) Glucose contain aldehyde group. Hence it
HO give positive fehling solution test.
CH2 OH
12. (a) Starch is also know as amylum which
O CH2 O occurs in all green plants. A molecule of starch
OH HO + H2O
O (C 6 H10 O5 ) n is built of a large number of
OH CH2
-glucose ring joined through oxygen-atom.
OH OH
OH 13. (b) Chlorophyll contain Mg.
Sucrose 14. (c) Fructose is the sweetest sugar.
Sucrose is a disaccharide of –D-Glucopyranose 15. (c) C 6 H12 O 6 6O 2 2 ATP
and –D-Fructofuranose.
4. (c) All reducing sugar shows mutarotation. 6CO 2 6H 2O 38 ATP Molecules
5. (a) Sucrose does not have free — CHO or CO Net total number of ATP molecules evolved =
group, hence it does not undergo mutarotation. 36 molecules.
6. (a) Fructose, a ketose as the substrate, under 16. (a) In the lipid metabolism, when palmitic acid
the alkaline medium of Tollen’s reagent, a part of is oxidised, two carbon fragments are removed
fructose is transformed into glucose and sequentially to form acetyl coenzyme. It enters
mannose, both aldoses. Then these aldoses give
the citric acid cycle for production of 130 ATP.
positive silver mirror test.
17. (d) Glucose reacts with one molecule of phenyl
7. (c) Cell membranes (Plasma membranes) con-
hydrazine to give phenyl hydrazone. When
stitutes bilayer of phospholipid with embedded
warmed with excess of phenylhydrazine, the
proteins. In humans, lipids accounts for upto secondary alcoholic group adjacent to the
79% of cell membrance. aldehyde group is oxidised by another molecule
8. (d) It is a common pathway for both the aerobic of phenylhydrazine to a ketonic group. With this
& anaerobic respiration in which 1 glucose ketonic group, the third molecule of
molecule is converted to 2 molecules of pyruvate. phenylhydrazine condenses to glucosazone.
EBD_7324
îêê ÝØÛÓ×ÍÌÎÇ
Therefore the value of X is 3 21. (c) Due to denaturation of proteins, helix get
CHO + H2NNHC6H5 CH = NNHC6H5 uncoiled and protein loses its biological activity.
CHOH CHOH 22. (c) Peptide bond
warm
(CHOH)3 CH(OH)3 — C — NH —
||
CH2OH CH2OH O
Glucose Glucose Phenylhydrozone 23. (c) When the proteins are subjected to the
action of heat, mineral acids or alkali, the water
H2NNHC6 H5 soluble form of globular protein changes to water
insoluble fibrous protein. T his is called
CH = NNHC6H5 CH = NNHC6H5 denaturation of proteins. During denaturation
H2NNHC6 H5 secondary and tertiary structures of protein
C = NNHC6H5 C=O destroyed but primary structures remains intact.
– H2 O

CH(OH)3 (CHOH)3 24. (c) 2R Thiol


S H R S S R
Disulphide
CH2OH CH2OH Example :
Glucosazone keto compound of glucose
phenyl hydrazone 2HO 2C CHC H 2SH
18. (c) The hydrolysis of sucrose by boiling with |
NH2
mineral acid or by enzyme invertase or sucrase Cysteine
produces a mixture of equal molecules of D(+)
glucose and D(–) Fructose. HO 2CCHC H 2S SCH 2 CHC O 2 H
| |
C12 H 22O11 H 2O
HCl
C 6 H12O 6 C 6 H12O 6 NH 2 NH 2
sucrose D glu cos e D Fructose Cystine
[ D] 66.5º [ D ] 52.5º [ D ] 92 º
ïììììììîìììììì í 25. (c) The bond formed between two amino acids
Invert sugar ,[ D ] 20º by the elimination of a water molecule is called
19. (b) -D glucose and –D glucose are the a peptide linkage or bond. The peptide bond is
isomers which differ in the ori entation simply another name for amide bond.
(configuration) of H and OH groups around C1 C OH H — N— — C– N— H 2 O
atom. | | || |
O H O H
Carboxyl group Amine group of Peptide bond
1 1 of one amino acid other amino acid
H —C —OH HO —C —H
2
The product formed by linking amino acid
H —C —OH H —C2—OH molecules through peptide linkages. —CO—
O O
HO —C3—H HO —C3—H NH—, is called a peptide.
26. (c) Haemoglobin acts as an oxygen carrier in
H —C4—OH H —C4—OH the blood since it reacts with oxygen to form
H —C
5
H —C
5 unstable oxyhaemoglobin which easily breaks
6 6 to give back haemoglobin and oxygen.
CH2OH CH2OH 27. (b) The -helix structure is formed when the
– D - glucose – D - glucose
chain of -amino acids coils as a right handed
20. (a) Manufacture - By hydrolysis of starch with screw (called -helix) because of the formation
hot dil. mineral acids of hydrogen bonds between amide groups of
the same peptide chain, i.e., NH group in one
(C6 H10 O5 )n (C6 H10 O5 )n H 2 O unit is linked to carbonyl oxygen of the third
Starch Dextron unit by hydrogen bonding. This hydrogen
C12 H 22 O11 2C6 H12 O6 bonding between different units is responsible
Maltose Glucose for holding helix in a position.
Þ·±³±´»½«´»­ îêé

28. (b) All proteins are not found in L-form but +


they may be present in form of D or L H
29. (a) Due to resonance C — N bond in protein
acquires double bond character and is smaller
than usual C — N bond. O2N NH CHCOOH
|
O O

R1 +
C NH C NH NO2 (DNP derivative)
H 2 NCHCOOH H 2 NCHCOOH
30. (b) There are 20 amino acid in man out of which | |
10 amino acids are essential amino acids. These R2 R3
essential amino acids are supplied to our bodies (Amino acid mixture)
by food which we take because they cannot be 36. (a) When antigens enter in to the body cells
synthesised in the body .These are (i) valine and destroy them, then antibodies being
(ii) leucine (iii) Isoleucine (iv) Phenylalanine proteins are synthesised in the body and
(v) Threonine (vi) Methionine (vii) Lysine combine with antigens and destroy these
(viii) Tryptophan (ix) Arginine (x) Histidine. antigens by forming inactive complexes.
31. (d) Na+, Cl– and HCO3– are the electrolytes Therefore antibodies protein destroy antigens.
present in blood plasma. Na+ and Cl– helps in 37. (b) Sugar in DNA is 2-deoxyribose whereas
maintainence of osmotic pressure and fluid sugar in RNA is ribose.
38. (c) The base pairs of the two strands of DNA
balance.
are linked together through H-bonds.
32. (d) Hb CO HbCO (stable compound) 39. (d) Enzymes are most reactive at optimum
carboxy haemoglobin
temperature. The optimum temperature for
enzyme activity lies between 40°C to 60°C.
Hb O 2 HbO 2
40. (b) The DNA sequence that codes for a specific
oxyhaemoglobin (unstable).
protein is called a Gene and thus every protein
33. (a) Fe is responsible for oxidation of water to
in a cell has a corrosponding gene.
O2.
41. (a) In DNA the complimentary base are
34. (b)
Adenine and thymine.
35. (d) 2, 4- Dinitroflurobenzene, also known as
Guanine and cytosine
Sanger's reagent, reacts with the H2N - group of
The genetic information for cell is contained in
the peptide to from 2, 4- dinitrophenyl (DNP)
the sequence of bases A, T, G and C in DNA
derivative of the peptide. The DNP derivative of
molecule.
the peptide is hydrolysed to give DNP derivative
of the single amino acid 42. (c) Each nucleic acid consists of a pentose
sugar a heterocyclic base, and phosphoric acid.
H 2 NCHCONH CHCONH CHCOOH +
| | | The sugar present in DNA is -D (–) -2-deoxy
R1 R2 R3 ribose and the sugar present in RNA is D (–)-
ribose. The chirality of DNA and RNA molecules
O2N F are due to the presence of sugar components.
O
NO2 HOCH2 OH
2, 4, Dinitrofluoro benzene
H H
O 2N NHC HCONH C HCONH C HCOOH
| | | H H
R1 R2 R3
NO2 OH OH
DNP derivative of polypeptide) D(–)–ribose
EBD_7324
îêè ÝØÛÓ×ÍÌÎÇ
O 50. (c) Basically digestion means hydrolysis.
HOCH2 OH 51. (b) Enzymes being biocatalyst can increase the
rate of a reaction upto 10 million times. Even
H H very small amount can accelerate a reaction.
52. (a) DNA consists of two polynucleotide
H H
chains, each chain forms a right handed helical
OH H spiral with ten bases in one turn of the spiral.
D(–) -2-deoxy ribose The two chains coil to double helix and run in
43. (b) Pepsin and Trypsin are two enzymes opposite direction held together by hydrogen
involved in the process (hydrolysis of proteins) bonding.
Pepsin 53. (c) Adrenaline is a hormone produced by
Proteins Polypeptides
Proteases adrenal glands during high stress or exciting
Trypsin situations. This powerful hormone is part of the
Amino acids
Chemotrypsin human body’s acute stress response system,
(Pancreatic juice Intestine)
also called the fight or flight response.
44. (a) The sequence of bases in mRNA are read 54. (b) Beri-Beri.
in a serial order in groups of three at a time. 55. (c) Thyroxine is the only hormone among the
Each triplet of nucleotides (having a specific
given choices that contains iodine. Its structure
sequence of bases) in known as codon. Each
codon specifies one amino acid. Further since, is as follows:
there are four bases. therefore, 4 3 = 64 triplets
or codons are possible. I I
45. (b) Triglycerides are lipids, hence these are
hydrolysed by lipases to glycerol and fatty acids. I
A G
46. (b) 1
T C
Amount of A = T and that of G = C.
47. (b) Enzymes are made up of protein with I
specific structure.
48. (d)
56. (a) Thyroxine is an amino hormone.
O Deoxyribose-Adenine ... Thymine -Deoxyribose O 57. (d) Vitamin B is water soluble whereas all other
OH P Deoxyribose-Guanine ... Cytosine -Deoxyribose
P OH are water insoluble.
O O 58. (d) Testosterone and Adrenaline are steroid
P P hormone, Thyroxin is a non-steroid hormone
OH Deoxyribose-Guanine ... Cytosine -Deoxyribose
OH
O O and glucagon is a peptide hormone.
OH P Deoxyribose-Adenine ... Thymine -Deoxyribose
P OH
59. (a) Vitamins are organic substances which does
The hydrogen bonds are formed between the base not provide energy but are essential for healthy
(shown by dotted lines). Because of size and growth and proper functioning of body. Vitamins
geometrics of the bases, the only possible pairing are not synthesized inside human body but they
in DNA are between G(Guanine) and C(Cytosine) are essential part of our diet.
through three H-bonds and between A (Adenosine) 60. (d) Insulin helps in converting glucose to
and T (Thymine) through two H-bonds. glycogen
49. (b) Peptide bonds are present in enzyme. 61. (c) Vit B12 also called Cyanocobaltamin, is
anti-pernicious anaemia vitamin.
H
62. (d) Progesterone (Gestogens) is a steroid
N C hormone, which controls the development and
maintainance of pregnancy. Thryoxine and
O
Adrenaline are Amine hormones.
б´§³»®­ îêç

ݸ¿°¬»®

29 Polymers

̱°·½ ïæ Ý´¿­­·º·½¿¬·±² ±º б´§³»®­ 8. Biodegradable polymer which can be produced


from glycine and aminocaproic acid is : [2015]
1. Nylon is an example of : [NEET 2013] (a) PHBV (b) Buna - N
(a) Polysaccharide (b) Polyamide
(c) Nylon 6, 6 (d) Nylon 2- nylon 6
(c) Polythene (d) Polyester
9. Caprolactum is used for the manufacture of :
2. Which one of the following is n ot a
[2015 RS]
condensation polymer ? [2012]
(a) Nylon - 6 (b) Teflon
(a) Melamine (b) Glyptal
(c) Terylene (d) Nylon - 6,6
(c) Dacron (d) Neoprene
10. Which one of the following is an example of a
3. Of the following which one is classified as thermosetting polymer? [2014]
polyester polymer ? [2011]
(a) Terylene (b) Bakelite (a) ( CH 2 C CH CH 2 ) n
|
(c) Melamine (d) Nylone-66 Cl
4. [NH(CH 2 ) 6 NHCO(CH 2 ) 4 CO]n is a ( CH 2 CH ) n
(b)
[2006] |
Cl
(a) addition polymer
(b) thermosetting polymer H H O O
| | || ||
(c) homopolymer ( N (CH 2 )6 N C (CH 2 ) 4 C )n
(c)
(d) copolymer
5. Which one of the following is a chain growth OH OH
polymer? [2004] CH2 CH2
(a) Starch (b) Nucleic acid (d)
(c) Polystyrene (d) Protein n
6. In elastomer, intermolecular forces are [1995] 11. Which of the following organic compounds
(a) strong (b) weak polymerizes to form the polyester Dacron?
(c) nil (d) none of these [2014]
(a) Propylene and para HO—(C6H4)— OH
̱°·½ îæ Ю»°¿®¿¬·±² ¿²¼ Ю±°»®¬·»­
±º б´§³»®­ (b) Benzoic acid and ethanol
(c) Terephthalic acid and ethylene glycol
7. Natural rubber has [2016] (d) Benzoic acid and para HO–(C6H4)—OH
(a) all cis-configuration 12. Which is the monomer of Neoprene in the
(b) all trans-configuration following ? [NEET 2013]
(c) alternate cis-and trans-configuration
(a) CH 2 C CH CH 2
(d) random cis-and trans-configuration
CH 3
EBD_7324
îéð ÝØÛÓ×ÍÌÎÇ
(b) CH 2 C CH CH 2 (b) Terylene;
Cl
(c) CH2 = CH C CH
(– OC COOCH2 – CH2 – O–)n
(d) CH2 = CH CH = CH2
13. Which of the following statements is false? (c) Nylon 6, 6;
[2012] –NH(CH 2 )6 NH CO(CH 2 ) 4 CO
(a) Artificial silk is derived from cellulose. n
(b) Nylon-6, 6 is an example of elastomer. (d) Teflon; ( CF2 CF2 )n
(c) The repeat unit in natural rubber is 17. Which one of the following statement is not true?
isoprene.
[2008]
(d) Both starch and cellulose are polymers of
glucose. (a) In vulcanization the formation of sulphur
14. Which one of the following sets forms the bridges between different chains make
biodegradable polymer? [2012 M] rubber harder and stronger.
(a) CH2 = CH – CN and CH2 = CH – CH = CH2 (b) Natural rubber has the trans -configuration
(b) H2N – CH2 – COOH and at every double bond
H2N–(CH2)5 – COOH (c) Buna-S is a copolymer of butadiene and
(c) HO – CH2 – CH2 – OH and styrene
(d) Natural rubber is a 1, 4 - polymer of isoprene
HOOC COOH 18. Which one of the following polymers is prepared
by condensation polymerisation? [2007]
(d) CH = CH2 and (a) Teflon (b) Natural rubber
(c) Styrene (d) Nylon-6,6
CH2 = CH – CH = CH2
19. The monomer of the polymer; [2005]
15. Which of the following structures represents
neoprene polymer? [2010] CH3 CH3
|
(a) –(CH 2 – C = CH – CH 2 –) n CH 2 C CH 2 C
| | is
CH3 CH3
Cl
CN CH3
|
(b) –(CH 2 – CH –) n (a) H 2 C C (b) CH3CH=CHCH3
CH3
Cl
| (c) CH3CH = CH2 (d) (CH3)2C = C(CH3)2
(c) –( CH 2 – CH –)n 20. Acrilan is a hard, horny and a high melting
material. Which of the following represents its
structure ? [2003]
(d) –( CH – CH 2 –)n
|
C6 H5 (a) —CH 2 — CH —
|
16. Structures of some common polymers are given. Cl
Which one is not correctly presented? [2009] n
(a) Neoprene;
(b) —CH 2 — CH—
|
CH 2 C CH CH 2 CH 2 CN
| n
Cl n
б´§³»®­ îéï

CH3 (c) Nylon-66


| O O
(c) —CH 2 — C — –NH–(CH2)6–NH –C–(CH2)4–C–O–
| n
COOCH3 CH3
n (d) PMMA – CH2–C
COOCH3 n
(d) —CH 2 — CH 24. CF2 = CF2 is a unit of [2000]
| (a) Teflon (b) Buna – S
COOC2H5 (c) Bakelite (d) Polyethene
n
25. Natural rubber is a polymer of [1999]
21. Which one of the following monomers gives the
polymer neoprene on polymerization ? [2003] (a) butadiene (b) isoprene
(c) 2-methylbutadiene (d) Hexa-1, 3-diene
(a) CF2 = CF2
(b) CH2 = CHCl 26. Nylon 6, 6 is a polyamide obtained by the reaction
(c) CCl2 = CCl2 of [1996]
(a) COOH(CH2)4 COOH + NH2C6H4NH2–(p)
Cl (b) COOH(CH2)4 COOH + NH2 (CH2)6 NH2
|
(d) CH 2 C — CH CH 2 (c) COOH (CH2)6 COOH + NH2 (CH2)4 NH2
(d) COOHC6H4 COOH–(p) + NH2 (CH2)6 NH2
CH3 27. Bakelite is prepared by the reaction between
(a) urea and formaldehyde [1995]
22. Monomer of — C — CH2 — is [2002] (b) ethylene glycol
CH3 n (c) phenol and formaldehyde
(d) tetramethylene glycol
(a) 2-methylpropene (b) Styrene
(c) Propylene (d) Ethene 28. An example of biopolymer is [1994]
23. Which of the following is not correctly matched? (a) Teflon (b) Neoprene
[2001] (c) Nylon-6, 6 (d) DNA
O O ̱°·½ íæ Ë­»­ ±º б´§³»®­
(a) Terylene –OCH2–CH2–C– –C – 29. Which one of the following is used to make "non-
n stick" cook-wares ? [1997]
(a) Polystyrene
— CH 2 — C = CH — CH2 — (b) Polyethylene terephthalate
(b) Neoprene
(c) Polytetrafluoroethylene
Cl (d) Polyvinyl chloride
n

ANSWER KEY
1 (b) 4 (d) 7 (a) 10 (d) 13 (b) 16 (a) 19 (a) 22 (a) 25 (b) 28 (d)
2 (d) 5 (c) 8 (d) 11 (c) 14 (b) 17 (b) 20 (b) 23 (a) 26 (b) 29 (c)
3 (a) 6 (b) 9 (a) 12 (b) 15 (a) 18 (d) 21 (d) 24 (a) 27 (c)
EBD_7324
îéî ÝØÛÓ×ÍÌÎÇ

Hints & Solutions


1. (b) Nylon is a synthetic polyamide polymer. radicals or ions to which monomers get added
2. (d) Neoprene is an addition polymer of by a chain reaction.
isoprene. nC6H5 – CH = CH2 Benzoyl Peroxide
3. (a) Polyesters are condensation polymers of a
diabasic acid and a diol. e.g., Terylene C6H5
4. (d) The given compound is a copolymer of (CH CH2)n
hexamethylene diamine and adipic acid. It is Polystyrene
actually Nylon-6,6.
6. (b) Elastomers are the polymers having very
5. (c) Chain growth polymers involve a series
weak intermolecular forces of attraction between
of reactions, each step of which consumes a
reactive particle and produce another similar the polymer chain. The weak forces permit the
particle. The reactive particles may be free polymer to be streched.

7. (a) Natural rubber is found to be a polymer of cis-isoprene i.e. it is cis-polyisoprene


CH2 CH2
H2C C — CH CH2 polymerisation
|
CH3 H3C H
Isoprene cis-polyisoprene
8. (d) H2 N—CH2—COOH + H2 N — (CH2)5—COOH (HN—CH2—CO—NH— (CH2)5—CO) —
Glycine Aminocapric acid Nylon-2-nylon-6

O O
H
NH N
9. (a) Nylon–6 n
(Caprolactum)

10. (d) Thermosetting polymers undergo chemical changes when heated and set to hard mass when cooled
e.g. Bakelite.

O O
|| ||
11. (c) ( OCH2CH2–O– C C)n
Ethylene glycol Terephthalic acid Terylene
It is resistant to mineral and organic acids. It is used for blending with wool to provide better crease, in
safety helmets and aircraft battery boxes.
б´§³»®­ îéí

Cl CH3
CH3
12. (b) CH 2 C CH CH 2 (chloroprene) is the –H2C C CH2 C CH2–
monomer of neoprene. CH3
CH3
13. (b) Nylon-6,6 is an example of first synthetic 20. (b) Acrilan is a polyacrylonitrile ( PAN).
fibres produced from the simple molecules. It is Cl
prepared by condensation polymer-isation of |
K 2S2O8
adipic acid and haxamethylene diamine. 21. (d) nCH 2 CH C CH 2
Chloropren e
14. (b) Biodegradable polymer is Nylon-2-
Cl
Nylon-6 which is copolymer of glycin e |
(H2N – CH2– COOH) and amino caproic acid ( CH 2 CH C CH 2 ) n
Neoprene
(H2N–(CH2)5 – COOH).
nH2N – CH2 – COOH + CH3
|
glycine nH2N – (CH2)5– COOH
22. (a) Monomer of — C — CH 2 —
amino caproic acid |
CH3 n
polymer is 2-methylpropene.
O O
23. (a) Terylene is prepared by condensing
–( HN – CH2 – C – HN – (CH2)5 – C –) n terephthalic and ethylene glycol in presence of
nylon – 2 – nylon – 6 a weak base, calcium acetate.

HOOC COOH + nHOCH2 – CH2OH


15. (a) Neoprene is a polymer of chloroprene
(2 – chloro – 1, 3 – butadiene). Terephthalic acid Ethylene glycol
16. (a) Neoprene is a polymer of chloroprene.
CH3 H CH2 —C — —C —OCH2 —OCH2—
C=C 300°C
17. (b) O O
– CH2 CH2 CH3 Terylene n
Benzoyl Peroxide
CH2 CH3 H 24. (a) n CF2 CF2 [—CF2 — CF2 —]
Ammonium peroxoSulphate
C=C Tetrafluoroethene Teflon
C=C (NH 4 ) 2S2 O8
H CH2
CHn2CF
– CF2 [—CF2 — CF2 —]
Natural rubber 2 n
Tetrafluoroethene Teflon
(All cis configuration)

18. (d) Copolymer of adipic acid (6C) and 25. (b) Natural rubber is a polymer of isoprene,
hexamethylene diamine (6C).
n CH2 =CH–C=CH2 Polymerisation
n HOOC(CH 2 ) 4 COOH nH 2 N (CH 2 ) 6 NH 2
Adipic acid Hexamethyl ene diamine CH3
Isoprene
O O –CH2–CH=C–CH2 –
|| || n
( C (CH 2 )4 C NH (CH 2 ) 6 NH ) n CH3
Nylon-6,6 (2-methyl Polyisoprene
butadiene) (Natural rubber)

CH3 26. (b) nHOOC(CH 2 )4 COOH nH 2 N(CH 2 )6 NH 2


Adipic acid Hexamethylene
19. (a) nH2C = C diamine
O O
CH3
525K
[– C – (CH2)4 – C –NH–(CH2)6 – NH –]n
Polymerisation Nylon 6, 6
EBD_7324
îéì ÝØÛÓ×ÍÌÎÇ
27. (c) Phenol and formaldehyde undergo OH OH OH
condensation polymerisation under two
different conditions to give a cross linked CH2 CH2 CH2
polymer called bakelite.
OH OH OH CH2 CH2

CH2OH
OH CH2
+ HCHO +
CH2 CH2
CH2OH OH OH
o & p-Hydroxy methyl phenol OH
(intermediate) (cross linked Polymer Bakelite)

OH OH 28. (d) DNA is a biopolymer.


CH2OH 29. (c) Polytetrafluoroethylene commonly known
n Polymerisation
+ n as teflon is a tough material. This is resistant to
heat and chemicals and have high melting point.
CH2OH Therefore it is used for coating the cookware to
make them non-sticky.
ݸ»³·­¬®§ ·² Ûª»®§¼¿§ Ô·º» îéë

ݸ¿°¬»®

30 Chemistry in
Everyday Life

1. Mixture of chloroxylenol and terpineol acts as: 7. Chloroamphenicol is an : [2012 M]


[2017] (a) antifertility drug
(a) antiseptic (b) antipyretic (b) antihistaminic
(c) antibiotic (d) analgesic (c) antiseptic and disinfectant
2. Which of the following is an analgesic? [2016] (d) antibiotic-broad spectrum
(a) Novalgin (b) Penicillin 8. Which one of the following is employed as
(c) Streptomycin (d) Chloromycetin Antihistamine ? [2011]
(a) Chloramphenicol
3. Bithional is generally added to the soaps as an (b) Diphenyl hydramine
additive to function as a/an : [2015] (c) Norothindrone
(a) Dryer (b) Buffering agent (d) Omeprazole
(c) Antiseptic (d) Softner 9. Which of the following is not a fat soluble
4. Artificial sweetner which is stable under cold vitamin? [2011]
conditions only is : [2014] (a) Vitamin B complex (b) Vitamin D
(a) Saccharine (b) Sucralose (c) Vitamin E (d) Vitamin A
(c) Aspartame (d) Alitame 10. Which one of the following is employed as a
5. Antiseptics and disinfectants either kill or tranquilizer drug? [2010]
prevent growth of microorganisms. Identify (a) Promethazine (b) Valium
which of the following statements is not true: (c) Naproxen (d) Mifepristone
[NEET 2013] 11. Which one of the following is employed as a
(a) Chlorine and iodine are used as strong tranquilizer? [2009]
disinfectants. (a) Naproxen (b) Tetracycline
(b) Dilute solutions of Boric acid an d (c) Chlorpheninamine(d) Equanil
Hydrogen Peroxide are strong antiseptics. 12. Which one of the following can possibly be used
(c) Disinfectants harm the living tissues. as analgesic without causing addiction and
(d) A 0.2% solution of phenol is an antiseptic mood modification ? [1997]
while 1% solution acts as a disinfectant.
(a) Diazepam
6. Dettol is the mixture of [1996, NEET Kar. 2013]
(b) Morphine
(a) Terpineol and Bithionol
(c) N-acetyl-para-aminophenol
(b) Chloroxylenol and Bithionol
(d) Tetrahydrocannabinol
(c) Chloroxylenol and Terpineol
(d) Phenol and Iodine

ANS W ER KEY
1 (a) 3 (c) 5 (b) 7 (d) 9 (a) 11 (d)
2 (a) 4 (c) 6 (c) 8 (b) 10 (b) 12 (c)
EBD_7324
îéê ÝØÛÓ×ÍÌÎÇ

Hints & Solutions


1. (a) Dettol is a mixture of chloroxylenol and 7. (d) Chloroamphenicol is a broad spectrum
terpineol which is a very commonly known antibiotic.
antiseptic.
8. (b) Diphenyl hydramine also known as is an
2. (a) Novalgin is most widely used as analgesic. antihistamine.
Analgesics are pain releiving
9. (a) Vitamin B complex is water soluble vitamin
3. (c) Bithionol is added to soaps to impart whereas vitamin A, D, E and K are fat
antiseptic properties. soluble vitamin.
4. (c) Aspartame is stable under cold conditions. 10. (b)
5. (b) Dilute solutions of boric acid and 11. (d) Equanil is an important medicine used in
hydrogen peroxide are weak antiseptics. depression and hypertension.
6. (c) Dettol is a mixture of chloroxylenol and 12. (c) We know that N-acetyl-para-aminophenol
terpineol. (or paracetamol) is an antipyretic which
can also be used as an analgesic to relieve
pains.
Ò«½´»¿® ݸ»³·­¬®§ îéé

ݸ¿°¬»®

31 Nuclear Chemistry

1. The half life of a substance in a certain enzyme- 7. If species ba X emits firstly a positron, then two
catalysed reaction is 138s. The time required for and two and in last one and finally converted
the concentration of the substance to fall from
to species dc Y , so correct relation is [2001]
1.28 mg L–1 to 0.04 mg L–1, is : [2011]
(a) c = a – 5, d = b – 12
(a) 414 s (b) 552 s
(b) c = a – 6, d = b – 8
(c) 690 s (d) 276 s
(c) c = a – 4, d = b – 12
2. A nuclide of an alkaline earth metal undergoes
(d) c = a – 5, d = b – 8
radioactive decay by emission of the a-particles
8. When a radioactive element emits successively
in succession. The group of the periodic table
one -particle and two -particles, the mass
to which the resulting daughter element would
number of the daughter element [1999]
belong is [2005]
(a) is reduced by 4 units
(a) Gr. 4 (b) Gr. 6
(b) remains the same
(c) Gr. 14 (d) Gr. 16
(c) is reduced by 2 units
3. The radioactive isotope 6027 Co which is used in (d) is increased by 2 units
the treatment of cancer can be made by (n, p) 9. Number of neutrons in a parent nucleus X, which
reaction. For this reaction the target nucleus is gives 14 nucleus, after two successive b
7 N
(a) 59
28 Ni (b) 59
27 Co [2004] emissions, would be [1998]
(a) 9 (b) 6
(c) 60
28 Ni (d) 60 27 Co (c) 7 (d) 8
4. The radioactive isotope, tritium, (13 H ) has a half- 10. Carbon - 14 dating method is based on the fact
life of 12.3 years. If the initial amount of tritium is that: [1997]
32 mg, how many milligrams of it would remain (a) C-14 fraction is same in all objects
after 49.2 years? [2003] (b) C-14 is highly insoluble
(a) 8 mg (b) 1 mg (c) Ratio of carbon-14 and carbon-12 is constant
(c) 2 mg (d) 4 mg (d) all the above
5. U 235 , nucleus absorbs a neut ron and 235 1 236
92 11. 92 U 0n 92 U
disintegrates into 54Xe139, 38Sr94 and x. So what
fission products + neutrons + 3.20 × 10–11 J
will be the product x? [2002] 235
(a) 3-neutrons (b) 2-neutrons The energy released when 1 g of 92 U finally
(c) -particle (d) -particle undergoes fission is [1997]
6. A human body required 0.01M activity of (a) 12.75 × 108 kJ (b) 16.40 × 107 kJ
radioactive substance after 24 hours. Half life of (c) 8.20 × 107 kJ (d) 6.50 × 106 kJ
radioactive substance is 6 hours. Then injection 12. One microgram of radioactive sodium 11 24
Na with
of maximum activity of radioactive substance a half-life of 15 hours was injected into a living
that can be injected will be [2001] system for a bio-assay. How long will it take for
(a) 0.08 M (b) 0.04 M the radioactive subtance to fall up to 25% of the
(c) 0.32 M (d) 0.16 M initial value? [1996]
EBD_7324
îéè ÝØÛÓ×ÍÌÎÇ
(a) 60 hours (b) 22.5 hours 16. If an isotope of hydrogen has two neutrons in its
(c) 375 hours (d) 30 hours atom, its atomic number and atomic mass number
13. Half-life for radioactive 14C is 5760 years. In how will respectively be [1992]
many years, 200 mg of 14C will be reduced to (a) 2 and 1 (b) 3 and 1
25 mg? [1995] (c) 1 and 1 (d) 1 and 3.
(a) 5760 years (b) 11520 years 17. The age of most ancient geological formations is
(c) 17280 years (d) 23040 years estimated by [1989]
14. In a radioactive decay, an emitted electron comes (a) Potassium–argon method
from [1994] (b) Carbon-14 dating method
(a) The nucleus of atom (c) Radium-silicon method
(b) The orbit with principal quantum number 1 (d) Uranium-lead method.
(c) the inner orbital of the atom 18. Emission of an alpha particle leads to a [1989]
(d) the outermost orbit of the atom. (a) Decrease of 2 units in the charge of the atom
15. India has the world’s largest deposits of thorium (b) Increase of 2 units in the mass of the atom
in the form of [1994] (c) Decrease of 2 units in the mass of the atom
(a) rutile (b) magnesite. (d) Increase of 4 units in the mass of the atom.
(c) lignite (d) monazite.

ANSWER KEY
1 (c) 3 (c) 5 (a) 7 (a) 9 (a) 11 (c) 13 (c) 15 (d) 17 (d)
2 (c) 4 (c) 6 (d) 8 (a) 10 (c) 12 (d) 14 (a) 16 (d) 18 (a)
Ò«½´»¿® ݸ»³·­¬®§ îéç

Hints & Solutions


1. (c) For a first order reaction Hence 32 mg becomes 2 mg in 49.2 years
Total time T = no. of half lives (n) × half life 235 1 139 94
5. (a) 92 U 0n 54 Xe 38Sr X
(t1/2)
92 + 0 = 54 + 38 + a a= 0
n
N 1 235 + 1 = 139 + 94 + b b = 3 So, X = 3 0n1
N0 2 i.e 3 neutrons.
where n = no. of half lives 6. (d) Remaining activity = 0.01M
Give N0 (original amount) = 1.28 mg/ after 24 hrs
n
N (amount of substance left after time T) 1
Remaining activity = Initial activity
= 0.04 m/g l 2

n n Total time 24
0.04 1 1 1 Used half life time (n) = 4
; ; T1 / 2 6
1.28 2 32 2
4
1
1
5
1
n So, 0.01 = Initial activity
2
2 2 Initial activity = 0.01 × 16 = 0.16M
n =5
7. (a) b b 0
T = 5 × 138 aX a 1 X 1e

= 690 –2
b 8 4
a 5 X 2 He
2. (c) When IIA group element (Ra) emits one
–2
-particle its group no. decreases by two b 8
a 3X
unit. i.e., go into zero group (Gr. 16) But as –
b 12 4
it is radioactive thus due to successive a 5 X 2 He

emission last product is Pb i.e., (Gr.14).


8. (a) Mass number is effected by emmision of
3. (c) [X ] 10 n 60 Co 1 H
27 1
particle while particle has negligible mass
does not effect mass number. e.g
Balancing the mass and atomic numbers on
A
both sides 60 1 60 1 ZX Z 2 XA 4 Z 2 1X
A 4
28 X 0 n 27 Co 1 H
60 A 4
Thus X should be 28 Ni ZX
4. (c) Given t1/2 = 12.3 years 14 14 14
9. (a) 5X 6Y 7N
Initial amount (N0) = 32 mg
No. of neutrons = Mass number – No. of
Total time = 49.2 years
proton =14 – 5 = 9
T 49.2 10. (c) By carbon dating method
No. of half lives ( n ) 4
t1 / 2 12.3
2.303 N0
Age of wood t½ log
n 4 0.693 N
1 1 32
now N t N0 = 32 = 2 mg
2 2 16
EBD_7324
îèð ÝØÛÓ×ÍÌÎÇ

n
ratio of C14 / C12 in living wood N 1
14 12
ratio of C /C in dead wood N0 2
Hence it is based upon the ratio of C14 and where N0 original amount of radioactive
C12. sustacnce
11. (c) 1 atom of 235 N = Amount of substance remain after n half
92 U on fission gives energy
lives
= 3.2 × 10–11 J
n n
6.023 × 1023 atom (1 mole) on fission gives 25 1 1 1
energy = 3.2 × 10–11 × 6.023 × 1023 J or
200 2 8 2
235 gm of 235 on fission gives energy
92 U 3 n
1 1
6.023 or ; n=3
= 3.2 1012 J = 8.2 × 107 kJ 2 2
235
now T = n × t½
1
n
N0 where T = total time
12. (d) N N0 ; here, N T = 3 × 5760 years = 17280 years
2 4
14. (a) When a radioactive elements emits or
N0 1
n
1
n
1
2 particle the new element formed may have
N0 n=2 unstable nucleus. It may further disintegrate
4 2 2 2
by emitting - or particle forming a new
Total time (T)= n × t1/2 = 2 × 15 = 30 hrs
element. This process of integration may
13. (c) Half-life of 14C = 5760 yrs; Initial weight of continue till end product formed is a stable
14C = 200 mg and final weight of 14C = 25
compound.
mg. 15. (d) The ore of thorium is monazite.
200 16. (d) As number of neutron =
Quantity left after 5760 years = 100 mg;
2 Mass number – atomic number
Similarly quantity left after another 5760 Give number of neutron = 2
Mass number will be 3 and atomic number
years (i.e 11520 years) 100 = 50 mg. will be one.
2
17. (d) Age of geological formations (i.e. predicting
Quantity left after another 5760 years
the age of the earth and rocks) is estimated
(i.e. 17280 years) 50 by U– Pb method, also known as helium
25 mg.
2 dating.
Thus time taken by 200 mg of 14 C to reduce Note : C-14 dating method is used to predict
the age of fossils or dead animals or a fallen
to 25 mg = (5760 + 5760 + 5760 ) years = 17280
tree.
years.
Alternative solution 18. (a) Emission of -particle ( 42 He ) leads to
As we know that decrease of 2 units of charge. e.g
238 234
92 U 90 Th 2 He 4

You might also like